You are on page 1of 736
COMPLETE SOLUTIONS MANUAL FOR ZILL’S A FIRST COURSE IN DIFFERENTIAL EQUATIONS WITH MODELING APPLICATIONS 77H EDITION AND ZILL & CULLEN'S DIFFERENTIAL EQUATIONS WITH BOUNDARY-VALUE PROBLEMS ‘STH EDITION BROOKS/COLE eE—e Australia + Canada + Mexico + Singapore + Spain + United Kingdom + United States Table of Contents 2 Inteoduction to Differential Equations 1 2 First-Order Differential Equations 2 3 Modeling with First-Order Differential Equations n 4. Higher-Order Differential Equations 104 5 Modeling with Higher-Order Differential Equations 194 6 Series Solutions of Linear Equations 240 7 The Laplace Transform 308, 8 Systems of Linear First-Order Differential Equations 370 9 Numerical Solutions of Ordinary Differential Equations 430 10 Plane Autonomous Systems and Stability 458, 22 Orthogonal Functions and Fourier Series, 491 12 Partial Differential Equations and Boundary-Value Problems in Rectangular Coordinates 538 13 Boundary-Value Problems in Other Coordinate Systems 616 24 Integral Transform Method 654 15 Numerical Solutions of Partial Differential Equations 695 Appendix? Gamma function a7 Appendix Introduction to Matrices, 78 1 Introduction to Differential Equations Exercises 1.1 1. Second-order; linear 2. Thisd-order; nonlinear because of (dy/dz)*. 3. ‘The differential equation is first-order. Writing it in the form 2(dy/dzx) + y? = 1, we see that it is nonlinear in y because of y®. However, writing it in the form (y? ~ 1)(dr/dy) += 0, we see that 4. The differential equation is first-order. Writing it in the form u(du/du) + (1+ ue = ue we see that it is linear in v. However, writing it in the form (v+ uv ~ ue¥)(du/dv) + u = 0, we see that it js nonlinear in u 8. Fourth-order; linear 6. Second-order; nonlinear because of cos(r +) 7, Second-order; nonlinear because of y/1 + (dy/da?* 8, Second-order; nonlinear because of 1/8? 9, Third-order linear 10, Second-order; nonlinear because of #? LL. From y= 2/24 esi? 212 we obtain y= —fe#/2, Then 2y!+y 12, From y= § ~ fer we obtain dy/dt = 24e"*, so that B Ys oy 8 ay = ne 20 (8 Ser) a 5 13. From y = ecos2z we obtain y/ = Se cose — 2e™ sin2x and y" = Se cos 2x — 12e sin 2z, s0 that y" — 6y' + 13y = 0. 14. From y = ~ cos n{secr + tan2) we obtain y/ = ~1 + sin ln(sec + tanz) and yf = tanz+ coszin(secs + tanz). Then y+ Exercises 1.1 15. Welting In(2X ~ 1) — In(X — 1) = ¢ dnd differentiating implicitly : we obtain ax 7X — WX 1) = (K-22. Exponentiating both sides of the implicit solution we obtain 2X a1 Cg . Sea ae mo ON 1a Kee =e (¢ -2) =f “YK SX Solving e! ~ 2= 0 we get t= In2, Thus, the solution is defined on (20, ]n2) oF on (In2, 00). “The graph ofthe schvion defined on (00, n2) is dashed, and the graph of the solution defined on (112,00) is soli. dou a3 16. implicitly differentiating che solution we obtain 2228 soy 4 ny = Pay —28y = z gxo dy ~2syde + ydy = 0 => days + (2? -y)dy =O. Using the quadratic formola to solve y?—224y— 1 = 0 for y, we get y= (022s fiated)/2 = 2s VeFT. Ths, two explicit solutions are yi = 2°+ VETFT and y= 2? - vztFT. Both solutions are defined on (90,00). The graph of yx(z) is solid and the graph of wp Is dashed. 17. Diferentiating P = eet/(1-+exe) we obtain ap _(i+ad)ae-aet-aet a (Fae) aet_{(t+ae!) -ae! = ae eae) 8) ~ pa - p), Tad Tat 18, Doering =e? [at ae on btn yao met [oat rez mane [PoP at — 2oyze 2 Exercises 1.1 Substituting into the differential equation, we have ee [ean taae? 420” [Pas te yl + 2ry = 2 2s ab 2 a en) 42a and PH = (ey deni dene” 19, From y = cie™* +eqze™ we obtain so that (dey + don — Bea ~ dep + 4eyJe™ + (dex — Ben + don}ze™ 20. From y= e127! + cpr + eyeing + 42° we obtain Ba eo? cnt Beart sqsostesinetée a PY arya eee 48, and so that by [Bey + dey + ey bey)a™? + (e+ 2c oon + on) +(e teslelne + (16-8 +4)2? = 122" 21. (a) From 4 = 2? we obtain df = 22, s0 20, ~ 26, = 2(2x) 20? =0 From dy = 2? we obtain 6 = ~2z,s0 26h 26 Thus, dy and dp are solutions of the difetial equation on (~00, 20) ae lr, 2<0 (b) From v-{2 2S) we obtain -{2 FSF so that ey ~2y=0, on 5, Thus, y'(x) 22, The function y(2) is not coutinuous at 2 = 0 sinee Ii does not exist at 2 = 0. 23. (a) The domain of the function, found by solving 2 +3 > 0, is [-3, 00), 3 (22) -2(-24) =0 24. 25. 26. a7. Exercises 1.1 (b) From pf = 1+ (243)-1/? we have (yra)y! yt 2-2 fet VES Halt + (1+ (2-3) ~ [e+ 2VEF3] 42-2 =2aF5+2-2-2VeF3+2-2=0 Since y(x) is not differentiable t x = ~3, is a solution ofthe diferential equation on (3,20). (a) An interval on which tans is continuous is ~*/2 < 5t < m/2, so 5tan5t will be a solution on {—m/10, 3/10). (b) For (1~sin)-¥/ to be continuous we must have L—sint > O orsint < 1. Thus, (1~siné)-!/? ‘will be a solution on (x/2, 57/2). (a) From y=" we obtain y/ = me", Thun y +2y =D implies (m+2e™ = 0. me™ +26 2. Thus y rt we obtain yf = me™ and "= mBe™. Then y" — Sy + 6y = 0 implies mem! — 5me™ + 62 = (mm — 2}(m —3)e™" = 0, Since e™ > 0 for all t, m = 2 and m= 3. Thus y = e* and (a) From y= t” we obtain of = mt”! and y/" = m(m — 1)t"-®. Then ty +24 tm(m — 1)e™2 4 met f(a ~ 1) + Qmfe™2 = (m2 + myeml Since &™ > 0 for all f, m= -* is a solution (b) From = e* are solutions implies 0. and m= —1. Thus y= 1 and y = ¢# are solutions. a mm +e Since &*-! > 0 for t > 0, m (b) From y = # we obtain y = mi"! and y” = m(m — 1)0"-?. Then Py!" — Tty/ + 1Sy = implies Palm ~ 1}e"-3 — Tema™# 4. 15E™ = [mlm — 1) — Tm + 15]e” (me? — 8m + 15)t™ = (m— 3)(m ~ 5)" = 0. Since #* > 0 for #> 0, m=3 and m=5. Thus y = ¢? and y = #5 are solutions. From +52 we obtain 34 3e% and y ie 2 18st 1 Botte we and Fh = Be + 30d Then Bt Sy = (o% + 30%) +(e + Set) a a = 224 18e = 26% 4 18 = and Exercises 1.1 Sat By = Se + Se) + 3(—e-™ + Be) = 26 + 306! 28. From x = cos2t + sia2t + Je! and y = — cos2t ~ sin2t — Jet we obtain de ie &y SE x asin 2 + 2005: and! = 2sin2t — 200821 - : GF = ~Bsin 2+ 2eos2t + Gh = Bsin2t ~ 20062 ét i ey Gams tt—dsinde + Zeb and Fh = 40828 + sin Then ay +e! = 4(—cos2t — sin2t — Bet) +e! Le ee = ~deos2t— Asin 2+ Pet = 0 +ie=G and : = A(cos2e + sin 24+ Ze) — et Le ey = 4c0s2t-+ asin at — bet = EY eos 2t + s- Ge 29. (y’)? +1 =0 has no real solution, 30, The only solution of (9)? + 9? since if y #0, 4? > O and (y/)? + y? > y? > 0. 1. The fret derivative of f(t) = ef is ef. The fst derivative of s(t) =e! jo keM, The dierentel equations are y' = y and y' = Ay, respectively. 82 Any function of the form y = ce! or y = ce~ is its own second derivative. The corresponding SAO) = ONAL) ~ vf Teal) where we assume that ¥1(0)vs(0) — ;(0)ya(0) # 0. For the first-order differential equation integrate (zx). For the second-order differential equation integrate twice. In the latter case we get y = f(/ f(t)dt)at + ot + c2 Solving for y/ using the quadratic formula we obtain the two differential equations Lp é 1 (a-a/iaai), (+2 Tra) andy’ 0 the differential equation cannot be put in the form dy/dt = f(t,y). ‘The differential equation yy'—ty = 0 has normal form dy/at = t. These are not equivalent beceuse 1y=(isa solution ofthe fist differervial equation but not a solution ofthe second, Differentiating we get v= c1 + ext? and y”" = Goat. Then cy = y"/6t and cy = y’ ~ ty"/2, 90 0) Coy te v= (v-£)e+ (Bean fev and the differential equation is #%y" — 3ty' + 3y = 0. When g(t) = y= 0 8a solution ofa near equation (a) Solving (10 ~ 5)/82 = O we see that y = 2s & constant solution (b) Solving y? + 2y—3 = (y-+3)(y— 1) =0 we soe that y = —3 and y = I are constant solutions, () Since 1/(y ~ 1) = 0 has no solutions, she differential equation has no constant solutions (A) Setting y' = 0 we have 1/’ = 0 and 6y = 10. Thus y = 5/9 is 2 constant solusion, ‘One solution is given by the upper portion of the graph with domain approximately (0,26). The other solution is given by the lower portion of the graph, also with domain approximately (0,26) ‘One solution, with domain approximately (co, 16) is the portion of the graph in the second quadrant together with the lower part of the graph in the fist quadrant. A second schution, with domain approximately (0,1.6) is the upper part of the graph in the first quadrant, The third solution, with domain (0,0), is the part of the graph in the fourth quadrant. 6 Exercises 1.4 43. Differentiating (2? + y*)/2y y(S2? + 30% ate sayy — ay - tyr ay v4 ‘any — 24 — apy = -30'y + ay ty vin 2rty _ yy! — 2x8) Depot ™ a7) 44. A tangent line will be vertical whore of i8 undefined, or in this ease, where 2(2y? gives 2 = 0 and 2y° = 23. Substitming v= 29/2 into 2° + y? = Say we get 0. This 8) 20, ‘Tis, there are vertical tangent limes at x = 0 and ¢ = 22, or at (0,0) and (229,249). Since 2/3 = 1.59, the estimates of the domains in Problem 42 were close. 45. Since > 0 for all xin J, ¢{2) 6 an increasing function on J, Hence, it can have no relative extrema on I 46. (a) When y = 5, y’ = 0, s0. y= 5 is a solution of y' = 5—y. (b) When y > 5, y’ < 0, and the solution must be decreasing. When y < 5, y/ > 0, and the solution must be ineressing. Thus, none ofthe curves in color can be sontions. (e) ” 0. pets 47. (a) y= and y= a/?. (b) Since dy/dz = yla— by} > for 0 < y < a/b, y= d(x) is increasing on this interval. Since dye < 0 for y <0 or y> aft. y= O(c) is decreasing on these intervals, 7 Exercises 1.1 (c) Using implicit differentiation we compute (a 48, The fomily of parabolas is plotted using ¢; = +1, +4, +10. The ellipses are plotted using cz = 1, 2, 3. It appears from the figure that the parabolas and ellipses intersect at right angles. To verify ‘this we note that the first differential equation can be written in the form dy/dx = y/2r and the second in the form dy/dr = —22/y. ‘Thus, at a point of intersection, the slopes of tangent lines are negative reciprocals of each other, and the two tangent lines are perpendicular. 49. In Mathematica use Clearly] y(x}:= x Exp[5x| Cos(2x} yes) yee] — 20 yr ]x) + 158 yr 50. In Mathematica use — 580 y'[x] + 841 y[x] // Simplify Clear{y] ylx_]= 20 Cos{S Log{x]]/x ~ 3 Sin|5 Log|x]}/x ybd 28 yr x] + 2x72 yrefx] + 20 x y'lx] — 78 ylx] // Simplify Exercises 1.2 — Exercises 1.2 ——_______________ >. The solution Is ee 3 ~eysint + cacost we obtain ¢, = ~1 and c2 =8. The solution is 2 = ~wast + 8sint 4 oy sin¢ + epco8t we obtain y= 0 and ~c) = 1. The Solution is z ~ —cost 5. Using x= ~cysint + ercost we obtain Led at ea ta+ Bano 8 1 8 Solving we find cs = YS and ca = 4. The solution is 8. Using 2! = cr sin# + epcost we obtain Solving we find ¢) = ~4 and ey = 3, The solution is = = ~ cost + Ssine 7. From the intial conditions we obtain the system tea) ce Solving we get cy = § and cy =~}. A solution oft initial-value problem is y = fet — fer 8. From the initial on tons we obtain the system qet ae ae~ met =e, Solving we get cy = J and cy = —Je2. A solution ofthe initial-value problem is 9. From the initial conditions we obtain ‘Solving we get c = 0 and cy = Se". A solution of the initial-value problem is y o Exercises 1.2 10, From the initial conditions we obtain Solving we get ¢; = cz = 0. A solution of the initial-value problem is y = 0. 11. Two solutions are y =O and y = 3° 12. Two solutions are y = O and y= 22, (Also, any constant multiple of 2? is a solution.) 13. For f(y) = fr we nave 3 = 2y 19. (Thus the differential equetion will have a unique solution in any rectangular region ofthe plane where» 0 eet on ie 14. Por f(z,3) = 29 we tare 2 = 1 FE, thus the dferetial equation will havea una aclution fas) = Ai wince Fo BE a . sn any rogion where > O and y > Dot where 2 < and'y <0 15. or fea) «2 vedne f=, taste diel cut i be ie hn ny region where 2 #0. af 26, For flay) <2 + y we have ZT = 1. Thus the difeental equation will havea unique solution in the entire plane arty ZELS. Thus the differential equation will have a unique @-ey solution in any region where y < ~2, -2 2 af | =a ay we have F = EY, Tus the differential equation will have a unique have 5 > agar Ts 2 qua i 18. For f(z.y solution in any tegion were y # 2 af 2% ¥ a we have SF PFU Ba OO By yt solution in any region not containing (0,0) 19. For (2,9) ‘Thus the differential equation will have a unique V2 vepme a ; 20, For flay) = #2 & 7 Ga solution in any cegon whore y <2 of where y > 2 ‘Thus the diferential equation will have a unique 21, The differential equation has a unique solution 2% (1,4), 22. The differential equation is not gusranteed to have a unique solution at (5,2). 23. The differential equation is not guaranteed to have @ unique solution at (2,—3) 10 Exercises 1.2 24, The differential equation is not guaranteed to have a unique solution at (—1, 1). 25. (a) A one-parameter family of solutions is y = or. Since y’ =e, 2y' = ze = y and y(0) = ¢-0=0. (b) Writing the equation in the form y' = y/z we see that R cannot contain any paint on the y-axis, ‘Thus, any rectangular region disjoint from the y-axis and containing (o,yo) will determine an interval around ro and a unique solution through (z0, ya). Since 29 = 0 in part (a) we are not, guaranteed a unique solution through (0, 0). (c) The piecewise-defined function which satisfies y(0) entiable at x = 0. is not a sclution since itis not differ. 26. (a) Since A tan(e +c) = sex%(2-+.0) =1.+tan%(e +c), we se that y = tan(a-+ 6) satisfies the diferettial equation (b) Solving (0) = tanc = 0 we obtain ¢ = 0 and y = tanz. Since tanz is discontinuous at 2 = r/2, the solution is not defined on (2,2) because it contains -tn /2 (c) The largest interval on which the solution can exist is (—7/2,/2} 1 js a solution of the differential 21. (a) Since £(- 1) me , we see that y equation, (b) Solving y(0) = -I/e = 1 we obtain € = ~1 and y = 1/(0 1). Solving ¥(0) = ~ wwe obtain ¢ =] and y = -1/(1 +1). Being sure to include ¢ = 0, we see thet the interval of existence of y = 1/(1.~1) is (00,1), while the interval of existence of y = ~1/(1 + ) is (ie) {e) Solving y(0) = —1/e = yy we obtain ~liye and 1 — Theo Since we must have ~1/yp +t 7 0, the largest interval of existence (which must contain 0) is either (oo, 1/yo) when yy > 0 oF (2/30, 0e) when yo < 0 (4) By inspection we see that y = 0 is a solution on (00, 60) 28. (a) Differemiating 32° — y# = ¢ we get Gx ~ 2yy' = O or yy = 30 n Exercises 1.2 (b) Solving 32? ~ y? = 3 for y we get y=dulz)=Y5@—1, L<2<0o, yedlz)=-y5@?=), 1 0, for y > 2 ~ 1, we see that solutions above the line y= x —1 are increasing. From y/ < 0, for y < 7 — 1, we see that solutions are decreasing below the line y= 2-1. From y” = 0, for y ‘that solutions have possible infection points on the line y = x. Actually, y= 2 is easily seen to be a solution of the differential equation, so the solutions do not have infection points. From y/" > 0 for y > 7 we see thet solutions above the line y = x are coucave up, From y"" < 0 for y < x we see that solutions below the line y are concave down, Exercises 1.3 ap oka Let bbe the rate of births and d the rate of deaths. Then b the differential equation is dP/dt = kP ~ kyP. 3. Let bbe the rate of births and d the tate of deaths. Then b= kyP and d = kyP?, Since dP/dt = bd, the differential equation is dP/dt = kyP ~ kyP?, 4, Let P(t) be the number of owls present at time t. Then dP/dt = k(P — 200 + 101). 5. From the graph we estimate Ty = 180° and Tm = 75°. We observe that when T = 85, dT/dt = From the ciffeential equation we then have aT /dt T-T, 5-3 6. By inspecting the graph we take Ty, to be Tm(t) — 30.cos mt/'12. Then the temperature of the body at time tis determined by the differential equation »P and d= kyP. Since dP/dt = b~d, k 01 Da gfr-(o-a0aeds)], 190 1. The mane fstodens wih he B65 andthe umber no ted s 1000 2, 0 det = x(1000 - z). By anlgy with frente equation modding he pedo tate we ssme ht te ae shih she echlga!inosin doped rpc ote uber fpr wo he doped he innrton nd ln ote mob f peal), who ar ote doped Mone pene wh ns doped the inno since int te poplin ten +4 = ad te Fa kaeti—2), 20) <1 13 Exercises 1.3, 9. The rate at which salt is leaving the tank is A A (3 gal/mia)« ( to/eal) = 5 o/min Thus dA/st = 4/100 10. ‘The cave at which salt i entering the tank is Ry = (3 gal/min) (2 1b/gal) = 6 {min Since the solution is pumped out ata slower rat, iis sccumulating at the rate of (3~2)gal/min = 1 gal/min. After ¢ minutes there are 300+ ¢ gallons of brine in the tank. ‘The rate at-which salt is leaving is =0 A t/a) = 24 a= (2 gal/min) Gas tb/eal) Hoey be/ein ‘The differential equation is aa 24 a 7°" Sort 11, The volume of water inthe tank st time ¢ is V = AyA, The differential equation is then Aa es BO Ae ae = Ay (avs) = S22 Using A 100, and gy = 92, this becomes dh __on/38 = 7 0g YOR = ~ a5 YF 22, The wolume of water in the tank at time tis V = rr = }dwh Using the formula from Problem 1! forthe volume of water leaving the tank we see thatthe dieental equation is ‘Using An = #(2/12)? = 1/36, 9 = 32, and c = 0.6, this becomes & Sea vem, : < AM? "Tp find Ay we let r be the ead ofthe tp ofthe water. Then r/h = 8/20, so = 2h/S and Aw = (24/5)? = 4rh?/25. Thus _ dt inh? 25, AVE we 25h? ay eq ae 04 Fagg + 's second law we o a 14, By Kichof's second law we obtain RSE + 29 = Bit) 13. Since aa Ae RG = B(t) we obtain LE + Ri = Bt) 14 46. 16. ar. 18. 19. 20. Exercises 1.3 From Newtons second low we obtain ml = kg We hav fm Archie ptt upward force of water on berrel = weight of water displaced = (62.4) x (volume of water displaced) = (62.4)n(s/2)y = 15.678%y. dy = _ : ae? 0, where Jc then fllows from Newton's second law that = 32 and w és the weight of the barrel in pounds. The net force acting on the mass is és Pm moa mq = -Ks42) +mg = ke + mg ~ hs Since the condition of equilibrium is mg = ks, the differential equation is es mig = nhs From Problem 17, without a damping force, the ifferential equetion is mds/dt? = ~kx. With a damping farce proportional to velocity the differential equation becomes @s de @s | ade moze kro or moe + oe Let 2(¢) denote the height of the top ofthe chain at time t with the positive direction upward. The ‘weight of the portion of chain off the ground is W = (r fe): (1 lb/ft) = 2. The mass of the chain is +ke = 0, m= W/a = 2/32, The net force ie F = 5-1 =5—z. By Newton's second low, ay2 dy, ye é@) or ihe =o) a2e “ins be ferential equation is @x dzy2 224 (BY + m2 = 100 ‘The force is the weight of the chain, 2, so by Newton’s second lew, mu} = 22. Since the mass of the portion of chain off the ground is m= 2(L-~ 2}/a, we have 2 22. 23, 24, 26. 26. ar. 28. 29. Exercises 1.3 |. From 9 = &/R? we Snd k = gf Using a = d’r/dé? and the fact that the positive direction i upward we get @r | oR? ore no. ete ‘The gravitational force on m ie F = —kMym/r®, Since Me = drdr®/3 and M = 4x6R8/3 we have My = OM/R? and Mem mM Pook aM Now from F = ma = d?r/dt? we have Pr __ ym er __ kM a a a Thee aon = ‘The differential equation is i Ay(Mf ~ A) — be ~ kat) where & > 0. a ‘The differential equation is 2'¢) By the Pythagorean Theozem the slope of the tangent line is y/ We see from the figure that 28+ a=. Thus eee __2tand 2 = tana = tan(s — 26) = ~ tan 26 = Since the slope of the tangent lines y/ = tan we have y/2 = 2y/1—(v’)] or y~vlv)? = 2ny’, which is the quadratic equation y(u')® + 22y —v =0 in yf Using the quadratic formula we get are (Ty ee PTH fy y Since dy/dz > 0, the differential equation is v dy ~tt+ ety a gee vZ-VFrPt+e=0 ‘The differential equation is dP/dt = kP, so from Problem 31 in Exercises 1.1, P =e“, and a ‘one-parameter family of solutions is P = ce®. ‘The differential equation in (3) is d7/dt = 4(T - Tm). When the body is cooling, T > Tm, 80 TT > 0. Since T is decreasing, d7'/dt <0 and k-< 0, When the body is warming, T < Tin 30 T-Tq <0, Since Tis increasing, dP/dt > 0 and k <0. 16 30. al. 32. 33. 34. 35. Exercises 1.3 ‘The differential equation in (8) is dA/dt = 6 ~ A/100. If A(t) attains « maximum, then dA/dt = 0 ‘at this time and A = 600. If A(t) continues to increase without reaching a maximum then A'(t) > 0 for t > Q and A cannot exceed 600. In this case, if4"(t) approsches 0 as ¢ increases to infinity, we see that A(t) approaches 600 as t increases to infinity. ‘The input rate of brine is 7; gal/min and the concentration of salt in the inflow is ¢;Ib/gal, so the input rate of salt is ricilb/min. ‘The output rate of brine is r, gal/min and the concentration of salt in the outflow is clb/gal, so the output rate of salt is rsclb/min, ‘The solution in the tank is accumulating at a rate of (7: ~ re) gal/min (or decreasing ifr, < 79). After t minutes there are Vo-+ (r1—re}t gallons of brine in the tank, and the output rate of salt is roA/[Vo+ (r,~ra}]lb/min. ‘The differential equation is Se @ "Ott ‘This diferenial equation could describe a population thet undergoes periodic Ructuatons a) Dear is tinea 2) aba to tncar @): Fawr) is tne @: 2 @: Za kla—x(G-X) isronlinar (8): (oy tha AR euotnar (02, Gh m4 Sstner oa) (05), mE 4 A = ng is nee (16) 2 0 is linear From Problem 21, d?r/dt? = ~9R2/r?. Since Ris a constant, ifr = Rs, then dr/dt? = ds/at? and, using a Taylor serie, ve get @ TR Thos for R much lars than s, the df ~oIO(R 4 9)? GHEY? = 26 +g Bt ential equation is approximated by ds/ai? = —g Hf pis the mass density of the raindrop, then m = pV” and dm tr) 9s a a aa If dr/at is & constant, then dm/dt = KS where pdr/dt = or dr/dt = k/p. Since the mdius is decreasing, k <0. Solving dr/dt = k/p we get r = (/p)t + ea. Since r(0) = 79, e9 = ro and r= ktip+ro adr. 7 36. Exercises 1.3 d From Newton's second law, mo] = mg, where vis the velocity ofthe raindrop. Then 4 8 ang oe ol Sar) + eter? met omy (gar) wasn?) Dividing by 4onr3/3 we get We assume that the plow clears snow at a constant rate of k cubic miles per hour. Let ¢ be the time in hours after noor, z(t) the depth in miles of the snow at time t, and y(t) the distance the plow has moved in ¢ hours. Then dy/dt is the velocity of the plow and the assumption gives d wrt ak where w is the width of the plow. Each side of this equation simply represents the volume of snow plowed in one hour. Now let tp be the number of hours before noon when it started snowing and let s be the constant rate in miles per hour at which s increases, Then for t > —to, x = s(t + to). ‘The differential equation then becomes ea at" us TF Integrating we obtain ve Epaletey +e ee or eee ry io wats) Final om the fact that when 2 and when t= 2, y= we obtain | |. 2y (+d (+) (+8) ‘Expanding and simplifying gives ¢3 + tp — 1 = 0. Since to > 0, we find ty = 0.618 hours = 2 minutes, Th started snowing at about 1:29 in the toraing 18 Chapter 1 Review Exercises Chapter 1 Review Exercises 4 ct meet, 1 dae mekets Baty 2 25) d et ae Dee me ~2(5 + ee -5); a d 3. Flercoske + cysinkz) = ~ho sinke + kepcos ks; Ho, cos ka ~ Hep sin ka = ~F(c; cos ka + en sin ke) @y a oo Hy on Sh+ By a0 4. Atercosh ke + casnh he) = hey sinb he + kop cosh ks ey a 5. ym equel toate tone, y= cre" + cate? + Denes Wl ty Ace? + ere!) + exe = ore + eazet + cnet) = By" y"— Iv + 6. y= ac sing + cet cose tae cose +e sin of = ~cyeF cos — ye? sing ~ ee sine+ ce cos Hoe sing + Ree cos. w #4 a Pyn0 Hsing boge™ cost + eye cos + one" Sint Deve? cos:r — Deze sine y yo Oy + Dy 7. ad Be ab 10. ac Mb 12, abd 13. A few solutions are y = 0, candy =e 3 15. The slope of the tangent line at (2,4) is y/, so the differential equation is y= 2? + y?, 14, Easy solutions to see are y = O and 16. The rate at whica the slope changes is dy//dz = y", so the differential equetion is y/” vv or 17. (a) The domain is all real numbers, (b) Since xf = 2/324! the solution y = 24/* is undefined at x = 0. This function isa solution of the differential equation on (20,0) and also on (0,0). 18. (a) Differentiating y? —2y = 2% — 2+ we obtain 2yy/ ~ 2y/ Lor (2y~2)y/ =27-1 19 Chapter 1 Review Exercises {b) Setting z = 0 and y = 1 in the solution we have 1~ of the initial-value problem is y® ~2y = 2? — 2-1. (c) Solving y?—2y—(z?-2-1) = O by the quadratic formula we get y = (24 4+ We —2—1))/2 = lt VeTaa = 14 se 1). Since z(z— 1) 2 0 for x <0 or 2 > 1, we see that neither y=1+ yae>1) nor y=1- y/2[e—1) is diferentiable at r= 0. Thus, both functions are solutions ofthe differential equation, but neither is a solution of the initial-value problem, Yo & =O-+e or e=—1, Thus, » solution 19. (a) oy (b) When y 27+ eee When ya? ay yf = ~2e and (/}? = 2 <0 230 (c) Pasting together 20, and —2?, 2 <0, we get y= te 20. The lope ofthe tangent line is |. 6V4+5(-1)® = 7. 21. Differentiating y = sin(In 2) we obtain y/ = cos(In2)/z and y" = -[sin(In2) + cos(In2)]/2® Then ay" sty" bay! + So(ins) 4 eosin) 220) 5 sn(inz) = 22. Differentiating y = cos(In x) In(cos(Inz)} + (In) sin(Inz) we obtain: sin(lna) sin(inz), |, eos = costing) ry (SAD) + con tnsy)(-S8U22)) + nS) _Hn(cos(in2)sin(nz) | (nz) coslin2) asd y= = [invonon sy 222 satan ot (—SBUB)] : + lo(costing) sn(na) + lny(—AE2) « U82)| Zn) oot sin?(nz) . |= In(cos({ln 2) cos(In zr) + ‘coalin) + In(cos(in z) sin(In x) ~ (Inz) sin(in 2) + cos(In 2) ~ (In:z) cos(In: 2) 20 Chapter 1 Review Exercises ‘Then sin®(In 2) ay! + ay + y= —In(cos(ina))oos(in2) + See + In(cos(In)) sin(in 2) ~ {In 2) sin(ln 2) ++ cos(ln2) ~ (in2)cos(tn2) — In(costin2))sin(Inz) + (nz) cosIn-z) + ecatn2)In(cos(tn2)) + (In2) sin(In 2) z)+cos?(Inz) 1 sin%(nz) cosine) eestinz) ‘oslin) cos(lnz) = see(In2) (This problem is easily done using Mathematica) 23, From the graph we see that estimates for yo and gy ate wo = —8 and yy = 0. 24, The diferential equation is ah cho the ooh aa, Using Ap = #(1/24)? = 1/576, Ay = (2)? = dr, and g = 92, this becomes 21 2 First-Order Differential Equations Exercises 2.1 Exercises 2.1 9 10 a. 12. 13. 4. 15. Writing the differential equation in the form dy/dr = y(1—y)(1-+y) we see that critical poi located at y= 1, y= 0, and y = 1. The phase portrait s showa below. a ee @) () ll © @ 23 16. a. 18, 19, 20. a. Exercises 2.1 Whiting the diferentia equation in the form dy/de = y2(1 ~ y)(U + y) we see that critical points are located at y = —1, y =0, and y = 1. The phase portrait is shown below. (a) 4 (b) ' ©) (a) Solving y? — 3y = yly ~ 3) = 0 we obtain the critical points O and 3. a ° 2 From the phase portrait we see that 0 is asymptotically stable and 3 is unstable Solving 9? ~y° = y2(1 — y) = 0 we obtain the critical points 9 and 1 a 2 t From the phase portrait we ste that 1 is asymptotically stable and 0 is semi-stable, Solving (y — 2)¢ = 0 we obtain the critical point 2 + 2 From the phase portrait we see that 2 is seri-stable. Solving 10 + 3y — y? = (5~y)(2+y) =0 we obtain the critical points ~2 and 6. From the phase portrait we see that 5 is asymptotically stable and ~? is unstable Solving (4 —y?} = 2 — WI + y) wwe obtain the critical points ~2, 0, and 2. a ° 7? 24 22. 23. 24. 25. 26. Exercises 2.1 From the phase portrait we see that 2 is asymptotically stable, 0 is semi-stable, and ~2 is wistable. Solving y(2- y)(4 — y) =O we obtain the critical points 0, 2, and 4. é 7 q From the phase portrait we see that 2 is asymptotically stable and 0 and 4 are unstable Solving yln(y +2) = 0 we obtain the critical points ~1 and 0. f : $ From the phase portrait we see that ~1 is asymptotically stable and 0 is unstable Solving ye? — 9y = y(e? ~ 8) = 0 we obtain the rtiel pein O and In9 at From the phase portrait we see that 0 is asymptotically stable and In9 is unstable. (a) Writing the differential equation in the form rs a ‘we see that a critical point is mg/k. From the phase portrait we see that mg/E is an asymptotically stable critical point. Thus, mgfk (b) Writing the differential equation in the form e Ame ee ame n sce that a etal pin eGR Timed Yak From the phase portrait we see that /mg/k is an esymptoticaly stable critical point. ‘Thus, limo v = mg/h (a) From the phase portrait we see that critical points are a and 8. Let X(0) = Xa. a e 6 If Xo 8, we see that X(¢) increases in an unbounded manner, but more specific behavior of X(t) a8 t ~ 00 is not known. (b) When a = 6 the phase portrait is as shown, yp If Xo a, then X(t) increases in an unbounded manner ‘This could happen in a finite amount of time, That is, the phase portrait does not indicate that X becomes unbouncied as # — oo. (c) When & = 1 and a = 8 the differential equation is dX/at = (a — X)*. Separating variatles ‘and integrating we have For X(0) = a/2 we obtain For X(0) = 2a we obtain For Xo > a, X(t) increases without bound up to = 1/a. For t > L/a, X(t) increases but Xs aast oo. 26 Exercises 2.1 27. Critical points are y = 0 and y = c. 20. At each point on the circle of radius ¢ the lineal element has slope c2 30. (a) When 2 = 0 or y= 4, dy/dr = -2 s0 the lineal elements have slope ~2. When y = dy/dz = x — 2, so the lineal elements at (x,3) or (2,5) have slopes 2 — 2 (b) At (0,40) the solution curve is headed down. As x increases, t will eventually turn around and hhead up, but it can never cross y = 4 where a tangent line to a solution curve must have slape ory=5, 0. Thus, y cannot appreech oo as z approaches 00 31, When y < 22, / = 22 2y is positive and the portions of solution ‘carves “inside” the nulleline parabola are increasing. When y > 32, y = 2? — dy is negative and the portions of the solution curves “outside” the mulleline parabola are decreesing. 82, For dr/at = 0 every real number is critical point, and hence all critical points are nonisolated, 83. Recall that for dy/d2 = f(y) we are assuming that f and f” are continuous functions of y on some interval J. Now suppose that the graph of nonconstant solution of the diferential equation 27 34, 35. Exercises 2.1 crosses the line y = If the point of intersection is taken as an intial condition we have two distinct solutions ofthe initial-value problem. This violates uniqueness, So the graph of any nonconstant solution mast lie entirely on one side of any equilibrium solution. Since fis continuous it can only change signs around a point where itis 0. But this is a critical point. Thus, fy) ia completely positive or completely negative in each region 2. If y(z) is oscillatory or has a relative extremum, then it must have a horizontal tangent line at some point (29,20). In this case yo would be a critical point of the differential equation, but we saw above that the graph of a nonconstant solution cannot intersect the graph of the equilibrium solution y = yo By Problem 33, a solution u(2) of dy/de = f(y) cannot have relative extrema and hence must be monotone, Since y'(z) = {(¥) > 0, x(x) is monotone increasing, and since y(z) is bounded above by cn, limo u(z) =f, where L < ez. We want to show that L = ca. Since L is a horizontal asymptote of y(z), limo /(c) = 0. Using the fact that /(y) Is continuous we have HE) = Him, v(e)) = Jim, fev) = Jim, ve But then Z is a critical point of f. Since ¢ < L < ca, and f has no critical points between ¢) and bee, (a) Assuming the existence of the second derivative, points of inflection of y(x) occur where y"(z) = 0. Pram dy/dz = gly) we have dy/de? = g/(y) dy/dz. Thus, the y-coordinate of a point of inflection can be located by solving g'(y) = 0. (Points where dy/dz = 0 correspond to constant solutions of the differential equstion,) (b) Solving y?—y~6 = (y—3)(y+2) = 0 we see that 3 and —2are critical points, Now dy/ds* = (2y— 1)dy/de = (2y ~ Vy ~ 3K +2), so the only possible point of inflection is at y = }, although the 2 and y=3. Since y"(2) <0 for y < ~2and } 0 for -2. 3, we see that solution curves are concave down fo y < -2and } < y <9 and concave up for -2 < y < band : 4 > Points of inflection of solutions of autonomous differential equations will have the same y-coordinates because between critical points they are horizontal translates of each other. concavity of solutions can be different on either side of y 28 Exercises 2.2 Exercises 2.2 In many of the following problems we will encounter an expression of the form In |g(y)| = f(z) +¢. To solve for g(y) we exponentiate both sides of the equation. This yields |g(y)] = e/(#)*¢ = eefl#) which implies o(y) = ke%e/™). Letting oy =e? we obtain gly) = cre 1. From dy = sinzdz we obtain y =~} eos5z +. 2. From dy = (x +1)?dr we obtain y= $(r+18 +e, 3. From dy = -e"* dz we obtain y= Je +e 4. Pom ped ade we dain dy = —spe de or sin yy ~ ~ cos? x de = ~B(0 + co8 22) de we obtain cosy — or deosy = 22+ sinde + 12, From 2ydy SESE ag = —tandzsne Sede wo obtain y? = —fec?ae +c Met sapere 13. From mo dye ere we ain (er) 14 om oy » eats ota (14) = (428) +e 35. From ZdS = dr we obtein $= cd edt we obtain In|Q - 70] = kt +cor Q—70= 29 Exercises 2.2 . =sinlet4lte ot &) soe 73, ee ee or a = —; } de we ot 20, om Et gym EEF ae or (1+ 25) arm (14+ 225) te we cin y ee a da t 2 2, Bo 2d = Fy we ota fo? very aun(Z +a) : 1 ae = —%— de we obtain ~1 = tan let +0 of y= -—p ee eel ee glee 28, om hip = tw bea tala Abe Ung 0) = Lm ad om =e Te solution of the initial-value problem is tan™ 1 1/ sh)e ale =1j-inje+1|+ine or 2 infe ~ 1) ~twje Il +ine or Fo = SA 24. From =) ar cian nly —Inly + Using 9(2) = 2 we find =1_g-1 ¢=1. The solution of the initial-value problem is 2—* = =—— or 1. The solution of tl I-value peat al sat 25. de we obtain nly) = ~2~ Inje| = cor sy = ae“. Using u(t) = -1 we find ¢1 =e", ‘Thr solution of the initial-value problem is zy = e~!-¥/, 26. From 75-dy = at we obtain —}In)1— 2y| = t-te or L— 2y = cre. Using u(0) = 5/2 we find 3f the initial-value problem is 1 ~ 2y = —4e¥ or y = 22" + J 27. Separating variables and integrating we obtain ds dy -1 . = =0 and sin? 2—sinmhy = ner 30 Exercises 2.2 Setting = = 0 and y = V3/2 we obtain ¢ e problem is sin“? ~ sin-?y -/3. Thus, an implicit solstion of the initial-value /8. Solving for y and using a trigonometric identity we get yosin(an* 24% 28. From Zl dy=—— dx we obtain ° TH yp” 14 ay $rantay=—Lean te? eo or tan”? 2y-+ tana? =e 2 3 Using y(1) = 0 we find ey = x/4. The solution of the initial-value problem is ten”! y+ tan“? 2? a 29. (a) The equilibrium solutions (2) (0) = ~2, respectively. Setting and y(z) Band y= 2 satisly the initial conditions y(0) = 2 and in y = 2(1 + cxf)/(0 ~ ce**) we obtain Lice 1 See, and c= ~5: TES, teem 24206, ‘The solution of the corresponiting initial-value problem is Lidet gate te eee eee eet wi (b) Separating variables and integrating yields 1 1 Maw—2-2 | + Ing pial 2] — Finly+ 2) +h Inly ~2)—tnly +2)+lne= 42 Solving for y we get y = 2[e-+<4)/(e—e*). The inital condition y(0) = ~2 can be solved for, yielding ¢ = 0 and y(z) = ~2. The initial condition y(0) = 2 does not correspond to a value of 6, and it must simply be recognised that y(2) = 2s olution ofthe initial-value probiem. Setting 2 = 4 and y = 1in y = 2(e+ ef*)/(c~ e*#) leads to c= ~2e, Thus, a solution of the ‘nitial-value problem is mete _g— ett ot 31 Exercises 2.2 142 ape Bae wotn n— ston ( bi seoy y az . Another solution is (a) If y(0) =1 then y (b) tf y(0) = 0 then y=. (e) If y(1/2) = 1/2 then y= ie (a) Setting 2 = 2 andy = § we obtain — a" imag" 1 2 Thus, y = a YS Tey 24ke 31. Singular solutions of dy/de = 2YT—y are y = ~1 and y= 1, A singular solution of (ef +e)dylde =P nyo. 32, Differentiating In(z? + 10) +eacy =e we get 2 ay Bay ~ ty ey # =O, 25 _ cosy 1 dy eosy tl dy ig F+10” sinySaydr 2zsin® ydz — (x? + 10) cosy dy = 0. Writing the diferenial equation in the fort dy ___2esin®y de” Bs Weosy wwe see that singular sotutions occur when sin?y = 0, or y= hm, where & is an integer. 32 88. The singular solution y = 1 sstisfies the initial-value problem. dx. Then zte-l ere 300. The solution is dy 95. Separating variables we obtain A m= de, Then Lets fan? -Isr+e anc =1+—7 tena Oran"! 10(y— 2) zte and yait teen Setting x = 0 and y = 1 we obtain c= 0. The solution is + duane io 0 36. Separating variables we obtain ey dr, Then . wai 001 10y — 11 ona rte Setting £ =0 and y = 1 we obtain = Sin 1 = 0, The solution is sia[2= 14 [Toy 33 Exercises 2.2 37. Exercises 2.2 Separating variables, we have dy = a v-nl+y) Integrating, we get 1 1 Info| $l yl Zin ty = 26 ‘When y > 1, this becomes L lag lng — ply +t) nes ate Letting 2 = O and y = 2 we find ¢ = In(2/ V3). Solving for y we get y_(2) = 2e*/VEEF=G, where > In(v3/2) When 0 < y <1 we have 1 1 dny— Fla 9) - 5294) Letting x = 0 and y= } we find c = In(1/v3). Solving for y we get yo(z) ~00< 2 In(V3/2), 34 Exercises 2.2 38. (a) The second derivative of y is @y ___dyjdz __Aty~3) A ee y-ip ‘The solution curve is concaw up when dy/de? > 0 ot y > 3, ‘and concave down when d?y/da? <0 or y < 3. From the phase portrait we see that the solution curve is decreasing when y <3 ‘and increasing when y > 8 (b) Separating variables and integrating we obtain (y~3)dy = ar a eee ied by + 9e2rbe W-3P sorta y=32VErq ‘The intial condition dictates whether to use the plus or minus sign ‘When 91(0) = + VmeFT When yo(0) = 2 we have cy = 1 and yo(z) -Ver When yo(2) = 2.we have cy = ~1 and y(2) =3- VHE=1 When ya(=1) = 4 we have cy = and ys(2) =3-+ VIEF3 wwe have ¢; = 1 and g(r) 38. (a) Seprating variables we have 2ydy = (2x + 1}d2. Integrating gives y? = 242+ When u(-2) = ~Lwe find ¢= 1, 90 y? = 2? 42~ Land y=—vaF +e 1. The negative square root is chosen because of the initial condition. 35 Exercises 2.2 (b) The interval of definition appoars to be approximately (~o0, ~1.65) (e) Solving 2? + 2 — {~00,-}- $y). 0 we get e =} £ JVB, so the exact interval of definition is 40, (a) From Problem 7 the general solution is 3e¥ + 22% = c, When y(0} = 0 we find © Be" 4 268% = 5, Solving for y we get y= —Jln }(5 ~ 24"), (b) The interval of definition appears to be approximately (—c0, 0.3) { 2 (6) Solving 4{5—2e"*) = 0 we eet 41. (a) While yx(z) = —¥25—@ is defined at x = -5 and », Yel) is not defined at these values, and so the interval of dfinition isthe open interval (5,5). 41n(Q), so the exact interval of definition is (~20, 4 n($)) {(b} At any point on he z-axis the derivative of y(z) is undefined, so no solution curve ean cross the r-axis. Since —z/y is not defined when y = 0, the initial-value problem has no solution, 42, (a) Separating variables and integrating we obtain 2? ~ y? =e. For c # 0 the graph is a square hyperbola centered at the origin. All four intial conditions imply c = 0 and y = siz. Since the ferential equation is not defined for y ~ 0, solutions are y = iz, r <0 and y= 41,2 >0 ‘The solution for y{a) = ais y= 2, 5 > 0; for yla) = -a'sy <0; and for y(a) = =; for y(~a) = ais y= —z, waisy=a7<0. (b) Since /y is not defined when y = 0, the initial-value problem has no solution (e) Setting x = 1 and y = 2in 2? —y® =e we get c= ~3, soy? = 2° +3 and (2) = VFS, ‘where the positive squsre root is chosen because of the initial condition. The domain i all real numbers since 2? +3 > 0 forall 36 Exercises 2.2 43, Separating variables we have dy/(\/1+ 92 sin?y) = dz which is not readily integrated (even by a CAS). We note that dy/dz > 0 for all values of z and y and that dy/dz = 0 when y = 0 and y = x, which are equilibrium solutions 44, Separating variables we have dy/(/@+y) = dz/(V/Z+ 2). To integrate f dt/(/t-+t) we substitute wat and get 2y we Integrating the separated differential equation we have 2in(l + yG)=2ln(1 + yz) +e or Inf + V9) =n + v2) +Ine1 Solving for y we get y= [er(l + vz) ~ JP. 45, We are looking for a function y(z) such that 2 dum [ -2du=2ln|1 +0) +e=2in(1 + vz) +e Using the postive square root gives we we ‘Thus a solution is y= sin(e +). If we use the negetive square root we obtain y= sin(o—2) = ~sin(s—) = ~sin(e +4) Note also that y = J and y = ~1 are solutions 40. (a) (b) For j2] > 1 and |y| > 1 the diferential equation is dy/dr = yy@—1/vZ™=T1. Separating variables and integrating, we obtain dy wig wt cab yn entree 37 Exercises 2.2 47. (a) () 48, (a) wwe find e = cosh"! 2 ~ cosh Setting 2 = 2 and y explicit solution is y ==. Separating variables and integrating, we have (i? + dy = (Ge +5)ér ond ty Using a CAS we show various contours of f(2,y) = y?+u-t4a*+ 52. The plots shown on [~5,5] x [~5, 5] correspond to c-values of 0, 45, £20, 440, 280, and 4126, 42? Srv0. ‘The value of c corresponding to y(0) = ~1 is f(0,~1) to u(0) = 2s £(0,2) = 10; to v(t) = and to y(-1) = ~2 is -31. O and cosh“!y = cosh"'z, An Separating variables and integrating, we have (y+ )dy 1 ely Using & CAS we show some contours of (2.9) = 24 — by? + 2x3 — 322, The plots shown on [-7,7] x {-5,5] correspond to c-values of —450, —300, —200, =120, =60, -20, =10, -81, ~8, ~08, 20,60, and 0 (ea \dx and 2 yt 38 Exercises 2.3, (b) The value of ¢ corresponding t0 (0) = $ is f(0,$) = =2. The portion of the graph between the dots cor- responds to the solution curve satisfying the intial con- dition. To determine the inteval of definition we find ddy/de for 248 — 6y? +22" — 302 = -%, Using implicit differentiation we get y! = (x ~ 23/(y? ~ 2y), which is infinite when y = O and y = 2. Letting y = 0 in 2y? Gy? + 24 — G2? = —2 and using a CAS to solve for x we get x = ~1.19282, Similarly, letting y= 2, we find 2 of definition is approximately (—1.13232, 1.71299. (c) The value ofc corresponding to y(0) = 2 8 /(0,—2) = 40. The portion of the graph to the right of the dot corresponds to the solution curve satisfying the initial condition. To determine the interval of definition we find aya for 2y° ~ 6y? + 203 ~ 302 = —40. Using implicit ifferentiation we get y= (x ~ 2°)/{y? — 2y), which is infinite when y = 0 and y = 2. Letting y = 0 in 2y5 — 6y? +223 — 32? = -40 and using a CAS to solve for 2 we get z= ~2,20551. The largest interval of definition is approximately (~2.29551, 0). Exercises 2.3 1, For yf ~ Sy = 0 an integrating factor is e~J #4" 00 <2 <00 2. Ror yf + 2y = 0 om integrating factor is 24 oo < 2 < 00. The transient term is ce~*, 8. For y/+-y= eon integrating factor is ef 00 <2.< co. The transient term is 2 4. For y+ 4y = 4 an integrating factor is ef 44? = for ~00 <2 < oo. The transient term is oe“ 8. Rory/48sty = 2? an inegratng factor is eS for oc < 2 < 00. The transient term is ce 39 Exercises 2.3 6. For of + 2zy = 2° an integrating factor is ef #4" = c* so that jx? — 1 + ce-#” for -00 d If y(0) = 0 then cy = —1/2 and for continuity we must have —f c= fe} a0 that 1 fa-e™), oszs% Heo ve™, 258 For y/-+y = f(2) an integrating factor is e* so that Y Gta, 0S251 w(t = If y{@} = 1 then cy = 0 and for continuity we must have cp = 2e so that L o. If y(0) = 2 then ey = 3/2 and for coatinaty we mst have on = jer} oo that u 42 Exercises 2.3 34. For an integrating factor is 1+ 2? so that (42}v-{ 1 y(0) = 0 then cy = 0 and for continuity we must have cz = 1 s0 that kta, OsrSh w}tten 254 7 1 rs 3~ Faas OS*SE en Wee 2 7?) 85. We need a PCa) aa [ret {Fe, 51 [An integrating factor is ef Plaids = {ee O21 ya, 2>1 and 4 (fue, osest} (ee os1 af, e>1 Integrating we get ee ee Osrs1 vist, 2 >1 4inz+ on, z>1 Using y(0) = 3 we find ¢; = 4. For continuity we must have cy =2—1+4e-® = 144e?. Then 2e-1+4e%, O 46 Exercises 2.3 is 27, Thus i fiety = 102E ae =y y=10f Mare y= 102~*5i(2) + e2"* 1USi(1). Thus 1Ox™*8i(1) = 102-*(Si(x) — Si(1)). From y(1) = 0 we get ¢ y= 10z"*8i(z) 4 4 a “4 ol (c) From the graph in part (b) we see that the absolute maximum occurs around 2 = 1.7. Using the root-finding capability of a CAS and solving y'(2) = 0 for + we see that the absolute ‘maximum is (1.688, 1.742) 50. (a) Separating variables and integrating, we have Y= sina? 2 ® canztae and sin ‘ a Injy|= [Pinta +e Now eting t= 7b wave : Ven [ontan Ep oejan, TT eye tin at «6 pV ATS snteu? 2) du (b) a7 Exercises 2.3 (c) From the graph we see that as z ~» 20, y(z) oscillates with decreasing amplitude approaching 9.35672, Since lime-o (2) =}, liso y(z) = SeV*F = 9.357, and since imy oo S(z) = 4 lng nao (a) = 5e-V578 = 2.672 (a) From the graph in part (b) we see that the absolute maximum occurs around x = L.7 and the absolute minimum occuss around t = —18. Using the root-finding capability of a CAS and solving (2) = 0 for 2, we see that the absolute maximum is (1.772, 12.235) and the absolute tninimua is (-1.772, 2.044). —— Exercises 2.4 —_________ 1. Let M = 22-1 and Ny) =3y +7, and Ay) 2. Let M=22+yand N 3. Let M = Sx + 4y and N = 4r ~ 8y? so that M, = 4 = Nz. From f, = 5x + 4y we obtain = $x? + day + Aly), A(y) = -8y°, and h(y) = -2y!, The solution is §2" + dry ~2y4 = 0 4. Let M = siny~ ysing and NV = cosz + zcosy —y so that. My = cosy ~ sinz = Na. From fz =Siny—ysinz we obtain f = rsiny+ycosz+h(y), Al(y) = —y, and Aly) = $y?, The solution is zsiny +yeos2 — fy? =0 5. Let M = 2y?s —3 and N = 2y2" +4 s0 that My = 4zy = Ne. From fe = 2y%x ~3 we obtain = aly? — 32 + Aly), W(y) = 4, and Aly) = dy. The solution is 2¢y? — 35 + 4y = 6. Let M = dz? — aysinds — y/s? and NV = 2y — I/z + 00832 $0 that M, = —3sin3x— 1/2? and N, = 1/2? ~3sin 3x, The equation is not exact. 7. Let M = a2 ~ 9? and N = 2? — 2ry 00 that My = exact 8. Let M=1+lnz+y/z and -y+ylnz+ Aly), 9. Let M = y3— sing — 2 and N = 32y? + 2ycosz 90 that My = 2y? — 2ysin = Nz. From fe =~ ysing 2 we obtain f = zy? + yPooss — fu? +-h(y), W(y) = 0, and My) = 0. The solution is 2° + yPoosz — da? = 10. Let M = 23 +48 and N = 3zy? so the f= fete ry +My), MG) =8,and My) 11, Let M = ylay — ¢-* and N= 1/y + zlny 00 that My = 1+ Iny + ye"®” and Ny = Iny, The equation is not exact +7 0 that My = O-= Nz. From fe = 22-1 we obtain f = 2?-r+A(y), Bu! +79. The solution is 2? 24? +70 2 ~ 6y. Then M, =1 snd Nz =—I, so the equation is not exact. 2y and Nz = 2x — 2y, The equation is nat =1+in so that My = 1/2 = Ny. From fy = -1-+Inz we obtain +Inz, and A(y) = ¢lns. The solution is ~y+ylnz +elnz =e. My = 3y? = Ny. From fz = 2° + y? we obtain 0. The solution is fx! + 243 = 48 12. 13. 14. 45. 16, a. 18. 19. 20. a. Exercises 2.4 Let M= Sety +03 and N= 2° +20) — 2y so thet My = 32? +6¥ = Nz. From fe =32%y + e¥ we obtain f = 24y + ze¥ +h(y), h'(y) = ~2y, and h(y) = —y?, The solution is z*y + ze” — y? Let M = y~ 6x? —2ze* and N = 2 sothat My = 1= Ny. From fz = y~ 62? — 2eet we obtain f = xy~22—2re" +26 + hly), Mly) = 0, and Aly) = 0, The solution is zy~22— 226" +2eF = ¢ Ne From fe =1~3/z+y = Sinly|. The solution is Let M = 1-3/2 +y and N = 1-3/y +20 that M, = 3 =, and Ai 7 (y) x Sia|e| +2 + Aly), Ay) = 2 ety tay- Slory Let M = fy —1/(1+90") and NV = 2'y? so that My = 32%y? = Ny. rom fe 1/ (1492?) we obtain f = 42% § arctan(32) + h(y), W(y) = 0, and h(y) = 0, The solution is iy — erctan(3e) = Let M = —2y and N = Sy—22 so that My = -2= Nz. From fz = —2y we obtain f = ~22y+h(y). W(y) = By, and Aly) = fy2. The solution is ~2ry + fy? = Let M = tanz ~ sinzsiny and N = coszcosy so that My = —sinzcosy = Ne, From fe = |sec|+coszrsin y+h(y),A!(y) = 0, and h(y) = 0. The solution we obtain f tanz—sinesiny we obtain f = is In| seca] + cose sin: Let M = 2ysinzcose—y+2yte%” and N + sin? + dye” so that My = 2sinzcosr— 1+ dzy'e™ + dyet™ = Ny From fz = 2ysinxeosz — y+ 29% we obtain f = sin” — xy +26 + hly), hy) = 0, and ‘y) = 0. The solution is ysic?2 ~ 2y-+ 265 le M obtain f it3y—15t?—y and N = tt4+3y?—t so that My = 4t°—1 = Nj. From fr ty — 51° ty-+h(y), My) = 2y?, and A(y) = v2. The solution is thy — Let M = 1/te1/i—u/(P +98) and N= yeh +t) (2+?) sorbet My = (VP) (24+) = Ni Rom fi= 1/4? y/ (2+ yf) we obtain f= nll ~ t—anetan (£) +A). 806) =e ‘and A(y) = ye — e¥. The solution is nid ~} -arcian ($) +2et- 8 =e t Y, Let M= 2+ 2ey+y? and N= 2ry-+22—1 so that My = 2e-+y) = Nz. From fe=2?+2ryt ie we obtain f= fx! +2%y + ay? + Aly), Ay) = —1, and My) = ~y. ‘The general solution is jet+22y-+ ay? — yc. yl) =1 then = 4/3 and the solution ofthe initial-value problem is Pescysat—ye4 49 22. 23. 24. 1%. 26. al. 32. + Bquating Wf, = 18zy?—siny and Ne = dkzy? —siny we obtain Exercises 2.4 Let Me +y and N = 2+2+ye¥ so that M, = 1= Nz. From fe = ef-+y we oblain f =e +zy+ Aly), Al(y) = 2+ ye¥, and A(y) = 2y + yeY~y. The general solution is e* + zy + 2y + ye ~e? Tf y(O) = 1 then c= 3 and the solution of the initial-value problem is e* + 2y + 2y + ye ~e¥ Let M = dy +24~ 5 and N = Gy +4~ 10 that My obtain f = ty + 1? — St + A(y), Ay) = 6y ~1, and Aly ty +2 —5¢+ 342 —y =e Ify(-1) = 2 then ¢= 8 and the solution of the initial-value problem is dty +0? — 5t+ By? -y = 8. Let M = ¢/2y4 and NV = (342) /v® so that My = ~2t/y* = i. From fr = Ny From fe = dy +25 we 3y? — y. The general solution is t/2y* we obtain i. 3 3 == a 1 A(y) = and Aly -5-q- The general solution is — — 55 Tfy(l)=1 J Ga TMU). HG) = yvand Aly) = ~The ae tons Gt ag Hat : #3 then em —8/4 and the solution of he ntinkvale problem Sy ~ 5p =~ Lat M = yPcos2~32%y 22 and NV = 2ysine +2? + Iny so that My = 2ycos— 92 = Nz. From fe = yPoosz—3x%y—22 we obtain f = y?sinz—24y—2*+hly), hy) =Iny, and Ay) = ylay—v ‘The general solution is y?sinz ~ 2°y~22+ylny~y=c, If y(0) =e then c=0 and the solution of the initial-value problem is y?sinz —2%y—s?-+ylny—y Let M = 9 + ysing and N= 2ey—cosz—1/ {1 +2) so that My = 2y-+sinz = Ne. From fe = = and Aly Tee) If y(0) = 1 then c = —1 — #/4 and the solution of the initial-value problem is 2y?— yoos — tan“y = -1— 7 tan“! y. ‘The general sw? +ysine we obtain f = 2y?—yooss + Aly), H(y) solution is 2y? — ycos ~ tan! y Equating M, = 3y2 + dkzy? and Nz = 3y? + 40zy? we obtain & = 10, 2 Let M = -z°y2sin + 2zy2 cos and N = 2z%ycos so that M, = -2a%ysin x + 4zycos: From fy = 2zycosz we obtain f = zy? cos=+ A(y), h'(y) = 0, and A(y) = 0. The solution of the differential equation is 24? cos Let M = (2? + 22y—y*) /(a2 + 22y+y2) and N = (y? + 20y 2) / (2 + 20y-r22) ao that 2 My = ~Aay/(2-+y)® = Ne. Brom fe = (22 + 22y + 9? ~ 242) /(e ty)? we obtain faa Aly), h{y) = -1, and A(y) = -y. The solution of the differential equation is 2? + y? = c(z + 9). We note that (IM, ~ Nz)/N = 1/z, so an integrating factor is el #/* = 2, Let M = 2ry! +32 and N = 2xy 50 that M, = dry = Nz. From f, = 2ry?+ 32? we obtain f = 27y? +23 + Aly), H(y) =4, and A(y) = 0. The solution of the differential equation is 22y? + 2: We note that (My ~‘Ne)/N a 1 0 an inegrating factor is ef me, Let Mm zye? +86 et 50 33. 34 35, 36. 37. 38. Exercises 2.4 and N= ze + 2ye" so that M, = ze + 2ye" + ef = Ny. From fy = 2e% + 2yet we obtain f = xye® + ye + A(z), A'(y) = 0, and Ay) = 0. The solution of the differential equation is yet +e =o. ‘We note that (Nz ~ M,)/M = 2/y, so an integrating factor is e/™¥/¥ = y?, Let M = 6zy? and = 4p +924? so thet M, = 18xy? = Ne. From fz = 62y? we obtain f = 32%? + Aly), H{(y) = 4y4, and A(y) = y$, The solution of the differential equation is 32°y! + 4 We note that (My ~ Nz)/N = —cot, s0 an integrating factor is e~S*=2 = esoz, Let M = cosesex = cots and N= (1 +2/y)sinzescz =1+2/y,s0 that My =0= Nz. From fe = cote wwe obtain f = In(sinz) +h(y), A'(p) = 1+2/y, and h(y) = y-+iny®, The solution of the differential equation is Gin z) +y+Iny? =. We note that (My ~ Nz]/N = 3, so an integrating factor is e 94 =e. Let M = (10 ~6y +e )e% = 1008 ~ Gye™ +1 and = 26%, so that My = —6e™ + Nz. From fe = 10e ~ Gye +1 we obtain f = Wei — aye ++ Aly), H(y) =D, and A(y) = 0. The solution of the differential equation is ¥e™ — 2ye* +2 = c, We note that (Nz — M,)/M so an integrating factor is e241 1/48, Let Ma (ita) =iy+e and N= (5y?— 2y+gsiny)/y? = 5/y—2/y? +siny, so that My = ~1/y? = Ny, From fz = 1/y-+2 we obtain f = 2/y+}2? +h(y), Aly) = S/y+siny, and Aly) = Sin |y| cosy. The solution of the differential equation is 2/y++ J2?-+5in|y|—cosy ‘We note that (My — Nz) /N ~2x/(4+ 22), so an integrating factor is e~2/=#/(4+=") = 1/(4 4-22), Let M = 2/(4+2?) and N = (2?y+4y)/(4+2*) = y, 80 that My =0= N, From f, = 2(4+2%) we obtain f = fin(4-+ 27) + A(y). Ay) = y, and h(y) = jy ‘The solution of the differential equation is $in(d+ 22) + jy? =e. We note that (My—N,)/N = -3/(1-+2), so an integrating factor is 2/42/0142) = 1/(142)°. Let M = (27+y?—5)/(1+2)* and N = -(y+zy)/(1+2)* = -y/(1+2)*, so that My = 2y/(i+z)° = Ns. From fy = -y/() + 9)? we obtain f = ~Jy?/(1 +2)? + A(z), W(2) = (@* = 9/0 +2)°, and A(x) = 2/(1+ 2)? +2/(1+2) +In|L +2. The solution of the differential equation is 2 2 ~aeapt Tee 2 2s sine sl~e Gey that 51 39. 4a 42. 43, Exercises 2.4 (a) Implicitly differentiating 29 + 22%y + y? = cand solving for dy/dz we obtain ay 30 tary a Ty Separating variables we get (dey + 3x*)de + (2y + 22")dy (b) Setting 2 = 0 and y= —2 in 2? +224y + y? =e we find find ¢= 4. Thus, both intial conditions determine the same implicit solution wre Beeps ben ad and setting = y= we also (e) Solving 23 + 20%y + y? for y we get (care a yilz) = =: and 4 wale) “To see that the equations are not equivalent consider de = (z/y)dy =O. An integrating factor is (2,y) = y resulting in yde-+ 2dy = 0. A solution of the latter equation is y = 0, but this isnot a solution of the original equation, ‘The explicit solution is y = y(@ coma) =a). Since 3+ cox? > 0 forall x wo must have 1-2? >0or—1 FE Let w= y= 248 so that dude = dy/ds — 1. Then $+ meta zteand ets rte L +e" or edu = dr. Thus +e du Let wo 2-4 y 90 that dude 1+ dy/de, Then MY ~ 1 cos and pL du L cosu _Lnconu Treosta = “sina OT SeHCOt Treo 58 30. 3 Exercises 2.5 sowehave [(csc?u—cseucotu)du = Jdr end —cotuteseu = x40. Thus —cot(ety)tese(ety) = +6 Setting z= 0 and y=-x/4 we obtain e= VB—1, The solution is cse(e + y) ~cot(a ty) = 2+. V2 te ut? du _Sub6 oy ut = 324 2y 5 . a= = leva set 2y soca dds w 24h Thm Zin oy = Sp ad Seg = te ut2 1, 4 iuh6 5 * Baro s0-we have [G+ ape) = and Ju+ gh In|25u+ 30] = 2-46, Thus 1 4. _ gle 20) + Ze In|T52-+ 50y + 30) = 2 +. Setting + 1 and y= —1 we obtain = $1n95, The solution is i oe 4 (82+ 20) + pg in| Tc + Sdy +30) = 2 + 2 In 95 Sy Sx +2in 752 + Boy + 30] = 101m98. We waite the differential equation M(z, y)dz +.V(z,u)dy = 0 as dy/dz = f(z,y) where May) N@w) ‘The function f(z, ) must necessarily be homogeneous of degree 0 when M and N are homogeneous of degree a. Since M is homogeneous of degree a, M(tz,ty) = t#M(z,y), and letting t = 1/z we have feu)= M(,y/2)= May) or MGz,9) = 2° (2/2), 2°M(Ly/t)__ MO,y/2)_ p(y NG) ~~ Nava) ~* (=) To show that the differential equation also has the form 4 g(t 2-¢(5) wwe use the fact that M(z.y) = y*M(zx/y,1). The forms F(y/z) and G(2/y) suggest, respectively, the substitutions = y/z or yu and y = 2/y on z= 0p. Base) 59 Exercises 2.5 32, As 2 —,—00, ef — 0 and y—22-+3, Now write (1-+c2)/(1 ~ cals) a8 (e+ oe" —e) ‘Then, a8 2 + 00, e~* 0 and y > 2r -3, 33. (a) The substitutions y= yi +u and ay an, du a ate lena to noe Bees P+Qly +t) + Rly +0)? =P +Qy + Rif + Qu+ In Rue RP SQ + 2nRu= Re “This o's Bernoul equation with n = 2 which can be reduced to the linear equation dw B+ (Q+2nRw by the substitution w= ut (b) Identity P(x) = ~4/22, Q(z) = -1/z, and R{z) = S)e--1 a integrating factor i 2 so that xt = —fat +eor w= [fet crt] "Ths, y=? +u 34, Write the diferential equation inthe form 2(//y) = Inz-+iny andetu = ny. Then dalde = o/y and the differential equation becomes z(du/dr) = Inz+u or du/dz ~u/z = (Inz)/z, which is first-order, linear. An integrating factor is e~/4*/* = 1/z, so that (using integration by parts) ‘The solution is Exercises 2.6 ——_—____ A. We identity f(¢,y) = 22 —3y-41. Then, for k= 0.1 Uo = tn + 0-120 ~ St +1) = 0.2m + 0.7m + 0.1, and WL) & yy = 0.211) +0.7(5) +01 =38 y(L.2) & yo = 0.2(1.1) + 0.7(8.8) + 0.1 = 2.98, 60 Exercises 2.6 For h= 0.05 Yast = te + 0.05(2tq ~ Wyn +1) = O.1sn + 0.8540 + 0.1, and (1.05) = yy = 0.(1) +0.85(5) + 0.1 = 44 QL) % vp = 0.1(1.08) + 0.85(4.4) + 0.1 = 8.895 ‘y(1-18) m yp = 0.1(1.1) + 0.85(3.895) + 0.1 = 3.47075 y(12) = yy = 0.1(1.25) + 8.85(8.47075) + 0.1 = 3.11514 . We identify f(2,y) = 2 +¥. Then, for h= 0.1, Yost =n + OMzn +98) = On + yn + OYE, and y(0-1) = yr = 0.1(0) +0+0.1(0)? = 0 y(0.2) © yp = 0.1(0.1) + 0 +0.1(0)? = 0.01 For k= 0.05 = Yn + 0.05(n + v8) = 0.05zn + Yn + 0.0542, and (0.05) © yy = 0.05(0) +0 + 0.05(0)? = 0 (0.1) ar = 0.05(G.05} + 0+ 0.05(0)* = 0.0025 ‘y(0.15) = ys = 0.05(0.1) + 0.0025 + 0.05(0.0025)? = 0.0075 (0.2) © ye = 0.05(0.18) + 0.0075 + 0.05(0.0075)? = 0150. - Separating variables and integrating, we have Yaar and inlyl=z+e ‘Thus y= cre® and, using y(0) = 1, we find e = 1, so y = ef is the solution of the initial-value problem. 61 Exercises 2. 0.1 0.05 Tue te al Tee eT Ba Yo value ero _“prvor vaiue _grver _Eevor TTB os oot Tasso 900e 0-00 0:19 1-100 112052 0:0082 0147 10533 a:0q13 0.12 0:30 32100 Tia oleae 193 Titos2 010027 0.24 0:30 3.3310 113499 lowes aa Tiueie ovaoaz 0036 old 24641 Llasia oc0a7? 18s azais 0.0053 0.48, 0150 16108 6487 ol03e2 2.32, 112640 010079 0:60, blo 1.7716 iles21 oles6 2.77 13492 0.0098 0.72, 0179 1.9487 2'0138 Ol0ss0 3123, Yale 910120 ols igo 2.1436 2'2285 o'aan0 3148 asia biota 9196 bso 2.3879 214895 012017 «133 a's683 0.0170 1-08 boo _3:5937 21714301125 4158 Lets; o:0198 i120 rings ola als Eleaaz o-ozes tea Tigiss 020259 iis6 zioiae 070338168 3iua9 oc03e1 a0 3iiass 010427 1192 Bihsee 010476 2106 2iagse 010530 2.15, 2'sa57 0.0588 2°27 2°7163_0:0650_ 3:9 rds and Inlyl=2?-+e ‘Thus y = ce” and, using y(1) $0 y = e**-1 is the solution of the initial-value problem, 1, we find ¢ = € peo. 0,05 x ee Tease syn value grea ‘error ) | x ye vate ero prror TOIT BIT ta Sor] [Towra ao tua 12000 312337 9.0337 2.73] | uios alzano 2.2078 0.0079 0.72, £120 ilaeaa f1ssa7 oloe7 $12] | 129 dlanss 112332 oleae fay 130 Liaise tesa7 o1a7ea als | | Las alaasa Lisece Olga 2.37 240 2267 zieii7 013263 12-42] | 120 aisoad 115527 0.0483 3.2 350 2320314303 _0.se28 ie't2] | ids tases 11958 0!0702 400, ria ilgass 119937 010982 ¢.93 ras 21q9 2.3762 071303 $190 iiag 2iasii 2.6117 O"1a0e 6.92 ras 2.74 30117 012403 9138 Esa 3:1733 314903 003a71 908 62 Exercises 2.6 bso. 9.05. m0 oe Se Ya Spe: ye a 1-00 Tar 0130 0:2000| | 0-05 0.0500 0.20 31000 | | 0:05 10500, 0120 0:3905 | | 0:10 c!o516 0:20 12220) |o10 "1053 esa 0.2732 | | 0.18 ocsa2s 0:30 113753 | | 03s i ies o'4o 0.3482 | | 0.20 o:3863 o.a 215738] | 0:20 2.2360 02g0_ 0498] | 0.28 L297 oiso_tlesa| | 0:35 shies 2:30 0.2676 0:30 aoae 253s 0°3086 0:35 i s0%e 0:40 013427 o'40 Leer o:as 013742 0.45 1.7690 bso 01434 05039332 beg 0.05 x se Ye mm | ay 0-0 OSD DoT aee| [Toons azo o.s250| | 0105 olsizs 0.10 1/2000 | 0:05 1.0500 6:20 o/saa1| | 0130 oleae 0:20 iizuss| | 010 1038 0:30 0:8ea8 | | 0.28 018309 0:30 T'ss0s) fous ims 0.30 0-613 | | 0.20 0.5385 0.40 1 sovz| | 0:20 2795 bso o.sera | | 0-25 O'ses2 0:50 _ivese2 | | 0°25 35923 030 015486 o.30 13653 0:35 015537 6:38 1aaa0 0:40 o1ssa7 01a 35233 0145 015559 0.5 e217 0:50 0.5865 050317233 p03 4 xe ye oe ye Ye TT Toot Poo a-s00r ay Freres oa0e 0/20 015280 | 0/05 015125 igo 7 Looe 0530 81sas8 | | 910 015250, 30 1 ileus? 0130 O'5ve7 | | 0118 015398 Veo i aloase ‘o:40 0.2991 | | 0:20 015438 iso) sagas 0'50_ocg#31} | $198 075623 iors 0130 o-st46 as 0:38 8.5068 isis 0:40 6.5988 12087 oes 0.6109 ates6 0:50 9.6228 wos Exercises 2.6 13. Using separation of variables we find that the solution of the differential equation is y = 1/(1— which is undefined at z= 1, where the graph bas a vertical asymptote, 2 Buler bx0.05 puler aed] RK ed. Ye mt mye fe me a ae LT o'19 1.0000] | 0.05 ::0000] | o'10 1-o102] | 0:05, 0120 i020] | oto ios | |o20 toa] | 010 0:30 toss | |ocs tloisi) } 9:30 09a] | os 01a 1.1292 | | o:20 10306] | o-ao 1.3905] | 0-20, bro 12513 | | olas iloste | | ocso aaa! | o.as bio 1.3828 | fala 1.0795 | o.g0 15695] J 9.30 aio tea | fora itiae | | 0:90 t3807] | 9.35 bred Ls7ea | foaa tiases| | oceo 2irm| | ovao 0130 aesoz | |o'4s t:2uzs| | o:s0 5.2308] | ous $iee_3iszar | jo'se igre) [Lieo_aaisani| | 0:50 oss 1382 035 060 1 ee0a 030 oles tsa bigs 9.70 17535 070 6:75 i969 Sus o.ad 2.2504 0:80 0185 2geea 688 0190 32708 8:30 bas danse 0.95 B00 5.9363 $006: Because the actual solution of the differential equation becomes unbounded at 2 approaches 1, very small changes in the inputs z will result in large changes in the corresponding outputs y. This can be expected to have a serious effect on numerical procedures. 64 Chapter 2 Review Exercises 14. (a) 10) (c) Using FindRoot in Mathematica we see that the solution has a relative minimum at (0.924139, ~0.541044) and a relative maximum at (0.924139, 0.541044), Chapter 2 Review Exercises 1, Writing the diferental equation inthe form y/ = (y+ A/E) we see thet the critical point ~A/i is a repeller for k > 0 and an attractor for k < 0. 2, Separating variables and integrating we have wu v Iny=ding+o=tnzt +e We see that when x = 0, y = 0, so the initial-value problem bas an infinite number of solutions for k= and no solutions for k #5 wu -3? vy -2Pw-4) 65 Chapter 2 Review Exercises 5. Wy Vy i 1 | | For dr /dt = 2".when n is even, Vis semi-stable; when nis odd, 0 is unstable. For d2/dt = ~2" when ‘nis even, 0 is semi-stable; when n is odd, 0 is stable. 6. The zero of f occurs at approximately 1.3, Since P(t) = f(P) > Ofor P< 13and Pi) = f(P) >0 for P > 1.3, limo P(t) = 13. 8. (a) linear in y, homogeneous, exact (b) tear ine (c) separable, exact, lines in z and y (a) Bernoul in = (e) separable (6) seperabte linear in 2, Bernoulll (g) linear in 2 (1h) homogeneous (@) Bernoulli (§) bomogencous, exact, Becnoull (&) linear in 2 and y, exact, separable, homogeneous (1) exact linea in y (mm) homogenous (n) separable Chapter 2 Review Exercises 9. Separating variables we obtain v cos? Leg Leinze = Lin(g?4) +e a ae tsin2e =2in(y?+1) +6 rT get jane Fin G41) +e Pe tsinde 2 (P41) + 410, Write the difexential equation in the form yin dz = (em S %) dy, ‘This is « homogeneous v i equation, 40 let z= uy. Then de = udy-+ du and the dierentel equation becomes yilnu(udy+ydu) = (uylnu—y)dy or ylnudu= —dy. Separating vaiahes we cbiin dy Inudus — > win|al - = a lu) 5 =Inlyl te -Injyjte => Zin ty) i v => a(lng—Iny) 2 = ~ylaly| + oy. ee 9 ve ota 11, The erential egstion Se =P oy = Sy? ig Bel Using w = 9! ea w, S27 An integrating factor is 62 +1 Bg Eom gO. aninteptin fr 6 +1, 9 a : 323 e Peon qellGe te Myu) = 90? wa SS (Get ay = 3a + (Note: The differential equation is also exact.) 12. Write the differential equation in the form (3y?+2z)dx-+ (4y? + Gzy)dy = 0. Letting M = 3y?-+2r and N = 4y? + Gay we see that My = 6y = Nz so the differential equation is exact. From fo = 3y?+ 20 we obtain f = Sry? +2? +h(y). Then fy = 6ry+ h'(y) = 4y? + Gry and h'{y) = dy? so h{y) = $y°. The general solution is 4 a4 any? +2? + 5 13. Write the equation in the form 42 19a) et zea eint ‘An integrating factor is e™* = t, so ad gitdl=tint = 1Q= 14, Letting u= 22+ y+ 1 we have Chapter 2 Review Exercises and so the given differential equation is transformed inta de du_2utl “le ee Separating variables and integrating we get le i ari) 1 Giutijsere 2u~In|ou +1) = 22+ er Resubstituting for w gives the solution p+ 2y +2-Inlde + 2y +3] = 2x + 2y+2~In|dr + 2y +3 oo |. 15, Write the equation in the fom 22 + 22 ym 22. An negating factor (22+ 4), 30 4 [+e] =2e(2 eg! > (P+a) y= F(a) ne ya toes) 16. Letting IM = 27? cos@sin9 +rcos@ and N = 4r-+sin@ ~ 2rcos? @ we see that: My = dr cos@sin8 + 088 = Ny so the differential equation is exact, From fy = 2r2cosdsin + reos@ we obtain f= reas +rsind + h(r). Then fy = ~2rcos'6 + sind + A'(r) = 4r + sind — 2rcos?@ and Ni(r) = 4r 50 Me] = 2r2, The general solution is a Pees o trsind $2? = 11. The ier quaon hs eh oom sine] 0. tmegaig hve lazy = €o y = c/sinx. The initial condition implies ¢ = ~2sin(7r/6) = 1. Thus, y = 1/sina, where 1 << 2n i chosen to inde 2 = 74/6 Chapter 2 Review Exercises 18. Separating variables and integrating we have dy. 2043) gen 20+) 1 -G+1ee eT re ‘The initial condition implies c= —9, 60 the solution of the initial-value problem is 1 ERE where -4et<2 eyHae 19. (a) For y <0, yf is not a real number. (b) Separating variables and integrating we have de and 2oe2 Va Letting y(zo) = yo we get ¢ = 2/96 — 20, 60 that QG=2+2VH—z0 and y= fle+ 2 ~ 20). Since Yj > 0 for y # 0, we see that dy/dz = $(e + 2/6 ~ 20) must be positive. Thus, the interval on which the solution is defined is (z9 — 2/78, 00). 20. (a) The differential equation is homogeneous and we let y= ur. Then (2 yP)de +aydy=0 (a? = ws?) de + ua"(ude +2du) = 0 de+urdu=0 2 ¥ =~] +0. ‘The initial condition gives cy = 2, so an implicit solution is y? = 2°(2~ 21) 69 a1. 22. Chapter 2 Review Exercises (b) Solving for y in part (a} and being sure that the initial condi- ton is still satisfied, we have y = —V2|x((1 ~In[zl)¥/2, where $x Sesothat 1-In|z| > 0. The graph of this function indi cates that the derivative i not defined at x = O and x = e, Thus, the solution of the initial-value problem is y= —V2:x(1—In-z)!/?, for0<2 20 the differential equation is 20 di/dt + 28 = 0 and + = cert. 74 Exercises 3.1 60 ~ 60e~? so that cp = 60 (c= 1). Thus 80 - 60e7/10, Osts 20; 0" Leola, 100 Att = 20 we want exe 28. Separating variables we obtain ay oo B= gi hr kt (B-a/0ye Talla + hath ter me pias =o Setting 4(0) = go we find 2 = (B= 9/G)-© _ (Bo-~w/Cy® _, Cit kaa oe g a Ab = &-E-(B-8) (x) _ oF R0+(@-H0)(-B) 28. (a) From mdu/dt = mg— kv we obtain v = gma/k-+ceH/™. 1f (0) = up then ¢= oy —gmm/k and the Sclution ofthe initial-value problem is (b) Ast — co the limiting velocity is gm/: (c) From ds/at = v and s{0) = 0 we obtain BB (0-FB)om eB (o-) 80. (a) Integrating d?s/at? = —9 we get u(t) = ds/ét = ~9t + ¢. From v(0) = 300 we find c= 300, so 32 + 300 the velocity is v(t) 0 we get s(t) = -16:? + 3002. The maximum height is 9.375. The maximum height will be s(9.375) = 1406.25 4. (b) Integrating again and using s(0} attained when y=, that is, at 31. When air resistance is proportional to velocity, the model for the velocity is mdv/dt = —mg — kv (using the fact that the positive direction is upward.) Solving the differential equation using separation of variables we obtain v(t) = —mg/k + ce“M/™. From v(0) = 300 we get Exercises 3.1 Integrating and using s(0) = 0 we find s(t) = Setting f= 0.0025, m = 16/32 = 05, and 9 = 32 we have e+ F(s004 M2) - Him a(t) = 1,340,000 ~ 6 4008 — 1,$40,000¢~095¢ and oft 6,400 + 6,700e0 ‘The maximum height is attained when v = 0, that is, at f = 9.162. The maximum height will be 5(9.162) = 1363.79, which is less than the maximum height in pact (5. 52, Assuming that air resistance is proportional to velocity and the positive direction is downward, the ‘model fr the velocity is m do/dt = mg — ku. Using separation of variables to solve this differential equation we obtain u(t) = mo/k + c27M™. Fram u(0) = 0 we get o(t)'= (mg/k)(1 — e-#/m), Letting & = 05, m = 160/32 = 5, and = 32 we have v(t) = $20(1 ~ el"), Integrating, we find s(t) = 320 + 3200e-°, At ¢ = 15, when the parachute opens, v(15) = 248.598 and (18) = 551402 At this point the value of k changes to * = 10 and the new initial velocity is vp = 248.598. Her velocity with the yasectste open (With time measured from the instant of ‘opening) is vp(t) = 16 + 232598". Integrating, we find 3p(2) = 36 ~ 116.200". Twenty seconds after leaving the plane is five stoonds after the parachute opens. Her wlocity at this time is v9(S} = 16.0106 ft/sec and she has fallen s(18) + 55(8) 5594.01 8. Her terminal velocity is ime vp{t) = 16, 0 she has very nearly reached her terminal velocity five seconds after the parachute opens. When the parachute opens, she distance to the ground is 15,000 ~ 5514.02 = 9485.98 ft. Solving ap(t) = 9485.98 we got ¢ = 592.874 s = 9.88 min. Thus, ic ‘wil take har approximately 9.88 minutes to reach the ground after her parachute has opened and a total of (592.874 + 15)/60 = 10.19 minutes after she exits the plane 833. (a) The aifeenial equation is Sesvorder, linear. Letting b = Jp, the Interating factor is eS dH) ry, Then ¢ 2. (roby llr +B} = ara tb? and (ro + bv = Biro +b) +e ‘The solution of the differential equation is v(t) = (9/46)(ro-+ be) +clro-+ta)-®. Using v(0) =0 wwe find ¢ = ~gr$/4b, 80 that ers wt 99 : Tk(r0 + kt oft) = 2fro+by - 8, O= we a) Ab(ro + bee ak k m+ ti) (b) Integrating dr /at = k/p we get r = kt/ps-c. Using r(0) = ro we havec = ro,s07(t) = kt/p-tro 76 Exercises 3.1 (©) Ur 0.0003 and (007 ft when t = 105, then solving r(10) = 0.007 for k/p, we obtain ki, ‘r(t) = 0.01 — 0.0003. Soiving r(t) = 0 we get ¢ = 33.3, so the raindrop will have evaporated completely st 38.3 seconds 24, Separating variables we obtain e ae Sone ere f= keost dt In|P|=ksint+e => P= cel a If P(0) = Py then , = Fy and P= Pyebsint = 85. (a) From dP/dt = (hy — kz)P we obtain P = Poel M2 where Fy = P(O) (b) fr > kp then P00 a5 t+ 00. If ki = he then P= Pp for every t. If hy < kz then P +0 ast 90. 36. The first equation can be solved by separation of varlables, We obtain x = ce“™*. From 2(0) = 20 wwe obtain ¢) = zp and so 2 = zoe", The second equation then becomes dy Me dy ut Wage — day ot 4 day = tone a7 OM uy cyt ay = 20d which i linear. An integrating factor ise, Thus a a pty) = zp ye Meat m zy yelteonlt Gitta) = 2 " ety = TOM 2 da ue BOM Goh 4 get Ye From y{0) = yo we obtain cz = (yod2 ~ yoAt ~ 20A1)/(%2 — A). The solution is 20M joist 4 WAR WM = FON pat hm 7a 87. (a) Solving ky(M — A) —hyA = 0 for A we find the equilibrium solution A = hy M/(ki + ha). From the phase portrait we see that lim-ce A(t) = hi M/(B + fa) st es Me co ‘Since kp > 0, the material will never be completely memorized and the larger ky is, the less the amount of material wil be memorized over time 7 Exercises 3.1 (b) Write the differential equation in the form dA/dt+(kt tha) = kyM. ‘Then an integrating factor is e+)", and 4 feng] mayen mete IO a i corththit 388. (a) Solving r — kx =0 for «we find the equilibrium solution x= r/k. When x < r/k, de/dt > 0 and when x > r/k, dz/ds <0, From the phase portrait we see that limo 2(t) = r/k. (b) From de/dt =r — kr and 2(0) = 0 we obtain so that 2 7/k as t= 00. If 2(T) =7/2k then T 1 ~ (r/Bje# (in 2)/& 39, It is necessary to know the air temperature arrives. We will assume that the temperature of the air is a constant 65°F. By Newton's law of from the time of death until the medical examiner cooling we then have up - Faure), To)=8 Using ineviy or separation of variables we obtain T = 65-+ee!. From (0) = 82 we ind ¢= 17,50 that T att 5+ Tel. To find k we need more information so we assume that the body temperature hours was 75°F. Then 75 = 65 + 17 and k = ~0.2653 and T(t) = 65 + 17602880 At the time of death, to, T(to) = 98.6°F, so 98,6 = 65+ 172-9765, which gives the murder took place about 2.568 hours prior to the discovery of the body. ~2.568. Thus, 78 Exercises 3.2 40. We will assume that the temperature of both the room and the cream is 72°F, and that the temperature of the coffee wlien it is first put on the table is 175°F. If we let Ty(t) represent the temperature of the coffee in Mr. Jones’ cup at time ¢, then whieh implies 7; reduces its temperature by an amount a, so thet T;(0) which implies ¢) = 103 ~ a, so that T;(¢) = 72+ (108 ~a)eM#. Att = 5, T1(6) Now we let T(t) represent the temperature of the coffee in Mrs. Jones’ cup. From Ts and T3(C) = 175 we obtain ¢7 = 103, so thet Ta(t) = 72+ 108e. At t= 5, To(5) = 72 + 1035 ‘When cream is added to Mrs. Jones’ coffee the temperature is reduced by an amount a. Using the fact that k <0 we have Fg < 724 1036 ~ ae* 2+ (103 ~ ae = 73(5), ‘Thus, the temperature of the coffee in Mr. Jones’ cup is hotter. 41. Drop an object from a great height and measure its terminal velocity, y. In Problem 29(b) we saw that ux = gm/k, 80 k = gm/v. 42. We saw in part (a) of Problem 30 thet the ascent time is ta =: 9.375. To find when the cannonball hits the ground we solve s(t) = —16t? + 900¢ = 0, getting a total time in fight of t= 18.75. Thus, the time of descent is tg = 18.75 — 9.375 = 9.375. The impact velocity is vy = v(18.75) = ~300, which hes the same magnitude as the initial velocity ‘We saw in part (b) of Problem 30 that the ascent time in the case of ait resistance is te = 9.162. Solving s(t) = 1,340,000~6,400t~1,340,000e~000 = 0 we see that the total time of flight is 18.466, ‘Thus, the descent time is ty = 18.466 ~ 9.162 = 9.304. The impact velocity is y = (18.466) = 290,91, compared to an initial velocity of wo = 300. Exercises 3.2 1. (a) Solving N(1 ~ 0.0005") = 0 for N’ we find the equilibrium solutions N = 0 and N’ = 2000. ‘When 0 < NV < 2000, dN/dt > 0. From the phase portrait we see that Tint. M(t) = 2000. ~—>_—_4 a 200 A graph of the solution is shown in part (b) 79 Exercises 3.2 (b) Separating variables and integrating we have an (i= 0.0008) = and ‘a InN ~In( — 2000) = ¢ +6 ad Solving for N we get N( solving for e° we find e* 2o00e** (4 + eF¥4) = 2000eFet (1 + ee). Using (0) /1999 and so N(#) = 2000e'/(1999 +e}. Phen. N10) 50 1834 companies are expected to adopt the new technology when t = 10 and 833.59, an . 00a From SE = Ne BN) and (0) = 500 we obtain W = aap. since im = $ 50,000 and (1) = 1000 we have a = 0,709, 6 = 0.00014, and N= ec 500 . From 22 = p{io-t ~10-7P) and P(0) = 5000 we obtain P = so that at P ~ 1,000,000 as ¢ > oo. If P(e) = $00,000 then t = $2.9 months, (@) We have dP/dt = P(a— bP) with P(0) = 3.929 million, Using separation of variables we obtain Tons + 0.0905—-oF 2.00 afb TOG + (a — 3.98DH|e-e * TF Ta/SaIse— Tee Tram Ne ‘At t= 60(1850) the population is 23.192 million, 30 To (e/S029 = ee 23.192 + 29.192(c/3.929 — 1}e"®. At t = 120(1910) Pie 23.193 = ¢ 912 = Ty epaaaa ee ‘or ¢ = 91.972 + 91.972(¢/3.929 — 1)(e-}2, Combining the two equations for ¢ we get (c= 23.199)/28.192 He c-1) = 2591 2/3928 =1 Ta oo 91,972(8.929}(e ~ 23.192)? = (28.192)"(c — 91.972)(c ~ 3,029). ‘The solution of t's quadratic equation is ¢ = 197.274. This in turn gives a = 0.0913, Therefore awit oS T+ d9.2fe-OaaT 80 10. a. 12, . From So 44150 - X)%, X(0) = 0, and X(5) = 10 we obtain X = 150 |. (a) The initial-value problem is dh/dt = —8A,VR/Aw, 19 Exercises 3.2 150 Tok +1 ‘k = 000095238. Then X(20) = 93.3 grams and X — 150 as t -> oo so that the amount of A+ 0 and the amount of B 0 as # + oc. X(t) = 75 then t = 70:minutes where’ (0) = H. Separating variables and integrating we have THOTT Hi we find c= 277, so the solution of the initial problem is Jh(@) ~ (AwVFT ~ 44yt)/Ae, where de H ~ 4Agt > 0. Ts, A(t) = (AuV FE ~ 4An8)4/A2 for 0S t < AuHl/4An, (b) Identifying H = 10, Ay = 4r, and Ay = 1/576 we have A(t) = 2/331,776— (\/5/2 /144)t +10. Solving A(t) = 0 we see that the tank empties in S76V10 seconds or 30.96 minutes ‘To obtain the solution of this diferential equation we use A(é) ftom part (a) of Problem 11 in Byercises 11.3 with Ay replaced by cAn. Then A(t) = (AuWH = deAnt)?/43,. Solving Ale) = 0 with c = 06 and the values ffomn Problem 11 we see that the tank empties in. 085.79 seconds or 90.6 minutes (a) Separating variables and integrating we have eA9dh= st and 2H? = ste ‘Using A(0) = 20 we find (8005 ~ 3t)*. Sotving Ale) 1431 minutes (b) When the height ofthe water ish, the radius ofthe top ofthe water is r= tan 30° = b/V3 and Ay = 7h?/3, The differential equation is th (2/12)? B= ah = 00g = aa Sepacating variables and integrating we bave 5h dh = —2dt and 2h%/? = -2t +0. Using h(0) = 9 we find c = 486, so the solution of the initial-value problem is h(#) = (243-1) Solving A(t) = 0 we see that the tank empriea in 24.3 seconds or 4.05 minutes. When the height of the water is A, the radius of the top of the water is $(20~) and Ay = 4r(20 — A)?/25. The differential equation is 38203, so the solution of the initial-value problem is A(t) = (we see that the tank empties in 3843 seconds or 22/12" ee _8_ VR = 08 5520 — APTS 82 Exercises 3.2 (b) jansus Predict = Year | Population Population Error Error 0 398 ‘389 0000 G00 10 5308 33a 0028049 810 720 722 = “os oma an {9538 ome 0108-112 aso | 12885 va090 = 024114 sso | 11059 14% 0405 238 asso | 23:92 wus 004901 wo | 31499 0341 ios 347 iso | 38558 m2 0 ome 185 10 | 50156 maou = 02022 wo | esis 6260 © 048055 100 | 75.096 wos = 06m 088 io | sism sites 0239025 ao | rosm) = noraag ag 85 io | ims = im) ts 02 rio | 131659 138058438934 asp0_| sso67 __nasass_ p57 149 ‘The model predicts @ population of 159.0 million for 1960 and 167.8 million for 1970. ‘The census populations for these years were 179.3 and 203.3, respectively. The percentage errors 128 and 21.2, respectively. 5. (a) ‘The differential equation is dP/dt = P(1—In P), which has equilibrium fe. When Py > e, dP/dt <0, and when Fy solution 0. (b) The differential equation is dP/dt = P(1-+n P), which has equilibrium == solution P = /e, When Pp > 1/e, dP/dt > 8, and when Po < 1/e, aP/dt <0. ealberee +01 $0 that PO) = ap 7 6. from = Plo bi) we bala Snot aa then Gand 1 te X= X() tee ant fC une ant 2 9 ax} 39. 110 X(8) = 10 then X = 0001259, and X20} = 29.3 grams. Now X — 60 a5 t+ 00, so that the amount of A —+ 0 and the amount of B — 30 as t— 00, 81 Exercises 3.2 14. We solve mi =—mg—iv?, —0(0) = 300 ra using separation of variables. Tis gives me o re v0 = (2 tan (stone -\2 we find Integrating and using s(0) = fa mil s(t) = Fin |oos [Ej lm )] +e. Setting t =O and « = so we find e2 = 60 — In(ooshey) 83 Exercises 3.2 follows that (0-H) ated nt s[0-ViF@) 0] ace teae udu “avin é Wee udu/VT+ a? so that James Solving for y? we have potor+dn4(2) (+2) which i @ family of parabolas symmetric with respect to the z-axis with vertex at (e2/2,0) and focus at the origin (b) Let um 2? 4 y? go that ‘Then 85 Exercises 3.2 Separating variables and integrating we have vay Parte 17. (a) From 20? — W? = WH(2— WW) =0 we see that WY = 0 and W = 2 axe constant solutions (b) Separating variables and using a CAS to integrate we get aw. dae and tant t(LVIZ BW) mat Tap te tod nt VITBP) we Using the facts that the hyperbolic tangent is an odd function and 1 — tank®2 = sedis we have AaB at(oe oa tare) Lu—20) = eandle +) 1 bw = tanb(e +0) AW = 1~tantlle te) = sale +0) ‘Thus, W(z) = 2sech*(z +c). (6) Letting x = 0 and W =2 we fad that sechc) = 1 and e=0. 18, (a) Solving + (10 — h)? = 10? for 7? we see that r2 = 20h A2, Combining the rate of input of water, m, with the rate of output due to evaporation, knr? = kx(20h — h®), we have dV/dt x ~ kx(20h— A), Using V = 10xh? — Jrh?, we see also that dV/dt = (20ch — xh?)dh/at Thus, ah a (20nn ~ 12) =m — kx(20h—H2) and 86 Exercises 3.2 (b) Letting & = 1/100, separating variables and integrating (with the help of » CAS), we get : 200K(h~ 20) 100(K? — 10h + 200) : Taio ah dt and a att Using {0} = 0 we Gnd ¢ = 1000, and solving for A we get |) = 0.005( VF7+ 4000% — t), where the positive square root is ‘chosen because h > 0. (©) The volume of the tank is V = $(10)8 feet, so at a rate of w cubic fee pet minute, the tank ‘will fil in §(10)° = 666.67 minutes ~ 11.11 hours. (a) At. 666.67 minutes, the depth ofthe water is h(666.67) = 5.486 feet. From the graph in (b) we suspect that liz. A(#) = 10, in which case the tank will never completely fill. To prove this ‘we compute the limit of A(t): 2440000 - 0.005 im (1/22 + 0008 — ¢) = 0.005, im 8 i (V8 + 400¢— 1) = 0.005 i = 0.005 fin —— 40004 ___ 6,095 Jim af a000/t + eo TeT 0.005;2000) 19. (a) [7% Fo Tay [> ITT Te 20 300 | 0.036 20 | 37240 | 0!033, 30 | S'ese | O°033, 0 eee | 0.093, 3 a | 0.036 co | 25!as2 | 0036 8 435 | 01033 a0 | 3elsse | 62030 50 | sorsse | 0.026 200 sae | oloat ito 396 | o.oat sao | ailgre | oon 130 | ves. | o:tas 140 | 1221775 | 0007 350 e63 | Olona 160 659 | 0.013 30 $00 87 Exercises 3.2 (b) The regression line is @Q = 0.034830 — 0.000168222P. (©) The solution of the logistic equation is given in equation (6) in the text, Identifying a = 0.034831 and 6 = 0,000168222 we have ah G= PO = Eee (a) With Py = 3.929 the solution becomes 0.196883, PO = paneGORTE 0.0341 TBI ETE oo: 4} (6) We identify ¢ = 180 with 1970, ¢= 190 with 1980, and ¢ = 200 with 1900. The model predicts P(180) = 188,661, P(190) = 193.735, and P(200) = 197.485. The actual population Sgures for these years are 203.303, 226.542, and 248.765 millions. As t + co, P(t) + 4/9 = 207.102. 20. (a) Using a CAS to solve P(1~ P) +0.3e~ = 0 for P we see that P = 1.00216 is an equilibrium solution. (b) Since /(P) > 0 for 0 < P< 1.09216, the sokution P(t) of dP/dt = P(1~P)+0.2e?, P(0)= Pe, is increasing for Fh < 109216. Since f(P) <0 for P > 1.09216, the solution P(¢) is decreasing for Ay > 1.09216, Thus P = 1.09216 is an attractor, 88 21. 22. Exercises 3.2 (c) The curves for the second initial-value problem are thicker. The cuibrium solution forthe lople model is P = 1, Comparing 5 1.09216 and 1, we see that the percentage increase is 9.216%. To find ty we solve ma =mg-m, (01-0 é sing sepacatin of variables. This ges fm ig sit) = YE ean He Integrating and using (0) = 0 gives { Py w= Fan(s (2) eee (a) Solving u = \/mg/k for k we obtain k = mg/v?. The differential equation then becomes mem Bt oo Sxe(1- 50) oe ae a Separating variables and integrating gives wetanh! 2 = gt, ‘The initial condition v(0} = 0 implies ¢ = 0, so = st v(t) = eetanh & We nd the nase by teptog t= [osam tar En (aan) +0 ‘The initial condition (0) = implies e» = 0, 50 oe at s=F In (coon 2) Jn 25 seconds she has fllen 20,000 ~ 14,800 200 fet, Solving 5200 = (v?/32) in (costs 22028) y 89 23. Exercises 3.2 for v gives uy © 271-711 ft/s, Then st) = 2 (cosh) = 207 0st 0.117772 (b) At t=15, s(15) = 2,542.94 ft and (15) = s/(15) = 256.287 ft/sec. “While the objet i inthe air ts velocity is modeled by th near diferente equation m/e = mg~ kv. Using m = 160, k= 4, and g = 32, the differential equation becomes du/dt + (1/840)u = 32. The integrating factor is ef4'/6 = ef/ and the solution of the differential equation is /5H0y =f 32et/40 ge = 20,480e/ +c. Using v(0) = Owe see that ¢ = -20,480 and v(t) = 20,480 — 20,4802~*/*0. Integrating we get s(t) = 20,480t + 13,107,200e~*/% + ¢, Since s{0) = 0, ¢ = ~13,107,200 and s(t) = ~13,107,200+20,480t + 13,107,200e-*/*°, To find when the object hits the liquid we salve s(z) = 500~75 = 425, obtaining t, = 5.16018. The velocity at the time of impact with the liquid is vq = v(t.) = 168.482, When the object is in the liquid its velocity is modeled by mdv/dt = mg—kv?, Using m = 160, 32, and k = 0.1 this becomes du/dt = (51,200—v*)/1600. Separating variables a integrating We have spats ot ont in PO | ee 51200 - 0? ~ 1600 640 “| w+ 160V2 |” 1600 164.482 we obtain ¢ = —0.00407537. Then, for v < 160V2 = 226.274, Jue MOvE | vayetaus 4, _ Ya 1OVE _ ya/s-to0s v+ 60V2 vt 160V2 Solving for v we get Solving v(0) = v4 (y= 189686 2008 29608 MO* GLTSS + 9.7507 ETHIE Invegrating we find (6) = 226.275¢ ~ 1600 n(6,.9237 + eV") + 0 Solving s(0) = 0 we see that ¢ = 3185.78, so s(t) = 3185.78 + 226.275¢ — 1600106 3297 + eM), Tp find when the object hits the bottom of the tank we solve s(¢) = 76, obtaining t, = 0.496273, ‘The time from when the object is dropped from the helicopter to when it hits the bottom of the tank is ta + ty = 5.62708 seconds Exercises 3.3 Exercises 3.3 1. The equation dz/dt = —Axe can be solved by separation of variables. Integrating both sides of dr/x = —Drdt we obtain In|2| = —Axt +c from which we get x = c1e**, Using 2(0) = z0 we find cia so that <= zoe"™". Substituting this result into the second diflerential equation we have dy at fy Nay = ayzoe ete ‘which is Hinear. An integrating factor is e™ so that, & [ety] = aizael2- 9 + op 220 Sy g Mat eget = MED_ge hit 4 gem bat Bete let ce oN en (0 we find ep = ~Ayz0/(A2~ 2s). Thus daz /,-at ae Substituting this result into the third differential equation we have Avdato (_ rat _ gore poe) Integrating we find Using 2(0) Owe find os = zo. Thus ena(t 2. We see from the graph that the halflife of A is ap x. ys proximately 4.7 days, To determine the halflife of B24 wwe use t of substance A is so small that it contributes very lite tle to substance B. Now we see from the graph that 29 y(50) © 162 and y(191) & 81. Thus, the halflife of Bis approximately 14] days dapat uu 0 as a base, since at this time the amount 230 3. The amounts of x and y are the same at about ¢ = 5 days. The amounts of 2 and 2 are the same at about t = 20 days. The amounts of y and = are the same at about ¢ = 147 days. The time when and 2 are the same makes sense because most of A and half of B are gone, so half of C should have been formed. 5 to Tas 18a a1 Exercises 3.3 Suppose that the series is described schematically by W mp —yX => -AY <> —As2 where hy “Aa and —As aze the decay constants for W, X and ¥, respectively, and Z is a stable element Let w(t), x(t), y(t), and 2(¢) denote the amounts of substances W, X, ¥, and 2, respectively. A model forthe radioactive series is ty i ean a Fw - de Ss a: di }. The system is a2-3¢se, : 2 +6 ja2stgn-dnda-dat hen 1 22 a-ha du-gn Lat 21,2, ad be the amounts of lt in tanks A, B, and, respectively, so that hal 1 13 = Gog Gye Os ye wT a ie at 1 oa sya dyer 6+ mn - gher-2- hase Sa ~ hare dis oe eee ‘s= 3997? 700"? ~ 106" (a) A model is day 2 a. Bas te -=1(0) = 100 2 ioe oo (b) Since the system is closed, no salt enters or leaves the system and 2y(t)+z9(¢) = 100+50 = 150 for all ime. Thus 21 = 150~ 22 and the second equation in part (a) becomes dq _2(150— 2) __32y_ __-300_ rp Sap “& ~~ O+t =~ 100-F 100 E” 10044 100-8 dy (2. 3 300 (eat) we which is linear in 2. An integrating factor is eBl(t00+01-81n( 100-0) = (109 + 4)2(200— 2) 92 Exercises 3.3 Sia00 + £)2(300— #}-z9] = 300(100 + ¢)(200 - 2)~3 Using integration by part, we obtain (200 + 17¢400 ~ £)-#rp = 200 [5100 +-1)(200 —1)-? = $o00-2"" +4 Tas tq = 3, fer190— 2) — (100 - 1)? + LG — = Tapeeap [A100 — 88 — 5200 — 2) + 5200 + 1100 »] 300 ee = Tae ~ 9° + 1000-0) Using 29(0) = 59 we find e= 5/3000. At t = 30, xp = (800/180)(70%e + 30 70) = 47.4 Ibs. 8. A moti 1 4 ain 15a) ~ (4 /) (54 ein) (oat) ~ (n/n) (2) atin (Boe et) ~ etna) (2.29 /) ee an ae asa a és ae Over a long period of time we would expect 21, 22, and 23 to approach 0 because the entering pure water should fush the salt out of all three tanks. 9. From the graph we see that the populations are first equal vy at about t = 5.6. The approximate periods of x and y are \10 both 45. Ene! 30 100 93 10. a 2. Exercises 3.3 (a) The population y(t) approaches 10000, while the population (0) approaches extinction s co (b) The population 2¢) approaches 5,000, while the population x(t) approaches extinction. (c) The population y(t) approaches 10,000, while the population, (2) epproaches extinction, (d) ‘The population x(t) approaches 5,000, while the population ay y(t) approaches extinction. eee ‘ry 0° () 4 eo 5 s 0 ot 2k 20 © 44 @ 9 s s 20 40 2» 4! In each case the population x(t) approaches 6,000, while the population y(t) approaches 8,000. By Kirchof’s frst law we have i = ip-+is. By Kitchofs second law, on each loop we have B(®) = Lil, + Ryda and E(t) = Li, + Rais+ 4g so that q = CRii2—CRais, Then ig =o = CRyi -CRaie so that the system is Lil + Lis + Riis = E(t) Repos Ruth Rah + Gis 94 Exercises 3.3 13. By Kirchof’s first law wo have i: = ig + is. Applying Kirchofl’s second Iaw to each loop we obtain Blt) =P +2 +i a and dey BU) = aiRy + La + aks, Combining the three equations, we obtain the system + (Ri + Radin + Rais = B niente mie 14, By KirchoHs first aw we have & Et) = Lif + Riz and E(t) = Li + i so that q-= CRip. Then ig = q’ = CRif so that system is in + ts, By Kirchoff’s second law, on each loop we have Lil + Riz = B) CR +ig— i = 0 15, We fst note that s(t) +i(t) +14) =n. Now the rate of change of the number of susceptible persons, s(t), is proportional to the numberof contacts between the number of people infected and ‘the number who are susceptible; that is, ds/dt = —kys;. We use —k1 because s(t) is decreasing. Next, the rate of change ofthe number of persons who have recovered is proportional to the number infected; that is, dr/dt = kai where kz is positive since r is increasing. Finally, to obtain di/dt we Sosi+ es ‘This gives di @ ‘The system of equations is then ksi hoi + si fr A reasonable set of initial conditions is 2(0) = ip, the number of infected people at time 0, (0) = n~ ig, and r(0) = 0, 16. (a) If we know s(t) and i(t) then we can determine r(#) from s+ i+ 95 Exercises 3.3 (b) In this case the system is a Ga -025 ai Fn WOT + 0.28%. We also note that when (0) = all values of t. Now ho/ky i, s(0) = 10 ip since r(0) = 0 and é{t) + s(t) + r(t) = 0 for conditions s{0) = 2, i(0) = 8; (0) = 3.4, 4(0) = 6.6; (0) a st if 5 Ss: - To Toe Dt ae ‘We see that an initial susceptible population greater thax ky/ky results in an epidemic in the sense that the number of infected persons increases to a maximum before decreasing to 0. On. the other hand, when s(0) < Az/ky, the number of infected persons decreases from the start and there is no epidemic. LT. Since zo > yo > Owe have x(t) > y(t) and yz <0. Thusde/dt<0 x and dy/dt > 0. We conclude that z(t) is derreasing and y(t) is increasing, As t — 00 we expect that (0) + Cand ylt) = C, *(} whore C is a constant common equilibrium concentration vio} 18. We write the system in the form F =my-2) aoe ETRE Exercises 3.3 where ky = «/Va and ky = a/Vp. Letting 2(¢) = 2(¢) — y(t) we have a dy So aya} hale) a Ga h(-2)— he as Et thje=0 ‘This ise first-order linear diferentiel equation with solution 2(t) = ee~(**4, Now az Se ily - 2) = hy =~ Gym hy 2) = z= hy and a(t) = re hth +9, Since y(t) = x(t) — 2(t) we have ecthyehale ten ‘The initial conditions 2(0) = z0 and y(0) = yo imply aS a ee = y+ ke ‘The solution of the system is aft) = COBO ete 4 Z0ka 30h (0 Re he a5 +h = W020) nyt Zoho + yo MO- "nam © + hth ‘As t+ 00, a(t) and y(t) approach the common limit, oki + yoki _ zon/Va+wxi/Va _ 20Vat we ky + he w/Vatn/Ve Va+ Ve Va Ve. VatVe eeva +Ve = 10 ‘This makes intuitive sense because the limiting concentration is seen to be a weighted average of ‘the two initial concentrations. 97 Exercises 3.3 19. Since there are initially 25 pounds of salt in tank A and. none in tank B, and since furthermore only pure water is being pumped into tank A, we would expect that 21(t) would steadily decrease over time. On the other hand, since salt is being added to tank B from tank A, we would expect z(t) to increase over time. However, since pure 50 i water is being added to the system at a constant rate and a mixed solution is being purnped out of the system, it makes sense that the amount of salt in both tanks would approach 0 over time. Chapter 3 Review Exercises —____________ 1. Pom 2-001? an = hag wba P= da PS) 8 on i ie Sp 12 =F and A(0) = 168 we obtain 5.789, the concentration is 0.017%. As t —+ 00 we have 2. Let A= A(t} be the volume of CO» at time t. From A= 48+ 112M Since ALO) A= 48%? oF 0.06%. 3. (a) For0 10 the equation for the current becomes 0.21 = 4 or t= 20. Thus at-}, 0 4 and 1 < Fo <4 the population approaches 4 as ¢ increases. For 0 < Fy <1 the population decreases to 0 in finite time. (b) The differential equation is ¢ = PS P)-4=-(P?-5P +4) = -(P-4)(P -1) Separating variables and integrating, we obtain ap wane" 41/3 1/3 (A )ar= ~at Setting t= 0 and P = Fh we find cy = (Fp — 4)/(Fe — 1). Solving for P we obtain Po Fo = den (Po=1) = (Po~ de® P= (©) To find when the population becomes extinct in the case 0 < Fh <1 we set P=Oin Pr4_ And a Pri a1 from part (a) and solve fort, This gives the time of extinction t= tn HB=-D 3°" RT 2. (a) Solving P(S - P) - = 0 for P we obtain the equilibrium solution P = §. For P 4 $ Pat < 0. Thus, if Fy <§, the population becomes extinct (otherwise there would be another equilibrium solution.) Using separation of variables to solve the initial-value problem we get. 101 Chapter 3 Related Exercises P(t) = [AP + (10) ~ 25)/[4-+ (UR) — 10}. To find when the population becomes extinct for Py < § we solve P() =0 fort. We see shat the time of extinction is t= 4P/5(5 ~ 2F)) (b) Solving P(5—P)-7 = 0 for P we obtain complex roots, so there are no equilibrium solutions. Since dP/dt < 0 for all values of P, the population becomes extinct for any initial condition, Using separation of variables to solve the initial-value problem we get Py a) _ va Solving P(t) = 0 for t we see that the time of extinction is, Flvatan-"(6/v3) + V3tan[(2R, —5)/v3}) 3. (a) Without harvesting, the population is governed by the logistic equation dP/dt = P(r ~rP/K). With initial “© population Py, the population was shown in Section 3.2 to be : 1A PO = IRS PR SS BIRT OBI ‘To find when P(t) = 4 we solve ae BIKY =k oo ATK Reto This gives 70,000 no) * 194 98 (b) Letting K = 400,000, Po = 72,000, and r = 0.08, and, solving = K~ JR =aKir . : for h, we get h = hg = 4620. 102 (c) The MSY is obtained when Ey 4. (a) (b) () @ yield in this case is BP, = KB(1~ B/r) 2 limiting population is K(1 — E/r) = 200,000, 04, and the 000. The Letting 1959 be year 0 and 1973 yeas 14, we obtain the graph shown. ‘Taking P(0) = 1.91/(c- 414 - 294) = 78.4613 and ex- perimenting with the other parameters, we find that the graph of the logistic function fts the data points reason- ably well for 500 and © oat ‘The graph is shown with Pp = 78.4613, K = 500, 0.11, ond hm fri 6.875. ‘The graph is shown with Po = 78.4613, K = 500, re O11, and B= fr = 0.056. 103 Chapter 3 Related Exercises eo) 7 ee 4 Higher-Order Differential Equations Exercises 4.1 1. From y = c1e® + exe"? we find y’ = oje* — ne. Then y(0) = e1 + er = 0, y/(0) =e) - cp = 180 that ey = 1/2 and 2 = 1/2. The solution is y 1c" + cp we find y/ = 4cye* — ge? Then (0) = c1 +02 2/5. The solution is y= et* + Be (0) = der en 2. From y so that ¢, = 3/5 and ep 3. From y= c1z-+epz nz we find yf = ot + on(1+In2). Then y(1) = = 3, y'(1) =c1 ten 3 and cz = —4, The solution is y ie —Azinz 4, Prom y = cy +0nc0s- +essinz we find yf = ~cpsin2+-cy cos and y” = cp cosz—cysinz. Then lr) = 0-02 = 0, ¥ (a) Y"(@) = 1, and cy = -2. The solution is y 1 s0 that ¢1 = 1, 62 ~cosz — 2sinz. 8. From y= ci + 02? we find yf = ors. Then y(0) = ex = 0, (0) = 202-0 = 9 and y/(0) = 1 is not 0, Theorem 4.1 is not violated. possible, Since a2(z) = 2 is 0 at 6. Im this ease we have y(0) =e: =0, ¥(0) Band y = 222 162 ‘Om 0 $0 c = 0 and cp Is arbitrary. Two solutions arey= 2? 7. From 2(0) = to = cy we see that x(t) = xocosut + opsinut and 2'(t) = ~zosinut + ow cosut. ‘Then 2'(0) = 21 = ogy implies ¢ = 21/u. Thus a(t) = ap cosut + = sin ut. 8. Solving the system uta) = 1 cosuty +2 sinssto (to) = ~crwsinuty + exweasuto = 21 for ey and cp gives Fr cosuto + wo sinsst ey = AROS lo TFS and ey = TSH Fotos ‘Thus : wzyeobuty— 21 5inu _ —er—~—r—._._.C:28szC¥isC¥ = ry(coswt cosuty + sinwt sinwto) + = sinwt coswto — coswt sin uty) = ronal) + Banat 8) 104 10. nL 12, 13. 14. 15. 16. 17. 18. 19. 20. Exercises 4.1 }- Since ag(x) = 22 and 29 = 0 the problem has a unique solution for -00 < z <2 Since ap(z) = tan and 29 = 0 the problem has a unique solution for ~#/2< 2 0 and let yo = —z° for x < 0. Ini? is a solution let yp = 2? 220 then yp =2 pn) =| (b) Ha20 then y= aad Wend = | "3 M2-<0 then y= sand Winn) =|, This does not violate Theorem 4.3 since az) = is zero at x = 0 () The functions ¥i = 29 and ¥_ = 2? are solutions of xy" ~ éry/ + 6y = 0, They ate linearly independent since W (23,22) = x* #0 for co < 2 < ov. (4) The funotion y = 2 satisfies y(0) = 0 and ¥(0 (2) Neither isthe general solution since we form a general solution on an interval for which n(x) 4 0 for every 2 i the interval Since & eFetlet mS, we see that & functions are linearly dependont ote 2 isa constant multiple ofe**? and the 108 Exercises 4.2 AL, Since Gyn + 0yp +--+ Ok + Yoke 42, The solutions are linearly dependent. Suppose n of the solutions are linearly independent (if not, then the set of n + 1 solutions is linearly dependent). Without loss of generality, let this set be v1, Wo Yae Then y = 0x + 0m +--+ ca is the general solution of the nth-order diferential equation and for some choice, ¢}, c.f, of the coeficients ga = Chui + cfun-+----+ ett. But ‘then the Set ¥1, 9, .., Uns Yous is linearly dependent. the set of solutions is linearly dependent. Exercises 4.2 —————__ In Problems 1-8 we use reduction of order to find a second solution, In Problems $16 we use formula (8) from the text. 1. Define y = u(z)e so ym Que 4 ule, aa oul + eu! + dey, andy’ — dy + dy = 4e™u" = 0. ‘Therefore u" = 0 and u= cx + cq, Taking ci = 1 and cp = 0 we see that 2 second solution is ware, 2. Define y = u(z)re"* so vin(l—z)e ut seed, yf ze tu" +201 —z)e ty 2— seu, and v4 Oy ty = eF(eu" + 2u) = 0 or w+ lu! =0. If w = w we obtain the first-order equation we? ‘w = 0 which has the integrating factor e?/ ¢*/* = 2", Now a qelttul=0 gives atu =e ‘Therefore w 5/2 and u = c1/s. A second solution is yo = tae 3. Define y= u(x) cose 0 usin dz + uv! cos4z, wl" cos 4x — Bu sin dr — 16ucosde. and yl" + 16y = (cosdz)u" — S(sindrju’ = 0 or ul" ~ 8{tan dru! = If w =v! we obtain the first-order equation w/ — 8(tan dz)w = 0 which has the integrating factor ertJisndeds — os? dz, Now E eostaa)uj = 0 giver (cos? azju 109 Exercises 4.2 ‘Therefore w = w! = csec?4z and u = cy tan-4z. A second solution is yp = tan 4zcos4z = sin te. 4, Define y = u(z) sin 3x s0 y= 3ucosaz+u/sin3z, "=u" sinds + Gu! cosdx — usin dr, ad xy" +9y = (sin3r)u" + 6(cos3z)u'=0 or wu" + 6{cot Iz)u! If w = w! we obtain the first-order equation w' + 6(cot 3c)w = 0 which has the integrating factor SJooseds sin? 3x, Now Litoin?s2)u} =0 gives (cin?3z)w Therefore w =u! = cesi?Sr and u = e; cote. A second solution is y2 5. Define y = u(r) coshz s0 ‘ot dr sin 3z = cos3e. ysusinhz+u'coshz, "=u" cosh + 2u'sinkz + ucoshr and y! ~y=(coshz)u" + 2(6inhz}u! = 0 or ul + 2(tanhz)u! = 0, If w =u! we obtain the first-order equation w/ + 2(tanh-z)w = O which has the integrating factor ePfiasheds = cosh? x, Now Ailcost?2yu) =0 giver (cost? zw =e ‘Therefore w =u! = csech and u= cy tanh, A second solution is yo = tanh zcosh = sinh = 6. Define y = u(z}e¥ so of = Seu bePul, yf mu! + 10eFu! + 2568 u and yf! — By = &(u" + 10u') fw = wwe obtain the first-order equation wu! +10w el, Now or ul" +10u' =0. hich has the integrating factor e!0 4 105, 10s, ‘Therefore w 7. Define y = u(2)e*/9 so 2-1 A second solution is yo v Lea AL, ofc Ps holy Sy!" — 12y/ + dy = 9e*/4u" = 0. 110 10. 1. 12, 13. Exercises 4.2 ‘Therefore u" = 0 and w= ez +0. Taking ¢) = 1 and cz = 0 we see that a second solution is = e208 meee - Define y = u(2)e*!? so 2 1 fm erly Pesityt 4 Leity vi gee + 5 and gh ao (6u" +5u')=0 or wl + Zu’ =0 fw = u! we obtain the first-order equation w’-+$w = 0 which has the integrating factor 6/9) 4 = &/®, Now a A (cul a0 gives ew me, & lo ‘Therefore w = u! = ce"! and u = cye"*/% A second solution is yo = e~S*/Se#/ = e-#/2 Identifying P(x) = ~T/z we have ce en f-tieyde 6 [LOM ara ot [Laem ile A second solution is yp = 24 In|z]. Identifying P(x) = 2/z we have er fltsyde nas [CPN aoa a fates ‘A second solution is yo = 2°9 Identifying P(e) = 1/2 we have =n [Gor A second solution is yp = 1. Identifying P(2) = 0 we have eine f A second solution is yz = 2/2, fet demas | eis ” Identifying P(2) = ~1/e we have ec S-#els eo seintns) |S dew saints | aia = (esin(ln2)][~cot(laa)] = —zeos(in2) |A second solution is yp = xeos(ln2). 111 Exercises 4.2 14, Identifying Pz) = ~3/2 we have eof -Sdeis 3 pesteaties |S dea stowton | ages = 2 cos(Inz) tan(Inz) = 2*sin(In2) A second solution is yp = 2°sin(ln) 15. Identifying P(z) = 2(1 +2)/ (1 - 22-2?) we have ei nt n=e+y [a se eer (e are eon [Fem ern/ |e ap-ie wesof teat A second solution is yy = 22+ +2, 16. Identifying Pl) = ~22/ (1~2%) we have vem fe Sesl-Pay = femePte = [ [A second solution is ya = la|(1-+2)/(1~ 2) 17. Define y = u(z)e* so Que syle, yl me — guile + sue and vay ru" —4e*u' =0 or u" — du’ = 0. Iw =u we obtain the fist order equation w’ ~4u = 0 which has the integrating factor e~4/#" = ee, Now dnt 4s ietuj=0 gies eHw ae. getul=0 a ‘Therefore w = u' = ce and u = cre? A second solution is yy = e~**e!* = e7*, We see by observation that a particular solution is yp = —1/2. The general solution is y= cre + ee — 2 18. Define y= u(z)- 1 80 yrew' and you's 112 19. 20. Exercises 4.2 / we obtain the ist order equation w+ = O which has te integrating factor ef Now a & and u = cye~#. A second solution is yp = 1-* Ful=0 gives fw Therefore 2 observation that a particular solution is yp = 2. The general solution is oe We see by yratact +z, Define y = u(z)e® s0 and aly! +2y = eu" eu! = 0 or w= If w = wv’ we obtain the first order equation w’~w = O which has the integrating factor e~ {= Now 0 gives & ‘Therefore w = u! = ce¥ and u = oe, A second solution is yy = efe* = e*. To find a particular solution we try’ yp = AeS#. Then y/= 34eS*, y! = 9Ael, and Ae" — 3 (3Ae%*) + 24c¥ = Bet ‘Thus A= 5/2 and yy = fe. The general solution o prada + Define y = u(z}e s0 and 0 or w= d= 0. yl = + 8y eu" — ee If w =u! we obtain the first order equation w!—2w = 0 which has the integrating factor e~?/ e-™, Now gives «wee. Therefore ce and u= ce. A second solution is yo = e*e =e. To find a particular solution we try yp = ar +6, Then yp =a, uf = 0, and 0—4o-+3{ar+8) = Saz—4a+3b= 2. Then 3a = 1 and ~4a+3b=0 so.a= 1/9 andb = 4/9. A particular solution is yp = }x+§ and ‘the general solution is = = ar) 113 Exercises 4.2 21, (a) For m; constant, let yx = e™=. Then yj = me™* and yf’ = mJe™*, Substituting into the differential equation we obtain af + by ten = omfe™? + bmye™™ + 06? (am bm +6) =0. ‘Thus, ») = e™ will be a solution of the differential squation whenever am?-+ bm +e = 0 Since a quadeatic equation always has a least one rea or complex root, the diferential equation ‘must have a solution of the form yy =e", (b) Write the differential equation in the form ng by gS vate and let y1 = e™ bea solution. Then a second solution is given by me f = os meme [erioramdegs 1 nm oe) lees (omy # ~b/20) ——} _e-(hjatm) a ‘Thus, when m # ~b/20, a second solution is given by ya = e%* where ma = —b/a— mi When my b/2a a second solution is given by nae fde= 20 {c) The functions 7 alee et ) cos = 3(e +e“) phe sinha Keto) cash = Het +e) are all expressible in terms of exponential functions. 22, We have yf = Land yf = 0, sozyf —2yf +y1 = 0—2-+2-= 0 and y(z) = 2 isa solution ofthe dilferential equation. Letting y = u(z)yy(2) = u(x) we get y= ava) tule) andy” =2u"(x) +202) "Then xy! ~2y/ + y = 22u" + 2eu! — 2% — eu + zu = au" — (2? — 2a)’ = 0. If we make the substitution w = u’, the second-order linear differential equation becomes zu" ~ (2? ~ z]w =0, 114 Exercises 4.2 whlch separable =(-})e (pe Inw=z-Inz+e (8 ale Then w = cyet/z and u =o, fetda/z, ‘To integrate e*/z we use the svies representation for €. ‘Ths, second solution is w= nule)= a2 [Zaz wan [isergede ee naz f(z tie des date ee vores ass? +) na(sines24 sh aay" aay An interval of definition is probably (0,00) because of the Inz term. 28. (a) We have y= y= €, 90 ay" — (2+ 19}y’ + 10y = xe ~ (x + 10}e + 10e* and y = e is solution of the differential equation. (b) By (5) a second solution is a elon gen a= ee epee a dene [eres = e*(—3,628,800 — 3,628,8002 — 1,814,002? — 604,802 ~ 151,200z4 — 30,2428 — 5,0402° — 72027 — 902° ~ 1029 — x!9)e-= ~3,628,800 ~ 3,628,800 ~ 1,814,402? — g04,8002° ~ 151,2002* 30,24028 ~ 5,0402° — 7202" — 9028 -- 102" — 2" 115 Exercises 4.2 {©) By Corollary (A) of Thosem 4.2, — yn = x 22" iso solution Exercises 4.3 From 4m? +m. . From m? — 36 From m? ~ m6 =:0 we obtain m= and m= 0 we obtain m= and’m ey tee-24 opel + ne, 2 50 that y = eye + one =1/4 50 that we obtain m = 6 and m = ~6 so that y . From m? - 3m +2 “1e* + cye™. From m? + 8m + 1 From m? — 10m + 25 = 7. From 12m? — Sn — 2 = 0 we obtain m= -1/4 and m = 2/3 so that y = cye7*/4 + ope, J. Fromm + 4m ~ 1 = 0 we obtain m = ~2 YB so that y = qel-2*¥8) + eqe(-P¥8)e From m? +9 = 0 we obtain m and m -3i So that y = cj Cos 3x + ey sin 3z. 10, From 3mm? +1 = 0 we obtain m= if /T and m = —i/V/3 so that y = oc082/V/3+ epsin/V5. AL. From m? ~ 4m +5 =0 we obtain m = 224 so that y = 2¥(ci cose + cpsin2). 12. From 2m? +2m +1 =0 we obtain m = ~1/2+i/2 so that wwe obtain m= Land m= 2-so that y ce + epre 0 we obtain m =~ and m = ~4 so that wwe obtain m= 5 and m= 5 s0 that y = cie* + epze™. y= e7!%(cy cosr/2 + cpsinz/2). 18. From m2+2m +1 =0we obtain m = —1/3-+ VBi/3 so that 218 (cy cos VBz/3 + epsin V22/3) ure 14. From 2m? ~ Sin 4 4 = 0 we obtain m = 3/4 + V3Bi/4 so that 4 (e008 VB x/4 + crsin VBz/4) y 1 so that 15. From m® — 4m? — 5m =0 we obtain m =0,m=5, and m yao toe + cye 0 we obtain m= and m= ~1/2 Vi/2s0 that y= cre +e! (ep00s VBx/2 + casin v32/2) 16. From m* = 17. From m! — 5m? + 3m +9=0we obtain m = —1,m=3, and m= 3 s0 that yea tee +e 116 Exercises 4.3 18. From m3 + 3m? — 4m — 12 = 0 we obtain m a y= ce + exe + ceo wwe obtain m= 1 and m= 144 so that ua ciel + e“epcost + eysint) ‘we obtain m= 2 and m = =1/2+ V7i/2 s0 that exe + eH (en c0s V7t/2-+ esi V 78/2) 21. From m3 + 3m? + 3m +1 = Owe obtain m = —1, m= —1, and m= 1 so that ye ce? tere? + carte 22. From m — Gm? + 19m ~8 = 0 we obtain m so that m= 2, and y= cye™ + core” + cgz?e™. ), m= 0, and m = —1/2+ V3i/2 so that y= orton +e? (oyc0s 32/2 + asin V3z/2) 23. From mé +m? +a? =0 we obtain m 24. From mé 2m? +1 = Owe obtain 1, m=1,m=-1, and m= ~1 0 that cet + opre® + eye + cate’ 25, From 16m +24mn? +9 = 0 we obtain m = 4-V3i/2 and m= +V3i/2 so that eycos Vi2/2 + egsin V3.2/2 + eyxcos V3.z2/2+ cersin V3z/2 v Tm? — 18 |, m= -3, and m = +V2i so that + e3cos VEz + cgsin VEz. 26. From m! ‘we cbtain m yee + oe" 27, From m® 4 Sm! — 2m! — 10m? + m-+5 = 0 we obtain m m= ~5 50 that Ama 1m and m= 1, and ue cye"T + epre? 4 oge’ tyre” +e, 28. From 2m! ~ 7m‘ + 12m9+ 8m? = 0 we obtain m= 0, m 2£2i so that , m= 1/2, and m zm ey tenet ose "!? + (ey 00828 + 0582s) 29, From mi? +160 we obtain m = -b4i so that y= cos 4x + cnsin dx, If y(0) = 2 and (0 2, € = =1/2, and y= 2e0s-4x — sin 2. then 30. From m?+1=0 we obtain m= i so that y = c1cos8 + epsiné. If y(x/3) = 0 and y'(n/3) V3, n= 1, and y = -V3 cos# sin = 2 31. 32. 33. 34. 35. 36. 37. 39. 40. au Exercises 4.3 From m? ~ 4m —5 = 0 we obtain m = —1 and m = 5, so that y = cie* + exe. IF y(l) = 0 and y/(1) = 2 then qe! +o = + Soe? = 2 50 cy = -e/3, 0 = 75/3, and ya adobe ges mae From 4m?—4m~3 =0 we obtain m = ~1/2and m = 3/200that y = o1e"#!? +oye/2, If (0) and y/(0) = 5 then e1+e2 =1, ~Jei-+Jea= 5, so = =7/4, en = 11/4, and y = —Je"*?74 Mee Fromm?-4m4+2 = 0 we btain m = -1/24VTi/2s0 that y= e-*!? (oy cos V7.z/2-+ asin V7 2/2) Tf y(0) = 0 and y(Q) = 0 then c; = 0 and cz = 0 so that y = 0. From m? ~ 2m + Land m = 1 40 that y = cje* + cgze*. If y(0) = 5 and y/{0) = 10 then cy 5, and y = 5e* + Sze”. From m!+12m?+36m = 0 we obtain m =0,m = —6, and m = —6 so that y = c1+c2e-* +eqre“! If y(0) =0, 4'(0) = 1, and y"(0) = -7 then 0 we obtain m 5, e1 +e = 10 80 01 = 5,02 6 ey +e2=0, —Ber+e3=1, 36c2 — 12c3 = —' 80 c1 = 5/36, c2 = ~5/26, ca = 1/6; and y= § - $e et From m? + 2m? — Sm — 6 = 0 we obtain m 8 fae =1,m=2, and m = 3 so that y= c1e* + ope + ope. 18 y(0) = 0, /(0) = 0, and (0) = 1 then er + 202 ~ dea atate 1/6, c2 = 416, ¢3 = 1/10, and e+ deg + Se v exe + cave, If y(0) = 1 and oF a 2c From m?— 10m +25 =0 we obtain m =5 ad v(t) = 0 then e = e168 + one? $0 ¢ = 1, 62 5. From m2 +4 = 0 we obtain m = £24 so that y = cy cos 2r + epsin 2e. If y(0) = 4 and y(n) = 0 then cr = 0 and y = cpsin 2z. From m? + 1 = 0 we obtain m = i so that y = then cy = ~2, e = 0, and y cosa + epsinz. If /(0) = 0 and y'(n/2) 2coss From m? — 2m + u(*) = 1 then cy no solution ‘The auxiliary aquation is m?—3 = 0 which has roots V3 and VJ. By (10) the general salation ts y = exe + ge", By (11) the general solution is y = c1cosh Vr + epsich V8. For y= cre¥ + cye~¥ the initial conditions imply cy +02 = 1, Ve; ~ Vee = 5. Solving for cy +2 = 0 we obtain m = 14% so that y = e*(c;cosz + cpsina). If (0) = 1 and and u(x) = e" cos =~", Since ~e #1, the boundary-value problem has 118 42. 43. 45. 46. 47, 48, 49. 50. 51. 52. Exercises 4.3 we find c = }(3 + 5V3) and cy = 4(3 — 5V5) so y = §(3 + SVS )eV™ + 113 - Sv3)e", For y= eycosh Viz + cy sinh Ve the initial conditions imply cx = 1, Vex = 5. Solving for cy and ep We find cx = 1 ond cy = §V3 s0 y = cosh Vx + § VSsih Vx. ‘The auxiliary equation is m? — 1 = 0 which has roots —1 and 1. By (10) the general solution is y = c1e* +c2e"7, By (11) the general solution is y = ci cosh z + cp sinh z. For y = ce" +@e* the ‘boundary conditions imply ¢: +e = 1, c1e—cye™? = 0. Solving for cy and cz we find c = 1/(1+e2) and cy = €2/(1+e2) soy = €*/(1 +62) +e2e#/(1 +62). For y = cy cosh +casinhz the boundary conditions imply ¢ = 1, ¢9 = ~ tanh 1, so y = cosh x — (tanh 1) sinha. ‘The auxiliary equation should have two positive roots, so that the solution has the form y = exeh* + cael, Thus, the differential equation is (f. - The auxiliary equation should have one positive and one negative root, so that the solution has the form y = ceh* + ge", Thus, the differential equation Is (3) ‘The auxiliary equation should have a pair of complex roots a bi where a < 0, so thatthe solution has the form e**(c cosbe + ez sin ba). ‘Thus, the differential equation ie) ‘The awsiiary equation should have a repeated negative root, so that the solution has the form y= cxe-* + eae, Thus, the differential equation Is (c). ‘The differential equation should have the form »” + ky = 0 where k = 1 so that the period of the solution is 2x. Thus, the diffeentil equation is (d). ‘The differential equation should have the form y/" + k*y = 0 where k = 2 s0 that the period of the solution is x. Thus, the éiferential equation is (b). (a) The auxiliary equation is m? — 64/L = 0 which has roots +8/YE. Thus, the general solutien of the differential equation is x = ¢; cosh(8t/VZ) + ca sinh(8t/ VL). (b) Setting 2(0) = 2p and 20) = & we have ey = 20, Be2/VE = 0. Solving for ey and op we get e1= 29 and oy =0, 20 2(t) = 20 cosh(8t/¥E). (c) When L = 20 and ap = 1, a(t) = cosh(4tV5). The chain will last touch the peg when 2(¢) = 10. Solving x(t) = 10 for t we get ty = }VEcosh™" 10 ~ 1.67326. ‘The velocity of the chain at this instant is 4/(fs) = 12y/T1/5 ~ 17.7989 ft/s. Both ~C{l] and c; represent arbitrary constants, and each may take on any real value Since (m-4)(m-+5)? = m?-+6m?— 15m ~100 the differential equation is y”" +6y”—15y'—100y ‘The differential equation is not unique since any constant multiple of the left-hand side of the 0, and the solution cannot approach 0 as x ~ co, Thus, for the solution to approach 0 we must have) > 0. Now, if¢ < 0 then VORA > b end —b+ VIF=ae > 0. Thus v(z) cannot approach 0. Finally, if ¢ > 0 then VOT < b and —b+ VP —4e < 0. In this case the solution has terms of the form e% where 8 <0. Therefore y(z) + 0 as x -+ 00 if and only if b > 0 and c > 0. (b) If 8? - 4e > 0, then y = cie™* + oye and the only solution satisfying y(0) = 0, y(1) = 0 is u=0 120 Exercises 4.3, We —4e isy=0. , then y= c1e™* + cate™. Again, the only solution satisfying y(0) = 0, y(1) =0 If - 4e< 0 then ys cre"! cos V4e— Bz + eye? sin Ve Bz, Now y(0) = 0 implies ¢ = 0 and e-Iin Jaca Bn If we are to have @ nontrivial solution, the condition y(1) = 0 implies V@e—BF = nn or 4c~ tt = nn? for n a positive integer v 57. The auxiliary equation is m? + = 0 and we consider three cases Case I When A= 0 the general solution of the differentisl equation is y = 1 +022. The boundary conditions imply 0 = y(0) = cy and 0 = y(r/2) = cxr/2, so that cy = cp = 0 and the problem possesses only the trivial solution, Case II When A < 0 the general solution ofthe differential equation is y = eeY=¥# + exe or alternatively, y = c1cosh Y=Az + epsinhV=Kz. Agein, y(0) = 0 implies q = 0 90 y = cosinh Y=Kz. The second boundary condition implies 0 = y(x/2) = cosink yAn/2 or cp = this case also, the problem possesses ouly the trivial solution Case II When \ > 0 the general solution of the diferential equation is y = oy 0s Vz + epsin Via, In this case also, (0) = 0 yields = 6, 90 that y= easin Ve. The second bosredary contition implies @ = cysin Vin/2, When Vin/2 is an integer multiple of 7, that i, when ‘Vi = 2k for k a nonzero integer, the problem will have nontrivial solutions, Thus for A= 4K? the boundary-value problem will have sontrivial solutions y = ezsin2kx, where k i a nongero integer. On the other hand, when VX isnot an even integer, the boundary-value problem will have only the trivial solution 58. Applying integration by parts twice we have [etiteae = ej0a)- a teye-tferrte—t fe i ales) aeera [ese 2 Collecting the integrals we get fe (1@-49'@) a= tenei-Zerte, 421 59. 60. 61. 62. Exercises 4.3 Ip order for the technique to work we need to have fer (#21-2s"@)) teak [esas Sta) - 3 s"ta) = kya), where k # 0, This is the second-order diferential equation Pa) + P(k - 1) F(z) = 0. If k <1, k #0, the solution of the differential equation is @ pair of exponential functions, in which case the original integrand is an expovential function and does not require integration by parts for its evaluation. Similarly, if k = 1, f"(z) = 0 and f(z) has the form f(z) = az+6. In this case a single application of integration by parts will suffice. Finally, if & > 1, the solution of the differential equation is f(z) =e cosavE= 12 + ensinavE= Te, ‘and we see that the technique will work for linear combinations of cosaz and sin a Using 2 CAS to solve the auxiliary equation m? ~ Gm? + 2m + 1 we find my = ~0.270594, ‘my = 0.658675, and mg = 5.61186. The general solution is, yer DMOSDE 5 gpg DSHBTEE 4 581A Using a CAS to solve the auxiliary equation 6.11m* +-8.59m? + 7.99 +0.778 = 0 we find my = ~0.110241, m = -0.647826 + 0.857552, and ms = ~0.647826 ~ 0.857532i. The general solution is y= eye OIF 4 547828 (0, 05 0,8575322 + cy sin 0.857532) Using a CAS to solve the auxiliary equation 3.15m! ~ §.34m? + 6.53m ~ 2.08 = 0 we find ‘my = 1.74806, mp = 0.501219, my = 0.62342 + 0.588965i, ond my = 0.62942 — 0.588966: The general solution is ye NTE 4 cgeD AMA 4 10H, cos 0.588065 + sit 0.588965) Using a CAS to solve the auxiliary equation m! + 2m? — m-+2=0.we find m = 1/2+ ¥¥4/2, mg = 1/2 ~ Y3i/2, mg = -1/2+ V7i/2, and mg = ~1/2— YTi/2. The general solution is £0 (aca Senin $e) $0 (neues n°) 122 Exercises 4.4 63. From 2m + 3m? — 16m? + 18m —4 = 0 we obtain m= 4, m= J, m= 1, and m= 1, so that y= cre + exe? + get + eae. If (0) = -2, (0) = 6, ¥"(0) = 3, and y"(0) = J, then atata=-2 1 wt +jatate a Wer + Feat es + 2e4 = 3 1 ~6ler + ga tert3a= 5, Esc = OF, cq = - 8, and 64, From m! — 3m} + 3m? —m = 0 we obtain m=0, m= 1, m= 1, and m= 1 so that yet eae + eare! + eq2te®, If y(0) = 0, v0 , and y"(0) = 1 then tm=0, ata=0, at2attanl, etderéeanl, 1/2, and so =2a=-2q=24= yereatene lie Exercises 4.4 —__ 2 and we assume and 1. From m? + 3m +2= 0 we find my = —1 and mp = —2, Then ye= c1e™* +026" yp = A. Substituting into the differential equation we obtain 2A = 6, Then A= 3, yp = yer + ee $3, 2, From 4m?-+9-= 0 we find my = —Ji and ma = Ji, Then ye = cy c0s32-+ opsin fz and we assume ‘yp = A. Substituting into the differential equation we obtain 9A = 15. Then A= §, yp = § and ercos e+ cqsin Se +3 veos get easinge + 3 3. From m? — 10m + 25 = 0 we find my = mz = 5. Then ye = cje™ + cpre™ and we assume up = Az+ B. Substituting into the differential equation we obtain 26A = 30 and ~10A +258 = 3. Then A=§, Ba $, yp=$e+§, and are + epze +8248 yoae® tee +5245 123 From dm? —4m—3 = 0 we find my = § and m: Exercises 4.4 |. From m? + m—6 = 0 we find my = 3 and my =2. Then ye = cre~* + oye and we assume up = Az+B, Substituting into the differential equation we obtain ~6A = 2 and A-6B = 0. Then Aa~$ B= —ki ys it~ hand Loa sce tgs te ys toe —3e- From fm? + m+ 1 = 0.we find my = my = 0. Thon ye = ce + cute and we assume Up = Ar? + Br +O. Substituting into the differential equation we obtain A = 1, 24+ B = 2, and JA+B-+C=0, Then A=1, B= —4,C= 5, yp= 2? 4244, and oye ene + 2? z arty From m? — 8m + 20 = 0 we find m, = 2+4i and my = 2~4i. Then ye = €** (cx cos4z +2 sin dz) and we assume yp = Ax? + Br +C+ (Dz + Be®. Substituting into the differential equation we obtain 2A-8B +200 =0 ~6D +138 =0 -16A +208 =0 Lap = ~26, 204 = 100, Then A “Hayy set tac + Wo (22 — Be and y un B aa + egsindz) + 52? aa + (-2-T)e. (eoste texsinde) +50 +404 Es (22-2) From m? +3 = 0 we find m, = V3i and mz = —V3i. Then ye = cicosV3x+ qsinv3r and we assume yp = (Az? + Br + C)e™, Substituting into the differential equation we obtain 2A + 6B + 12C = 0, 12A+12B = 0, and 12A = ~48, Then A = -4, B = 4, C = -4, ap = (~t2? +42 — fe and y= c1e0s VB2+ opsin VO2+ (a2? ar ~ 3) 3 =. Then ye = exe%/?-+ep6°#/? and we assume tp = Acos2e + Bsin2x, Substituting into the diferential equation we obtain —19— 8B = 1 and 84-198 <0. Then A = 3k, B= —Zfe, yp = 7 cos 2x ~ gfe sin2z, and 19 8 rst 1S cosae ~ sine. 425, S 425 : oer y + one 124 10. nn. 12. 13. 14. ). From m? — m = 0 we find my = 1 and my Exercises 4.4 0. Then ye = cie + e2 and we assume yp = Az. Substituting into the differential equation we obtain —A = -3. Then 4 = 3, yp = ax and veel tert &. From m?+ 2m = 0 we find m = -2 and mz = 0. Then ve = cre" + ep and we assume tp = Ax? + Br + Cxe-™. Substituting into the differential equation we obtain 2A + 2B = 5, 4A =2, and -20=-1. Then A=}, B=2,C=4, yp = Ja? +2r+ hae", and 1 1 em 3t 4204 dee yacie™ + ont 5284 22 + 5 From m?— m+} = 0 we find my = m2 = 4. Then ye = cre + epze*/? and we assume p= A+ Batet/®. Substituting into the differential equation we obtain 1A = 3 and 2B = 1. Then Aq12, B=}, yp = 124 hater, and yeae? senet sins teen From m? — 16 = 0 we find m, = 4 and mz = —4. Then ye = exe! + cxe™ p= Are, Substituting ito the diferental equation we obtain 8A = 2. Then A= 4, y, and and we assume det 1 as ae ae + 2 y ae E47 From m?+4= 0 we find m: = 2i and my = 2%, Then ye = ¢ cos2x + crsin2z and we assume yp = Axcos2x + Brsin2z. Substicuting into the diferensial equation we obtain 4B = 0 and 4A =3, Then A=—9, B=0, yp = ~Jxcos2z, and 3 Y= ¢ cos 2r + cpsin 22 ~ jrooste. From m? +4 = 0 we find m, = 24 and mp = —2i. Then ye = ¢) cos 2z + cz sin 2x and we assume p= (Az!+Bz?-+Cz)cos2s-+(Ds'+ 22+ Fa) sin 2x. Substituting into the differential equation we obtain 2R+4P=0 6A+8E=0 1D=0 ~4C42B=-3 -8B+6D=0 -2A=1. 125 15. 16. ar. 18. 19. 20. Exercises 4.4 Thon A= =, B= 0,0 =D and B= thy P= 0, ip = (dye? + er) cose + fe2%sin2n, a5 =e Ay p= erate tysinte + (Lat + Be) covae+ Latnae From m?+1=0 we find m =i and my =i, Then ye cy cos +e)sinz and we assume yp (Az? + Br)cosz + (Cz? + Dz)sinz. Substituting into the differential equation we obtain 4C = 0, 2A+2D = 0, -4A = 2, and -2B8+2C = 0, Then A= -},B=0,C =0,D=}.y= —Joteose + Jzsing, and = creose + cpsing ~ fr%cosz + tesinz y= cicose +ensinz ~ f ; From m? — 5m = 0 we find m; = 5 and my = 0. Then ye = cie™ + cp and we assume yp = Ax‘+Br'+Cz?+Dz, Substituting into the differential equation we obtain -20A = 2, 12A-15B = =4, 6B ~ 10C = 1, and 20-5 = 6. Then A=—, 8 = i, C= 3h, D = -, apa —iprt + Hot + Bho? — Ble, and wey ty — hats Mas, 32 _ 7 ical aes) +35 * 250" 625 = Owe find my = 142% and m2 = From m?-2m-+: = 2%, Then ye = e*(cy 008 22 +e sin 22) and we assuine Yp = Are* cos 2x + Bze* sin 2x. Substituting into the differential equation we obtain 4B = 1 and 4A =0. Then A= 0, B= 4, yp = 420"sin22, and ynelaste+qsnts) + Lee inte Fromm? — 2m +2=0 we find my = 1 +4 and mz =1—i. Then ye = e*(e1cos + epsin2) and we assume yp = Ae" cos-r + Be*sinz. Substituting into whe differential equation we obtain A4+2B=1 ond -24+B=-3 Then A=§,B 4, up = Je cos ~ fe sinx and, = 7 ae, 2s, y= eo cose + opsinz) + Ze™ cosz ~ Ze sins From m?+ 2m +1 = 0 we find m1 = mz = ~1. Then ye = cie* + care“ and we assume up = Acosz + Bsinz + Ceos2z + Dsin2z. Substituting into the differential equation we obtain 2B 1, -80-+4D = 3, and -4C~3D =0. Then A= -},B -$,D=8. gp = —}oosz — Bcosde + HBsin 22, and 1 a ~ Foosz — Fonsae + Y sind From m? + 2m —24 = 0 we find m = -6 and mz = 4. Then ye = c1e™® + ope! and we assume yp = A+ (B2? + Cz}e!. Substituting into the diferentil equation we obtain ~24A = 16, 126 21 22. 23. 24, 25. 26. Exercises 4.4 2B+10C and =2,and 208 = =1. Then A= -9, B= —2,C=—3B up yeoe® rad—2- (Lite Ba) From m? — 6m? = 0 we find m; = mg = 0 and ms =6. Thon ye = cr +cat + eye and we assume up = Az? + Boosz +Csinz, Substituting into the differential equation we obtain 124 = 3, 6B-C=-1, and B+6C = 0. Then A=-], B= -$,C= 4h, wp =—}2?- fases desing, and ao i jal- poset ane. From m3—2m?—4m+8 = 0 we find m; = mz = 2 and m3 = —2, Then ye = c1e™ + opze™* +-cye™ and we assume yp = (Az*+ Bz?)e™*, Substituting into the differential equation we obtain 244 and 64+ 8B (je? fe) 2, ana 32) & Gi From m! ~ 3m? + am —1 = 0 we find my = mg = mg = 1. Then ye = cie® + caze® + exe and wwe assume yp = Ar + B+ Cz%et, Substituting into the differential equation we obtain A = 1, 3A—B=0, and 6C = -4. Then A=-1, B= ~3,C =~}, yp=—2—3- $a%e*, and yeatar tee — 2 get tonne + atet —2~3— Fae From m*—m?—dm-+4= 0 we find m, = 1, mp = 2, and mg = =2. Then ye = ae? +e +e3e7% and we assume yp = A+Bze*+Cze™, Substituting into the differential equation we obtain 4 = 5, 3B = =1, and 4C=1. Then A= §, B=}, C=], yp = $+ fact + de, and = eyet ope + ese + 5 + Bet + Le, meet + exe + ese 4 E+ Saet + F From m! + 2m? + 1= 0 we find my = my =i and mz = my = =i. Then ye= 1 e082 + epsins + eqrcosz + cersinz and we assume yp = Az? + Bz+C. Substituting nto the differential equation wwe obtain A= 1, B=~2,and44+C=1. Then Awl, B=—2,C=-3, y=s'— 22-3, and 1cost + sine + esr cos + oyrsinz +2? — 2x —3. v From m!—m? = 0 we find m; =p = 0, mg = 1, and my =—1. Then y= 1 beret eget texe"* and we assume yp = Az? + Ba? + (Ca? + Daje"*. Substituting into the differential equation we obtain —6A = 4, -2B = 0, 100 - 2D = 0, and -4C = 2. Then A= -3, B= 0,C = -}, D= Fy up=—§a?~ (fa? + fx)e™* and yee tort ove? + cue La 5.) . a? + 32) e 2 127 ar. 28, 29. 30. 31. 32, 33. Exercises 4.4 We have y, = er c0s22-+ cp sin 2 and we assume tp = A. Substituting into the diferential equation ve find A= J. Thus y= c10082z + eysin2z~§. From the initial conditions we obtain r= 0 and c= V3, 20 y = VB sind ~ § We have ye = cie-® +ere™!® and we assume yp = Az?+ Bx +C. Substituting into the differential equation we find A= ~7, B= 19, and C = -91. Thus y = c1e® + epe*/? — 72? ~ 192 ~ 37. From che initia! conditions we obtain e, 1 5 =} and co = 88, s0 te Wee rt ioe We have yo = cre™*!9 +0 and we assume yp = Ar? Br. Substivuting into the differential equation we find A= ~3 and B = 30. Thus y = cyeW*/* + ep ~ 3x" + 302. From the initial conditions we ‘obtain cy = 200 and cp = ~200, so y= 200-85 ~ 200 — 95? + Soz, We have yo = cxe"® + cpze™** and we assume yy = (Az + Bae Substituting into the dliferential equation we find A= 4 and B= $. Thus y= ce-™ + onze™™ + (fx5 + fa? From the initial conditions we obtain cy = 2 and e = 9, so yore sone + (late de) ote Wo have y= e°*(c1cos2 + ersin2!) and we assume yp = Ae. Substituting into the differential equation we find A = 5. Thus y= e-%(cy cos + opsin 2) + e~**, From the initial conditions we bain cy = ~10 and cy =9, 50 y=er™ (cose + 9sinz + Te). We have ye = c1coshz+czsinh and we assume yp = Az cosh2 + Brsinh 2, Substituting into the ifferential equation we find A= 0 and B= 4. Thus a y= croub2 + ersinhs + 52sinkz, From the initial conditions we obtain ¢y = 2 and ep = 12, so ye teas +tasahs + Lesh We have x, = ¢1 cosut + esinut and we assume zp = Atcoswt + Btsinut, Substituting into the differential equation we find A = —Fy/2u and B = 0, Thus x = oy cosut-rensinwt—(Fo/2u}tcosut. From the initial conditions we obtain so and on = Fy/2u’ B= (Fo/2Psinut ~ (Fo/2u}teosut 128 34. 36. 36, 37, Exercises 4.4 We have 2¢ = 1 coswt-+ep sinut and we assume zp = Acosyt+B sin ot, where 7 # w. Substituting, into the differential equation we find A = Fo/(w? ~ 72) and B = 0. Thus Fo 1 cosut + epsinut + cost A From the initial conditions we obtain cx = Fo/(u? ~ 7) and 2 = 0, 50 our eA i cost yh cont iz + Bate + Ce. Substituting into the }. Thus We have ve = ct + ene? + egze and we assume Yp differential equation we find A= 2, B = —12, and C i. per eae + ane +20 12 Le From the initial conditions we obtain ¢y = 11, ¢2 = ~11, and ca pati -tie +800 420 12st Be We have ve = c1e-2 + e*(cpecevz + cssin VE) and we assume tp = Az +B + Cre. Substituting into the differential equation we find A= 3, B= —f, and C= 2. Thus ya ee™ +et(epcos Var esin V32) + fe 3 + Fee From the intial conditions we obtain c= ~#, “Hf anda = Bv3, 90 2 et (2 u 2,5 4 Duet y= Bert so (Boos Van + 2 asin V2) + 2 2 + Fe ‘We have ye = e1 cosz-+eysin.z and we assume yp = A?-+ Br+C. Substituting into the differential equation we find A= 1, B = 0, and C= —1. Thus y= cy cost + cpsing +2? —1, From y(0) = and y(1) =0 we obtain ce (cos 1)e1 +sin(1)e = 0. Solving this eystem we find «1 = 6 end ¢ = ~Gcot 1. The solution of the boundary-value problem y= Goss ~ 6{cot1)sinz + 2 - We have ye = e*(c1 cos + e2sinz) and we assume yp = Ar + B. Substituting into the differential equation we find A= 1 and B = 0. Thus y = e*(cy cosz-+ersin.z)+z. From y(0) = Oand y() =» wwe obtain aso nae 129 39. 40, Exercises 4.4 Solving this system we find cy = O and pis any real muruber. The solution of the boundary-value problem is yroetsing +2 We have ve = ¢1 00s 2r + cpsin 2r and we assume yp = Acosz + Bsinz on (0,/2]. Substituting Into the differential equation we find A = 0 and B=}. Thus y=c1 osx +ensin2r + $sinz on [0,/2]. On (r/2,00) we have y = 1 cos2z + cysin2z, From y(0} = 1 and y/(0) = 2 we obtain 1 free? Solving this system we find c1 = 1 and cx = §. Thus y = cos2e+ $sin2e + $sinz on [0,7/2) Now continuity of y at 2 = /2 implies ci + 2sing + tin’ = ccost + esine conn + Bains + Fsin = eos +c or 1+} = ~cs. Hence y =}. Continuity of y’ at 2 = /2 implies fe ~2sinz + Secosn-+ Seon or ~$ = 2c, Then cy = § and the solution ofthe initial-value problem is ees Oszen/2 Jeos2e + fsin22, a> 2eqsin # + 2e4 cos via) We have ve = e*(c1cos3z + crsin3x) and we assume yp = A on [0,7]. Substituting into the iflerential equation we find A =2, Thus, y = e¥(c1 cos 32 + ¢2sin3z) +2 on [0,7]. On (x,00) we have y = e*(cy008 32 + c45in32). From y(0) =O and y/(Q} = 0 we obtain an-2 a+3e 2 and c = }. Thus y = e*(—2cos3z + 9 sin 32) +2 on (0, implies Solving this system, we find ey Now, continuity of y at e"(-2e0s8r + 2sine) +2 = o¥(eycos8x + 24 sin3e) of 24+ 267 = ~cye™ or cy = —2e-*(1 +e"), continuity of y’ at = implies 20 Spe sind = eT [(ey + Seq) cas Se + (—3ey + c4) sin Sx] or ~ege" — Seqe = 0. Sinoe cy = ~26°*(1 4+ €7) we have cy = Je¥(1 +e), The solution of the initial-value problem is ta = fC Pomae Finds), oczer (1 +eteP*(-2eende+$sindx), 2 > x 130 41 42. 43. Exercises 4.4 (a) From yp = Ae! we find yf, = Ake and yff = Ale, Substituting into the differential ‘equation We get aAKeM + bake! + ode = (ak + bk+ Act = eM, 0 (ak? + BR -+5)A = 1. Since kis not a root of am? + bm +e 0, A = I/(ak? + 0k) (b) From yp = Aze# we find yf = Akzel* + Ae and yf = AKzel* 4 2Aket, Subetituting into the differential equation we get aAlze! + 2atkel + bAkze + bac + cAze™ (ak? + bk + c)Ane™* + (2ak + b) Act = ()Aze + (2ak-+ 5)Acl = (Pak +0)Ael where ak? + bk +c = 0 because k is a rot of the auxiliary equation, Now, the roots of the auriliary equation are ~b/2a: VPF= dae, and since kis a root of multiplicity one, k# —b/2a and 2ak +6 #0, Thus (20k-+B)A=1 and A=1/(2ak +0) (c) If is a root of multiplicity two, then, a8 we saw in part (b), k= —b/2a and 2ak +b = 0, From yp = Are we find yf, = Akzte + 2AxeK* and yf = Ase + 4Akzel = DAU. Substituting into the diferentil equation, we get aAka7e + daAkzel +20Ae + bAkzte + 2bAzel + cAstelt = (ah? + bk-+ c)Aze + 2(2ak + b)Aret + Dae fe = (Arte + 2(0)Aze!* + 2aAel* = 2atel Since the differential equation is second-order, a # 0 and A = 1/(2a). Using the double angle formula for the cosine, we have nz — 2sin* x, sinro0s 22 = sin (cos? x ~ sin? z) = sin (1 - 2sin®) Since sin is a solution of the related homogencous differential equation we look for a particalar solution ofthe form yp = Azsinz + Bxcosz + Csin'z. Substituting into the differential equation ‘we obtain 2a eos + (66 ~ 26) sin ~ Sesin® x = sinz — 2sin® x Equating coefcients we find a= 0, ¢= ‘Thus, « particular solution is 1 i tee jrcose+ Hsin? (2) se) = elsint. We see that yp — 00 as t+ 00 and yp 0 as t+ 00. (b) (0) = ef, We see that yp — 00 a5 t = 90 and yp — 00 a5 t > —o0. 131 Exercises 4.4 (6) §(€) =sin2t. We seo that yp is sinusoidal (4) Jt) = 1. We see that up is constant and simply translates vertically 44, The complementary function is ye = e2*(c) cos 2r-+ o25in 2x). We assume a particular solutian of the form yp = (Az? + Ba? + Cxje% cos2r + (De? + Ez? + F)e™ sin2z. Substituting into the diferential equation and using a CAS to simplify yields [12D2? + (64+ 8B)2 + (2B + aF)je™ cost + [-12A2? + (-8B + 6D)z + (40 + 2B )]e* sin 22 = (22? — 32)e™ cos 22 + (102”~ 2 ~ t)e™ sin2z, ‘This gives the system of equations 2D=2, 6A+8B=~% 2B+4F =o, -WA=10, -88+6D=-1, ~40 +28 1, from which we find A= -£, 8 =},C=$,D=}, B=}, anh P=}. Ths, « particular sion fee ot of Ags, La 1) 2 gin w= (4 feits? ge Je cose + (Fa + Ze 3) sin 22, 45. The complementary function is u eos +epsing +eyzcos + cxzsin, We essume a particular solution of the form yp = Az*cosz+Br'sinz. Substituting into the differential equation and using a CAS to simplify yields (Ba +240) ccs2-+ be sins = 2e0s2~ 3rsinz. This implies ~8a + 246 = 2 and —24 = —3. Thus = ha=f, and yp = fotos + Jotsing Exercises 4.5 (9D? — d)y = (3D = 2)(3D + 2)y = (D8 5)y=(D~ v8\D+v8\y (DP ~4D - 12)y = (D~6)(D+2y 22-8 (2D? — 3D = 2)y = (2D + 1)(D 2) (D3 + 10D? + 28D)y = D(D +5) (D3 + 4D)y = D{D? + aby =F cose (D3 + 2D? ~ 13D + 10)y = (D - 1)(D - 2)(D + 5)y = ze“ (D° +402 + 3D)y = DID + 1)(D + 3)y = 2 cose ~ Be 432 10. a 12, 13. a4. 15. 17. 19. a. 2, 23. 2, 28. 26. ar, 28, 29. 30. 31. 32. 33. 34. 36. . (D4 + 8D)y = DID +2)(D?— 2D +4)y Exercises 4.5 (D* — 8D? + 16)y = (D- 2)(D + 2)?y = (z* — 2z)e* Diy = DA(1023 — 22) = D*(302? - 2) = D602) = D(60) = 0 (2D - 1)y = (2D ~ 1)4e*/? = 8e#/? — get/? = 4e#/? — 4e7/? = 0 (D~2)(D+5)(e*+3e78) = (D~2)(262*~15e-* +50? +1528") = (D-2)Te2* = Me™ —14e™™ (D? + 64)(2cos 8x — 5sin8x) = D(—16sin 8¢ — 40 cos 82) + 64(2co88z - Ssin 8x) =128cos x + 920sin 8x + 1280s 82 ~ 320sin 8x = 0 DS because of 2? 16, DP because of 24 D(D ~2) because of 1 and e® 18, DD — 6) because of x and ze DP +4 because of cos 2 20, D(D? +1) because of 1 and sinz D9(D? + 16) because of 2? and sind D°(D? + 1)(D? + 25) because of x, sin, and cos5z (D+1)(0 ~ 1) because of e°* and x%e* D(D~1)(D 2) because of 1, , and e* D(D? ~ 2D +8) because of 1 and e* cos2z (D?+2D+2)(D? ~ 4D +5) because of e-* sin and e% cos 1a, 272,24 D?+4D=D(D+4), en ek DP-9D-3%6 = (D-12(D+3); el, e- cos ¥5z, sin V5r DP~6D +10 = D?-28)D+ (9 +19); econz, sine DP — 10D? + 25D = D(D~5)%; 1, e, ze 12,€,e% Applying D to the differential equation we obtain D(D* - 9} Then Sac te 36. a7. 38. 39. Exercises 4.5 and yp = A. Substituting yp into the differentia] equation yields —9A = 54 or A= ~6. The general solution is ysaet tae 6, Appiying D to the differential equation we ebtain DQ 7D +5)y=0. Thea SI + cae? 03 and yp = A. Substituting yp into the differential equation yields 54 = 29 or A = —20/5. The ‘general solution is 29 y = cre! + ope — i Applying D to the differential equation we obtain D(D* + D)y = D(D + ly =0. ‘Thea yeatoe™ tear und Yp = Az. Substituting yp into the differential equation yields A = 3. The general solution is yer toe + 3x, Applying D to the differential equation we obtain D(D* +202 + Diy = 04D +1)*y =. Then rota tae tee and yy = Az. Substituting yp into the diferential equation yields A =10, ‘Tae general solution is ysert oe + egne™? + 10s. Applying D? to the differential equation we obtain DYD? + 4D +4)y = DD +2)" Thea cue beget op tear 134 40. 41. Exercises 4.5 and yp = Az + B. Substituting yp into the diferential equation yields 442 + (44 +4B) = 2246. Equating coefficients gives dane 44s 6B x6. ‘Then A= 1/2, B= 1, and the general solution is 1 ya cre + epee + 5241. Applying D? to the differential equation we obtain D'(D + 8Djy= D(D+3)y Then veatge® tar? +c and yp = Az? + Br, Substituting yp into the differential equation yields 64z + (24 +38) = Equating coefficients gives 6A=4 2A+3B Then A =2/3, 19/9, and the general solution is yratacts dt Applying D® to the differential equation we obtain D9(DS + Dy = D(D + 1)y = 0. Then ext one + get + ect! + egz? +627 ond ap = Ast + Br? + Ca?. Substituting yp into the diferential equation yields 12Az? + (244 4 6B)x + (6B +20) = 8c? Equating coefficients gives 2A=8 ALOR =O 6B +20 135 42. 43, Exercises 4.5 ‘Then A= 2/3, B= -8/3, C= 8, and ve general solution is Qe pret exe tose + 52! Applying D* to the differential equation we obtain Di? 20 + t)y = DD 4y=0 Then y= cre tense” +032? + eye” + e52 +6 and yp = At? + Ba? +Cz+ D. Substituting yp into the differential equation yields Aa? +(B~64)x? + (6A ~4B + C)z + (2B -2C + D) 3 ae Equating coefficients gives, As B-6A=0 6A-4B+0 28-210 +D=0. ‘Then A=1, B= 6, C=22, D=82, and the general solution is y= ce +cgre® +2) +62? + 222 + 32. Applying D~4 to the diferestial equation we obtain, (D-4(D8- D-2y= D ~4)*(D + 3)y =0. Then y= cre + ce + ogze % and yp = Aze*, Substituting yp into the differential equation yields 7Ae* = e. Equating coefficients gives A = 1/7, The general solution is og = eye +00 + dre! ya eel toe + 7: - Applying D —6 to the differential equation we obtain (D-6)(D? +20 +2) ‘Then Foy cost + cpsinz) + ce 136 45. 46. 47, Exercises 4.5 and yp = Ae, Substituting yp into the differential equation yields 50Ae%* = Se. Equating coefficients gives A = 1/20. The general solution is *(eycose + casinz) + Def. Y= ee cons + ersinz) +7 Applying D(D ~ 3) to the diferential equation we obtain D(D = 1)(D? - 2D -3y = DID 1)(D + 1)(D-3y = 0. Then and yp = Ae® + B. Substivuting yp into the differential equation yields ~4Ae* ~ 5B = de® ~ 9. Baguating coeficients gives A = ~1 and B=. The general solution is ae +e 7 +8. Applying 7°(D + 2) to the differential equation we obtain DUD + 2)(D? + 6D + 8yy = DD +2)°(D + 4y=0. Then 10 + ce + ogae’ artes a and yp = Aze“™* + Bx +C, Substituting yp into the differential equation yields 2Aew™ + 8B e+ (6B +8C) = 3e* +27, Bquating coefficients gives 2A=3 sB=2 6B +8C=0. ‘Then A= 3/2, B= 1/4, C= -3/16 , and the general solution is Applying D? +1 to the differential equation we obtain (D? +1)(D? + 25)y = 0. Then ym crcosSs tepsinSs + epcosz + cysing 137 Exercises 4.5 and yp = Acos + Bsin. Substituting vp into the differential equation yields 2A cos + 24Bsinz = sin, Equating coefficients gives A = 0 and B = 1/4. The general solution is 1 cx cosSr + ensinSz + 7 sinz. 48. Applying D(D? + 1) to the difexeatial equation we obtain Did? +10? + dy =9. Then 10828 + op sindz + c9 0082 + ey sin + 05 and yp = Acosr + Binz +C. Substituting y, into the differential equation yields 3Acose +3Bsinz +4C = doosz + 3sinz—8. Equating coefficients gives A = 4/3, B = 1, and C = -2, The general solution is 4 y= cycos2e +opsinds + 5 cos2 +sinz ~2 49. Applying (D ~ 4)? to the differential equation we obtain (D~4)°(0? + 6D +9)y=(D- 4% +3P'y =0. ‘Then, le se ym cre + ogre + cyre® + oye and yp = Axe + Be", Substituting yp into the differential equation vields a9 Aze! + (144 + 49B)e = ~2e Equating coefficients gives 494 A+ 498 ‘Then A= 1/49, B = 2/343, and the general solution is se = cxe™™ + eyae — Lote a oo get 50. Applying D*(D ~ +)? to the diferential equation we obtain D'D ~1)(D? + 3D ~ W)y= XD ~ )*(D-2\(D + Sly =. 138 Exercises 4.5 ‘Then vais + xe" + cnet + c4e” + o50 +06 and yp = Aze* + Be® + Cxr+ D. Substituting yp into the differential equation yields ~6Aze* + (5A — 6B)e* ~ 10Cx + (3C ~ 10D) = 207 +2. Equating coefficients gives ~6An1 5A-6Be0 -100 = 1 iC - 0D =0. Then A = ~1/6, B = ~5/36, C= -1/10, D = ~3/100, and the general solution is oo es 6 36° ~ 10" 100 51. Applying D(D —1)8 to the differential equation we obtain D(D = 1)? = 1) yao + ene" D(D-1)(D+ Ay Then y= giet + cre" + sate + cane? + esse? +05 and up = Azte® + Bz®et + Ce + D. Substituting tp into the differential equation yields GAnte® + (6A + 4B)zc" + (2B 420) ~ Dawe +5. Equating coefficients gives 6A=1 6A+4B=0 2B+20=0 -D=5. Then A= 1/6, B= ~1/4, C= 1/4, D = ~5, and the general solution is tenets ite tates bee yee tee? + Latet — Late + inet =. 52. Applying (D + 1) to the differential equation we obtain (D+1)(D? +204 Iy = (D+1)'y=0. 139 88. 54. Exercises 4.5 ‘Thea ym qe teases teacte® + cyrhe? + oge%e™ and yp = Azte-* + Ba¥e-* + Caer, Substituting yp into the differential equation yields 12Az*e-* + 6Bre"* +200 e Equating cosficients gives A = fh, B =O, and C=0, The general solution is - eles yooe* tere + Sate iy Applying D® ~ 2D +2 to the diflerential equation we obtain (DP - 2D +2(D?-2D+5y=0. Then y= eF(ecos2e + epsin2z) + eF(cycos2 + eysinz) and yp = Ae® cos + Be®sin-z. Substituting yp into the differential equation yields BAe* cost + 3Be* sine =e sine Equating coefficients gives A= 0 and B = 1/3. The general soluion is *(c1 cos 22 + e2sin2z) + Se? sins. y= e(e1c0s2e + esin2z) +} Applying D? — 2D + 10 to the differential equation we obtain (Dt-20 +10) (D?+ D+ 2) y= (0? - 20+10)(D+})"y Then ne? + cyze tl + cge* cos 3a + cye* sin Sz and yp = Ae* cosz + Be? sin3z, Substituting yp imo the differential equation yields —€ cos3z +e sin (9B ~ 27A/A)et cos 3x — (9A + 27B/4)e* sin3 Equating coefficients gives +9B=-1 a 9-2 B21 oa 2 Then A= —4/225, 28/225, and the general solution is eel? ogre? as 28 ppt conte ~ See sinSe. 140 Exercises 4.5 55. Applying D? +26 to the differential equation we obtain (DP + 25)(D? +25) = (D? +25)? = 0. Then U crease + opsinSe + cyrcosSx + eur cosiz and yp = Azcosiz + BrsinSz. Substituting yp into the differential equation yields 10B os bz — 10Asin 52 = 20sinSe. 2 and B = 0. The general solution is Equating coefficients gives A = y= c)cos6z + cysin Sz ~ 22 cos5z, 56. Applying D? + 1 to the differential equation we obtain (D?+1(D? +1) = (DP +1? Then Y= gueosz + esing + egress + cxncosz and yp = Arcosz + Brsin. Substituting yp into the diferential equation yields 2Beosz ~2Asinz = deoss—sinz. Equating coefficients gives A = 1/2 and B = 2. The general solution is i = c1cose + opsing + Azqwsz ~ 2rsinz. y= ccoz+qsing + 5208: 57. Applying (1D? + 1)? to the differential equation we obtain (p+ 10? + D+1)=9. Then yee? cose reins] eqeese + egsin + ogzcosc + cgrsin a a . . . . . ... (B+C+2D)cosr+ Drcosz +(—A —2C + D)sinz — Czsinz = zsinz. Equating coeflicients gives B+C+2D=0 D=0 ~A-20+D=0 -C=1 141 58. 59. 60. Exercises 4.5 ‘Then A= 2, B= 1,C = =I, and D = 0, and the general solution is 2 = }(1 +c0s2z) and applying D(D? + 4} to the differential equation we obtain D(D¥ +4\(D? +4) = D(D® + 4)? = 0. y ‘Wirting cos? ‘Thea y= crcosQr bepsin2e + cyrcos 2x + cur sin 2x + o5 and yp = Arcos 2z + Bzsin2x +C. Substituting yp into the differential equation yields pole ~sAsin2e + 4B cos2e +40 = 5 + 5 cos2e. Equating coefficients gives A= 0, B= 1/8, and C = 1/3. The general solution is 0 1 ys evcosde + oysinde + Sain + 2 Applying D® to the differential equation we obtain D3(D* +80") = DD +8) = ‘Thea exbege + oye +042" + cg2* + ops y and Yp = Az?-+ Bz! +-Cr', Substituting yp into the differentia equation yields 16A + 6B + (48B + 240)2 +9602? = 2-492 ~ 62" Equating ccefficiants gives WA+6B =2 48B +240 =9 960 = ~6. ‘Then A= 11/256, B = 7/32, and C = —1/16, and the general solution is, sateetoe + dats Tet Lot ee aes Applying D(D ~ 1}%(D +1) to the differential equation we obtain D(D ~~ 17(D +1)(D8 — D? + D~ 1) = D(D- 1D + 1)(D? +1) = 0. Then ret topcosz tossing +4 + ce" * + ogre + crz%et 142 Exercises 4.5 and yp = A+ Be-* + Cre* + Dr*e*. Substituting yp into the differential equation yields tT HT. 4Dze® + (20 + 4D)e* — 4Be"* A Equating coefficients gives wer westD=0 -B=-1 21/0 12 and D=1/4 nthe evr ition ad tamestasns 7+ tee taet ste 61. Applying D®(D—1) to the differential equation we obtain D*'(D-1)(D* = 8D? + 3D - 1) = D>(.D ~1)¢ ‘Then ym cue t epee s egaet + 04 + cor t caste and yp = A+ Bz + Caer, Substituting yp into the differential equation yields (-A+3B)- Breed = 16-242 Equsting coefficients gives -A+3B=16 -B=-1 6c=1 Then -15, B= |, and C= 1/6, and the general solution is yead tar teste Bee ite, 62. Writing (e+ e-#)? = 246% 46% and applying D(D ~ 2)(D+2) to the diferentil equation we obtain D(D ~ 2)(D +2)(208 - 3D? - 3+ 2) = DID- XD +2(.D +1)(2D- 1}=0. Then ym ret teat + cael? 4 c4 + ogre + eg 143 Exercises 4.5 and yp = A+ Bre™ + Ce-%, Substituting yp into the differential equation yields s peg ete 2A + 9B — 2000 = 240 + 0% Equating coefficients gives A = 1, B= 1/9, and C= -1/20. The general solution is sp ya ae bone + ore Lae +14 Gre — . Applying 9(D — 1) to the differential equation we obtain D(D ~ 10" ~ 208 + D2) Then vm atoe t eset + eyset + 052? + con7e? ‘Ax? + Bz%e¥, Substituting yp into the differential equation yields 2A + 2Be* = 1 +e*. Eaquating coofcente gives A = 1/2 ond B = 1/2. The general soltion is ans tert toset + bat + 5a Applying D'(D — 2) to the differential equation we obtain DD ~2)(D4 — 4D?) = DD ~ 2)*( +2) =0. Then . 2 4 eye 3 pera! + ogne™ Umer tege + ege™ + eye ™ +052" + og2! + cra! + cone? a and yp = Az? + Ba" + Cx! + Dre. Substituting up into the differential equation yields (-8A +240) ~ 24B2 ~ 48C2? + 16De% = 52? — e™. Bquating coefficients gives “844240 =0 -UB=0 880 25 16D = -1, Then A= -5/16, B = 0, C= -5/48, and D = ~1/16, and the general solution is vrei tons +ese + oye 144 65. 66. 67. Exercises 4.5 ‘The complementary function is ye = ce + e®*. Using D to annihilate 16 we find yp = A. Substituting yp into the differential equation we obtain ~644 = 26, Thus A= —1/4 and aie tone uf = Bee ~ Sexe* ‘The initial conditions imply ate Ber Bey Thus ¢ = 02 | ate a ‘The complementary function is ye = ¢1 + qe-, Using D? to annihilate 2 we find yp = Az-+ Ba? Substituting yp nto the diferental equation we obtain (4 ++2B) + 22s ~ =, Thus A= —1 and B=1/2,and v yeatee yl = -qe* - 142, ‘The initial conditions izmply Thus ¢) = 2 and op yore ‘The complementary function is ye = c1 +e2e*. Using D* to annihilate 2~2 we find yp = Az+Ba?, Substituting yp into the diferential equation we obtain (—54-+2B)-10Bz = -2+-2. Thus A = 9/25 and B = -1/10, and ‘The initial conditions imply atest a Sea = Hp 68. 69. 70. Exercises 4.5 ‘Thus oy = ~41/125 and cp ‘The complementary function is ye = cieF + e~*, Using D ~ 2 to annihilate 106" we find up = Act, Substituting yp into the differential equation we obtain 8Ae™ = 10¢*. Thus A = 5/4 and 5 oc ,, 5,22 +a ya ae? tone 5 # — Gye Bt 4 528 ie — Gone + 3 ‘The initial conditions imply 7 ata=-t 3 4-6-5 ‘Thos 61 = 8/7 and 2 = 5/28, and fy es yaaa e Bee + 2 ‘The complementary function is ye = cycosz + casing. Using (D* + 1)(D? +4) to annihilate 8cos2r ~ 4sinz we find yp = Azcost + Brsinz + Ccos2z + Dsin2z. Substituting yp into the differential equation we obtain 2B cos 2 ~3C eos 22 — 2A sin x ~ 3D sin2: A=2,B=0,C = ~8/3, and D=0, and Boes 22 4sin x. Thus ee 6 ysing + epcosz + 2eosz — 2zsing + sin 22. ‘The initial conditions imply ‘Thus ¢ = ~ and oy = ~11/3, and n 8 = -meggs — sing + 2x cos.2 ~ 5 c0s2x, y En 5 ost - 5 ‘The complementary function is ye = ex + oe + eptet. Using D(D ~ 1)? to annihilate E45 welfind yp = Ar + Bale! + C%e*, Substituting yp into the differential equation 146 n. 72. Exercises 4.5 we obtain A+ (2B + 6C)e* + 6Cze* = ze" +5. Thus A= 5, B=—1/2, and € = 1/6, and =a tect + eye a meget + elec + et) 4 5-20 4 PE Pea salee sn) -e nate dts doe ‘The initial conditions imply ataa2 atat a+ 2c Thus o = 8 ep = ~6, and ¢ = 3, and Oe te te 8 6eF + Bue + Bx — 5 2%e + ge ‘The complementary function is ye = €*(cy cos2x + epsin 22). Using Dé to annihilate 2° we find yp = A+ Bx+ Ca +Dz®, Substituting yp into the differential equation we obtain (8A —4B +20) + (8B ~ 80 + 6D)z + (8C~12D)2? + 8D2° = 2°, Thus A= 0, B= 3/32, C= 3/16, and D = 1/8, and - st y= &(q,c00224 csinde) + Bas Sat 4 bed v © fe\(2cos2e ~ 2sin 22) + ex(20 in2z))+ op bet 302 feu(Qcos2e ~ 2sin2z) + co(2eos2x+ 2sin2a)] + 3+ E24 F ‘The initial conditions imply 3 do tat a4 Thus ¢: 2 = ~3/64, and ed 3, 52,18 = &(2eosa2— 2 sina) + S24 Sota be 7 ¢ 64 V+ a +67 +8 ‘The complementary function is yo = 1 + cpr +eg2? + cqe®. Using D®(D — 1) to annihilate E+E we find yp = Ax! + Bat + Cze". Substituting yp into the diferential equation we obtain 147 Exercises 4.5 (64+ 2B) ~24B2 + Ce = ater. Thus A= 1/6, B= 1/24, and @ =1, and 1 Hoy tert et? + qet 9 — Aat a ret veo teartosa?+ ee? ~ 529 — sat + 1 . = 1 2 Yl ata teat pete ae tae y" —etSe tae ‘The intial conditions imply ctee=0 atati=0 Wat qt2=0 2+e=0. Thus o = 2, = 1, ey = 0, and oy = ~2, and A ye gett 73. To see in this case that the factors of L do not commute consider the operators (xD — 1}(D +4) and (D +4)(zD ~ 1). Applying the operators to the function 2 we find {2D ~ (D+ 4)x = (2D? + 42D - D~4)z ya2ta- = 2D + deDz~ Dr—4e = 2(0) + 42(1) - adr and (D+4(2D~-1)2=(0+4\(2D2~2) =(D+a(e-1=2) ‘Tiss, the operators are not the same Exercises 4.6 ‘The particular solution, yp = uit + vayn, in the following problems can take on a vatiety of forms, ‘especially where trigonometric functions ere involved, The validity of # particular form can best be checked by substituting it back into the differential equation. 148 Exercises 4.6 1. The auxiliary equation is m?+1=0, 50 ye = 6 cos + epsinz and cose sing 21 2 ldentifying f(2) = seo we obtain ‘Then uy =In{eose|, v= 2,and y= cycost bepsins + cos in |cos| + zsinz, 2, The auxiliary equation is m? + 1s 0, 0 ye =o) cosz + c28inz and | cost sinz [-sinz cose Identifying f(2) = tan we obtain cost uj =-singtane = cos — sec uy =sinz, ‘Thon uy = sinz —In|secx + tan 2], up = —cosz, and y =o) cosz +o sinz + cosx(sinz ~ In |secr + tanz}) ~ cosrsinz. 3, The auxiliary equation i 2 +3 =6, 00 ye= cont +exsine and corz sing w ‘ldentifying f(z) = sinz we obtain shan? ub = cos x sin. Then 1 it 1 w= jsinde— 52 = 5sinzcosz 52 and + : 2 2 feos? sins excosz + epsing +5 sinzcos? 2 — Fa cos ~ 5 cos 1 = crcosz + sing ~ 52¢0ss. 149 Exercises 4.6 4, The auxiliary equation is m? +1 $0 ye = cy cost + epsin.x and cost sine w Identifying f(2) = secz tan we obtain uj + sin zx(secztanz) = ~tan?x = 1— sec? ub = cos2(secrtanz) = tan z 2 tang, w= —In|cosz|, and y= crcosz +esinr + reese ~ sing —sinzin| cos) = q.ewsz +epsinz + 200s ~ sin in| cos3| 5. The auxiliary equation is m?-+1= 0, so ye= 1 cos + epsins and W| tee cael Identifying f(x) = cos? we obtain uy = —sinz cos? 4 ca (1 sin?) ‘Then uy = Jos, up = sine ~ Join? 2, and cosz+epsing + 2 costs +sin?z ~ Zsint exons +epsing + cost gaint sw cyoosr + egsine + 5 (costs + sina) (cos? —sic?2) + sin?x 12 eyc0sz + epsina + 5costx + Fain queost+esinz +14 1sints, coset epsing 5 +S 6. The auxiliary equation is m? +1 , $0 ye ex c08 + o28in z and Wentifying f(2) = sec?.z we obtain Exercises 4.6 ‘Then up = ln|see + tan and y= crcose+epsinz ~ coszsecz + sin zIn|secz + tana] = cycosz + easing ~ 1 +sinzln|sece + tang], ‘The auxiliary equation is m?—1= 0, so ye = cye* + cpe"* and Identifying f(z) = cosh ‘Then and ae tee — exe ege? + Hl 4) = ose? + e4e* + Resin sacsae tt ‘The auxiliary equation ism? - 1 Idectifying f(z) = sinh2z we obtain 10. The auxiliary equation is m' Exercises 4.6 Thea and as mae tone + j sinh? = 0, 80 yom oye + opem% and mek We|ya pert Identifying f(z) = e%/2 we obtain = 1/Az and 1 gill ape eG ht and oe) pe elt yaa tacts t(einis|e% (at), >a ‘The auxiliary equation is m?—9= 0, so ye = exe + ene°* and eet w ae% 363 ing #(2) = 92/e™ we obtain v{ = $re~® and uh =~}. Then and oe yea tee - 2 i se + ert = Heer 32) 152 LL. The auxiliary equation is m?+9m +2 12. as. Exercises 4.6 te (m+ 1)(m+2) and nem + oe w [rent =2e-2 Identifying f(z) = 1/(1 + €*) we obtain “oe 2 +e *, and +e ‘Then 1 = In(t +e), up = In(2-+ e*) ym ce tone + oF In(1 +e) +e nll +e cae F + ope + (1+ e°*)e™* n(1 4 €*) ‘The ausiliary equation is m? — 2m-+1= (m—1)? =0, 90 ye= ce +oazeF ond w.[f = . jet sett Teentifying f(2) = e%/ (1-+22) we obtain “n-gage ee 1 4 “Rare ie Then uy = fin (1+ 22), wy = tant, and vee teazet — Zein (142%) + ae tans ‘The auxiliary equation is m? + 3m +2= (m+ 1)(m+2)=0, so ye=c1e-* + ope" and of et | as w j-e# ~2e% Identifying f(2) = sine* we obtain 1 etsing® je SS ec eme p sine? 2 sine? See ‘Then uj = ~cose®, up = e* cosa — sine®, and ys ce" + exe" ~ €* coset + eF coset — e* sine? mae +e — sine’, 153 Exercises 4.6 0,50 ye = exe! + cate and t | 14, The auxiliary equation is mn? —2m-+ 1 = (m ~ 1)? et : ‘ were Identifying f(¢) = ef tan~! ¢ we obtain a ,_etettan tt uy = aa = tan Then tE eaotend wae eatee’ : 42 pa(t+?) and vee (EE tanto Se (0007 (ite yee (1+2)] 15. The auxiliary equation is m? + 2m +1 = (m +1)? = 0, $0 y sexe! + cate! + Ze! [(@? ~1) tan ret sextet and Identifying F(e) ‘Then and y=aet sete t peatent + dpdet = cyet + eate + Stet int 16. The auiory equation is 2n? 2m +1 = 0, 90 ys= e¥7(eo82/2 + ensin2/2) and 154 a. 18. Exercises 4.6 wel ees e*aing a ~fer*? cosh —Jer*Msing Jer*/Peos § — fer"sin g |" 2 Identifying f(2) = 2/2 we obtain #/2 sin(x/2)2VE aA z uw ORRIN set sind i #2 coslz/2)2VE _ 45/2 2 Uy AE 0085. Then , mina [eV bt vant [el Feast ct and #/2(/e cos = + esin2) —4e"72 cos [* et/2VMisin tt + de-/2sin = [* et/2VE cos t at (cx 008 5 + exsin 5) ~ 4 ; isin 5 dt + 4 ae Vi cos 5 ‘The auxiliary equation is Sm? — 6m +6 = 0, so ye = €*(c}cosz-+ cysinz) and we| cose esing JeFcost —Fsing cost + efsing Identifying f(2) = Je*secz we obtain 1, __(etsing\leteeca)/3_ 1, | uy asslieenys Then uy = $n(cos2), up = fx, and 1 aD Y= cre cos2+ cx coz + ZIn(cosz}e* cos + Ze" sins (2m — 1)? = 0, 80 ye = cre*/? + onze??? and en ae? Welt faetlt 4 er? eR VTRF we obtain The auxiliary equation is de — dm + Identifying f(z) Exercises 4.6 ‘Thea and yaad seals Leh (1 22)4 Lenina Leelee 8 fm —1)(2m-+1) =O, S0.Ye = ee"? + cae 2)? and 19, The auxiliary equation is 4m' en esl jet jean deotifying f(z) = ze*/2/4 we obtain uf = 2/4 and ub = —2e/4. Thon uy 22/8 and aeF/A+ e7/4, Thus : n2/2 4 Lae) ym e184 oge"2/? + Fate — 1 etl? 4 hysit a 7 AP gga tty Bytes? Lagslt +F ne = 67? + ce and ' ' : Lye? Leggnslt Lately Lapel? esr 2 2 +76 a 4 ‘The initial conditions imply ‘Thus ¢3 = 3/4 and ep 1 L 1 Byairy Lynnig Largs? Lael, ya jet es 2 pe 20. ‘The auxiliary equation is 2m +m eye?! + ope-* and (2m —3)(m-+1) =0, s0 y. po yer — Wa Identifying f(x) = (x + 1)/2 we obtain 21. Exercises 4.6 ‘Thea Thos and ‘The inital conditions imply ‘Thus oy = 8/3 and cy = 1/3, and ‘The auniliary equation is m? +2m—8 = (m~2\(m +4) = 0, 80 ye = cre + ce and a ee Wa lgee nae Identifying f(z) = 2% ~ e# we obtain ‘Then wa ‘Thus y= ce + oe — = ae tee and 157 22. 23. Exercises 4.6 The initial cqnditions imply 5 at angel 1+ Oa 1 6 fn + Pe ent Thus ¢; = 25/96 and ox = 4/9, and -%, v= Be ‘The auxiliary equation ism? — 4m +4-= (m~ 2)? = 0, s0 ye = cre™ + eze!* and eae “ae aze* + Identifying f(2) = (1227-62) e we obtain w a= 62-1228 uf = 122? 6x. Then uy = 22° — 324 uy =a! — 322 ‘Thus y= cre + opee™ + (20 ~ Sat) e% + (42° — 32) ae mre togrel +e (24-23) and Y= 22 4 (220% +2) + 22 (429 — 302) + 20% (c— 25) Te initia conditions imply ao ta ta=0. ‘Thus o1 = 1 and op = ~2, end y= 208 +02 ( 0 and has z-intercepts where In = x/24+kr for k an integer or where x = e*/2+#*, Solving 1/24 br = 0.5 we get k= ~0.34, 50 e/2#* < 0.5 for all negative integers and the graph has infinitely many 2-intereepts in (0, 0.8). ‘The auxiliary equation is 2m(m — 1}{m ~ 2) — 10.98m{m ~ 1) + 8.5m + 1.3 = 0, so that my = ~0.053299, ma = 1.81164, my = 6.73166, and oy POS 4 yg ABE 4. gS TIGS ‘The muxiliary equation is m(m — 1)(m— 2) +4m(m ~ 1} + 5m —9=0, so that my = 1.40819 and ‘the two complex roots are ~1.20409 + 2.222911, The general solution of the diferential equation is y= cya s8819 + 2120095 co5(2.29201 i 2) + ey sin(2.22291 In) ‘The auxiliary equation is m(m—1)(m—2)(m—8) + 6ra(m~1)(m—2)+3m(m—1) -3m+4=0, 0 that my = my = 3 and ms = my = —V3. The general solution of the differential equation is VE soe Vln year toring tae + cue ‘The auxiliary equation is m(m—1)(m~2)(m~B)~6m(m-—1)(m~2)+33m(m—1)—105m-+169 = 0, 0 that my = m2 = 342i and my =m =3—2i, The general solution of the differential equation y= 2%[e1cos(2inz) + cpsin(2in2)] + 2°In-x{cscos(2in x) + cy sin(2in=)) 170 Exercises 4.8 Exercises 4.8 1. From Dz = 22—y and Dy = z we obtain y = 22— Dr, Dy = 22~ Dz, and (D?-2D+1)z =0. Thea zacye'tete! and y= (cy) — eget + cxte’. 2, From Dz = 4+ 7y and Dy = x ~ 2y we obtain y = 4D2 — fz, Dy (D? - 2D - 18): Then . 1 oe readme and y= Fae oe. 3. From Dz = —y+tand Dy =2~t we obtain y=t— Dz, Dy=1— D's, and (D?+1)r= 144, ‘Then ex cost-+ cpsiné +1 +t and qqsint ~ epeost t= 1, 4. From Dz—4y = 1 and 2+ Dy = 2 we obtain y= }D2—4, Dy = }D*s, and (D? +1)z= 2. Then eycost + esint +2 and 5. From (D?+5)z—2y = 0 and ~2r-+(D?+2)y = 0 we obtain y = $(D?+5)z, D¥y = f(D'+5D%)z, and (D? + 1)(D? +6)z = 0. Then €; c08t +2 sint + cy.c0s VBE + cx sin VEE and 20s cost + 2epsint — Hegcos VEt — Seasin VEt 6. From (D+ 1)z + (Dy = 2 and 32 + (D+ 2)y = —1 we obtain 2 = -- 4(D+2)y, Dz = -}{D? + 2D)y, and (D2 + 5)y=—7. Then y= creasvt+ epsin VEt~ E and oan V5, 2a 3 e=(Go-Bo cavit+ (Ba - je sin v5e+$ pots — fe and 7. From Diz = 4y +e! and DYy = 4o'— et we obtain y = JD% ~ je, Déy amt 10. ). From Dr + Diy Exercises 4.8 {D?+4)(D~2)(D +2)z = ~3et. Then 1 co82t + opsin 2t + eye + eye and 1 C08 2t ~ cp sin 2t + ge + eye — y 5 From (D? + 5)z+ Dy = 0 and (D+ 1)z + (D ~4)y = 0 we obtain (D - 5)(D? +4)x = 0 and (D~5)(D? +4)y = 0. Then exe + epcos2t + egsin 2t and y= cue! + 05 cos 2t + og sin 2t Substituting into (D +1) +(D~ é)y =0 gives (Ger + ca)e® + (cn + 2c ~ des + 2eg) cos 2t + (—2ea + 3 — 2es — eg) sin 2t = O 80 that cy = —6e1, ¢5 = $es, 5 = —}ea, and exe + By cos 2t — Sep sin Gaye + Feycos3t ~ Fepsin 2. and (D + 1}x + (D—1)y = 4e™ we obtain D(D* + 1)z = 34e™ and D(D? + 1)y = ~8e™. Then 4 at y=ortensint + cpcnst — 736% and ae = cyt opsint + cocost + 1 Baca essint + cocost + Te Substituting into (D+ Iz + (D~ Uy = 4e% gives (e401) + (ep — 05 — 09 — eg) sin + (05 +05 + 02 — og) 005) so that q = ¢1, 05 = ¢3, ¢5 = ~cp, and = or ~cpcost +eysin + te Beane 7 5 From Dz — Dy = t and (D +3) +(D + 3)y = 2 we obtain D(D + 1D + 3) = 1+3¢ and D(D+1\D +8)y =-1—3t, Then nae i emataettae™ t+ 50 and 2 yr cp teset tose bt 2 172 nL 12. 13. 14, Exercises 4.8 Substituting into (D +3) + (D+ 3)y = 2 and D?z— Dy = t gives Shey +64) + 2en + eset = 2 and (cot es)ert + 8(8ea+ cle = 0 so that 04 2, 05 = ~8c3, and ee se td yo ran eet sae +t From (D? ~ jz ~ y= 0 and (D~ 1)z + Dy = 0 we obtain y = (D® ~ 1)2, Dy = (D®— D)z, and (D~1(D2+ D+ 1}z=0. Then read set? [as y= ($a-Ya) anther (Fafa) taal From (2D?-D~1)z—(2D+1)y = lend (D-1)z4Dy = —1 we obtain (2D+1)(D-1)(D41} 2, Then reg eget ese 4 ys age + cet 2. Substituting into (D = 1)2 + Dy = ~1 gives eee (-Ja~ jeu) et + (200-03) Bey, ¢5 = —2en, and so that ca Beye? — ope“! = 2. From (2D—5)z+Dy = ef and (D—1}z+ Dy = Set we obtain Dy = (5~2D)r-+e! and (4—D)x = 4et Then Sere + 5e so that and Dy From Dz+Dy =e! and (-D?4+D+I)r-+y = Owe obtain y = (D?~D-1)2, Dy = (D°-D®—D)z, and D*(D- 1)z~ et. Then cut eat eget tet and 173 16. 16. ar. 18. Exercises 4.8 y= ne erent — aye! tel bel Multiplying the frst equation by D +1 and the second equation by D* +1 and subtracting we obtain (D! — D2)z = 1. Then za oy boat tye! + oye! Multiplying the first equation by D +1 and subtracting we obtain D'(D + 1)y = 1. Then L yacptattoet = Substituting into (D~ 1) + (D? + 1)y = 1 gives (ei tea tes — 3+ (=204-+ Rene + (“1 en + os)t 21 40 that 65 = ¢1 — 62 +2, ¢5 = cn +1, and or = cg, The solution ofthe system is exe teat tone! + cue ~ 5 i y= (er~ en +2) +(er4 te + oe — 54 From Dz - 2(D? + D)y = sint and z+ Dy = 0 we obtain z = ~Dy, D’r D(D? +2D +2)y = —sint. Then ey tone" cost + oyetsint + Post + 2 sin 5 5 and -t 1 2 (ex+esle™*sine + (ea — ese cost + sint ~ = cost. From Dz = y, Dy =z. and Dz = 2 we obtain s = D¥y = Dr so that (D ~ 1)(D*+D+1)z =: sea +e? [asin Ye cor Se] Be eee Dz = —D'z + et, and the system (~D? + D ~ 1)z + Dy = et and (-D? + lz +21 y= HD? ~ 1x, Dy = }D(D* - 12, na (D - 2)(D? + 1) race" +ocost + eysint tel, and 2et so that a y= zie —encost — epsint, and 174 Exercises 4.8 2 = —2eye™ — cy cost + on sint. 19. Write the system in the form Dz~6y=0 2-Dy+z=0 pty-Dz=0. ‘Multiplying the second equation by D and adding to the third equation we obtain (D+1)z~(D? ~ ly =0. Eliminating y between this equation and De ~ Gy = @ we find (0) D~ 6D-6)z = (D+1)(D-+2\(D -3jz=0. Thus pace tt ne + one, and, successively substituting into the first and second equations, we get 20. Write the system in the form 2-y+Dz=0. Multiplying the third equation by D-+1 and adding to the second equation we obtain (D + 1)2+ (D°4.D-1)2 = 0. Eliminating = between this equation and (D+1)2—2 = 0 we find D(D+1)?%x = 0. ‘Thus zat cnet besten, ‘and, successively substituting into the fist and third equations, we get y= cpt (crea) + ogte* atae” 21, From (D4+8)z+y = Oand 42 (D+1) ‘Then dr +(D?+5D)x-4(D+5)2= Owe obtain y = -(D+5)z 8 that Dy and (D+3)°2 =0. Thus (Dt+sD)x. and Exercises 4.8 Using (1) = 0 and y(1) = 1 we obtain ata=0 201 +309 = = The solution of the initial value problem is A249 yp 3t+3 ‘Thus ¢1 = e* and ~L and 32-+(D~2}y = 0 we obtain x = ~}(D ~2)y so that Dr Then -4(D? = 2D)y = y ~ 1 and (D?~ 20 + 3)y = 3. Thus =e! (ey cos V3e-+ensin VBA) +1 4(D?-2D)y. and 22 4e [(a- Voex) cos v2t+ (YBa +0) sin v2") +2 Using (0) = y(0) = 0 we obtain ati=0 ae La-via)+3e0 1 and = 3/2. The solution of the intial value problem is a a 2 ane (~Jeosvae—YPsavar) +2 Thus 1 = vod (-onvit+ Bainvit) +1 23, Bquating Newton's law with the net forces in the 2- and y-ditections gives mie —mg, respectively. From mD#z = 0 we obtain 2(t) = c1t-+c2, and from mD*y we obtain y(t) = ~hot? + ext +65, 24, From Newton's stcond law in the z-direction we have In the y-direction we have éy ae Exercises 4.9 From mD?z+|c|Dz = Gwe have DimD + |dl)z = 0 so that (mD + lel) = cx. This is a first-order linear equation. An integrating factor is el HAt/melet/™ so that A ildting) = eyelet™ Setting] =e and els = (erm/l))#/" +n, The general solution ofthis equation is 2(¢) = ep-+exe#™. From {mD? + jeD}y = ~mg we have DimD + leljy = —mg so that (mD + [ely = ~mgt +e This iso first-order linear equation with integrating factor el*/", Thus Siebting = (mgt tayedtm 2 4, 9 gctin , S getim 4 SU gon Tel e let eetiny o and mg mi lett vO) = eet Tae tet exer 25. Multiplying the frst equation by D +1 and the second equation by D we obtain D(D + 1)z-2D(D + 1y= 2+? D(D + Ie-20(D + 1)y =0. ‘This leads to 2¢ + 12 = 0, so the system has no solution. Exercises 4.9 ‘1. We have yj = yj =e, so (ui? = (er? sinz and yf = -cosz, so ws? However, if y = cu + oxun, we have (y/')? = (cre® — enc0sz)? and y? = (ce + epcosa)?, Thus war 2. We have vj = vf =0, 80 Also, vf, (— cos)? ayy = Hu Allo, yh = 2e and yf =2, 20 7 1 diy sav} = 27(2) = 22? = Lan)? = Leap? However, if y = cig + cnyp, we have yy” = (cy -1+ cpt*)(cy - 0 + 202) = 2ea(er + 22?) and BW)? = Flor 0+ ea(22))* = 242", Thus wy” # Bly 17 Exercises 4.9 =v so that w! =p”. The equation becomes u’ = —u~1 which is separable. Thus atte => ystenley—2) — y= la|cos(e)—2)) +07. - Let u = yf so that u’ = y". The equation becomes u/ = 1+u?, Separating variables we obtain du A pope tater te > ue tenlet a) <= y=—Inlealeteyita |. Let u = y/ so that w’ = y/'. The equation becomes 24x’ + u? = 0, Separating vasiablee we obtain a w+ 2 1 a tqatttl yg 2 2 une = a (x) a 1 1 = ys gblas+t|- Sete, du 5. Let w= so that of! = ? du The equation becomes (y + 1)u 7 = w obtain Separating variables we Ta pei BM =mWtilsing = vay +2) dy ty aw p= 8 ear yt) = Peas ae = ly tiysarte = ytlaaet. ie aoe at us y so that y= ut. The equation becomes uf + 2yu? = 0, Separating variables we Yoo that yf =u The ea $e + 20 = 0. Separating obtain 1 Pra Let v= y! so that y/ 4 o du= = dy=dz ¥ ay 1 1 =i = de (for 6 #0) => Ay+iniy-ij=2 Esau) az (forex #0) —> Sut gin =zte , then ydy = —dr and another solution is $y? = —2 +e 178 Exercises 4.9 (b) Let u= y/ so that obtain i dy eee 5 gp ty pete sn(le+a) |. y= 1 80 1 = taney and ¢2 = 1/4, The solution of the initial-value problem is rem (GH) When 2 = ‘The graph is shown in part (a). (©) The interval of defintion is —n/2.< 4/4 — 2/2 < 4/2 or —n/2< 2 < 3/2. 179 Exercises 4.9 10, Let u = y' so that u! = y’, The equation becomes (u')?+u? = 1 » which results in w! = tVT=WF. To solve u’ = VI=u? we 2 separate variables: = sin useta = ussin(rt+c) ‘ a 2 = yf =sin(e +a) eee (5 ~§) tea -3 + e7 and x = 1. The solution of the lakabalve problem sy = 1 ~cos(«~ 2). : ‘To solve u! = —VT=w? we separate variables: andr = ostuszta = us cos(r ter) => yf = c0s(z +1) so Bw cos(% sei) and er = -F. Thos yeos(e-2) = yesa(e-Z)+0 asin (3 ~ $F) +02 = j.4ey ond ep = 0. The solution of the initial-value hs0 : problem is y= sin (c - 2) 1a S whic sro Using 2 2. Am integrating factor is 22, so LL, Let u = yf so that of = y/ The equation becomes u' a du 3 w=? we obtain 2 + Zu = a Ju] = 22 Peg =e unt ta me wat +9 = uses 180 Exercises 4.9 12. Let u = y/ so that w’ = y". The equation becomes w! — which is Bernoulli. Using the dw we obtain $2 4 Ly = 1. An integrating factor is, s Str ‘An integrating 0 substitution w 20 ea afer ~23| In Problems 13-16 the thinner curve is obtained using a numerical solver, while the thicker curve is the graph of the Taylor polynomial 18, We lok fora solution ofthe om v(2) = 0)-+ 910) + Fo") + Au"0)+ Lie) + BV) From y"(z) = 2+ y? we compute (x) = 1+ 2yy' ya) = ay +20")? f 91(2) = 2yy"" + 6y/y" Using y{®} = 1 and y'(0) = 1 we find a | ¥O)=1, MOH=3 YO yO) = 12. ‘An approximate solution is Tress wa) mts fete the Jets tt 14, We look fr a solution of the form via) = v(0) + 9/0) + 500) + Ay") + Eye) + Sul, ‘ From y"(z) = 1 — y? we compute w"(2)=-20/ yl (z) = ~2yy" — ay? - * (2) = —2yy/"— 6y/y" ‘Vsing y(0) = 2 and y/(0) = 3 we find vO) =—3, Q)=—12, y9(0)=-6, yO) = 102. 181 16. xe. a. Exercises 4.9 ‘An approximate solution is wa) 22432 — $e? 203 — Bat 4 We look or a solution of the form le) = 0) +90) + 500) + ZeyO) + From y/"(z) = 2? + y? —2y/ we compute ¥"(x) = Qn + Quy’ — Ay” ya) = 2+ oo + Quy!" — 24" (a) = 6/y! + 200" — 24 Using y(0) = 1 and y/(0) = 1 we find wO)=-1, yO)=4, YO) = ‘An approximate solution is, (0) = 14 Vaya lta fate Be We ook ora ton fe em . ve) =200)4¥()+F O+ Ay" (O)+ DIME) + Aya) + Fi 4 fom a) =o wesompte ; vz) = ery! ye) = ey"? + ely” (a) = ey) + Seby/y! + ey | Hy) + Bet(y')*y! + Bey")? + de/y + ety 4 Using y(0) = @ and y/(0) = -1 we find . vO=1 Y=, yO)=2 yO) ¥(0) = 16. 1 ‘An approximate solution is, ya) Lag dete date et Wa) =-24 ja? dah + hate he Ee We need to solve [1+ (y)3)"* = y" Let u = yf so that w = y/'. ‘The equation becomes (14 ut)" =u or (14ut)"? = 2. Separating variables and using the substitution w= tand 182 Exercises 4.9 wwe have au a+wy (for2>0) = y= ie fx iu iu _ —K? 18. Let u= & so that $3 = 0S .55 ee Ga ~ ¥Z- The equation becomes u SE = I. Separating variables we obtain 2 1p # 1g_# udus 3dr = sate = —— 2 ce 2 z 2 2 a e 2 When ¢= 0, = 29 and v=0300=" + cande= ©. Then = % jtee(i-2) ws $--1a BS 2 20, ¥z9 Separating variables we have NB) fea" ee oe INF [Wee Bear 5) | 1 % ao 2s -1/2 [Vana Fs POR. PA Px For q+ sinz = 0 the motion appears to be periodic with amplitude 1 when x; = 0. The amplitude and period are larger for larger magnitudes of x3, 183 Exercises 4.9 -t se0 ] Pz de For Sp + G+ sin 0 the motion appears tobe period with decreasing amplitude, The dx/at term could be sald to have a damping effect. sinz, and ww"? 20. When y = sinz, y! = cosz, y/" ‘When From (y")? ~ y? = 0 we bave o' = sy, which can be treated as two linear equations. Since linear combinations of solutions of linear homogeneous differential equations are also solutions, we see that y= ce? + ee"? and y = cycosz + cysinz must satisfy the diferential equation. However, linear combinations that involve both exponential and trigonometric functions will ot be solutions since the differential equation isnot liear and each typeof function satisfies a different linear difeential ‘equation that is part ofthe original differential equation 21. Letting w /", separating variables, and integrating we have a ViTe, Gly nds and aikteneta +i ‘Thea uso! =sinh(z +e), cosk(e$e)) $6, and y=sinh(e-ter) tore tes Chapter 4 Review Exercises ce? + one", a particular solution for 9!” At Bret, yalteisy ‘They are linearly independent over (oo, 00) and linearly dependent over (0, c0). (a) Since fa(z) = 2inz = 2/i(z), the functions are linearly dependent. 184 10. u. 2. 13. Chapter 4 Review Exercises (b) Since 2"*" is not a coustant multiple of 2", the functions are linearly independent. () Since +1 is not a constant multiple of 2, the functions are linearly independent. (@) Since fiz) = cosx-cos(n/2) ~ sin: sin(r/2) = —sinz = —fo(z), the functions are linearly: dependent. (e) Since fi(z) = 0- fol2), the functions are linearly dependent. (£) Since 2z is not a constant multiple of 2, the funetians are linearly independent (s) Since 3{z*) + 2(1 ~ 2*) ~ (2+ 22) = 0, the functions are linearly dependent. (h) Since xe! + o(4x — S)e* ~ exe* = 0, the functions are linearly dependent. + (a) The auxiliary equation is (m~2)(m-+5)(m~1) = m!+m?—17m-+15 =O, so the differential ‘equation is y"" + y" = V7y'+ 16y = 0. (b) ‘Tae form of the auxilisry equation is m(m —1)(m— 2) + bm(m — 1) +om+d= m? + (b—3)m? + (c~ b+ 2)m+ d= 0. Since (m ~ 3)(m-+ 5)(m~1) = m°+m? - Vim+15 = 0, we have )-3=1,c—b4+2=~I7, and d= 15. Thus, b= 4 and e= ~15, so the differential equation is y"+4y" — 16y'+ 15y = 0. - Variation of parameters will work for all choices of g(z), although the integral involved may not. always be able to be expressed in terms of elementary functions. The method of undetermined coefficients will work for the functions is (8), (c), and (¢) . From m? — 2m = 2-= Owe abtain m= 1 V3 so thet yn eye 4 eyllVO, From 2m?-+ 2m +3 = 0 we obtain m = —1/2:t V5/2 s0 that - wert (0, cos Ee + eosin, yn er? (om Be renin S From m® + 10m + 25ne = 0 we obtain m= , m= =8, and m= —5 30 that yercrt eae care, From 2m? + 9m? + 12m +5 = 0 we obtain m = —1, m= —1, and m= —5/2 so thet e782? + ope + cgze*. fe From Sm + 10m? + 16m-+4=0 we obtain m = ~1/3 and m = ~3/2 v7/2 0 that wee 4 28 (ep con Me + egsin Me years (ae er ses) 185 14. 15. 16. a. Chapter 4 Review Exercises From 2m + 3m! + 2m?-+ 6m — 2, and m = V3 so that = 0 we obtain m= 1/2, m ym ere? exe + 0y008 VEx + egsin VEr 0. Then Applying D* to the differential equation we obtain D*(D? ~ 30 +5) pear (er tein Fa) seq tesrt es! ta! and yp = A+ Bz + Cx? + Dz. Substituting yp into the differential equation yields De + 4a" (5A - 3B +2C) + (5B - 6C +6D)x + (SC — 9D)z” + 5Dz° Equating coefficients gives A = —222/625, B = 46/125, C = 36/25, and D = 4/5. The general solution is = 020 (acm Hie gin V,) 224 8 ya e2? («00 Pies sin Ye) FP + a Applying (D — 1)° to the differential equation we obtain (D ~ 1)(D ~ 2D +1) = (D-1)° = 0. Then yrat tare parte betel tesetet and yp = Ante? + Bale! +Ca'e*, Substituting yp into the dferential equation yielés 1202" + 6Bze* + 2A? = ae? 1/12. The general solution is Bquating coefficients gives A =0, 8 = 0, and C oe te pratt +d ‘Applying D(D? + 1) to the differential equation we obtain. D(D? +(0? — 50" +60) = DD? +1)(0 = 2)(0 -9 ‘Then tne +90 +042 + e50082 + sine and yp = Az + Beosz +Cssinz. Substituting yp into the differential equation yields 64 + (5B +5C)cosz +(-5B +50)sinz = 8 + 2sinz. Equating coefficients gives A= 4/3, B= ~1/5, aid C= 1/5. The general solution is = oy + cnet? ese + Ae — Pecos + deine. yor cael tse + 5 +t 186 Chapter 4 Review Exercises 18. Applying D to the differential equation we obtain D(D® — D") = D*{D—1) = 0. Then yeatartac +z" and yp = Az?. Substituting pp into the differential equation yields ~2A = 6. Equating coefficients ‘gives A= —3, The general solution is yratar tact 32 19. The auxiliary equation ism? ~2m+2 [m= (1 +i))fm~ (1—8)] = 0, so ne = exe sina + ene" cose and esinz Foose Scosrtesing —e% sine + ef cost Identifying f(s) = ef tan we obtain » __(€c0s2)(e* tana) ina ~In|secx + tani, and ym ce?sing + ce" coss —e*sinzcosz + €* sin cosz — e* cos zin| sec + tans = exe sine + exe" cost — e* cos zn |secz + tan zl 20. The auxiliary equation is m? —1 = 0, s0 ye = cre® + o2e°* and e wales Identifying f(2) = 26/(& + e"*) we obtain —— ie ‘Then u: = tan“, up = ~e8 + tan le, and pace? toe 8 + ef tan het 1+ en tan Fe 21, The auxiliary equation is 6m? —m—1= 080 that 12 gee! +718 187 22, 23, as. 25. 26. ar. Chapter 4 Review Exercises ‘The auxiliary equation is 2m? + 13m? + 24m +9 = (m+ 3)*m + 1/2) = 0 s0 that 1 ne) + ope Ping + 42 y +00 +3 ‘The auxiliary equation is m? at sting so that Sm+6 = (m—~2)(m—3) = 0 anda particular solution is yy = yaoxt baste ating ‘The auxiliary equation is m?—2m-+1 = (m~1}? =0 and a particular solution is yp = $2 so that y L yeteeing + 523 ox 7 (a) The auxiliary equation is m? +? = 0, s0 ve ary Acosat + Bsinat and 1cosust + cpsinwt. When w # a ty = y= cucoswt + ersinut + Acosat + Bsin at. When w = 0% Yp = Atcosat + Btsinat and y= cosut + eysinut + Atcosat + Btsinat (b) The awsitiory equation is m? —u? = 0, so ye= cie™t + ce, When wf 2, up = Ae and. yaoi tenet Act When w= a, yp = Ate® and y= cre + oye + Ate (a) Ie y = sinz is a solution then so is y = 282 and m? +1 is a factor of the auxiliary equation m4 2m! 4 Lim? + 2m +10 = 0. Dividing by m? +1 we get we 4 2m_+ 10, which has roots 14%, The general solution of the differential equ: ion is e1cos + esinz + &*(e3cos3z + cusin32), (b) ‘The auxiliary equation is mm +1) = mit += 9, so the associated homogeneous diferential equation is y+ y/ = 0. Letting y = 1 + xe~* + $a? —z and computing y’ + y’ we get 2. ‘Thus, the differential equation is y" + yf =<. (a) The auxiliary equation is m4 ~ 2m? +1 = (m? 1)? = 0, so the general solution of the Aiferential equation is ye cvsinh + cp cosha + cursinh 2 + oe cosh 2 (b) Since both sinh and sinh 2 ace solutions of she associated homogeneous diferental equation, a particular solution of "9 ~ 2/4 y = sinh z has the form yp = Aa*sinh s + Br? cosh 188 Chapter 4 Review Exercises ‘and yf = 0, zy! — (2? + 22)y{ + (2 +2)yn = -2?~ 22 +274 27 = 0, and yy = 2 is of order, we let 28. Since af = ‘solution of the associated homogeneous equation. Using the method of reduct pave. Then of = tu! + uand y= zu" +20, 90 ay! ~ (2? 4 2e)y + (a4 y= ou" + aru! — 28! — One! — 22a Dou tzu + Ore Pu! ~v). To find a second solution of the homogeneous equation we note that u = ¢ is a solution of w' =u =0. Thus, ve = ez + e2e?. To find a particular solution we set 2%(u" — u’) = 29 s0 thot w” —u’ = 1. This differential equation has @ particular solution of the form Az. Substituting, we find A = —1, s0 a particular solution of the original dfitential equation is yp = ~2? and the ‘general solution is y = cy + cgze ~ 2? 29. The auxiliary equation is m?~2m4+2=0 so that m ue /2) = 0 and vn) 80. The auxiliary equation is m? + 2m-+1= (m-+1) = 0, s0 that y u(-1) = 0 and v(0) = ge = Oand ~oy + 62 = 0. Thus 61 = ep andy is a sclutin of the boundaty-value problem for any real number 1. The ausiliry equation ig m?~1 = (ex — 1}(m-+1) = 060 that m= al and y = aie? + xe Asouming yp = Ar +B-+ Csinz and substituting into the differential equation we find A = —1 seiand y 0. Thos, y (cx 6082 +cpsin.2). Setting eT cos. 1 we obtain oy =e and cp ce" + egzen*. Setting tee we get cy jal-value problem is 82. The auxiliary equation is m?+1= 0, so ye =e) c082 + epsinz and ose sing lentifying f(2) = sec? 2 we obsain 189 Chapter 4 Review Exercises ‘Then Ths _ yao osing—heosrwcs + satan = qcosz+esine ~beecs + LOPE 1 momsztasns+ loecs aod {= -cysine + excosz.+ bsecxtans, Baa cos + 5 seer 8 ‘The initial conditions irmply +}ei a ‘Thus oy = er = 1/2 and deine +} cose + 58inz + Seer 2 du 33. Let u= yf so that w/ = y". The equation becomes u SY = 4, Separating variables we obtain vdenteds — Waatta <= autre When x= 1, y'=u=2,s04=440p and =0. Then dy waa — & or : ae = yatta or When 5 =1, y= 5,s05= 1 4c5 and §=—1+e4. Thus cy and y = -27 +6, Note however that when y= —22 +6, y! solution ofthe initial-value problem is y = 22 44 34. Let uso that y” =u. The equation becomes 2u 2 = Sy. Separating variables we obtain iy xy 2udy = 3? dy => way ter 190 Chapter 4 Review Exercises When x =0,y=1and y'=u=1s01=1+c) and cy =0. Then Be ayP me lyn te = era? Wiens = y= el ee ee e-P and y'(0) = —1 #1. Thus, the solution of the cs however that when 9 = —fa.0! = ~Eap initia-value problem is y= —*5 oR 35. (a) The auxiliary equation is 12m* + 64m* + 59m? — 23m ~ 12 = 0 and has roots ~4, -3/2, ~1/3, ‘and 1/2. The general solution is exe eye? 4 egeF + oe (b) The system of equations is atetataqet ey ~ Sen - beg + bey = 2 wto) — Jar Goat fea= ; ee arte 7 ~Giey ~ Fer~ Foes + Feu = 0. Using 2 CAS we find. = —9, 2 The solution of the initial-value problem is 3 495 191 36, 37. 39. . From (D = 2)2—y = Chapter 4 Review Exercises Consider 2y/" + / = 0 and look for a solution of the form y = 2". Substituting into the differential equation we have ay" + yf = m(m = 12"! + ma) = mz, ‘Thus, the general solution of 2y/"'+ y/=0 is ye= 1 bealnz, To find a particular solution of ay" + y/ = —y/% we use variation of parameters The Wronskian is 1 inz\_1 Jo a2] Identifying f(2) = -2~'/? we obtain y_o-V ing i aE Pea vine and so that _ ua 28(Ene-4) ‘Then yp = 2°/2(} In 2— §)~ §r°/?inz = —Gz¥/? and the general solution of the differential equation a+ ela ~ $2°/7, The initial conditions are y(1) ~ 0 and y/(1) = 0. ‘These imply that A, isy e1 = {and cy = §. The solution ofthe initial-value problem is y = §-+ 9Inz — $s" From (D — 2) + (D ~2)y = 1 and De + (2D ~ t)y = we obtain (D~ 1)(D ~ 2)y = ~6 and Dr =3-(2D-1)y. Then yace™+met—3 and 2 Substituting into (D — 2) + (D —2)y = 1 gives cs = 5/2 80 that and From (D = 2)z—y =e and ~32-+ (D —4)y = ~7e! we obtain (D ~ 1)(D — 5)z = —4e so that cre! + ge tet Then. (D~2)z +e! = ~cyet + Bene — tel +264 192 Chapter 4 Review Exercises 40. From (D+2)2+(D-+1)y = sin 2t and 52+(D+3)y = cos 2t we obtain (D?45)y = 2eos2t~Tsin 2t ‘Then 2 z ym ciout+epsint — 5 cost + Zsin2t and 1 1 Bx -p(D+3)y+ Zeon = (Ja~Se)ane+ (Ha $e) ost Sonar Fone 193 5 Modeling with Higher-Order Differential Equations Exercises 5.1 1. Prom Ja" + 162 =0 we obtain 1 cos8VEt + oysin 8Y2t 0 that the period of motion is 2n/8V2 = y21/8 seconds. 2. From 202" + kr =0 we obtain 1 fe 1 gitensin5y so thatthe frequency 2/1 = f/R[Br and k= 320 Nfm. I S02" + 9800 = 0 then 2 = 1002 + ‘op8in 2 so that the frequency is 2/2 = 1/1 vibrations/second, 3. From Je" + 722 =0, 2(0) = ~1/4, and (0) = 0 we obtain z =~} cos4v6e 4, Brom fe" + 72z = 0, (0) =0, and 2'(0) = 2.we obtain z = ¥8 sinavB. 5, From $2" +402 = 0, x(0) = 1/2, aud 2(0] = 0 we obtain 2 = }cos8 (a) 2(/12) = -1/4, (0/8) = -1/2, 2(/6) = ~1/4, 2(%/8) (b) 2 = —dsin8t so that 2/(3x/16) = 4 ft/s directed downward (©) Wa= }oos8t = 0 then t = (2n + 1)#/16 for n =0, 1,2. 6, From S02” +2002 = 0, 2(0) = 0, and 2'(0) = -10 we obtain z = 7. From 20z" + 20 = 0, z(0) =0, and 20) = ~10 we obtain 2 (a) ‘The 20 keg mass has the larger aniplitude, (b) 20 ke: 2/(n/4) = ~5VE m/s, 2'(5/2) = 0 m/s; 80 ket 2!(n/4) =0 m/s, 2/2) = 10 m/s (c) If ~Ssin 2¢ = —10siné then 2sin (cos¢—1) = 0 so that t = ns for n = 0, 1, 2.» placing both masses at the equilibrium position. The 50 kg mass is moving upward; the 20 ky mass is 4) 2(0n/32) = V3/4 Sin 2 and 2! = ~10cos2t. 10sin¢ and 2 10cost. moving upward when n is even and downward when m is odd 8, From 2" 4 16r = 0, 2(0) = —1, and 20) 10. u. 2. 13. 14, Exercises 5.1 ‘The period is 7/2 seconds and the amplitude is V5/2 feet. In 4 seconds it will make 8 complete vibrations. "+ = 0, 2(0) = 1/2, and 2"(0) = 3/2 we obtain 1 ss ze oos2tt Ssinat = AF sniar + 0.588), From 1.62" + 40r = 0, 2(0) = ~1/3, and 2'(0) = 5/4 we obtain 1 i 5 Bm ~Fcosht + jsindt = sin(st + 0.927) Ve 2=5/24 then t=} (§-+ 0927+ 2nr) and ¢ = § (38 +0.997-+ 2nz) for n= 0, 1,2, From 22” +2002 = @, 2(0)) = -2/3, and 2'(0) = 5 we obtain (a) 2 = —}cos 00+ } sin 10t = f sin(10% ~ 0927). (b) The amplitude is 5/6 ft and the period is 2/10 = 7/5 (c) 31 =k/S ond k = 15 cycles (a) If 2 =0 and the weight is moving downward for the second time, then 10t ~ 0.927 = 2r or 12070 (€) Ifa! = % cos(101 ~ 0.927) = 0 then 10¢— 0.997 = n/2-+ nx o t= (On +3)n/20+ 0.082" for n=0,1,2, (f) 2(3) = -0597 & (g) 2"(3) = 5.814 ft/s (h) 278) = 50-702 f/s? @) Ie= then ¢ = $h(0.927 + nn) forn=0,1,2,..-anda'(é) =2% A/s. G) I2= 5/12 then t= fh(a/6-+0.927 + 2ne) and ¢ = $y(5n/6-+ 0.927 + Ine) for n= (6) 12 = 5/12 and 2 <0 then t= h(61/6-+ 0.927 + 2n) for n= 0, 1,2, From x" + 9x = 0, 2(0) 1, and 2'(0) = —V3 we obtain en nonnt-‘Banar= aan (ore 2 and 2! = 2V3cos(3t +4n/3). If 2’ = 3 then t = —Tr/18 + Inn/3 and t = —n/2+ 2nn/3 for n=23, From fi = 40 and kp = 120 we compute the elective spring constant k = 4(40)(120)/160 = 120 Now, m = 20/92 so k/m = 120(32)/20 = 192 and 2" +1922 = 0. Using 2(0) = 0 end 2/(0)=2we obtain z(t) = ¥f sin8V3¢. Let m denote the mass in slvgs of the first weight. Let ky and kp be the spring constants and Je = Ahsha/(hi + ha) the effective spring constant of the system, Now, the numerical valve of the 195 15. 16. ar. 18. 19. 20. a1. Exercises 5.1 first weight is W = mg = 32m, so =h(2) and 32m = e(P son=t() wi son -4() so w = 30. Since the mass of an 8-pound weight is 1/4 slug, we have from w? = k/m = 7/18, k oe or k= whe Atk or kas We now have the system of equations sivky mem 2, = ha Solving the second equation for ky and substituting in the Brst equation, we obtain A(Bkn/2)h _ 12KY 1k Sh/2+ ha ~ Sh“ S Thos, kz = 375/4 and by = 1125/8. Finally, the value of the frst weight is atom = M2 NOSIS _ 375 W=som =H 18/8 _ 3 soso 225, For laige values of t the differential equation is approximated by 2 ‘equation is the linear function decayed to the point where the spring is incapable of returning the mass, and the spring will simply ‘keep on stretching. = 0. The solution of this ext +2. Thus, for large time, the restoring force will have [As t becomes larger the spring constant increases; that is, the spring is stiffening, It would seem ‘that the oscillations would become periodic and the spring would oscillate more rapidly. It is lkely ‘that the amplitudes of the oscillations would decrease as t increases, (a) above (b) heading upward (a) below (b) from rest (a) below (b) heading upward (a) above (b) heading downward Boom |" +2’ + 2r = 0, 2(0) Bev = 16te If x = 0 then displacement is 2 = e feet =1, and 2/(0) = 8 we obtain 2 = dte~t —e- ond 2! = = 1/4 second. If 2" = 0 then ¢ = 1/2 second and the extreme 196 22. 23, 25. 26. a7. Exercises 5.1 From 2" + V2.2! + 22 = 0, 2(0) = 0, and 2'(0) = 5 we obtain x = Ste" and a = be-V7 (1 —2V3t). If 2 = 0 then t = V2/4 second and the extreme displacement is VB en¥/ feet. ia) From 2" 4.102" + 16r ), 2(0) = 1, and 2'(0) = 0 we obtain x = $e~™ a (b) From 2" +.2!+ 162 = 0, 2(0) 12 then 2 = ~$e and 2!(0) (a) 2 = Jer (4e8 1) is never zero; the extreme displacement is ¢(0) = 1 meter. (b) 2 = Jo (5~2e%) = 0 when + = Jn§ ~ 0.153 seconds if a” = fer" (e— 10) = 0 then th fln10 = 0.384 second and the exzeme displacement is 2 = ~0.282 meter (a) From 0.12" + 0.42! + 22 = 0, 2(0) = —1, and 2/(0) = 0 we obtain 2 [cost - sins] -a¥5 [2 2 () roe [Feo Bet sin(te + 428) (©) He =0 then at 42 so thatthe fst time heading upward is = 1.294 seconds (@) From fe" 42/4 5e=0, x(0)=1/2, and 2(0) =1 we obtain z= e-* (} osst +} sind) . (2 Fans) -> (©) Wz = Othen 4¢44r/4-= x, 2r, Sn, ... so thatthe times heading downward are ¢ = (7++8n)x/16 for n= 0,1,2, (a) From §2"4 6a"+5x = 0 wefind that the roots of the auxiliary equation are m = ~$a§ 4g? — 25 (a) 16.46? - 25 > 0 then > 5/2. (b) 1846? - 25 =0 then 6 = 5/2. (©) 1f46? 25 < 0 then 0 <8 < 5/2. 197 28. 29. 30. a1 32. 33. Exercises 5.1 From 0.752" + do! +62 = 0 and @ > 3V2 we find that the roots of the auxiliary equation are 90 + 9/8? — 18 and poet le cosh $/®—W8t + casitn 3/5 — 184] If (0) = 0 and 2'(0) = ~2 then o = 0 and oy = ~3//?— 18. If jo" + fo! + 62 = 10cos3t, 2(0) = —2, and 2(0) =0 then vi seasin Ft) and 2p = '(cos St + sin3t) so that the equation of motion is Cenc 10, resin Set) + Oicosae + sin’ ante ee) + Pleat ten (a) Ifa"+20'+52 = 12008 2+ 3sin 2t, 2(0) = 1, and (0) = 5 then 2: = and 2p = 3sin 2t so that the equation of motion is >~*{c1 cos 2¢-+02 sin 2t) \ Hceanaient From 2” + 8x + 16r = Ssindt, 2(0) = 0, and 2/(0) = 0 we obtain 2. = ce“ + cate and p= —}cosdt so that the equation of motion is eae Get tte — Fost From 2" + 82' + 16z = e“tsin4t, 2(0) = 0, and 2'(0) = 0 we obtain ze = cye~* + opte and Bhertoosdt — gBgetsin dt so that oo oe As ¢ 00 the displacement z ~ 0 From 2z" + $22 = 68e~% cost, 2(0) = 0, and 2'(0) = 0 we obtain ze = e1cos4t + cnsindt and fe7* cos dt — 2e“™' sin At so that 4 = Lert(ateos: it 2 (24 +1001) ~ sLeer*(Qdcosdt + 7sindt) 1 9 Lea 2 poosat + Toinat + 56° cost — 26% sin 198 34, 35. 36. 37. 38. 39. Exercises 5.1 Since « = 8 sin(4t —0.219) — Xfhe- sin(4t — 2.897), the amplitude approaches VB5/4 as t ~ oo. (a) By Hooke's law the external force is F(t)= kh(t) so that mz" + fix! + kx = kh(t), (b) Brom Jo"4-2r'+42 = 20cost, 2(0) = 0, and 2/(0) = 0 we obtain xe and zp = cost + Msint so that (-Seor Zant) + Boone = %(c; cos t-+epsin 2) & (a) From 1002" + 16002 = 1600 sin 8t, (0) = 0, and 2/(0) = 0 we obtain 2. = cy cost + en sin 4t and zp = sin 8¢ 90 that 1 2 = eindt— 5 sing. 41 5 sing 3 (b) Iz = }sin dt(2— 2c084#) = 0 then t = nn/4 for n= 0,1,2, (6) Ita! = §.cos4t—§ cos 8t = §(1—cos.t)(1+2cost) = O then f = #/8:4nn/2endt = 1/6-+nn/2 for m= 0, 1, 2,...at the extreme values. Note: There ere many other values of ¢ for which 220. (A) 2(/6 + 26/2) = V8/2 em. and 2(r/3 +nx/2) e) + - Viren, From 2" + 4y = —Ssin2¢ + 3cos2t, x(0) = -1, and 2!(0) = 1 we obtain r- = cy cos? + cysin 2t, Htsin2e + §teos2t, and 1 3 5 Bm — cost ~ 5 sin + Ftsin2t + Ftcos2e From 2" 492 = Ssin3t, 2(0) = 2, and 2/(0) = 0 we obtain xe = 1 cos 3t-+ en in 3, zp = ~ftcos3 and 2 5 = Doostt-+ F singe — Fteost (a) From s" +o = Focost, 20) = 0, and 2/(0) = 0 we obtain ze = cy coswt + epsinut and (Fo cos 7t)/ (u? — 17) so that Exercises 5.1 Fy ., —Fotsinyt — Fo 2) my ya (cos 7 cost) = Jy PESO 8 Foes () oy SpE eos ~ cost) = Ji, RPT = Pest 40. From 2” +w%r = Fycosut, 2(0) = 0, and 2/(0) = 0 we obtain 2, = oy cosut + crsinut and fa B Zp = (Fot/2u) sinuit so that 2 = (Fot/2u)sinust and jim, gotsinut = Feesinut 41, (a) From cos(ut — v) = cosucose + sinusiny and cos(ti + v) = cosweosy ~ sinusiny we obtain sinusiny = }[cos(u — wv) — cos(u + v)]. Letting u = $(y—w)t and » = Hy +u)t, the result follows (b) We = 4(7—w) then 7 =w so that 2 = (Fo/2ey) sinet sin yt 42, See the article "Distinguished Oscillations of a Forced Harmonic Oscillator” by TG. Procter in The College Mathematics Journal, March, 1995. In this article the author illustrates that for Fy = 1, 2 = 001, 7 = 22/9, and w = 2 the system exhibits beats oscillations on the interval [0, 9x], but that this phenomenon is transient as t ~ oo 43. (a) The general solution ofthe homogeneous equation is a(t) = e1e-™ cos( i? — AP) + exe™* sin = Ae sin( Yi Pt + 4), where A = Yofrq, sind = ex/A, and cos = on/A. Now Fal? — Fo(-2: alt) = ote sinnt op G cnt = Asin +0, where me) Fa(-209) Bayar r jaa Pea Fu? =) cond = = PPS OP £ Vata eae (aaa 200 sind = and wt-9/ Exercises 5.1 (6) Ue gy) = 0 then > (72 +29? a?) w 0 50 that 7 = 0 or y= VaTH OM, The frst derivative test shows thet g has # maximum value at y= Vu? — 207. The maximum value of g is 9 (Yat 28) = Ryan (©) We identify u? = k/m =4, \-= 8/2, andy = VP OM = JT B72. As BO, 2 and the resonance curve grows without bound at “1 = 2. That is, the system approaches pure g resonance. 21a B00] Tar] 0.58 aloo | 3187 | 203 Orr} 1193 | 1.36 0:80 | 1197 | 2.02 0:25 | 399 | 402 44, (a) For n= 2, sin? yt = }(0 ~ cos2yf). The system is in pure resonance when 21/2 = w/2n, ot when 7 = 0/2 (b) Note that Sih data = tine — sn tessa ca sin(A + B) + sin(A — B) = 2sin AcosB snrtecett = Handy —sn . ee pee] os Oe ee 2 sate Sent 1 snare ‘The frequency of free vibration is w/2x. Thus, when /2e = w/2n or 4; = w, and when Sra/2r = w/2r oF Iya =u oF m9 = w/3, the system wil be in pure resonance 201 45, a. 49. Exercises 5.1 oO: wea x vel x as 4 ad V Solving dy" +2 + 1009 = 0 we obtain qt) = ¢~**(y cos 40 + cin 4. ‘The intial conditions (0) = 5 and q(0) =O imply cx = 5 and cy = 5/2. Thus alt) =e (Scosd0t + 3 sinagt) = 25+ 25/4e°™ sin(4or + 1.1071) and 4(0.01) = 45676 coulombs. The charge is zero for the first time when 40¢ + 0.4636 = = or # % 0.0500 second Solving 4g" + 207 + 300g = 0 we obtain a(t) = c1e-™* + ene, The intial conditions a(0) and g'(0) imply cy = 6 and cz = ~2, Thus aft) = 66-2 — 20e wwe find e4% = 1/3 which implies ¢ < 0. Therefore the charge is never 0 (cy cos 3¢-+cesin 3t)+10. The initial conditions Setting Solving fa" +104’ +309 = 300 we obtaia alt) (0) = 4'(0) = 0 imply cy 10. Thus a(t] = 10 10e~*(eos3t + sin3e} and alt) = 60e% sin Solving é(¢) = 0 we see that the maximum charge occurs when t = x/3 and g(n/3) ~ 10.432. Solving q" + 100q' + 2500q = 30 we obtain q(t) = cre" + cate" + 0,012, The initial conditions (0) = 0 and (0) = 2 imply cy = ~0.012 and oy = 1.4. Thus a(t) = 0122 + 1.4te-™* + 0.012 and ft) = 2e®" — OLE Solving it) = 0 we see that the maximum charge occurs when t = 1/35 and q(1/35) ~ 0.01871 Solving ¢" + 2q' +4q = 0 we obtain ye = e~" (cos V3t + sin V3t), The steady-state charge has the form yp = Acost-+ Bsint, Substituting into the diferential equation we find (34 +28) cost + (3B — 24) sint = 50e0st. Thus, A= 150/13 and B= 100/13. The steady-state charge is 150, , 100 sult) = eose+ Maine auc the steady-state cute is 50 g(t) = 252 sine + 8 cone 13 3 202 Exercises 5.1 50. From ig) = 3 (Reine -% cosrt) ZAZ and Z = VX7F FF we see that the amplitude of i(¢) is [EgR BX? _ Eo 51. The differentia! equation is $q"-+20y'+ 10009 = 100sint. ‘To use Example 10 in the text we identify Eq = 100 and = 60. Then Ta 1 Xela = 9) - TapIT6o 2 Z = X24 FE = [x2 + 400 = 24.0370, v Eq _ 100 Bap x s160.. = 13.3833, and From Problem 80, then ‘g(t) = 4.1603(60e + 6) where sin §= —X/Z and cos = R/2Z. Thus tan d= —X/R m ~0.6667 and ¢ is a fourth quadrant angle, Now ¢ * ~0.5880 and p(t) = 4.1603(60t — 0.5880). 52, Solving $q" + 209’ + 1000q = 0 we obtain ge(t) = (cx cos 40% + cp sin 40t). ‘The steady-state charge has the form gp(t) = Asin 60t + Bcos 60t + Csin 40t + Deos Ut, Substituting into the differential equation we find (—16004 ~ 24008) sin 60t + (2400A — 16002) cos 60 4. (400¢ ~ 1600) sin 40¢ + (1600€ + 400) cos dct = 20 sin 6 +400 0840. Equating coeficients we obtain A = -1/26, B = ~3/52, © = 4/17, and D = 1/17. The steady- state charge is 1 3 L wlt)= 2 cnoar ~ Seosear + Ssinaae+ 3 cosant and the steady-state curren i 0 4 0 sp(t) = ~Fj cos 60 + F sin Got + > cos 404 ~ sn 4 203 53, 54. 55. 56. 87. Exercises 5.1 Solving "+ 10g + 100g = 160 we obtain gt) = e™(o cos 10¢ + cxsin Ot) + 3/2 The initial ‘conditions g(0) = 1 and q/(0) = 0 imply ¢ 1/2. Thus: tia fee rsa +3 Ast — 00, glt) —* 3/2. By Problem 50 the amplitude of the steady-state current is Eo/Z, where Z = VXTFRE and X = Ly=1/Cy. Since By is constant the amplitude will be @ maximum when Z is @ minimum. Since Ris constant, Z will be a minimum when X = 0, Solving Ly — 1/Cy = 0 for 7 we obtain += UVEG. The maximum amplitude will be o/R. By Problem 50 the amplitude of the steady-state current is Fy/Z, where Z = VXTFRE and X = Ly~1/Cr. Since By is constant the amplitude will be a maximum when Z is a minimum. Since Ris constant, Z will be a minimum when X = 0, Solving Ly — 1/Cy = for C we obtain C= 1/Ly?. Solving 0.19" + 107 = 100sin-y¢ we obtain g(t) = 1 c0810¢ + opsin 10¢ + gp(t) where gp(t) = Asin é + Beos yt. Substituting dp(t) into the differential equation we find (100 —4"}Asin yt + (100 — 7?) B cos »¢ = 100sin 4. 100 ‘coefficients we obtain A = 7) and B= 1S, = ‘sin 1 Equating coeficionts we obtain A = 100/(100~ 9?) and B = 0. Thus, apt) = spon st Th initial conditions q(0) = ¢(0) = 0 imply cy = 0 and c2 = ~10y/(100 ~ 32). The charge is, 10 4) = spqergaltosin tt ~ sin 108) land the carrent is 1007 8 = sppaleos cose) In an L-C seties circuit there is no resistor, so the difeential equation is a 1 Git gon 20. ‘Then a(t) = 1 €08(t//EC)-+casin (t/VEC) rapt) where alt) = Asinrt+ Beast, Substitating p(t) into the diferensil equation we find L (m1?) asiare + (2-177) Deore = Foot Equating coefficients we obtain A =0 and B = EyC/(1— LC), Thus, the charge is +epsin Tz t+ Fee 204 att) cost Exercises 5.2 ‘The initial conditions 9(0) = qo and q/(0) = ia imply cy = go — EyC/(1 ~ LO?) and ep = iV EC. ‘The current is Exe toe = Whe sin Tt + 2 cos sin = —Tre" yre'* Tie Vie 7 o ae Vie ie \~ t= te?) Ste Y= te 58. When the circuit is in resonance the form of gp(t) is gp(t) = Atos kt-+ Btsin kt where k = 1/V/EC. ‘Substituting gp(t) into the differential equation we find f+ gm ~2kA sin kt + 2B eoskt = 7 coskt Bqueting coefcients we obtain A= 0 and B = Ey/2kL. The charge is alt) = cost + epsinkt + 22 teint ‘The initial conditions g(0) = go and q'(0) = iy imply c1 = qo and cp = io/k. The current is Py aL" i(t) = ~eaksin kt + ogk cos kt + 2% (kt cos kt + sin kt) % Ze ~t)snte ious Bent Exercises 5.2 1. (a) The general solution is va) <1 topes at sae + 5B! ‘The boundary conditions are y(0) = 0, ¥/(0) = 0, y"(L) = ‘conditions give ey , VME) = 0. The fist two ‘and c= 0. The conditions at z= L give the system we eq + Geol + SEU woe 6a + EL = 0. Solving, we obtain ¢3 = uoL2/4EI and cx tL /6EI. The defection is 2) = UO (6122? — 40a + 2 (a) = SEB (Oe? — abs? + 205 Exercises 5.2 (b) waoe oe a* : \ 2, (a) The general solution is ttt, ET , y(L) = 0, 9"(L) = 0. The fist two conditions sive the system Wr) =e, tee teat? tear + ‘The boundary conditions se y(0) = 0, y"0) sive cj =O and cy = 0. The conditions at 2 f4 MO ps cob + el? + sel wo 72 bab + Rt Solving, we obtain ¢ = wpL3/24ET and cy = —upL/12B1. The deflection is (b) Ue ate 4 5M va) = SEB ADs — 228 + 4). 3, (a) The general solution is oa RET ‘The boundary conditions are y(0) = 0, y(0) = 0, y(L) = 0, y"(L) = 0. The first two conditions. and cp =0. The conditions at x = L give the system, we) =o tere te + ao + cal? gl + 5a 13 + Beal + 2? Dey + Geub + pL 206 Exercises 5.2 Solving, we obtain ¢ = uoL?/16ET and cy = —SupL/48B1. The deflection is a) = iB (onte ~ 512+ 23h y 4. (a) ‘The general solution is ‘ ws on ula) = e+ eaz + ea? + cg? + FI sin Fa, ‘The boundary conditions are y(0) = 0, (0) = 0, y(L) = 0, v"(L) = 0. The frst two conditions L give the system 2+ 6e4b = 0. Solving, we obtain ¢j = 3uoL?/2EIx3 and cy = —woL/2ETn3, The deflection is = th, (-avte aust 2+ Bante vio) = Bt (ait ste 8+ Ean (b) TATETS (c) Using a CAS we find the maximum defiection to be 0.270806 when 2 = 0.572536, 5. (a) The general solution is (a) ser teetes? teat + ot st we) =e taster +cat + aoe 207 Exercises 5.2 ‘The boundary conditions are y(0) = 0, y"(0) = 0, x(L) = 0, y"(L) = 0. The fist two conditions sive: = 0 and cy =0. The conditions at x = L give the system eb tele+ Mo ps Gosh + el = ~upL2/36EI. The deflection is Solving, we obtain cy = Tuol/360B7 and cx we) = (tebe ~ 00% + 35) (b) {c) Using a CAS we find the maximum deflection to be 0.234799 when x = 0.51933. ‘woL4 6. (a) tm = Vil) = Ser (b) Replacing both Land z by L/2 in y(2) we obtain woL4/128E7, which is 1/16 of the maximum deflection when the length of the beam is £. . Sunlt (©) hoax = WE /2) = Fo (d) ‘The maximum defection in Example Lis y(L/2) = (wo/24EV)L4/16 = woL*/S84E, which is 1/5 of the maximum displacement of the beam in Problem 2. 7. The general solution of the differential equation is [Pm gt, WEL ya eonty Eas epson Fae BB wel Setning y(0) = 0 we obtain et = —woE/P2, so that y= EL coat \E seas (E Setting y/(L) =0 we find a (Ban 208 Exercises 5.2 8. The general solution of the differential equation is P yo) 008: = EL og fF yen pr VET Setting y(E) = 0 we find _( Fett, fF ,_t\ / fF, om \ Vere Vere) f VET VET 9. For A. 0 the only solution of the boundary-value problem is =0. For A> 0 we have ya cos Vis-+epsin Vi Now (0) = 0 implies c= 0, 0 Ws) = sia Vie 0 gine Vinenn or A=n?,n=1,2,3, The eigenvalues n? cotrespond to the eigenfunctions sin nz for n = 1, 2, 3, 10, For 1.<0 the only solution ofthe boundary-value problem is y = 0. For A> O we have y= cos Viz ein Vie. Now 0) = 0 implies = 0, 0 1 (2) =asnvazwo ives mor A=I6n?, n=1,2,3, ae =1.23, 0, For 2> Owe have ‘The eigenvalues 16n? correspond to the eigenfunctions sin 4nz for LL. For 4 <0 the only solution of the boundary-value problem is y= qcosViz+ osiavaz, (a) = ~0.VF sin Vie + eaV cos Vz implies ¢) = 0, so iL) = eyeos VAL = 0 209 12. 13. a. Exercises 5.2 ives Vira CRUE gy z n= UR ioe The eigenvalues (2n ~ 1/8/41? correspond to the eigenfunctions cas P=. nai 3, For A $0 the only solution of the boundary-value problem is y = 0. For A > 0 we have ya cveos Vie +azsin Viz Now y(0) = 0 implies e: Vre0s VAS ives or As(2n-1,n ‘The eigenvalues (2n — 1)? correspond to the eigenfunctions sin(2n ~ 1)z For \ < Othe only solution of the boundary-value problem is y= 0. Fox 4=O we have y = 12402 Now y/ = ey and y/(0) = 0 implies cy = 0. Then y= o2 and y/(r) = 0. ‘Thus, = 0 is an eigenvalue ‘with corresponding eigenfunction y For \ > 0 we have creas VAz +easin Vx. Now (2) = ~oVKsin Viz + caV Recs VT and 1/(0) = 0 implies c2 = 0, so ives n=1,2,3, 2 0,1,2, For) > 0 we have ‘The eigenvalues 1 For \ <0 the only solution of the boundary-value problem is y= ercos Viz + eosin Vaz. correspond to the eigenfunctions cose far n 0 implies 1008 Vix ~eysin VE = 0 @ 008 VA +opsin Vax 0. 210 15. 16. Exercises 5.2 ‘This homogeneous system will have a nontrivial solution when cos Vix =sinvIe a eae 2sin Vir cos Vie = sin2VAn = 0. Then 2h When n = 2k ~1 is odd, the eigenvalues are (2k — 1}2/4, Since cos(2k ~ 1)r/2= 0 and sin(2k ~ 1)r/2 # 0, we see from either equation in (1) that cz = 0. Thus, the eigenfunctions corresponding to the eigenvalues (2k ~ 1)8/4 are y = oos(2k ~ 1)2/2 for k = 1, 2, 3,.... Similarly, = 2k is even, the eigenvalues are k? with corresponding eigenfunctions y = sin kx for 2,3, ‘The auxiliary equation has solutions 1.2.3, sar or when 24 (2-447) = 14 V=%, -* (cy cosh V=Rz + exsinh V=Kz) For \< 0 we have ‘The boundary conditions imply 0) = (8) = ee sinh5V=K = 0 2 = 0 and the only solution of the boundary-value problem is ya + gee and the only solution of the boundary-value problem is y = 0. For A> 0 we have (ey cos Viz + epsin V2) Now y(0) = 0 implies ex = 0, s0 (5) = exe" sinVI = 0 aa iq? SVimene or he rani ‘The eigenvalues n?x?/25 correspond to the eigenfunctions For 4 < —1 the only solution of the boundary-value problem is y = 0. For A= -1 we have ysciz+ex, Nowy =e, and y'(0) =0 implies ¢ = 0. Then y= cz and (1) = 0. Thus, A= -1 isan eigenvalue with corresponding eigenfunction form =1,2,3, 1 2 ar. 18. 1s. Exercises 5.2 For \> ~1 we.have y= eyeos VIF Is + exsin VEG Ie. Now y= no/T4F Tsin VIF Te + VN eT 008 VET and y/(0) = 0 implies ca = ¥(Q) = -avi¥Isin VIF =0 gives ViFIsne oe X= 0? 1, 021,23, The eigenvalues n?x? — 1 correspond to the eigenfunctions cosnmz for n= 0,1, 2 For A= 0 the only solution of the boundary-value problem is y = 0. For A 0 we have y= cy 008 Az + ensin Az Now y(0) = 0 implies cy = y(L) = eosin AL = 0 sis : Abana oe AaB, ne .23, The gels n/ corespend to the igefunctions sn x em =, 2,% For A= O he ony solution ofthe boundary pele is y=. Fr 30 we have y Now (0) = 0 imps = 0,50 ¥'(3n) ors Qa=te gy etal 2 a. ‘The eigenvalues (2n ~ 1)/6 correspond to the eigenfunctions sin ™™="s for n= 1,2,% For A> 0a general solution ofthe given differential equation is y= crcos(VX inz) + ogsin(V3 In2) Since in = 0, the boundary condition y(1) = 0 implies =. Therefore casin(V3 In). 1 we find that y(e") = 0 implies asin Vie 212 Exercises 5.2 or Vin =nn,n=1,2,3,.... The eigenvalues and eigenfunctions are, in turn, de m1 23, andy =sin(ning) For \ <0 the only solution of the boundary-value problem is y = 0. ‘To obtain the seadjoint frm we note thatthe integrating factor i (1/2")el#/* = 1/x, ‘That i, the se-sdjint form is ce hevittv= Identifying the weight function p(e) = 1/z we can then write the orthogonality relation ff Aswinina)sin(mina) de =0, man 20. For A= O the general solution is y = e1 + ezlnz. Now yf = ca/2, $0 o/(e™ ‘Then y= cy and y(1) = 0 gives c= 0. Thus (2) = For <0, y = qa-Y* + eg2V=4, The initial conditions give ¢ = c1e*V—* and ¢, = 0, 50 that y= O and y(z) = For A> 0, y= ey c0s(V/X Inz) + epsin(VE Inz). From (1) = 0-#e obtain ¢, = 0 and y = cgsin(VX Inz). Now 9! = co(VA/z)cos( VA Inz), so y/(e“!) = exeVXcos VX = 0 implies cos VI-= 0 or A= (2n~1)?2/4 for n= 1,2, 3,.... The corresponding eigenfunctions are 2 yesin (2 tntnz) 21, For \ = 0 the general solution is y = cr +calnz. Now y' exe = 0 implies eo/, 0 y'(1) = @ =O and y = ey Since y'(e*) = 0 for any cy we see that y(z) = 1 is an eigenfunction corresponding to the eigenvalue v=o. Fer <0, y= cy2-¥* + eV, The initial conditions imply ¢ = a = 0. s0 y(z For A> 0, y = ¢1 c0s(VX Inz) + o9sin( V3 Inz). Now / cB sinVK ina) + 62 coi V5 tn, and y/(1) = covX = 0 implies cp = 0. Finally, o/(e?) = —(crVT/e2)sin(2V%) = 0 implies A= ntn?/4 for n= 4,2, 3, .... The corresponding eigenfunctions are —) 22, For A> 1/4 a general solution of the given differential equation is ee ‘= soz) tenon ( 213 Exercises 5.2 Using Ine? == 2 we find that y(e2) = 0 implies cxe"tsin (VAN=T) =0 1 2, 3, ‘The eigenvalues and eigenfunctions are, in turn, nz) or VIK= (P41), 22 1,23,.0 and pee lain Por 2 <0 the only solution ofthe boundary-value problem is y = 0. For A= 1/4 a general solution of the differential equation is ee + ge Ming, From y(l) = 0 we obtain e; = 0, 90.y = cx2"V#lnz. From y(e®) = 0 we obtain 22ye~ cz = 0. Thus, there are no eigenvalues and eigenfunctions in this case ‘To obtain the selhadjoint form we note that the integrating Factor is (ifs pel/0)42 = (1/2%) 28 = 1, ‘That the slain form i a /] Zfev|+weo. [dentfyng the weight function pe) = 1 we can then write the orthogonality relation ff 1-2 ¥in (Eta) 2 sin (tna) dr= men, [f ssn (SZinz)sa (Mtns) aro, mon 23, If restraints are put on the column at z = £/4,2 = L/2, and x =3L/4, then the hx crcl oad wil be ) (24, (a) The general solution of the differential equation is x iP et yacoefoetasefEer 214 Exercises 5.2 ‘Since the column is embedded at this implies thet ¢ O and v(z) (b) 165 0, the initial conditions give, in turn, ey = 6 and cp = 0. Then : v=1{i-en/) : * ws(i-mft) = mefBe This gives \/P/EI L = nt/2 for n= 1, 2,8,.... The smallest value of Pa, the Buler load, is ‘then , the initial conditions are y(0) = y/(0) = 0. If 0, That is, there is no deflection Bla 7 Owe find put/PL = nr, n= 1,2,3,.... Thus, . 2,3, ... . The corresponding deflection curves are faa 7 (2) 25. The general solution is v From y(0) = O-we obtain ¢) = 0. Seting u( critical speeds are uy = na VT/Lyp.n = ua)= eosin 2, n= 1,2,3,-..5 where cr # 0. 26, (a) When T(2) = 2? the given differential equation is the Cauchy-Buler equation ty! + 22y! + pu®y = 0. ‘The solutions of the auxiliary equation m(m—1) +2m+ pu? =m? + m+ pu? =0 when pu? > 0.25, Thus y= ex" cos(Alnz) + oor? sin(AInz) where X= po? 1/2, Applying y(t) = 0 gives e y= gr? sin(QInz). 0 and consequently, 215 Exercises 5.2 ‘The condition y(e) = O requires qe~!/2sin\ = 0. We obtain a nontrivial solution when Aner m= 12,5). But da = ane 1/2= ne Solving for uy gives win = 5 Vana? + Do ‘The coreesponding solutions are alt) = e207 ¥? sin(na Ina) i m(m +1) = 0 s0 that ulr) = ert + cp, The boundary 1 yield the syster cya! +0) = tp, 18°! ben = tu, Selving gives 27, The auxiliary equation is m? +m conditions u(a) = vo and u(b) = ‘Thus a 28. The auxiliary equation is m? =0 so that u(r) = ¢ +ezInr. ‘The boundary conditions u(a) = up and u(b) = uy yield the system cy +cz!na = up, ¢1 + e2lnd = w1. Solving gives ulna wind w= aay 4 2 Ine7ay Thus fe) = lta = woIMb vans, olntr/t)— uy infr/a) In(aje) Tnfaje) ™ Infa/b) 7 29. (a) ‘The general solution ofthe differential equation is y = ¢1 cosd4z-+cpsin4z. From y= (0) =e wwe see that y = yo.cosdz + opsindz. From v1 = y(/2) = up we see that any solution must satisfy yo = a1. We also see that when yo = a1, y = 9ocos 4z + cesin dz Is a solution of the boundary-value problem for any choice of cz. Thus, the boundary-value problem does not have ‘ unique solution for any choice of yy and yt (b) Whenever yo = 1 there are infinitely many solutions. (e) When yo # ui there will be no solutions. 216 Exercises 5.2 (4) The boundary-value problem will have the trivial solution when yp = a1 = 0. This solution will not be unique. 30. (a) The general solution of the differential equation is y wwe see that y = cosdr +epsindz, From 1 = y(L) (1 ~cos4L)/sin4Z. Thus, 1 cos4r-+epsin dz. From 1 = y(0) = cos4L + epsin4L we see that 6p CSEe) ante will be a unique solution when sin 4 #0; that is, when L ¢ k/4 where k= 1, 2,3, (b) There will be infinitely many solutions when sin4Z = 0 and 1 ~ cos = 0; that is, when L = bx/2 where k= 1, 2,3, (c) There will be no solation when sina 0 and 1 ~ cosa Or tha i, when L = kr/A where F=1,3,5, (a) There can be no trivial solution since it would fail to satisfy the boundary conditions. 31. (a) A solution curve has the same y-coordinate at both ends ofthe interval [-n, x] and the tangent lines atthe endpoint ofthe interval are parallel (b) For A= 0 the solution of y” is y= ciz + cp. From the first boundary condition we have ul-n) = arn #2 = y(n) = ern ben or 2eyr = 0. Thus, 1 = 0 and y = cp. This constant solution is seen to satisfy the boundary- value problern. For ) <0 we have y = c; cosh Ar+ cp sink Ar. In this case the first boundary condition gives =n} = ex cosh(—An) + cpsinh(—An) = ycosh Ax ~ cp sinh kr = vl) = er cosh Ar + cysink Aw (or 2egsinh Aw = 0. Thus ¢2 = 0 and y = ¢ cosh Ar. The vecond boundary condition implies in 2 similar fashion that cy = 0. Thus, for \ <0, the only solution of the boundary-value problem isy=0. For > 0 we have y= cj cos Az + cpsin Az, The first boundary condition implies ul—n) = c1 co6(—An) + epsin(—An) 1608 Ar — epsin de Exercises 5.2 (ce) 32, (a) (b) (©) or 2epsin Ar = 0. Similarly, the second boundary condition implies 2q)Asin Aw = 0. If ey = eq = 0 the solution is y = 0. However, if cr 4 0 or o2 # 0, then sin dx =O, which implies that 4 must be an integer, n. Therefore, for cy and ce not both 0, y = cy cosnr+-ep inne isa nontrivial solution of the boundary-value problem. Since cos(—nz) = cosnz and sin(—nz} ‘we may assume without loss of generality that the eigenvalues are dy = Integer. The corresponding eigenfunctions are y, = cosni and y, = sinnz. for n a positive y=2sinde y= sinds ~ 2c0832 For A > 0 the general solution is y = c cos VXz +c sin YXz. Setting y(0) = 0 we find c = 0, s0 that y = ensin Viz. The boundary condition y(1) + (1) = 0 implies eatin VI + oVe08 VI = 0. ‘Taking co # 0, this equation is equivalent to tan VX = —V%. Thus, the eigenvalues are Dn = 22, n= 1, 2,3,..., where the 2 are the consecutive positive roots of tan VX = —V3. We see from the graph that tana = ~z has infinitely many roots. Since Jy = 23, there are no new eigenvalues when tq <0. For =D, the differential equation y” = 0 bas general salution y = oyr-+e2. ‘The boundary conditions imply 41250, s0y=0. Using a CAS we find that the first four nonnegative roots of tan = —r are approximately 2.02876, 4.91318, 7.97867, and 11.0855. The corresponding eigenvalues are 4.11586, 24.1398, 63.6591, and 122.889, with eigenfunctions sin(2.02876z),sin(4.913182),sin(797867=), and sin(11.08552) PSC Exercises 5.3 Exercises 5.3 ——___ 1, (0) = 1 is approxi x 2 12 2'(0 1, The period corresponding to 2(0) mately 8.6. The period corresponding to z(0) is approximately 6.2 2. The solutions are not periodic 3. The period corresponding to 2(0) = 1, 2/(0) = 1 is approx- imawly 5.8 The second initial-value problem does not have 2 petiodic solution. 4, Both solutions have periods of approximately 6.3 219 Exercises 5.3 5. From the graph we see that jeu| 6, From the graphs we see that the interval is approximately (-08,1.1) 7. Since 2091 w afl 400s + Z(0012)2 + Je Pe for small values of z, a linearization is 5 +2 = 0, = 1 the oscillations are symmetric about the line x = 0 with amplitude For 2(0) = 1 and 2/0) slightly greater than 1 For 2(0) = -2 and 2/(0) = 0.5 the oscillations are symmetric about the line x = —2 with small amplitude. For 2(0) = V3 and 2'(0) = 1 the osilations are symmetric sbout the line = ~0 with amplitude a 220 Exercises 5.3 Tittle greater than 2. For 2(0) = 2 and x'(0) = 0.5 the oscilletions are symmetric about the line x = 2 with small amplitude. For 2(0) = ~2 and 2'(0) =0 there is no oscillation; the solution is constant. For 2(0) = —v/2 and 2'(0) = ~1 the oscillations are symmetric about the line x = 0 with amplitude a little greater than 2. 9, This is a damped hard spring, so all solutions should be oscillae xc tory with 2 0 as t+ 00, Hl 10. This is a damped soft spring, so so we expect no oscillatory. solutions 221 Exercises 5.3 Pre ke 15) -aq as a x ki = 20 x ki = 100 Fy FT | F y y J AVE we 7 5 -q = ‘When ky is very small the effect of the nonlinearity is greatly diminished, and the system is close ‘ pure resonance. 12. (a) jx x ~ Lalhgattat —- vee k= -0.o00t7t ve = 000872 ‘The system appears to be oscillatory for ~0.000871 < ky <0 and nonoscillatory for () yz x k= -0.079 k= 0.078 ‘The system appears to be oscillatory for ~0.077 < ky < 0 and nonoscillatory for ky < 0.078, 222 Exercises 5.3 13, For 32 =u? > 0 we choose A= 2 and w = I with (0) = 1 and 2'(0) = 2, For 2? ~ u? < 0 we choose = 1/3 and w = 1 with 2(0) = ~2 and 2/(0) = 4, In both cases the motion corresponds to the over~ amped and underdamped cases for spring/mass systems, 14. (a) Settiog dy/dt = v, the differential equation in (13) becomes dv/dt = —gR?/y?. But, by the chain rule, du/it = (du/dy)(dy/dt) = vdu/dt, so vdu/dy = ~9R®/y?. Separating variables and integrating we obtain dy 2 vdv= oR and Setting v= vp and y= R we ind ¢ (0) As y— 00 we assume that v0", Then tf = 26R and v9 = VIR. (0) Using 9 = 32 e/s and R= 4000(5280)f we fad to = /2(82) (4000) (E280) ~ 967652 ft/s = 25067 mir (A) w = 2(0-165)(92)(0080) = 7760 ft/s = 5291 mi/hr 15. (a) Intuitively, ome might expect that only half of a 10-pound chain could be lifted by a 5-pound force. (b) Since 2 = 0 when f= 0, and v = de/dt = /160— 64z/5, we have e(0} (©) Since z should always be positive, we solve 2(t) = 0, VIGO 12.65 ft/s. setting t [52 = 2.3717. Since the graph of x(t) is a parabola, the maximum value occurs and t at im = 25/2. (This can also be obtained by solving P(t) = 0.) At this time the height of the chain is (tm) © 7.5 ft. This is higher than predicted because of the momentum generated by the force. When the chain is 5 feet high it still has a positive velocity of about 7.3 ft/s, which keeps it going higher for a while. 16. (a} Setting dr/dt = v, the differential equation becomes (L~z)dv/dt~0 = Lg. But, by the chain rule, do/dt = (du/dz)(ds/dt) = vdv/de, so (L~ 2)vdv/dz ~ v* = Lg. Separating variables 223, Exercises 5.3 and integrating we obtain = Lin(o? + Lg) = ~In(L—2) + Ing, rine dz and $in(e? + £9) = ~In(L—2)+ Inc, so fF 4 Lg = o/(L~ 2). When 2 =0, v =0, and c= LYTG. Solving for v and simplifying wwe get or _ sey) _ VaR Cy [Agnin, separating variables and integrating we obtain Loz viz=F dz = dt and atte [igiatz ==) vig : Since 2(0) =0, we have oy = 0 and v2Lz = #/ TG = t. Solving for z we ger x) =L-YP— Ig? and y= #5 (b) The chain will be completely on the ground when z(t) = £ or t= y/E/9 (c) The predicted velocity of the upper end of the chain when it hits the ground is infty 17. (a) The weight of z feet of the chain is 2x, so the corresponding mass is m = 2r/32 = 2/16. The only force acting on the chain is the weight of the portion of the chain hanging over the edge of the platform. Tirus, by Newton’s second law, ate Hm) EU) = and rdv/dt + v? = 322, Now, by the chain rule, du/dé = (du/dz\(dz/de) = vdu/dz, so seudy/éx + v? = 322 (b) We separate variables and write the differential equation as (v? — 322) dr + zvdv = 0, This is not an exact form, but u(x) = 2 isan integrating factor. Multiplying by 2 we get (zo? - 3222) dz + 2"vdv = 0. This form is the total differentia of w= bx%v ~ 3223, so an Implicit solution is $a? ~ %Pz4 =o, Letting x = 3 and v = 0 we find c= ~288, Solving for v we get dr a (0) Seping ach nd neg ein ee fe a va Ss Ve = 27 v3 224 Exercises 5.3 Since 2 = 3 when t = 0, we see that ¢= 0 and ‘We want to find ¢ when z= 7. Using a CAS we find t(7} = 0.576 seconds. 18. (a) There are two forces acting on the chain as it falls from the platform. One is the force due to sgavity on the portion of the chain hanging over the edge of the platform. This is Fj = 2e. The second is due to the motion of the portion of the chain stretched out on the platform. By Newton's second law this is zdv 1 a * 16 (b) Integrating vdv = 4rd we get Ju? = 22?-+e. Since v= Owhen x = 3, we have c= —18. Then Fa 4a? — 36 and u= VigT— 36, Using v = dz/dt, separating variables, and integrating we obiain pte =2dt and cost tbe Solving for 2 we get z(t) = 3cosh(2¢ +1). Since x = 3 when t = 0, we have cosh) = 1 and 10. Thus, a(t) = Scosh2t. Differentiating, we find v(t) = dz/dt = 6sinh2t (©) To find when the back end of the chain will leave the platform we solve 2(t) = 3cosh2t = 8. This gives ty = feosh"!§ ~ 0.8184 seconds. ‘The velocity at this instant is v(ts) = Gsinh(cosh™! §) = 2V55 = 14.83 ft/s 225 Exercises 5.3 (d) Replacing 8 with L and 32 with g in part (a) we have L du/ds = gx. Then 2 Integrating we get Ju? = 22? +c, Setting x = zo and v = 0, we find c= — 223. Solving 2B 2E o(e) = ($0 28 Then the velocity at which the end of the chain leaves the edge of the platform is w(t) = P28) 19. Let (2,y) be the coordinates of Sz on the curve C. The slope at (2,y) is then for v wo find dyide = (uit = y)/(O0-2) =(y—mit}/2 or ay! -y= ut. Differentiating with respect to x and using r = v/v» gives af =n LVF WP) atin = y/ and separating variables, we obtain Letting Abt =0, dy/de =0 and x = a, s0.0 = ca" ~ 1foa", Thus c= 1/a" and zal) -@1-3[G-@] Ifr > Lor r < 1, integrating gives Exercises 5.3 When t= 0, y=0 andr =a, s00= (a/2)[1/(1+r) —1/(1—r)] +1. Thus ey = arf(1— [EO . () |+2 “To see ifthe paths ever intersect we frst note that ifr > 1, then vy > ve and y ~+ oo a8 = ~+ 0* In other words, Sp always lags behind S;. Next. ifr <1, then m <% and y = ar/(1—r?) when = = 0. In other words, when the submarine’s speed is greater than the ships, their paths will intersect at the point (0,ar/(1—r?)). Finally fr =1, then integration gives (1/2)la/2— (1/a) ina] and we] -38 Since y +o as.2—+ 0*, Sp will never catch up with St ). (a) Let (7) denote the polar coordinates of the destroyer Sj, When 5) travels the 6 mies from (6,0) to (3,0) it stands to reason, since Sz travels half as fast as Si, that the polar coordinates ‘of Sp are (8,62), where 62 is unknown. In other words, the distances of the ships from (0,0) are the same and r(t) = 15¢ then gives the radial distance of both ships. ‘Ths is necessary if Si isto intercept $2. (b) The differential of are length in polar coordinates is (ds)? = (rd)? + (dr)2, so that ds)" _,2 (08)? , (ar)? dt at 2 dt Using ds/dt = 30 and dr/dt = 15 then gives do a 1112) 4)int [Ree +ime ; woven (2) sa wo 8 ae H(t) = V3 int-+e= VEIN +e. When r = 3, 8= 0, s0.0= —V3 In(1/5) and at) = v8 (in In) ving 227 Exercises 5.3 (eo) ‘Thus r = 3¢/V5, whose graph is a logarithmic spiral, ‘The time for $i to 40 from (9,0) to (8,0) = § hour. Now Si must intercept the path of Sp for some angle 8, whe t= e9/¥8/5, The total time is then +184 clase) <8 <2n. At the time of interception f2 we have 15t2 = 3229 or 21. Since (dz/dt}* is always positive, it = necessary to use |dx/it|(dx/dt) in order to account for the fact that the motion is oscillatory and the velocity (or its square) should be negative when the spring is contracting, 22, (a) The approximation is accurate to two decimal places for () 23. (a) 1 =0.3, and accurate to one decimal place for 8 = 0.6. aa ‘The thinner curves are solutions of the nonlinear differential equation, while the thicker curves ace solutions of the linear diferental equation. Write the differentia equation as > Fy rePsind =0, where u? = 9/£. To test for differences between the , 9(0) = 1, and (0) = 2. Using g = 32 om the earth and 9 = 5.5 on the moon we obtain the graphs shown in the figure. earth and the moon we take £ Comparing the apparent periods of the graphs, we see that the pendulum oscillates faster on ‘the earth than on the moon. 228 Exercises 5.3 (b) ‘The amplitude is greater on the moon than on the earth (c) The linear model is #8 Fi rsteno, where w? = 9/é. When g = 32, £ and (0) = 2, the general solution is (0) = 1, 9(¢) = cos 3.2664 + 0.612sin3.266 When 9 = 5.5 the general solution is A(t) = cos 1.354 + 1477sin 1.354. |As in the nonlinear case, the pendulum oscillates faster on the earth than on the moon and suill has greater amplitude on the moon. 24. (a) The general solution of @e et en0 is 6(¢) = ercost + cxsint, From 6(0) =7/32 and 6'(0) = 1/3 we find 4{t) = (7/22) cost ~ (1/3) sint Setting @(t) = 0 we have tant = 7/4 which implies t) = tan“1(nr/4) = 0.66577. (b) We set 6(t) = 6(0) + (0}e + J>"(O}t + Jo"(O}E+--- and use H(t) = —sinO(t) together with 9(0} = x/12 and #(0) = -1/3. Then 6"(0) = —sin(x/12) = —V3(v3— V)/4 and °"(0) = ~ cos 6(0) - (0) = ~ cos(x/12)(—1/3) = V2 (v3 + 1)/12. mm VBWV3= 1), VEV340) x1, vBv8-9 Biv3+1) a) =F je- Se ee (6) Setting x/12 — 4/3 = 0 we obtain ty = 7/4 ~ 0.78539, (a) Setting BW5-)) 3 and using the positive root we obtain t) = 0.63088 (e) Setting z_1,_v2W3-) m1, VUVE-1) 2, VED _ 1203 8 oy 72 a) 229 25. Exercises 5.3 wwe find t = 0.661973 to be the first positive root. (8) From the output we see that y(t) is an interpolating function on the interval 0 < ¢ <5, whose graph is shown, The positive raat of y(t) =O near ¢ = 1 is t= 0.666404, (g) To find the next two positive roots we change the interval used in NDSolve and Plot from {20,5} to {1,0,30}. We see from the araph that the second and third positive roots are near 4 and 7, respectively. Replacing (¢,1} in FindRoot with {t,4} and then {ts} we obtain t = 3.84411 and ty = 7.0218. -0.4 -o.4) « od 0. “od From the table below we see that the pendulum frst passes the vertical position between 1.7 and 18 seconds, ‘To refine our estimate of t, we estimate the sclution. of the ifferential equation on (1.7, 1.8] using @ step size of f= 0.01. From the resulting table we see that fy is between 1.76 and LTT seconds. Repeating the process with h ‘the pendulum is approximately 4¢; = 7.068. The error when using fy ‘and the percentage talative error is (0.785 /7.068)100 = 11.1 0.1 e001 te TI] iin | o!oss7q 172 | 0.05424 airs | o!0a275 394 | s.03ta us | o-o193q 1176 | 000755 1177 | -9-00438| ilza | Zo‘o16« ilts | 0/02854 10 | “008076 “OO3R] tinge] 000279} 11765] 0.00169) 11766| 0.00040) 1767] -0:00073 1:768| -0:00199 aises| 0.00320 1770] -0'00438 230 001 we conclude that fy ~ 1.767, Then the period of 2 is 7.068 — 6.283 = 0.785 Chapter 5 Review Exercises Chapter 5 Review Exercises —_________ 1, 8 ft, since & 2. 2/5, since }o" +6252 = 0 3. 5/4 m, since 2 = —cosat + sind 4, True 5. False; since an extemal force may exist. 6. False 7. overdamped 8. From 1(0) = (V2/2)sind = 1/2 we see that sing = -1/VB, s0 ¢ is an angle in the third or fourth quadrant. Since 2/(t) = /cos(2t +4), 2/(0) = V2cos¢ = 1 and cos @ > 0. Thus ¢ is in she fourth quadrant and 6 = 2/4 9. The period of a spring mass system is given by T'= 2n/w where w? = k/m = kg/W, where kis the spring constant, W is the weight of the mass attached to the spring, and a isthe acceleration due ‘to gravity. Thus, the period of oscillation is T = (2n/yFg) VW. If the weight of the original mass is W, then (2n//Eg)VW = 3 and (2n/VEg)VW— 8 = 2. Dividing, we get VW//W—E = 3/2 or W = 4(W ~ 8). Solving for W we find that the weight ofthe original mass was 14.4 pounds. 4 we obtain 10, (a) Solving $x" + 62 = 0 subject to 2(0) = 1 and "(0 2 cosit —sindt = Vin (4t + 36/8) (b) The amplitude is V3, period is »/2, and frequency is 2/n. (c) Ifz=1 then t = nn/2 and t = —n/8 + nm/2 for n = 1, 2, 3, (@) Iz = 0 then ¢ = £/16-+ nn /4 for n= 0,1, 2,... The motion is upward for n even and downward for n.odd. (6) 2/16) =0 (8) Ifz! = 0 then dt + 30/4 = m/2+ nx or t= 35/164 nm, 0, =(0) = 1/8, and 2(0) =0 we obtain x = Jer — Jen Owe see that oscillatory motion results if 8? — 256 < 0 or 0 < |G] < 16. 13. From ma" + 4z’ + 22 = 0 we see that non-oscillatory motion results if 16 ~ 8m > 0 or 0 < m < 2. 14, From Jo" + 2'+ 2 = 0, 2(0) = 4, and 2'(0) = 2 we obtain x = 4e~™ + 10te™. If a(t) = t=1/10, so that the maximum displacement is c= 5e~°? = 4.094. 15. Writing J2" + $2 = cosat + sin in the form 2” + = Bcosyt + Bsin yt we identify A= 0 and 1? = 64/8, The system isin a state of pare resonance when y = = y4/3 = 8/V3 n 12. , then 231 Chapter 5 Review Exercises 18. Clearly 2» = A/iu? suffices 17. From fo" +2! +3z = e+, 2(0) =2, and 2(0) = 0 we obtain x er (cye0s2VBt+ epsin2V%1), spa et and S cosavit+ BY anova) =e 18. (a) Let k be the effective spring constant and x and zp the elongation of springs ky and 4a. The restoring foroes satisfy ky21 = kyza 90 22 = (h1/kz)z1. From k(z1 +22) = hiz1 we have *(a+zn) sha kath) *(B EY) =a ike tothe aw koa: (b) From ky = 26 and ky = AWW we find 1/k = 1/2W +1/4W = 3/4W. Then k-= 4W/3 = dng/3 ‘The differential equation ma” + kx = 0 then becomes 2” + (49/3)z = 0. The solution is = cos a att) = e,cos2fFe + sina ‘The initial conditions 2(0) = 1 and 2/(0) = 2/8 imply ey = 1 and 2 = 1/ V3. (©) To compute the maximum speed of the weight we compute a 7.42 H 4 y= 2fFoinay Fes Foonal%e and j'nl= Yab+d 19, From q" + 10g = 100sin50t,g(0) = 0, and ot) = 0 we obtain ge = = dsin50t, and (a) (b) i= ~Jos 1008 + 30850, and Sy + 1 008 100¢ + cpsin 100t, psn 200e + sine, (c) @=0 when sin50¢(1 — cos 50t) = 0 or ¢ = nx /50 for n= 0, 1, 2, 20. (a) By Kirchoff's second law, eq 1 phere Bis FO Using q'(t) = i(¢) we can write the differential equation in the form, it Le Bit). Ri +RiAE 232 Chapter 5 Review Exercises Then diferentaing we obtain & Liew #4 lie ew Gi rrdsdie eo, (b) From Li'(t) + Rift) + (1/Cyg(t) = Blt) we find L4'(0) + Ri(O} + (1/C)q(0) = E(0) or Li'(0) + Rig + (1/C)qo = E(0). Solving for 3"(0) we get #0) = 5 (B00) ~ 3 a0- Rie 21. For A > 0 the general solution is y = cyoosVIz+ opsin Vz. Now y(0) = cy and y(2n) = cycos2nVF+ ensin2rVi, so the condition 40) = yn) implies 1 =e c082nV2 + cpsin Inv which is true when VX =n or =n? for n = 1, 2,3, Since y= Vie sin Via-+ Veg0os Vie = ~ney sine + neg eosne, ‘we see that y/(0) = nea = ¥(2n) for n = 1, 2, 3, ‘Thus, the eigenvalues are n? for'n = 1,2, 3,..., with corresponding eigenfunctions cos mz and sinnz. When A = 0, the general solution is y = cuz + & and the corresponding eigenfunction is y = For A-< 0 the general solution is y = cx cosh V=Az+ cpsinh V=Kz. In this case y(0) = a and (Qn) = o1 cosh 2rV/=H + cpsinh 2x V=N, 80 y(0) = y(n) can only be valid for A= 0. Thus, there are no eigenvalues corresponding to A < 0. 22. (a) The differential equation is dr /dt?@—u2 te= exe + eget. A particular solution has the form rp = into the differential equation we find ~2Aw?sinut — 2BuPcoswt = —gsinut. Thus, B 0, A = 9/20, and rp = (o/2)sinut. ‘The general solution of the differential equation is r(t) = cet + ce“ + (9/2u*)sinwt, The initial conditions imply ¢1 +2 = rp and 9/240 — ey + wea = vo. Solving for ¢, and ca we Bet —gsinut. The auxiliary equation is m?—w = 0, s0 sinwt + Beosut. Substituting 1 = (2u*ro+ Dump ~ 9}/4u® and cp = (2u%ra — Que + 9)/4u*, so thet at 4, 9 e+ So sinut (= 2H 2am = Oe, Wo D9 ia i ee = ae ae a (b) The bead will exhibit simple harmonic motion when the exponential terms are missing. Solving 61 = 0, c2 = 0 for rp and np we find ro = 0 and w = 9/2e. ‘To find the minimum length of rod that will accommodate simple harmonic motion we deter- mine the amplitude of r(t) and double it. Thus L = g/u?. 233 Chapter 5 Review Exercises (c) As t increases, e approaches infinity and e~™ approaches 0. Since sinwt is bounded, the distance, r(t), of the bead from the pivot point increases without bound and the distance of the bead from P will eventually exceed L/2. (da) (e) For each vp we want to find the smallest value of ¢ for which r(t) = £20, Whether we look for r(t) = =20 or r(¢) = 20 is determined by looking at the graphs in part (d). The total times that the bead stays on the rod is shown in the table below. wey i i Ter 7 [20 =20 20) 20 20 | rrsso07 | 2.asasa | 3.43008 | 6.at627 | 4.22339 ‘When 1 = 16 the bead never leaves the rod. Chapter 5 Related Exercises 1. (a) The auxiliary equation is m?4+-4 = 050.20 = cy cos 2-hensin 2. Letting xp = Asindt+B cost and substituting into the dilferental equation, we get ~12Asin dt ~ 12Beos4t = sin dt. Thus, A= yh, B= 0, and 2, = —jhsindt. The general solution of the differential equation is 2 01 cos2t-+e1Sin2t fh sind. The initial conditions imply cy = O and ey = }(a-+4). Thus a(t) = f(a f)sinze- 4 sinat = }(a+$)sin2e- (asin 2tcos2t) snaff(o+}) Lea], ogee For 2 +2 = sindt the auxiliary equation is m? +1 = 0, so 2. eyeost + ensint. Letting ap = Asindt + Boost and substituting into the diflerential equation we get —15A sin 4t — 1Beos4t = sindt. Thus, A The general solution of Chapter 5 Related Exercises the differentiel equation is x = cy cost + cpsind — 3 sin4t. The initial conditions 2(/2) = 6, 2'(n/2) = (a+ 9) imply cg = 0 +2 and c= 0. Thus a= (+2) cost Banat = (0+ 3) oot fant = (a+ 2)ant- Pranenten = (a+ Dent ~ Zeruntcetenen) wcost(0+2)-Aaintcaai], Exe (b) The velocity at the start of the secand cycle is 2/(8r/2) = a+ & (©) Using results from part (a), x = ey 00s2t + cpsin2¢— 7h sin 4t. The inital conditions are now 2(9n/2) =0 and 2'(Sr/2) = a+ , which imply oy = 0 and o = —}(a+ fk). Thus 1) saat Lina (0+ 2) snar— eesin2tcos2r) py Sr =sin2[-F(or 2) —2eos2t], Besar, sinae[-F (0+ 2) —Zeosat], Fess For? 2 2r we have 2 = c;cost + cpsiné ~ qf sin 4t, as in part (a). The initial conditions are 22x) = 0 and 2'(2n) = ~(a+ $), which imply ¢1 = O and ep = (a+). Thus 2) =-(a48)sint- Lane (a+ 8) sat Heanaienm) = -(2+5) sins Zeasintcostos2) wenf-(or 8) ~ feats], arse eoe @ Chapter 5 Related Exercises 2. (a) {b) 3. (a) We have 1, = cycost + cpsint and we let zp = Atcost + Btsint. Substituting into the 0. The solution of 2!-+ 2 = sind? is 2(2) = cycost + opsint ~ fysindt, as seen in Problem (a). The inital conditions 2(0) = 0 and 2/0) and cy = #8, so the solution of the initial-value problem is 1 imply er 19 4 a(t) = jesint — * sina ok. be 19 1 = point — Gg(2sin 2teos24) 9 2 Point — 2 (rsintenst cos t — Gglsimtonsteos 24) 5 sain (24 = (nn(8-A ‘The first positive value of ¢ for which 2(t) is zero is =m, at which time 2'(r) = -2 <0, 0 2 <0 immediately after {= x. The new initial-value problem is then 2" + dr = sinde, x(t) =, 2'(n) = —2. The solution of this initial-value problem js seta), 0¢ten eae x 2) = ~bennt- Lsinat= (6n29(—2- Loon), vets % ‘The next value of ¢ for which 2(t) = 0 ist = 3n/2, at which time 2'(3n/2} = #8 > 0, so > O immeditaly after ¢ = 35/2. ‘The net initis-vloe problem i ten a? +2 = sind (8/2) =0, 2(3n/2} = ff. The solution of this initial-value problem is xt)» Heat Sint (oa)(- Lantasn), Sees ‘The next value of t for which x(t) = 0 is t = 52/2, at which time 2!(5n/2) = J <0, 90 2 <0 immoditely ate ¢= 5/2. ‘Tb next intil-aie problem i then 27 +42 = snd 3151/2) = 0, 2(6n/2) = -2. The solution ofthis inta-value problem is L 7 1 1 a(t) = qantt= (sinay(Z sin 26 236 Chapter 5 Related Exercises where b is the value of ¢ for which a(t) = 0. In this case, y ~ fos 2t is 0 for t = 8.64514 whichis between Sn/2 and Sr, Thus, for the fist «wo cycles, (in ~ jheosteos2t), OS tS a at [OPE fomm), rts # ©) (cost)(Z - sintcos2t), Fst ¥ (sin2N(h Joos), Exes Se4s14 = sind, ‘The first part of the next cycle is the solution of the initial-value problem 2” + 2(884514) = 0, 2/(884514) = 0.28. This solution is approximately 2(t) = ~0.013c0st — (0.209sin¢ ~ 0.067sindt and is positive on (8.84514, 9.42478) with amplitude 0.042. In the second case, the differential equations are 2” + 642 = sin dt when x > O and 2" +4: sindé when 2 <0, Since the initial velocity is positive we solve fist for 2 > 0. [The following computations are done with the aid of a. CAS having a differential equation selver and graphing capatility.] The solution of 2” + 64x = sin 4t, 2(0) = 0, 2"(0) = 1 is 2 415458 is found using the root-finding capability of the CAS. The next initial- , (rs) = —0.909091. The solution is x= E (masts Bons), ostsn, ‘here value problem is 2” + 4z = sindt, x(1) Z(t) = 0.403cos2t ~ 0.255 sin 2t — 0.083sindt, my Stra, 0, s!(r2) = where rp = 2.1403. The next initial-value problem is 2" + 64x = sin4t, 2( 1.16206. The solution is alt)= 1BBcos$t + 0.021 sindt ~ 0.008sinSt, rp St S79, where ry = 2.53479, The next initial-value problem is x” + 4x = sind, 2(rg) = 0, 2(r3) = =1.28086. The solution is 2(t) = ~0.737cos2t —0.217sin 2 —O.083sindt, StS ra, where ry = 4.0453. In the third case, the differential equations are 2" + 36 «: sindt when x 2 0 and 2" +25 = sin dt when 2 <0. Since the initial velocity is positive, we solve first for z > 0. The solution of 2" +362 = sin4t, 2(0) = 0, 2/(0) = 1is poinat+ Asinet, ostsn, 3) = 35 iB 237 Chapter 5 Related Exercises where ry = 0.571447. Subsequent solutions are (¢) =0.115cos5t +0.L1Lsindt +0.098sin5t, ry St < rp = 1.24203 (8) = —0.068c0s6¢ + 0.05 sin4t + 0.082sinGt, rz 0, in each case the first differential equation used is x” + fa! + 4x = sindt, foliowed by a" + Oa! +x =sindt, and alternating thereafter. ie) 4 B=001 g=01 6=05 [As ¢ increases, the amplitude appears to increase for §) = 0.01 and decrease for @ = 0.1 and 5=05. 239 6 Series Solutions of Linear Equations Exercises 6.1 rHanttyin+1)| Ea L Reyne |e at] “The series is absolutly convergent for 2[z| < 1 ot fz| < 1/2. At 2 = —1/2, the series S= converges by the alternating series test. At x= 1/2, the series 3 + is the harmonic series which diverges. Thus, the given series converges on {—1/2,1/2) Loo + 7) /(n + 1)! 100 i poor eNO F UN ig 10 og i 2 im, |= | Toe + TP fal Heit a0 ‘The seis is absolutely convergent on (0,00) L L im je-5| = bie —5| : gl? I= ak | < 10, of on (~5, 15), Ate eS 15). Exercises 6.1 Since cos(r /2) = cos(—7/2) = 0, the series converges on (—7/2, 7/2) Seo Bee 2” 4° * 7 ~ ig" * Since the function is undefined at z = —2, the series converges on (~2,2). 9. Yo anes"! + 3° Goya" = 21-0124 So 2neae™™ 14S Gott 7 =) = Tor a = 2a + Ak + Newsa2” + Yo bea iz fas Ei = Pe + S12 + Tees + 6ex-als* ei 10. Erle Des” +255 n(n Yoga" 243 nega +2 lege? 42-3. 2es0! 43-1-c42 + Sol Dens™ 4255 nf Mega? 48 Se nega & = = oa ra = den + (1205 + (Ian + Sex)2-+ Se A(R Tags +23 (+264 Neat +E tes! i= = den + (Ser + 1eg} + 5 ((k(k 1) + Sklog + 2(k + 2)(E + Denaa)e* at = dea + (Ser + 12eq)z + Sophie + Dog + 2k + 1Y(R> Persale® Ey nee Beat, Scum (tly sy seen Scum net Sey 2 Seatac gents Eeyetins nents Seen! = = a Pats Raymon yet Fearn ett Beanie = = = ken-1 = kan? ken-1 241 Exercises 6.1 = Sytteets Rayer asnats Katee = = fi * ie [antes — (aye (2 + (9 2 = 0 et Jha yf tay = Se Mreaon afsvtay= scum 5 ete So oe , De en a = [twin , cute tet TEE} Buy * PES TE f(a yay? (at Jon “PRR PEE q Met € = Set [HRB] ae : carl DRIP | : 18. Substituting y= Dg coz" into the differentia equation we have yl = ay= Sala Yea? ~ Eesettt = Sb + 2) Yenyaet— Sect ae aan 22m + Fle-+ 20k+ ener — eile =0. 4 ‘Thus a=0 (k+2)(b+ Venez — ce. and L ray Choosing a = 1 end c1 = 0 we find a= ba 1,2.3, 1. Exercises 6.1 aud 90 on. For 9 =O and ¢; = 1 we obtain ee te ee nop ee i. an marty 504" ae cleat & 7 a0 Substituting y= C%Lo cq” into the differential equation we have " of +8 F nln Tega 24 F oa = So (E+ D+ Dengzat + To at & fd ra i ae et id ed = ent Gere + FU + 2)k-+ Vengo + eral & ‘Thus and eka = eats 1 Eee Choosing co = 1 and c =0 we find 243 Exercises 6.1 ‘and so on. Thus, two solutions are 15. Substituting y= Co qa” into the differential equation we have of = 2xy! y= So nln— Vena? ~ 25° nege" + Sena” & Pie ct ras a = Eh +2)(b+ erage! 2 hers + gst f=) =A & = eg + co+ FKk + 2VE-+ Nicege — (2k ~ Negla® #0 a ‘Thus 2ep to = (&+2)(b+ Vegas — (2k — Deg and 1 a=-j0 2-1 E+ Dee oxen b= 1,2,3, Choosing cp = 1 and cr = 0 we find cas cya cr Exercises 6.1 fand so on. Thus, two solutions are 7 oe % ad needa date tere 16. Substituting y= C%o cas" into the differential equation we have y= ay! + 2y = nfo — Yea? FS noga® +25 ona" & ei FE ket! +25 axa a mb = E+ 2) + Doragst % = den +2204 S [Ok + 2k + Dense ~ (= 2DJaalet & Thus dey +200 = 0 (i+ 2)(k-+ Vensz ~ (R= 2he = 0 and @ - 2 he? TES RSD) Choosing cp = 1 and ey = 0 we find aan emo cres m0 a=0 For ¢ = O and cy = 1 we obtain and so on. Thus, two solutions are 2 Exercises 6.1 17, Substituting y = 2.9 oz" into the diferente! equation we have yl tty + ry = Do n(n — Wenz“? + > neaz™*! + Yo on™ e = f=} a oa fe eee «Fusauernmaste Sedat s Bont = e+ (Ger + oy) + Se +2)(k + Veqea + hoy _a]s* = 0. Fay ‘Thus oy = Oey + 09 = 0 (8+ 2)(R+ Dewan + hee ty b= 23/4, & 2 = “TERS Choosing a = 1 and ¢; = 0 we find a=-. ates co 26 and So on. For ¢y = and c= 1 we obtain =0 i ang esos 5 ma and 246 Exercises 6.1 418, Substituting y = C29 cu2" ito the diferentil equation we have Yi + Day +2y = Sonn Vega”? +2 F nega” +2 Fo eas” = et = Tie eee = SU + 2k + Meesatt +25 east +2 Yeast ima fet i 2+ Rey + SU +2) E+ Necaa +20k-+ Dealat Thus 2ep + 2eg = 0 {B+ De + Deas + 2k+ Dee = 0 and 2 eee pepe FRL23, (Choosing co = 1 and ¢; = 0 we find 2 1 0 ‘and so on. For ey =0 and ¢, = 1 we obtain and so on. Thus, two solutions are ea mala a4 gat aah + Exercises 6.1 19, Substituting y = D2 oz into the differential equetion we bave (e-Dy += n(n t)eqa”"* = SS n(n = Tega + nega?! = = mar = Sc + Dhersrat — Fb + 2) + Teneatt + Sok + Denes a 3 ist tener + Slee hear — (+ K+ Devea + (b+ Deval = 0 #4 ‘Taus =2eq +0 = 0 (b+ APeney — (b+ 2)(8 + Neko and +1 cee = Type = 12,3, Choosing c = 1 and c} = 0 we find ep = = 0. For cy = 0 and cy = 1 we obtain and so on, Titus, two solutions are liawls 1 =1 an arthety tity lt m det pat get ate 20. Substituting y ext Into the differential equation we have (2429 bay —v= Sonn Noga So Anl— Dgs"2 + FS ens” & Tt et D+ Dhkensat’ + Do 2k + (b+ Yowsar® + So kez! a ms Bi mea co + Sl(h+ Whey +2k-+ (e+ Dense + (k= Deal ra ‘Thus 4e2— = 0 (e+ Dhenys + 26+ 2)(8+ Deny + (k= Dex 248 Exercises 6.1 and oe a= je (+ Wkensy + k= Dew aga = — ke, a 2k 2+ Choosing ep = 1 and ¢; = 0 we find 1 v ‘and s0 on. For co = 0 and cy = 1 we obtain 150 4 a=0 ‘Thus, two solutions are ly neofis et 2A. Substituting y = Do ea” into the cierential equation we have YP let thy y= So nln thena™?— ¥ nega — F nena? ~ Fee a ei es a mot 2 ec oe = Dll +2) (b+ Nenszt — F° keg — Sik + Vewaz*- Dart it at = = 2a 01 — e+ Sb + 2)0k-+ Dower — (E+ Vena ~ (b+ Dealt Thus 2eq~ 4 —c0 = 0 (E+ 2)( + Nekao ~ (F~ I) (enss +04) = 0 and Choosing y= 1 and ey = Exercises 6.1 and so on, For ¢y = 0 and cy = 1 we obtain =f ‘ =p 8 aay and so on. Thus, two solutions are are dete tat dete ext lat tata tty Walt sats Gate oat and poet jah jet oat 22, Substituting y= Cy cn2” into the differential equation we have (2 +1) yf = 6y = Salm — 2heya" +S. nin~ thoga" 6 F° ons” = = = rs ent aad = Sele east + Sk +2) + Nessa 6 Saat & & & = Pea — 60+ (Bey ~ Bez + So [(KP K-86) cu + (+2) + Dowsa] 2* im Thus 22 ~ 6a = 0 Gey ~ 60 =0 (b-3)(k + 2c + (K+ ME Versa = 0 and asta asa ones eu = Epo Choosing ¢ =} and ¢ = 0 we find Exercises 6.1 and so on. For eg = 0 and) = 1 we obtain on cal Ths wo ston ae wetitit dhe mk peete 23. Substituting y = C2 cuz” into the differential equation we have (224 2)y" + 7 Ynlr= Mens” +2 yn = Vena"? +3 Pie - yee" sy oa = Fae yast 42 EF esayes vast +s Etat Beet = (den — 09) + (1205-4 2er)2-+ Fo [2(k +2) + enya + (K+ 2k~ 1) 4] 2 PB Thus faa ey +26 2(k-+ 2)(k-+ Leesa + (HP + 2k ~ 1) ck = 0 snd Choosing cp = 1 and ¢) = Exercises 6.1 land so on, For q) = 0 and ey = 1 we obtain === 1 6 1 To and so on. Thus, two solutions are ; et ie-3 od mare dale 24, Substituting y= D92p Caz” into the differential equation we have (27-1) y+ ay! — y= YF nf pega” — Yo nlm — tens? + Yo neaz™ — Yo cn” = & Foy ma 7 TT = Ebb Neyo! — Fo (e+ 2) + Nensart + F kegs ~ Feast ib fo fa r=) Pas Thus = (-2e2 ~ 00) ~ Gene + Fo [-(e-+ 2)(8-+ Uewea + (K? -1) og] 202-09 = 0 6c = 0 (E+E Hees +E Dik + Nee =O ero cna = Ee, b= 23,4, 2 ee b= 2,34, Choosing ¢9 = 1 and e1 = 0 we find Exercises 6.1 and so on. For 69 = 0 and cy = 1 we obtain Thus, two solutions are and wer 25, Substituting y= D%g c,2% into the differential equation we have f= ay! ay! tym Ye nln 2egat b= F nf Dee? = So nena + ent = = =m ao a a = Feet Meas! — Tet DE Deca — Se heat + eget a & Fee ~2ey + e+ Flm(-+ 2K + Dewss + (RF Deeg ~ (Dea Thus rate =8 Ch + DK + Denso + (k= Whkenes — ( — Tee and ex Fe2” ERT k= 1,2,3, Choosing @ = 1 and cy = 0 we find and so on. For ¢ = (and c = 1 we obtain @ = oy = a Laslsy.) +o veais} ) and ¥ao (rt jets +e: ‘The initial conditions imply C) = ~2 and Cy -a(te deta det 4) eee se De 253 Exercises 6.1 26. Substituting y = S229 on2" into the differential equation we have (atly"-(2—a)y' ty = Fon net Fan poet <2 anaes Beat Soe at et fon Ore Ge ah = ea — Der + 09+ Sol + 2)(K-+ ensa ~ (+ Nener + (b+ ede =o. a Thus 2ca —2e1 +e {k+ Ik + Uegee — (+ Deke + Uk + ck = 0 1 casa = EE eet Choosing ey = 1 and c; = 0 we find es and s0 on. Thus, an) eea(eret= fete.) sn J+On(it22-2 ++) ‘The initial conditions imply Cy = 2 and Cz = -1, so eee) a Exercises 6.1 7. Substituting y = Cio cas" into the differential equation we have vf =20y +8y= Fn ~ Vega? = 25 na" +85 ons” ad En en = Fk 2k + Nessa! —2 Shot +8 F xe! B = & = den + 800 + Fell + 2) + Dense + (8 2Jeao* a ‘Thus 2ea + Boo = 0 (6+ 2)(k + lense + (8 -28}ey = 0 and a=—sa 2k-8 eas EERE FTN2S Choosing ay = 1 and cy = 0 we find arama a0 A ang wasps For q = 0 and ¢ = 1 we obtain and so on. Thus, and ©, (+ Be) 404 (1 acts bets) 255 Exercises 6.4 The ti contitons py Cy = an C= 0, . (nate) 28. Substituting y = Eq cna” into the differential equation we have vas? +424 y (22+ Dy! + aay =F nf Da” + Fe nl ~ toga”? + Fe rns” & & eo a we = Fete est + Ke + ME Newezel + Faken! & & a = ep + (609 + Dei)z + SelM + Ten + (k-+ 2)(K-+ Venyalat «0 fo Thus 2 = 69 +20 Kk Neg + (+2) + Nena and k ee k= 234, mat 3, 4, Choosing a = 1 and cy = 0 we find cg = c4 = = 0, Bor @ = 0 and oy = 1 we obtain oe and so on. Thus wara(e- tts} bre) ; yeava(s- tty hry v (Petty) 256 Exercises 6.1 ‘The initial conditions imply cy = 0 and ex = 1, so feb gad= late ey 29. Substituting y = C%q cn2” into the differential equation we bave -+)(@tartact+-) [seas aaa +g +] + foe tats (a-ha) 20+] = Dea (Gey eglz+ (12ee + ea? + (200s + 62 - feo) 2 4-0-0. Ths and Choosing cy and ¢; = 0 we find a=, =O oe 720 and so on. For ¢ = O and cy = 1 we obtain e @ asa and so on. Thus, two solutions are i Hoan, nang ttt and een" Exercises 6.1 30, Substituting y= C39 ext" into the differential equation we have feed y= Senin eet Ea ( Leste.) (a 2an ese? + 4eur! +.) — Sane” + (lee ts Bate) (1 btear + Se + de )- de [aoa + bye + 12eaz? + 200! +--] 1 : + [a+ Qaras+ (0=2+ha)22+--]-lasa2tac'+-+] : Lge = (2oy-her 0) + (Gea + Dene + (1204 +369 Heat 5e1) 2 fe, ‘Tins des +01 ~ 6g +2. = 0 a +8atert go ond Choosing op = 1 and c= 0 we find e wwe obtain and so on. For @ = 0 and cy 1 e=-} 4 and so on. Thus, tw solutions are Exercises 6.1 BL. Substituting y = Dp cna” into the first differential equation leads to of aye Senin Dena? Segal = So (k-+ 2) b+ Menzel — Fe ees er 2 2 rn = den + F[lh+ Nb + Dees — cele 2 Thus (k+ 2k + Vees2~ cx-1=0 and 2 2 7 oa : a Epa FR NAS Let qp and cy be achitrary and iterate to find and so on. The sétution is fl 1 scotartttttastt hart+ bet yecosart it im Be wo (tebten)ralerdven)ebta dete Suberivuting So cna into the second differential equation leads to Exercises 6.1 = DE + lk + Degsat” — So dhegt* — Fo deg F= Bi es = ey = dy + F1G > 24+ Denes — Ak + Dealt Fe gt weet Sie +h ‘Thus 2eq = deo = 1 (b+ 2k + Yewia — 4k+ Vee and 7 ans 20q Too . Gee Let ep and cr be arbitrary and iterate to find 4138 261 4 oa GO TS 1 4it 64 409 ot Tal * ot" HY ios oy and so on. The solution is 260 32. 33. 34. 35. Exercises 6.1 yratars (J+20)2?+ (4 +B efe+( a ‘ aya, ft pera aaete Lat] rae det Be sata tenJen[ea feel Me aylis We identify P(2) = 0 and Q(z) = sin2/x, The Taylor series representation for sin x/x is 1—22/S1-+ 4/3! —--+, for |z| < oo. Thus, Q(z) is analytic at z = 0 and = 0 is an ordinary point of the ifferential equation ‘The differential equation zy" = Qhas a singular point at 2 = 0. It also has two solutions, yx = and yo = 2, that are analytic at 2 = 0. If. > 0 and y > 0, then y” = ~zy <0 and the graph of a solution curve is concave down, Thus, ‘whatever portion of a solution curve lies in the first quadrant is concave down. When 2 > 0 and ¥ <0, y= —zy > 0, s0 whatever portion of a sotution curve lies in the fourth quadrant is concave up. (a) Substituting y= Dig cuz" into the differential equetion we have n= News? + nega + Se os = = yitnyty 7 YGe+ 2b + Denso + x keg + = cast = (202 +60) + Sle + Ih + Versa +(k + Veele* = 9. es Tms ert oo = 0 (b+ 2)()e + Deseo + (+ Te and cha = —Fpyee Choosing «9 = 1 and ey = 0 we find 261 Exercises 6.1 and so on. Thus, two solutions are ze (-1)hotet & uy a , -Ee5 ad m=} Gere (b) For 11, Ss = Sp and Sy = Ss, 50 we plot 5, Sy, Ss, Se, and Sto Exercises 6.1 (c) ‘The graphs of yn and yp obtained from a numerical solver are shown. We see that the partial sum representations indicate the even and odd natures of the solution, but don't really give a very accurate representation of the true solution, Increasing N to about 20 gives a much more accurate representation on (4, 4] (4) From e = Cfo z#/A! we see that e-#7? = S¥o(—22/2)K/k! = DEg(—1k2%/2kKI. From (5) of Section 4.2 we have 2p (Se dene [Pas ste) E/eu+) = 1 2h 2) (Eacme™") ee 2y era 36, (a) We have + (cosz)y = 2en + Gese + W2eq2? + 20052° + Bega + A2eqz Te 40-4 5 - Ft lloras ens? + eye" + cae! + ott +--+) 1.),3 ha)! an 1 — 3¢a)a + = (2ea + on) + (Gea + c1)z + (12c4 + 02 Ja? + (200s + 1 + tes +a Exercises 6.1 ‘Then BWepter+ tata =0 and Wertopt her ~ fe eat gga ga mot which gives cy == —en/80 and cr = ~19e1/5040. Thus ee pie, nee edd - i. as aby ds 19 or nla) +e- gat =0, (b) From part (a) the general solution of the differential equation is y = crm + caye. Then y(0) = cr +e2-0 = cy and y/(0) ie vautmeles— pit © : ; F 4 z , 7 » seraftlr* sara / i (a) y 264 11 -O-+e2 = ¢2, 90 the solution of the initial-value problem is, Exercises 6.2 Exercises 6.2 ——___ 1. Irregular singular point: 2 = 0 2, Regular singular points: 2 = 0, 3 3. Irregular singular point: x = 9; regular singular point: x= —3 4. Irregular singular point: 5. Regular singular points: 6. Irregular singular point: 2 = 5; regular singular point: 2 = 0 7. Regular singular points: 2 = -3, 2 8. Regular singular points: 2 = 0, 9. Irregular singular point: 2 = 0, regular singular points: = 10. Irregular singular point: x = —1; regular singular points: x 11. Whiting the differential equation in the form 5 2 fe y+ yee Weer taal! wwe see that 29 = 1 and zo ees a e— y+ te — py + EV yao, a r+1 In this case p(z) = 8 and g(z) = 2(z- 1)?/(e +1). For zo = -1 the differential equation can be a oy _ (e+ Py" +54 DEAE y + (2+ 1y=0 Ie this as fe) = (2+3)/¢—1) and ole) = afe +2, 12. Weiting the dlfereptal equation Inthe form +8 ot ye ty ‘we see that ao = 0 is a regular singular point. Multiplying by 2°, the differential equation can be pt in the form ay! bale Sl + tety We identify p(z) = 2 +3 and g(2) = 72°. 13. We identify P(z) = 5/32 +1 and Q(z) = —1/32*, so that p(x) = zP(z) = § +2 and g(z) = Q(z) = —}. Then ap = §, by = -4, and the indicial equation is = 45? 4 2r—1) = Err ey = Fr? + 2-1) = 3Gr- Nees) = 265 Exercises 6.2 The indiil roots are and ~1. Since these do not differ by an integer we expec to find two series solutions using the method of Frobeni 14. We identify P(z) = 1/z and Q(x) = 10/z, so that p(x) Then ap = 1, B =O, and th indicia equation i Land a(2) = 22Q(2) = tor -P(2) rr-Derer=0, ‘The indicial roots are 0 and O. Since these are equal, we expect the method of Frobenius to yield a single series solution, 15. Substituting y = D%2> cn2™7 into the differential equation and collecting terms, we obtain tr (21? 31) eget) +S [2(e tr = 1)(K + reg — (BE ree + Dena a which implies ot ay (Qr-3)=0 and (b+ YOK + Or — B)en +204 ‘The indicial roots are r = O and r = 3/2. For y = 0 the recurrence relation is b=1,2,3,.-., ‘The general solution on (0,0) is yaoi (ede ~28 ++) ert (1 +) 16. Substituting y= Tp ca2™* into the differential equation and collecting terms, we obtain (2°? 437) coat 1+ (01? +748) a2" ay" + Sy tay 4+ Se + Nk = en + 5(k+ re + og-alatt i 266 az. Exercises 6.2 which implies 2r? + 3p = r(2r +3) =0, (24 trese no, and ‘The indicial roots are r= and co For r= 0 the recurrence relation is and The general solution on (0,00) is rin (a At Lat aa Coe 9 (a— at seat) + (Im att gett”) Substituting y = C25 ce" into the differential equation and collecting terms, we obtain Paes = oe = 1 day" + oy +y = (47? — or) oa 3+ J4(k + r)(k+r — leet (b+ rey + cp] att pv tue (HF) aa 4 F [een eat Sk + ree +1] 2 which implies ad Me snieit—nareci=0 Then oo ae rand r= 7/8. Pr 20 he reunion RELI and 267 18. Exercises 6.2 For r = 7/8 the recurrence relation is dent GREE oS and ‘The general solution on (0,00) is oc! 2p 4 3, vee (tom Bites?) 43 ai8(,-25 ge yet) ten (1 Bat Substituting y = D° ant! — ay + (2841) 1 2"*" into the differential equation and collecting terms, we obtain = (27? art ope + (2? +e)era*t + Sete nik er tee (b+ ree tee + eealatt? & which implies art ares (2r~ IVP) =0, (Pere =0, and ‘The indicial roots are r = 1/2 and r = 1, 50 ¢ ee “Fk — 1) and a= =0, For r= 1 the recurrence relation is __ tet > FGR+1) and ae ‘The general solution on (0,20) is 19. 20. Exercises 6.2 Substituting y= Dio cn2%* into the differential equation and collecting terms, we obtain Bey" + (2—z)y! (37? =) coe’ + Spacer -yht rl + 2k ree (+ re a}et! which implies 3? -rerGr~1) and (E+ NBR ES Nee (E+ Fen = 0. ‘The indicial roots are r = 0 and r = 1/3. For r = 0 the recurrence relation is et = cee By) FNAB on and 1 3a For r = 1/3 the recurrence relation is and ana 3 ‘The general solution on (0,00) is weird hats aha) + cae? (14 et ate Bart) 10” * 30" Substituting y = D%2o c.2™* into the differential equation and collecting terms, we obtain v3) = (-r42) r+ Elicenittr at jana} which implies and ener dla on 269 Exercises 6.2 ‘The indicia roots are r = 2/3 and r = 1/3. For r = 2/3 the recurrence relation is Bey a Ei and oS 27 =e ref, asfa ante ‘The general solution on (0,00) is 392, 9 3.92, 9 2 (14324 Sat4 2s 28 (14 ber Paty 28 Cunt (1+ See Bate Soe) cael (1s ber tes Soren) . Substituting y = Tq e2™*" into the differential equation and collecting terms, we obtain Day" — (B+ 2z)y! +y = (2 — Sr) cpa! + Yo lalk + r)(k+ 7 — ow a = 3k rey Aker New ees) which implies 2? Sr = r(2r—5) = and (b+ r)[2k + 2p — 5)oy~ (Ze + 2r — B)eea = 0. ‘The indicial so0ts are r = 0 andy =5/2. For r = 0 the recurrence relation is and For r = 5/2 the recurrence relation is 2k + Yes cp = 2+ Dee Ba FAL2B and 22 Exercises 6.2 ‘The general solution on (0,0) is =0:(1+h0 ads (iter Bets Leabely..)scu@ (afer dete Bora.) a? * ws é Substituting y = Co cux"*” into the differential equation and collecting terms, we obtain S)y=(#- San + (#42748) ae a ea/+(2- +$ernerr-naretne- fara which implies 4a (142) (r-2 a C+8) 6-5 hor a (P+2r+2) nd [went-Jatareo The indicia roots are r= 2/8 and r= 2/3, 80 2/8 the recurrence relation is and a For r = 2/3 the recurrence relation is cao: aang, be AG. and ‘The general solution on (0,00) is yecarta (nfs 23. 24. Exercises 6.2 Substituting y = Dg ent" into the differential equation and collecting terms, we obtain, Ory! + ony + 2y = (Or? ~9r +2) a2? + Slot + lk tr ~ Dog +2e4 + 9k +r Doe fet? a which implies Gr — Gr + 2= (Br —1)(3" - 2} and (9 Arye + r= 1) + len +9[k +7 Lowa =0. ‘The indicial roots are r= 1/3 and r = 2/3, For r = 1/3 the recurrence relation is (3 ee. and ‘The general solution on (0,00) is 1 1 mit ore (1-9 +3 = By Bet) scnt (rte Bede) Substituting y= D%.p cnx‘ into the differential equation and collecting terms, we obtain 22y" + Say! + (22 ~A)y = (21? +r = 1) eos” + Foe ete er — shop + 3b + ren — oe + epee & which implies Warn (rar + and [(k + r)(2h + 2r +1) — Tee + 2eq1 = 0. 272 25. Exercises 6.2 ‘The indicial roots are r= —1 and r = 1/2. For r = 1 the recurrence relation is 2 Fae- 3" ba123.0., and a= 20, ay, os For r= 1/2 the recurrence relation is 2m 8 Fak +3) and c= gage ‘The general solution on (0,00) is : 4 2,294 ee 218 (p Peg a2 ya Cy! (14 20 ~ 2084 Ba 4) + Cet? (1 Fat Eat eat) Into the diferetial equation and callocting terms, we obtain Substituting y = Ep ene ay! + 2y —2y= (12 +r) cpa” 1+ (F237 +2) qa" 4+ Silks Nk +r Dee + 2k + roe — cgea}e c= which implies (Pears 2)a= and (k+ rib + 0+ Teg — 4-2 = 0. The indicial roots are ry =O and r 1, 0 cy = 0. For m1 = 0 the recurrence relation is as Ree! oe and Exercises 6.2 For ry = —1 the recurrence relation is and ‘The general solution on (0,00) is oe wr LoS 1 ett gy Bd tas oie +X Gayl a fp = Hecisinn2 + Cros} 26, Substituting y = 32% cna" fato the diferential equation and collecting terms, we obtain Ay tav+(#-}y= (#- Sar’ + (Arte Sa a 1 bee feet cana] 24 qt +3 fetter tet (b+ ree = which implies and [ietet-jaraa=o. ‘The indicial roots are r; = 1/2 and r2 = -1/2, $0 ¢1 cone For ry = 1/2 the recurrence relation is ke cs BAe e+” 274 27. Exercises 6.2 and =o ona Gn+iy For rp = —1/2 the recurrence relation is 1 ke Resp) EPR and anak =F ane =o VAC; sin x + Cp c083] Substituting y= Dip a2"*" into the differential equation and collecting terms, we obtain af maf ty= (Prego 14 Eller t 1)k+ ror — (E+ ree tale =0 is ‘which implies and (ker Ihara = (k +r = Tg = 0. ‘The indicial roots are 7) = 1 and ry = 0. For ry ie t= Tey aE SA) the recurrence relation is b=01,2--5 275 28. Exercises 6.2 and one solution is 11 = eye. A second solution is 1 fa E+ lne + 52+ ett rot ‘The general solution on (0,20) is Substituting y = Bie ena vt fy -y y= Cr+ Crmls). into the differential equation and collecting verms, we obtain = (Pears (Pears a) oe + Selle bt Dow + ak + Pl —Benaalat? a which implies Pe arer(r +2 Peirtda ao (barber Bex ~ 20, ‘The indicial roots are r1 = 0 and rz = ~2, so cy = 0. For ry =0 the recurrence relation is and ‘The result is ] Exercises 6.2 A second solution is - | “1 SGrhes gers ye ag rete at ‘The general solution on (0,00) is y= Cim(z) + Cnnla). 29. Substituting y = C&p cn2™” into the differential equation and collecting terms, we obtain bird ay"4 (l= ay y= Poe + Sle + rk tr Dee t (b+ re = (+ roel mi which implies r? = 0 and (b+ reg = (ht ronan = 0. ‘and the recurrence relation is ‘The indicial roots are ry = 2 = b= 1,23, One solution is a(iter} esis ) coe’. A second solution is wean [EM arnt | Ele ef (inet het dae fase 72 ‘The general solution on (0,00) is Exercises 6.2 30. Substituting y = D3Zp cy"*7 into the differential equation and collecting terms, we obtain sled ryatar t+ Sikes = Veg + (+ roe + ceai]zt! = 0 which implies r and (be rater 20. ‘The indicial roots are 71 O and the recurrence relation is oa WEE, = 1.23, One solution is ie Sou" nwo (tns+ fe ~ gine sige) Se A second solution is = fuse _ ey = a(l-r+|2- ges) de 20 | eR Ea i =n Fie ) ae Pad +o 153" ‘The general solution on (0,0) is y= Ciyi(s) + Caynla), B1. Substituting y = Dp cq2™* into the differential equation and collecting terms, we obtain ay! + (2—6)y/ —3y = (72 ~ Trae 1+ Ske nh T— eet kT eg a = 6h tree = Seuaslet = 0, which implies Exercises 6.2 and (btn Ta s(t r—do1 =O. “The indical roots are m = 7 and r2 = 0, For ry = 7 the recurrence relation is (e+ Tox + (E+ Bena = ke 1,2,3, Taking co # 0 we obtain 1 a=-j0 5 en= ge 2} a= Zo and so on, Thus, the indicial root r; = 7 yields a single solution, Now, for rp = 0 the recurrence relation is Rk=Ta+(b~d)eqr=0, F128, Then 6 -8e=6 0 o 0 = o=0 0 => eri arbitrary and 32. Exercises 6.2 Taking cp #0 and e = 0 we obtain Teking 9 = 0 and er £0 we obtain Substituting y = E%p cyx"™ into the diferental equation and collecting terms, we obtain ae — Ny! + 8y/ — 2y (=P) 4 Vike Vlkp r= Wer —(b eer ee + 3(K + roy — 2c a)eter—t which implies ar —Per(a—r)=0 and (ht ryt r = Meg + [lke +2) — Pear =O. ‘The indicial roots are 7: the recurrence relation is and ry =. For ry (b+ Aken + (+) +2) —Blonn = sem (K+ Ven b= 12,3, 280 Exercises 6.2 ‘Taking co #0 we obtain 1 = 20 2 = 3leo c= Alen, and so on. Thus, the indicial root r; = 4 yields a single solution. For ra = 0 the recurrence relation, RK — Bog + K(k Seam To : (b= Aey (b= Seay =0, B= 1,23, ‘Then 3e1 ~ ep 2a~e, at0q=0 = o=0 Oey + eg =O = cy is arbitrary and k=5,6,7, ‘Taking 0 = 0 and ce # 0 we obtain ato eye Dey een te In this cae ve chain he te acon yaitbesdet and ypeetezabeaeb a+: 33. (a) From -#. Then A we have dt/dz = —1/2? 281 Exercises 6.2 ay a and Py d (dy) d (_pdy at) ay a a (2) £(-#2 gg) mgt ww A a 2ay Ot ye 3 (#Ge+208 Fe | o becomes BB a nyeo () y= E&oent™™ into the differential equation and collecting terms, we obtain ‘Fe _. ty = (Pt reat + (7? + Br + leat” $+ SMlk + kr — Dee + 2b + ree + Ace alter which implies erent) =0, (P+ar+2)a=0, and (Kt rete + tee + Aca = 0 ‘The indicial roots are ry = 0 and rp = —1, so 4) = 0. For ry = 0 the recurrence relation is = 234, and Exercises 6.2 For rp = —1 the recurrence relation is qe Mist «= FE) ‘The general solution on (0,00) is v= E LM hem + cor ES vay 1 1 om a5 Eline = HIOisin Vit + Croos Vt] (c) Using t = 1/, the solution of the original equation is \Oresin D4 Cree”? 34. (a) From the boundary conditions y(a) = 0, y(b) = 0 we find vi Cisin 2 + crcos A =o vy va Crsin SS + Croos = 0. Since this is a homogeneous system of linear equations, it will have nontrivial solutions if Exercises 6.2 ‘This will be the case if 5 (tna mdab _ dab vi(Gt) =n a vin ee or, if neta? Pybt ae el ‘The critical loads are then Py = n'nr¥(a/t}*Elo/L?. Using C2 = ~Cy sin(V/A/a)/ cos(VA/a) we have Tn y= Gxa[sin Soya 4] = Crefoin cos — con sin A = Gyrsinv3(3~2), ond wwe) = Caren 2 (2-2) = cyan Be (b) When n = 1,6=11, and a = 1, we have, for Cy =1, we) = zante 35. Express the diferential equation in standard form: "+ Play! + Qle)y + Rly = Suppose 2y isa singular point ofthe diferenial equation, Then we say that 29 isa regular singular point if (2 ~9)P(x), (x — 29)Q(2), and (x —z9)°R(zx) are analytic at x = zo, 36, Substituting y= CPpcqa"*" into the fst diferential equation and collecting terms, we obtain Py! ty = cor" + Dla + (kt — Wk tr ~ Deere” =0. a It follows that co = 0 and cee tr DR +r Deane ‘The only solution we obtain is y(z) = 0. 284 Exercises 6.3 Substituting y= DSp cua" into the second differential equation and collecting terms, we obtain By! + Be— yl tys rat Elba rs ee (kr + Nereike = 0, & which implies rep =0 (ears 1fey— (e+ rt ene = 0 Icy =0, then the solution of the differential equation is y = ‘we obtain TTious, we take r = 0, from which au = (E+ Tee, k= 0,1, 2, Letting co = 1 we get c1 = 2, = 31, cy = 4!, and so on. The solution of the diferential equation Js then y= Do(n +1)!2%, which converges only at 2 = 0 87. We write the differential equation inthe form =2y" + (b/a)zy + (e/a) and fy = c/a as in (12) in the text. Then the indicia equations, 0 and identify ap a we-stratno 2 eteGeurtens, hich is also the auxiliary equation of az"y" + bey! + cy = Exercises 6.3 _____ Since v? /9 the gensrl solution is y = ey 9(2) + ¢od-1/3(2) Since v? = 1 the general solutin is y = e1Jy(z) + ex¥i(2). Since v? = 25/4 the general solution is y = crJeya(z) + e2J-sjx(2). Since 2 Since v4 = 0 the general solution is y = e1Jo(2) + e5¥0(2). /16 the general solution is y = ey g(t) + erJ-rya(2) the general solution is y = eyJa(z) + e2¥a(2) ‘the general solution is y = erJo(32) + 62¥2(32). . Since v4 ~ 1/4 the general solution is y= e1dy/a(62) + enJ-1/a(82) ey = afc) then oma Maa) ~ 32 o(2), y a ¥yt(g) 2 Pula) + Fa ol2), 285 10. a 12. Exercises 6.3 tad BYP Dey + Naty = PPV Pd 4 (ata — Lao a ‘Multiplying by 24/? we obtain Futsal +( 2! whose solution is v 2 pate) + ea V7J_1j0(A2). Jae) +eadja(Aa). Then y = 2 From y = 2"Jy(z) we find gaa Vytna ly, and oa" S04 na", t nfm = 1)", ‘Substituting into the differential equation, we have aM 4 ana + ls 1)a™My + (1 Ona Jt + nay) +2", STI + (Qn 1 Bn) + (n? mtn — Qnty + 2 TMP + al, — ny + 27 Jq] =P EU taht (2 -)l "1.0 (since Jp is a solution of Bessel’s equation) =o. Therefore, 2°Jp isa solution of the original equation From y = 27"Jq we find fm eI a ne, and of 2 — ana “Utena tay Substituting into the differential equation, we have ay! + (14 2n)yf bay = fat + 2ll + (28 ~ 2?) J 0 (ince Jp isa solution of Besse’s equation) ‘Therefore, 2°*Jy isa solution ofthe orginal equation. From y = VE Je(X2) we find of = VFU02) + 02) and. : yf = PVE Ma) + oJ (2) — F2-Y2UL 0) 286 13. 14. 15. 16. ar 18. 19. 20. a1. Exercises 6.3 Substituting into the differential equation, we have ay (4 22 ay = VE [222 UN(A2) + d2dL (da) + (922" —?) L02)] = VEO (since J is solution of Bessel's equation) =0. ‘Therefore, 7 J.(22) is solution of the original equation. From Problem 10 with n = 1/2 we find y= 2"/*J,(z), From Problem 11 with n = yal? s po(z). From Problem 10 with n= 1 we find y = xh(z). From Problem 11 with n = ys rJa(s) = ~2h(2) From Problem 10 with n= —1 we find y = 2-1J.y(z). From Problem 11 with n = 1 we find parle) =-2dil2), From Problem 12 with A= 2 end v=0we find y= 2 Jo(2z). From Problem 12 with and v = £3/2 we find y = VF Jgy2(z) and y = WE J_gy2(2). From Problem 10 with n = 3 we find y = 24Jy(¢). From Problem 11 with n = -3 we find y= e%es(2) = -25(2) (a) The recussence relation follows from Se (ery payin muha) + 2deale) = — 2 a gy + Sage (@) lam @) Las OG) 1/2 we find 4 we find SB UManey) zy" -Zearrrea a) 72) (b) The formula in part (a) is a linear firsteader differential equetion in J,(z). An integrating factor for this equation is 2”, so a “Yall =2"da(0) ‘Subtracting the formula in past (a) of Probe 19 from the formula in Example 4 we obtain Om Qrdsle)—Shele)—zdiai(2) of Bshfe) = shoal) + Bd ale) Lasting v= 1 in (5) inthe text we have zh(e)= Zieh) [ratr)dr rir) [7 = rdi(s). 287 Exercises 6.3, 22, From (14) we obtain J{(z) = —J4(2), and from (15) we obtain Jg(x) = J.y(2). Thus Ji(2) = Jala) = Ala) 23. Since wwe obtain _ eeprnr Junta) = 3. tam 24, (a) By Problem 20, with v = 1/2, we obtain Jyja(2) = 2Jaya(2) +2J-1/2(2) so that Je) = 2 (S22 ~cons)s with v= 1/2 we obtain —J_1/a(2) = 2Jyjo(2) + 2_3/o(2) so that Jesja(z) = ~ (2 +sinz) ; aad with v = 3/2 we obtain Nyala} = 2yale) + 2a) so that oy 25, Letting we have Exercises 6.3 and. Then or, since s = (2/a)y/k/me~e#/?, ob Eae 26. Differentiating y V2 (Jax%/) with respect to Jaz”? we obtain ary (Bas?) aa? + deMy (Jez) . y= are” (Fac) a2" +00 (2ea"®) ; 3 + fou! (Jez*”) ‘Then, after combining terms and simplifying, we have Wi eetay faa Sus (aa?? ey) | Letting ¢ Jox%? or a2°/ = ft his difecensial equation becomes fevers (e-uu]=4 t>0 27. The general solution of Bessel’s equation is ul) =adyalt)+easialt), +> 0. ‘Thus, the general solution of Airy’s equation for x > 0 is y= 2 (Zas!2) eB (Zo) + 02.1 az”) 289 Exercises 6.3, 28. Setting y = yEJ,(2yZ) and differentiating we obtain VEN OVE ig + 5g HOVE) = HOVE) + 5g IOV) and 2 fot aaa gape) eM OVE) + Fe MOVE) ~ ap lev) Substisuting into the oe ‘equation end letting ¢ = 2V we have ay += VENOVE) + Ni av) ~ 5m (WE) + VE (OVE) “3 [psteova + Z.novay + (-jaea] ee =? [fto+$t0+($-2) 20] = FICO +e) + — AC) Since J(t) is a solution of #2y/" + ty? + (1? — I)y =0, we see that the last expression above is 0 and y= VE A(2V) is a solution of xy” + y = 0. 29. (a) Using the expressions for the two linearly independent power series solutions, yn(z) and ya(z), given in the text we obtain Ae) = 3 (tone —a1804 + 1050? ~ 5) and Pie) = 2 (020e" ~ 08805 + 2180" ~ 352) (©) ala) satis (1~ 24) y"~ Day +40) = 0 and A=) sais (1 22)" — any + 56y =0 0, The ecurence relation canbe written Feah la) Ele =, 3,4 Pes(2) = 31 32, 33. Exercises 6.3, a5: Petey = Me (8p8_ 38,9, 18,) 8 (384 Wye 1050 8 keds Pola)= ge aot es) age eet 16° 6 - _3. 5) 5 (63.5 35,5, 15, bee Ree) Fe (pe Spt ee ae) aCe aot a) 29,7 _ 8,5, M52 167 "167 * 367 If x = cos then and oe That is, fy cael éy ay sin gr + c0s 655 + n(n + 1)(sin dy = sin8 (1 - cos? 6) Gyr ~ 2008 + n(n + I)y] = 0 n& 4 (0-2) ne From y = u/ Vi we find Qru'—u yp ar5/? — 4x 3/2y! + 32M Py ya agg yt ae oe Then ir 8/2y" — 4z9/7y! + 322y Qzu’ —u Aye pee + (ei Cee abaya ite Multiplying by 24/? we obtain For large values of 2 this equation can be approximated by u” +-u = 0 whose solutions are sin and cosz. Thus, we expect the solutions to oscillate with increasing frequency for larger =. Rolle’s theorem states that for a differentiable function {(2), for which f(a) = f(8) = 0, there exists a number ¢ between a and b such that f‘(c) = 0. From Problem 20 with » = 0 we have (2) = Ja(z). Thus, if a and b are successive zeros of Jp(z), then there exists a c between a and b for which Ji(z) = J§(2) = 0 291 Exercises 6.3 34, Since [jf f(x) dz| < (2 1f(2)] dx, we have Mala) < =f" |cos(asint ~ nt} at < 38. (a) We identify m .& anda = 0.1. Thea a(t) = eydo(1e 248) + op¥o( 069%) and 0.5e2¥4(10e- 0!) 2'(0) = -0.5e,5(10e" Now z(0) = 1 and 2/(0) = ~1/2 imply exJo(10) + ex¥5(10) e1J5(10) + e2¥§(10) Using Cramer's rule we obtain J5(1O) FOC} and = (09) 2 TUTOVHGCIO) = Jet) ¥OC TO) Using ¥f = Ys and Jp = ~J. and Table 6.1 we find o1 = ~4.7860 and cp = 3.1803. Ths x(t) = —4.7860J9(10e~%) — 3.1803¥e(1de" 9%) (») 36. (a) Identifying @ = }, the general solution of 2" + tr =0 is 209715 (1x9?) + ope pg (12%? 1245 ($249) + xen ($29) Solving the system 2{0.1) =1, 2/(0.1) =~} we iad e; = ~0.809264 and cp = 0.782397. (b) Pe 10 150 292 Exercises 6.3 87. (a) Letting t= L— =, the boundary-value problem becomes on rene #0) = 0, (L) where o? = ég/ET. This is Airy’s differential equation, so by Problem 27 its solution is 2 ya at hy Fat) eats (Gat?) = cate) + eatato (b) Looking at the series forms of 8 and 6 we see that 6)(0) # 0, while 6(0) = 0. Thus, the boundary condition 6'(0) = 0 implies ¢ = 0, and so A) avr yn (2a) From 6(Z) = 0 we have av sys (Gaus?) =a s0 either cp = 0, in whick case @(t) = 0, or Jyya(GaZ%2) = 0. The column will just start ‘to bend when Z is the length corresponding to the smallest positive zero of J-y/a- Using Mathematica, the fist positive root of J_yjg(2) is 2) * 1.86635. Thus JaZ%!? = 1.86635 implies Wy Le (case) . [z ie ce =| sons] '9(2.6 x 10")m(0.05)4/4 [eSgasass O5F 38. (a) Writing the differential equation in the form zy” + (PL/M)y = 0, we identify A = PL/M. From Problem 28 the solution of this differential equation is veh (2yPiaiM ) + vey (2yPla/it) 1" casesa57) = 76.919 y Now Jy(0) = 0 so (0) = 0 implies ea = O and yravts (2, /Pizjit) (b) From y(L) = 0 we have y = J1(2LVPAT) = 0. The first positive 2er0 of Jy is 3.8317 so, solving 2Ly/Pi]M = 3.8317, we find P; = 3.6705M/L*. Therefore, n(e) saves ( a ) movin (22) 293 I Exercises 6.3 (©) Pore; = Land £ the graph oft = VEASSIITVE) is shown, 02 a 02 04 06 08 1 ie Zeros and differences of Joy Jiy and Ja a t ah, b a. 2.4088 0.0000 9.0000 5.5201 3.1153 3.8317 3.8317 $.1386 5.1356 8.6537 3.1336 7.0156 3.1839 8.4172 3.2816 31.7915 311378 1011735311579 11.6198 3.2026, 4419309311394 13.3237_3.1502__14.7960__ 31762 Successive 2er0s of Jq for n = 0, 1, 2 appeat to be approximately equally spaced for larger values of 2, Furthermore, this spacing appears to be approximately x. ‘To check this conjecture a CAS can be used to determine that successive roots of Jp near 2 = 200 are 200.277156 and 203,418739, with difference 3.141583. This is consistent with the observation in Problem 32 that the frequency of the solutions for large 2 is approximately 1/2 40. (a) Using a CAS we find 1 Hest) Liss 3062-32) 1 ssa - aps? j(sse4 - 9027 +3) LigaeS — 70a! 3( 6805 - 7029 +152) Hone? ~ 3152 + 1052” - 5) tant one 180 294 Exercises 6.3 (b) a Py a Pe Siydq (c) Zeros of Legendre polynomials AG): 0 Pale): 20.57735 Aa): 0, 0.77460 Pa(z): 0.33998, +0.86115 Fe(z): 0, +0.53847, +0.90618 Paz): 2023862, 0.96121, 40.9347 Pr(z): 0, +0.40585, +0.74153 , £0.94911 Piola) + £0.14887, +£0.49340, 40.67041, +£0.86506, 0.097391 ‘The zeros of any Legendre polynomial are in the interval (—1, 1) and are symmetric with respect 00. 295 Chapter 6 Review Exercises Chapter 6 Review Exercises 1. The interval of convergence is centered at 4, Since the series converges at ~2, it converges at least on the interval 2, 10). Since it diverges at 13, it converges at most on the interval (~5, 18). Thus, at Tit does not converge, at and 7 it does converge, and at 10 and 11 it might converge. 2. We have fe) = 3 Bat 8. Solving 2? ~ 22+ 10 =0 we obtain = 1+ VIT, which are singular points. ‘Thus, the minimum radius of convergence is 1 Vi] = VII - 4, Setting 1—sinz = Owe see the singular points closest to 0 are ~35/'zand r/2. Thus, the minicourn radius of convergence is «/2. 5. The diferential equation (2° ~ 22)y" + y/ +y = 0 has a regular singulae point at x = 1 and an v y icregular singular point at and, 6. The differential equation (x — 1)(z+3)y” +y = 0 has regular singular points at 2 7. Substituting To onc into the diferential equation we obtain any ty ty= (2? ras + SR Mik +r Neat tra tail =o a which implies oe (2r~1)=0 and (et FY Qk + 2r = Deg + cea = 02 ‘The indicial roots are r = and r = 1/2. For r =0 the recurrence relation is oF k2,2,3, a aden Yor r= 1/2 the eecurcence relation is cee gays REND Farsi) F128 296 Chapter 6 Review Exercises ‘Two linearly independent solutions are 8. Substituting y = Ceecn2" into the differential equation we have v-ay/ a sn = Nott Fa” = F ot mae roan rn = Svat nocuet~ Etat East = en — 09 + [lk +2)(k+ Denaa — (h + 1)ea}a* = 0 fi Thos 20 ~ = 0 (6+2)(k+ Dewsa ~ (+ Dog = 0 and casa pipes R=123, m2 = EEO 2,3, Choosing 9 = 1 and ¢; = 0 we find 297 10. Chapter 6 Review Exercises and s0 on. Far ey = 0 and cy = 1 we obtain and Substituting y = L22q on2" into the differential equation we obtain (a= Uy" + By = (~2ea + Ben) + Yo (b = 1)(k — 2oea ~ BR = Neg + 3ee-a]a*? = 0 Fa ‘which implies ¢2 = deo/2 and (R= AY(K = Degen + Sera E= 1) ee Choosing eo =1 ade) =O we Bnd ee 2 8 and son, For ey = 0 and a ¢ ast i and 0 on, ‘Thus, tw solutions are (143s? bed 4 Sat ge n=O (14 Se bat + Baty ) = 1 1 waa(erostte...) na(etheehts Substituting y= D2 cnz" into the differential equation we obtain Yl 2 + 2y = 2ea+ Goa tele + Sk + AUR 2eaga~ (bale = 0 a which implies cp = 0, c3 = ~c0/6, and cuss un Chapter 6 Review Exercises ‘Choosing cp = 1 and cy = 0 we find and so on. For ¢9 = 0 and ¢) eso = 0-20 eee =a = e=a=c= and so on, Thus, two solutions are 1 wo (1-}e- da...) and peas. naa p age) atm Substituting y= E229 eq" into the differential equation, we obtain ay" ~ (242) +2y= (P—aeoe™! + Tilt Mk tr) = (ker Seale = 0, which implies ar =rir—3) and (ka r)(k-tr=S)on~ (+r Seq = 0 ‘The indical roots are r1 = 3 and m = 0, For ry = 0 the recurrence relation is K(E~ See (k- Doxa 0, k=1,2,3, ‘Then e1-c=0 2a-a=0 Oy — Oey = 0 => 65 8 arbitrary ond 1 cya Geen B= 4,56, 299 Chapter 6 Review Exercises Taking o #0 and cy =0 we obtain Taking cy = 0 and oy #0-we obtain ‘and so on. In this case we obtain the two solutions = _. malted sat : 64, 85,9 ooo oe = 6 — wre Gat y Gabe abt (1 12, Substituting y = 5%p.e,2" into the differential equation we have (cosy +y ) (2en-+ Sexe + 1Deqz? + 206524 + dopa +») +See" Ss 2 1 4 = [Ben + Genz + (124 ~ cn}o + (2005 ~ Sen)ot + (3006 - bea + Zyen) 24 +--+] tle teie tens? +eye? tayet + --] . 4 = (+20) + (+69) + ae +(e — Dae + (000 ~ 54+ en) + Thus cy +202 e+6e3 Tea 300 Chapter 6 Review Exercises =O ae a and so on, Thus, two solutions are 1 5 ttt and y= a- Synz into the differential equation we have 1 : a ast é 13. Substituting y = Ye +2y = Fo nla = Noga 2+ So nega” + 23> en" 4 f= i Tae aa es = ORF 2k + Vesa + So Regt +20 cy Bi i = don + 2a + Sell + 2)(K-+ Doea + (+ Dewlo & ‘Thus 2ep + 2c = 0 (b+ 2)(k+ Donan + (b+ 2)en = 0 and 2 ae oe F=1,2,3, 301 Chapter 6 Review Exercises Choosing « = 1 and ¢1 = 0 we find ‘and so on. Thus, the general solution is ei ma(I-ate jet dite.) ea(e- yra(t-2sie ioe and 42 = 3224 St 28 eo (-22+ $22 + ---) ter (1-5? + at yeh ) Setting y() = 3 and y'(0) = —2 we find ap = 3 and cq initial-value problem is 2, Therefore, the solution of the e320 tah +a! 14, Substituting y = C2. cn2" into the differential equation we have (e+ Dy" + 3y= Fo nln Dene! +2 FS mln egz™2+ 3S ona = ot ed a it & den + Sep + Folk + Deas + 20k + 2)(K + Dena + Seale 2 ‘Thus ey + eq = 0 (k + Wheugs +2(k + 2)(6 + Loken + Seq =O 302 15. Chapter 6 Review Exercises and Wk 3 Were” BESTEST) Choosing ep = 1 and o1 = 0 we find Ph cy and s0 on. For ep = O and 1 = J we obtain and s0 on. Thus, the general solution is 1 ma(i-t24% a eo taty tt pea (I- pts gos dat Babe )eg (ete s Late) meg (—3e4 8224 ,8— 9, aa (1- Sey het vreo (rade ies Sate )ea(i- fete it+ Setting y(0) = 0 and y'{0) = 1 we find cp = 0 and c, = 1. Therefore, the solution of the initial-value problem is ) } La ‘Writing the differential equation in the form 1 = cos y+ xf +2v=0, and noting that oor t_ 2 22 tT is analytic at 2 = , we conclude that 2 = 0 is an ordinary point of the differential equation 303 Chapter 6 Review Exercises 16. Wilting the differential equation in the form and noting that “8 ies 27 we see that z= 0 isa singular point of the differential equation. Since a? at at e ee "G4 zie wwe conclude that 2 =0 is regular singular point. LT. Substituting y = D%9¢q2" into the differential equation we have yl + a2y! + 22y = Fo nln — Tega"? + F negate! +25 ear i rot © = Ft + Neat! + Fk Nees’ +23 zt oy roma & me a = on + (Gey + Dog) + Fick + DKF Dena + (E+ Yesalz = 5 — Br + 102! Tins / 2-5 65 +20 ea +301 = 2005 +4e2 = 10 (42K + Newest (E+ Deer =0, K=45,6.005 and Chapter 6 Review Exercises Using the recurrence relation, we find ea aot ea i Taw i 3 2 18. (a) From y ‘Then dy/de = 2? +y? becomes 1déu 2 ate ful 0 True. (b) If 2 w(}2?) then veoty (2) +te¥e (22) and 305 Chapter 6 Review Exercises tava att [tv (12) sav (le) +(e Je) u(2e)] 0 Latting ¢= J? we have VF fore) + 20't) + (24-42) we] <0 ute sw+ (2-2) ww =o. ‘This is Bessl’s equation with v = 1/4, 50 w(t) = erdyalt) + erd-aya(0) (c) We have 1 @ an, Teagan eo [and dt tug Pree Now dy & AE wo tt Flea) + ond-salt] + erdyalt) + ordyald) mater (att) ~ Layald) +02 (-Zoaale) ~ ytd) terdyalt) + e2d-yalt) dextd aya) ~ dato) =m (te) -200%a (12), Jaya 2?) + o2dayal $22) evdiyalge?) + ead yal?) 306 Chapter 6 Review Exercises Letting ¢ = cx /oa we have 1. Let no= efn(0+s) in —2))-1 so that i + pln +2) —In(a ~ 23} and 1 “> sar * oT Li = 2° Gat Then (1 a)( +2 )uh — 2094 + 2y2 = 0. 20. (a) By the binomial theorem we have Sa ae 1 : [i+ (2-200)} L (eat) +8 (@ an) eet tated (set a) + (b) Letting ¢ = 1 in (1 = 2rt + £2)-¥/?, we have Gate Peay te ratte eee. 0 5. 2s} = ["(sinsjerat = (- 1s) 1 =0+3 ae 6. ZO} = [loose dt = a aot (orgigen®)=-gljen, ooo raveft Sst eEO) = [Pret a= (te Leta det, soo 0 Octet B10 {on 1 eu f (t= De *aa2(-te-1 10. nL. 12. 13, 14, 15. 16. Exercises 7.1 1-t, Oo . FU} = ftetane [Peroae ete = (eet LU} = [Pet ernat Hs Ops [Pretertare [rnd = (Ae . aor") . 1 se LUC) = [Pe Mersta mf terete (1 prertortn 2 sertooms_ 2 erteer) (Pg Be apt a) P= aR LUM} = [etsintyertat = [* sinsjer erat «(ephemera 1 1 * Gripsi” Syaee? 977 240) = [Petonetae [bonne 7 ELA cog 1 __sa-sitgint) | ate on) iss “Gage” Pager! SP! 309 Exercises 7.1 ar. 21900) = fC teosnente = (-sh rt s=4) (cost)e™* + ea. = S24, soo ery °? 18. 2{fO} = [ tsintjemee wc «(stg Ah) conga)” ~ [Con ath) oe (aaa) oo s>0 20. £105} a1. eue-p= 5-8 a ¥n+a-3 23. £(P + 61-3} a 2 9 24, L(A 4 e+ 9} = 45 + B+ S 3 25. £8497 +341 26. 2(81~12°+64-1 ar. e(.sety 28. ef 29. L{L+ 2% +e} 30, fe ai 4 panne) = 3 - 33. {sinh kt} -F 35. £{csinht} -¥ {« 38. Lfe“teosht} = ¥ {5} 37. £{sin 2tcos 21} = #{} sinat} = Es 310 Exercises 7.4 39. (a) Using integration by parts for a > 0, Ta+i = [Pre ‘d= lent dt = al (a). (b) Let u-= st so that du = sdt, Then sey = Coteas Pee (2) hae caret), a> 40. (a) (rv? () ey?) = POP. (©) 20%) = EOP)» ve 41, Identifying f(t) = we have (8) = ne}, n= 1, 2, 3,-... Then, since f(0) = 9, m2} A200} asH(0) wd Ste) @ Beery Forn=3, ¥(t) 42, Let F(t) = 1. Then F(t) is of exponential ceder, but f(t) = F(t) = }172/ is unbounded near t= 0 and hence is not of exponential order. 3 43. 44, 45, 46. Exercises 7.1 Let 7() = 212" case!” = 4 sine”. This function snot of exponential order, but we can show that its Laplace transform exists. Using integration by parts we have eee ome) = [Pee (Zine) de im, [etaind” +s fetsine” a] a =s f° etsine” at = 5 H{sine”} Since sin eis continuous and of exponential order, {sine} exists, and therefore ${2te" cos e"} exists. ‘The relation will be valid when s is greater than the maximum of ey and c2 Since et isan increasing function and t? > In M+ et for M > O-we have e! > els = Met for ¢ sufficiently large and for any ¢, Thus, e is not of exponential order. By part (c) of Theorem 7.1 1 1 (s-o)+ib_ s~a +a 20) = ap 7 Ura Gra) ea” GaP By Euler's formula, e! = cos +isin#, so Wl) = 2 (tte) Pecos be + isine)} = £(e* cost) +12 (0% sintt} — yee “poaeR Equating real and imaginary parts we get ead Boar re 2 sine} e We want faz + Sy) = af(2) +5f(y) oF L{e* cost m(az + By) +b = alms +0) + Aimy +8) = mlax + By) + (a+ B)b {or all real numbers a and 8. Taking a= 0 = 1 we see that b= 2, so b= 0. Thus, f(z) =mr+6 0. will be a linear transformation when 312 Exercises 7.2 Exercises 7.2 313 16. Vv. 18. 19. 20. 2. 22, 23. 24, 25. 26. a7. 28. 29. Exercises 7.2 Boe FOR wae} a 5 +e ‘ val fii aponeeneca} 2G ees — {asin = 4492" + cost + 3sint Pn ese ae 1 Jee {se-3 gogann ve che ia ae 2terierah 30. sl. 32. 33. 34. Exercises 7.2 643) i i spmesa}7! ig tae eye ae 1 = Deast + sint — 2eos2t— 5 sin 2 ‘The Laplace transform of the diferental equation t i 1 sity} v0) - 2) =2 Solving for {y} we obtain £0) Thus ya-lte ‘The Laplace transform of the differential equation is 2s£{u} ~ 2y(0) = L{y} =0. Solving for £{y} we obtain ,-6 3 4W- a ap Thus eo? y ‘The Laplace transform of the differential equation is s£{u) — v0) +6-£{y} Solving for £{y} we obtain Stu} -DS+8) ‘Thus ‘The Laplace transform of the differential equation is s£{y} - £{y} Solving for £{y} we obtain te oe TS 43 13 F435 Thos 5 cosSt + 2 sinSt. s+ Exercises 7.2 35. The Laplace transform of the differential equation is Lu} ~ sy(0) ~ v'(0) + 5s L(y} — ylO +440} = 0. Solving for #{y} we obtain eu} 1 a ‘Thus y= 36. The Laplace transform of the differential equation is #£{y} ~ o4(0) ~ (0) - 4[s L(y} ~ ¥O)] Solving for <2{y} we obtain 6 3 *0)= Gaye” eB} Thus 37. The Laplace transform of the differential equation is L(y} - sy(0) + £(u} 2 Pee Solving for é{y} we obtain 2 1 | Ws | 22 Aub = Gepetey teri” Pai a1 eT Thus y= l0cost + 2sint — V2sin v2t. 38. The Laplace transform of the differential equation is PLY} +920) = > Solving for $(y} we abtain “org Thus Exercises 7.2 39. ‘The Laplace transform of the diffesentiaequetion is 2[s {ad ~ £20) ~ s¥'(0) - vO] +9? ¥(u}-sH0)-¥ (0)]~31s fy} —viO]-2 240) = > Solving for ¥{y} we obtain las wetness nite 1 2eei +2 ‘Thos 40. The Laplace transform of the differential equation is 8 Ly} 20) yO) -¥"(0) +218 Lu} ~ 910) -¥(0)]- [6 tv) - 0) -2Z = gg Solving for ¥{y} we obtain t+ 12 =U(sF Ils + 2V(s?+9) gee sl 2{u} Besi wse2t I FTO PTO ‘Thus 13 13 16 3 1 va Be Beta Betty & cosa Eins 1, 30, #1 41, Le f= rand gt = {0 P20 81 then 2{s(0) #1010} = bo 10 4 0 42. The Laplace transform of the differential equation is _8t3 s43 20) + £0) = Caigi = Freer Solving for fy} we obtain eH S+68413 Dt “grea leegeed” Gayrea =m, Law —eMeosat + e°™ sin2e 43, For y" ~ 4y/ = 60% — Set the transfer function ia W(e) = 2/(s# — 4s). The zero-input response is 2G Ea w= 2 { gS} = 10. AL . L{(1 =e + 367) cost} = 2 (cos st — Exercises 7.2 and the zero-state response is 6 3 nae {s Hea GFT Ta} Exercises 7.3 L ee) = age 2 at ty sett} = 1 2 poe fiet sat rte) = at et gp fem - at ee) esa 2 = 2} a 2h ae eh a fe 3 ¥{etsn3}} = eg 242 ¥ (Mest = St cost + 3e~ cos 5t) 10 1 3s+4) “FB Ge GH 5 3 t Gays 2(e"(0-s+ 0808} teem {00 02" 10% sa} Exercises 7.3 id «tel ener 15. cost ~ 2 sint se of eta ef ta Ls = 2° cost ~ Le" sin St 246TH GSR +R 5 CTH Tw age nd 18. jee ea at ip} s se ioe w. 3 ee 20. “teeter ey Lett 21, The Laplace transform of the differential equation is s¥{9)-vo)+ 42) = hy Solving for P{y} we cbtain L{y} = Thus iG wry ta patent 4 2H, 22, The Laplace transform of the differential equation is s4ty} - 21) Solving for {y} we obtain, e{yb= ‘Thus 23. 24. 25. 26. 27. Exercises 7.3 ‘The Laplace transform of the diferential equation is $*£{u}- su(0} ~¥ (0) + Is2(v} ~ v(O)] + L{y} =0. Solving for <2{y} we obtain 343 40- Tar ‘Thus poets tet ‘The Laplace transform of the differential equation is 3° £(u} ~ su(0) ~ (O) ~ 4s ¥{u} - v00)] +42) 1 Solving for S2{y} we obtain E(u) = 55 ‘The Laplace transform of the differential equation is F£(y} ~ 540) -YO)~ 5s 2} - vO] 49:20) = 5 Solving for £(y} we obtain ti iF Thus i Be ‘The Laplace transform of the differential equation is #2} - s4(0) - VO) ~ 410-L(9} - vO) +420} = S Solving for £{y} we obtain 3 G-3F ‘Thus ‘The Laplace transform of the differential equation is 3? £{y} ~ sy(0) — (0) — 6 [2 L(y} ~ y(0)] + 13-2} Solving for ¥{y} we obtain A 20h aE 320 Exercises 7.3 Thus 28, The Laplace transform of the differential equation is 2s?-£{y} — sy(0)] + 20{s £{y} — y(0)] + 51 F(y) = 0. Solving for £{y} we obtain - ass P= FEEDS TE TESS GHP aT” HEP HIA Thus y = 2e-™ cos(t/ v2) + 10V2e~® sin(t/ V2), 29. The Laplace transform of the diferential equation is s-1 {9} ~ sv(0) ~¥'00) ~ 6 #9} - VO = GEG Solving for {y} we obtain 1 20) Farm Thus 80. The Laplace transform of the differential equation is Fy} ~ su(0) — y'() -2[s¥{u} - vO +5. L(y) = Solving for £{y} we obtain ¥{u} B45 sl 2 2 (s-1eee Thos Slt int cos2t + Feet sin2 81. Taking the Laplace transform of both sides of the differential equation and letting c = y(0) we 321 Exercises 7.3 obtain LU) + (ay) + (a) =0 #82 (y} — ald) - ¥(0) +25 209} - 2010) + £09} =0 PY(g} ~co— 2428 Ly} ~ 20+ Lu} =0 (842941) 200) =e 42242 : cs | a+? 0 Geet GaP Lstict, tra =Car t Gre oe ‘Therefore, tip} rete et anet To find © we let y(1) =2. Then 2= ce"! + (c+ Net = (e+ Ie! and e=e 1. Thus ult) = (e— Det + (e+ 1)te* 32. Taking the Laplace transform of both sides of the differential equation and letting ¢ = y/(0) we obi, Ely"} + £{8y'} + 2 {20} = 0 PL {y) — yO) + 8s L{y} + W¥iy} = 9 L(y} ~ c+ 8s L(y} + 2L{u} = 0 (2 +85 +20) 2{y} =e ne Fse00” (er arsa Therefore, uaF in 2. (ori To find c we let y/(r) = 0. Then 0 = yr) = ce and c = 0. Thus, y(t) = 0, (Since the we write 4 208-4 a2 m (o43}#~ (A? uw), 9 (Cealing that o sta x 1 Yah eac(} ~ GF=OFa ” Gea aa) ‘Ths for A> w, a = = eM cosh YP ut = sinh (Po? at) ac(1 YIP ~ Toph i ‘) For Ac wwe write 2420940 (644)8+ (u4— 2), 20 os , 20-8) -aaytteay sao) ‘Thus for (S074) Thus €) = 100(e-!9 — eH) U(¢ — 1) — s00(e20-) ‘The differential equation is i 005d e+ ahs fete = 200} —(¢- Ue — H] a | : +2001 + 10,000 fi(r}dr = 20,000[¢— (¢— 1) %(¢— 1), : ‘where s(0) =0. ‘The Laplace transform of the diferential equation is TEI 10,000 s£{i} + 200£(3} + {8 ) = 20000 5 - % -# Solving for fi} we obtain c 20,000 2 200 20 = ey roo 9 = |S — Sp ~ Go 10] Thos At) = 2 — 2er 10 — 2004-1" — Deut — 1) + 26-ME—D aye — 1) + 200¢¢ — Te Y(t — 1), 342 Exercises 7.4 47. The differential equation is a Fai Bl, where i(0) = 0, The Laplace transform of this ‘equation is S28) + £43) = ZEW} From Problem 21 we have Z{AUW) = 3 Thus and 0-H =(}-ph)a-en- ett aon) = (bey )on tet tetany) Therefore (0) = [1 — 24 ~ 1) + 206-2) — 243) +24 4) — +) ets Be Nae — 1) — 22-9 49) +26 MNay— 3) — 2-4) 4) sen ot ~(t-0)) (2 — n). 343, Exercises 7.4 48, The differential equation is a Sri= 20) where i(0) = 0. The Laplace transform of this equation is s£(i) + 2) = (50) From Problem 23 we have L{E(t)} = Tos = 7 1 a sel) siete 1 Ly 4 Gre) Gg evect = (R-dtaa)- Gear terete ‘Therefore ~ Leet) (1 eA 1) = (1 eM U(E- 2) (1-23) ae 3) - (Le DUC = 4 ~ == 1He)— Fa eee ~ 49, The diferenial equation is "+2 + 10x = 20/(t), where f(¢) isthe meander function with o = x Using the initial conditions 2(0) = 2/(0) = 0 and talking the Laplace transform we obtain ) = net 4 20 26 4 =248 Fcapem 344 Exercises 7.4 ‘Then 20 40 = yn ns He) = Serra sa * HPF ast Pe 2 med Bid 8] awe t, When a <¢, then 4 rt =7)? it [e-nuc-oyar= fie-nar = fe beat When a >t, then (ra) =0 since + <¢< and fe 7) (r -a)dr = 0. ‘Therefore eu(e-ayn (BOO #0 Ly gra oy teaue-ay= {Ht (rea de-aPae~o). 58, First method: By the definition of the Laplace transform and integration by parts we have {firmed = [eo (Lrre)a [rae Pewed Second Method: Let a(t) = ff f(r}dr; then g(t) = (4) and g(0} = 0. By Theorem 7.8 £{G(8)) = 8 Z{9(8)} — 910) = s Z(H}, % eu =se[ fine}, ond offre} =teue)-“2 346 Exercises 7.4 SA. Let u=t—7 so that du= dr and Seam [s(elote—ner= [Ff vatudu = 9 55. (a) ‘Theorem 78, the Laplace transform of the differential equation is a @ . ~ZIPY — oul0)~ v (0) + s¥ — vo) + Stsy — yi0)} + mY din 4 =-glevisey + Zpyieny InY =nla(s—1)-(1ta)lns+e Since the differential equation is homogeneous, any constant multiple of a solution will still be a solution, so for convenience we take cy = 1. The following polynomials are solutions of Laguerte's differential equation: neo: 1y=27{3} nel: Leet {i “f (b) Letting f(t) = tet we nate that /(0(0) = 0 for = 1, 2,8,..., 21 and (0) = nt 347 Exercises 7.4 Now, by the firs translation theorem, {5 Zerert) wb ets} = EEL} hee [rewey *4{0} — 8-9 /'(0) — --- — FO-N(0)] = dreert] where ¥ = {Ln(t)}. Thus t= 5 Fee}, n=0,1,2, 56. (a) The output for the first three lines of the program are ove] + 6u'ie| + yt 1-25 OY +5¥ 46(-24 s¥) sin 2s aH 43 — 229? — 438 ~ 11st ~ 255 (1+ 57)7(0 + 63 + 5?) ‘The fourth line is the same as the third line with ¥ — removed. The final line of output shows ‘a solution involving complex coeficents of e and e~. To get the solution in more standard form write the last line as two lines: euler = {B°(I t)-> Cost] +I Sinft], E°(-I t)-> Cos(t]-1 Sin{t}} InverseLaplaceTransform|Y,s,t]/.culer//Expand ‘We see that the solution is ue) = (So + he) ets Se ascost — iBheost —9sint + 20sint) (b) The station is Neots yiB 1 — VEE ot in VE 348 Exercises 7.5 (6) The solution is Exercises 7.5 1. The Laplace transform of the differential equation yields Lu} so that y= at — 9. 2. The Laplace transform of the differential equation yields sue be so that ya2eteeNae—7) 8. ‘The Laplace transform of the differential equation yields Ye}=a5 (+ tm) so that y= sint +sint%(t— 2n). 4, The Laplace transform of the differential equation yields ane so that nd(t — 2n)AU(t ~ 2m) 5. The Laplace transform of the differential equation yields 4 1 a n8/2 4 9308/3 sin (em eon) 349 10. u. Exercises 7.5 so that vasa = = cost 9 (t- s)en(-§) 5. The Laplace transform of the differential equation yields 5 a Luba git gem te so that y= cost + sin [%U(t — 2r) +U(t - 4n)} ‘The Laplace transform of the differential equation yields Jars iS-iss so that : anda — 5-4 fue - 0. . The Laplace transform of the differential equation yields 20 a aT so that ‘The Laplace transform of the differential equation yields i = Gay so that ~%-29) sin £91 (¢ — 2), y ‘The Laplace transform of the differential equation yields i #0 Gripe so that y= (t— erage - 1). ‘The Laplace transform of the differential equation yields dts m4 eo See" FFTs 23 oo 2 1 a Geree* Gees pane ete) 350 12. 13. ua. Exercises 7.5 so that Hedatt re teas 4 HEB ane ME) + Jot singe — 32) Ut ar). ‘The Laplace transiorm of the differential equation yields so that 24 Lead —2 4 Loea] ago ‘The Laplace transform of the Cae equation yields 20} =F4v'0+2 So +2 so that 1 24 1ymgyg3 41 ym fu0)2? + a" Oa? + oe (x 0 and y"(L) = Dwe obtain Using y"(L) BG? 6 Ae Ly L GET i ~w) Fsesh From Problem 13 ve know that 1 wq)q? + A yyoyz3 + 2 2 Lys L pu Or?+ "2 +2 (x3) a(e- 3) Using y(L) = 0 and y/(L) = 0 we obtain Exercises 7.5 15. The Laplace transform of the differential equation yields Lty}= = so that y(t) = sinwt. Note that ¥/(0) = 1, even though the initial condition was (0) = Exercises 7.6 1. Taking the Laplace transform of the system gives s£(z} = -¥(2} + 2} s¥{y}~1=22{2) so that L L Hh= Gage 735-135 and Fh 5+ Tae Then 2 2 2a dae and y= Jett ze 3 2. Taking the Laplace transform of the system gives s¥{r} sL{u} so that ret 1 Se-D@ =) 6s 160 344 18 Be 58, iB° Ie Then ly e+ en, SB, ae oe 192 20 8. Taking the Laplace transform of the system gives o£ {x} +1=2{z}-22{y} sL(y} {a} ~ £{y} 352 Exercises 7.6 s0 that ess = eg Bes and 7 za —cos3t— Zsindt ‘Thea 4, Taking the Laplace transform of the system gives, jl 1 (+3) £(z} + s£{y) =| {s— 1) £{z} + (s- DFG} so that and feted Thea 5. Taking the Laplace transform of the system gives @s-2)2{2}+s¥(y)= 2 (6-9) £{2} + (6-9) 4(y) =? so that sad Then so cbabeead aed yo 6. Taking the Laplace transform of the system gives (s+ £(e) - (6-200) Zz} + (s+2L{y} =1 353 Exercises 7.6 so that . era) s41/2 fhm ay eai ” Gaia iF = 5/2 -3/2 20h ay” Gai Then V5, =e ons Ys and ze y cos sin 2 7. Taking the Laplace transform of the system gives (+1) 2{2}- Ly} = -2 ~ #2} + (P+) Ly so that ape tat Ab 8 a MTS Sa 2eyD = 1,3 2 ~ pga. ‘Thea tere —tes Doin yest ere tee ss sinvit 8, Taking the Laplace transform ofthe system gives (+N ¥G}+ 20 42 {2} — (s+ H L(y so that 842 at ed YO)~ aeaes3 * Genlee * 2 Gee F and +3 +l 2 20> ar ye68 Ga? GT Then 1 ‘cos2t-+ 5e"tsin2t and cos 2 + 2¢~* sin 2 9. Adding the equations and then subtracting them gives 10. a Exercises 7.6 Taking the Laplace transform of the system gives and so thet ‘Taking the Laplace transform of the system gives (9) Aa} +S 20} = 5 (s+2) 4(2} — 289 fy} =o so tat ‘ cad #8) = Gry) "53-3 8 aT RT ond re Ay} = S(s— ee +1) Ten and ‘Taking the Laplace transform of the system gives P La} +354 Ly 2 B{2) + 34{y) so that 2e41 #6b=-se Then le eeltteitne and ly i 355 Exercises 7.6 12, Taking the Laplace transform of the system gives (5-4) £6} +22} = -3.¥(2} + (s+ D0} =F so that oe 1 40} = Gays 5 +8" Ge and Then sade dete Lert eev}ae—n and 4 2ev]ae—n 13. The system is af = 32 + (20-21) Y=-%n-n) (0) =0 xi(0)=1 n(=1 2(0) =0. ‘Taking the Laplace transform of the system gives (0? +5) {21} ~22{22} =1 ~2£{1} + (8? +2) Lf} =8 so that Pn ee a eS TET SSL SRST STE VE +E and 356 Exercises 7.6 ee eee 2 Fes SSF Se 5Ve +6 Thea oe: 4 cost-+ aint — Zone VBE + he in gots geint— = Vets vot = 2 1 = som Zeost-+ 5 eint + 5 cos VEt~ 5 sin Vb. 14. In this system 1, and 2 represent deplacements of masses my and my from thelr equilibrium poikons, Since the net forces acing on ry ad mp xe hay + hy(e2~ a) and — k(z2 21) ~ kaze, respectively, Newton’s second law of motion gives mya = —hy21 + Felz2 - 22) ‘mast = —ko(29 ~ 21) ~ keze. Using kt = ka = ky = 1, my = mo = 1, 21(0) = 0, 21(0) = taking the Laplace transform of the system, we obtain (249) #{2))- (ah =-1 £{n)— 2+) 2{22} = -1 1, 22(0) = 0, and 24(0) = 1, and 0 that 1 roa) An} 1 gig tnd Sle) = Then Fyinvit md 22m Fpsinvie 15. (a) By Kirchofl’s first law we have i; = ig + ig. By Kirchoff’s second law, on each loop we have E(t) = Rin + Lit and E(t) = Riz +Lais or Lyih+ Riot Rig = E(t) and Lat + Rig+Ris = E(t). (b) Taking the Laplace transform of the system ti + Big + Big = 100 0.01255 + 5ig + Sis 100 gives (s+ 500) L{in} + 5002 fis} = od 4002) + (6+ 400) i) = 28 357 Exercises 7.6 so that —_— 3 3900 +9008 ~ ‘Then 200 100, 0 Td and ip = 20~ 0.00254 —ig (6) ty in tig = 20 ~ 206-9 16. (a) Taking the Laplace transform of the system, a) +i + 10ip = 120 - 1204(¢ -2) ~10i} +54) +55 =0 ves 120), sm (s-+ 10) {00} + s{ia) = (1) 10s {i} + 5(5 +1) {i} = 0 so that wet 7 anf 8 a #0) = Griese "= [Seep asa t a|C ad 240 en -[ 240 240] ae 206) ~ srpiees 2) = [gp 9] Oe) ‘hen fp = 12 + 4828/9 — g0e-* — [12 4 4Be-HH/9 — Gye 2-2] u(t — 2) ana fin = 2407/9 ~ 24ge-2 — [24de"HI-2/9 — n4ger 2-2) a4(¢ — 2), (b) iy = in +iy = 12 + 2885/9 — 3006 — fia + 288—-M-2/9 — gope-4-2)] 1 ( ~ 2) 17. Taking the Laplace transform ofthe systern ih Lig +6%5 = 50sint + 6h +6%3 = 0sint gives (s+ 12} Lia} +82 (ia) = ao 5L{in) + (6 +6) ¥ {in} 358 a8. 19. Exercises 7.6 #2 * ideo e258 * iis TT tn FT 508 m1 am 1 Mes Ael= Gays TaD 7 Mam, FTO aw, MO, 80 2 HRM pe pete on inn Bane Bg Baym Mery 2 ort 4 BE ny oe ee et 13° Ta63° m3" * 3 ‘aking the Laplace transform ofthe sytem Oi + 80% = 60 0.005% + ig - i = 0 ves 6th) +100.4(¢2) = ~200 {1} + (8 + 200) {2} so thet 24,000 616 s4100 100 a) = pq 2OO_ 2b) ~ Terms + WH) Thea 3 (es 100)? + 100? ~ 5 (oy TOO + OF in = § — $6100" cos 100 - Se sin 1004 Lad and 7 iy = 0.00845 + i = ¢ — £21 coe 1004, 0.008% + fa = BF ‘Taking the Laplace transform of the system 2, +50in = 60 0.008% + in i, = 0 — 2se2{in} +502(i0} = 2 ~200 {i} + (5-+ 200) Ain} = 0 - #00) erm FETT) 61 6 s+ 100 6y2 50y2 “55 5 (e+ lo — (Sova 8 (e+ 100) (GOVE 359 Exercises 7.6 ‘Then and ing Kirchofs first law we write iy = iz-+ig. Since ip = dg we have iy ~iy = dg/dt. Using Kirchof’s second law and summing the voltage drops across the shorter loop gives 2 FU) = iB a) so that ' : = q8O- gat Then : : i Raen 8 and ma atts dye min =E0) Somming the voltage drops across the longer loop gives = inky + £23 + Rat BO = ik + LES + Rata Combining this with (1) we obtain aig 1 AR, +12 + Riek + Za . dig 1 193 + hain (b) Using L = = C= 1, Bt) = 50e-F(e — 1) = 50e-Me#-Y Ye — 1), gS} = (0) = 0, and taking the Laplace transform of the system we abtain (o+1) Stab + # (4) = (s+ 1) £fig} - £ {9} =0, so tha deters £0)= Gearai and a(t) = Sete ging — a} eU(¢— 1) = 50er*sin(t — 1)4(¢ 4), 360 Exercises 7.6 21, (a) Taking the Laplace transform of the system 461 + 3 +80, =0 Of +f + 26 = 0 gives 4(s?-+2) 2{6)} +? 2{H) = 38 P26) + (242) L(G} =0 0 that (558-44) (58 +4) 2062) = 334 Op IFT 2244 Then 3 “ Foosat and so that (b) ‘Mass mz has extreme displacements of greater magnitude. Mass my first passes through its equilibrium position at about ¢ = 0.87, and mass mp first passes through its equilibrium position at about ¢ = 0.66. The motion of the pendulums is not periodic since cos 2t/V/3 has period /3 r, cos2t has period x, and the ratio of these periods is V3, which is not @ rational umber. 361 Exercises 7.6 (c) The Lissajous curve is plotted for 0< ¢ < 20. (d) maeey a -0.2112 2 10.655 3° 0.4830 4 0.1325 5 7014113 6 7 a 3 — 0.8327 010458 -0/3639 0.3534 10 0.4370 (@) Using a CAS to solve 64(t) = #o(¢) we soe that 6 = Op (so that pendulum is straight out) when t is about 0.75 seconds. 362 10. nL 12, . £40) Chapter 7 Review Exercises (f) To make a movie of the pendulum itis necessary to locate the mass in the plane as a function of time. Suppose that the upper arm is attached to the origin and that the equilibrium position lies along the negative y-axis. Then mass m is at (z(t), yy(t)) and mass mg is wt (22(0).20{8)), where y(t) = WsinGy(e) and y(t) = -16c0864(t) and u(t) + 16sinG{t) and salt) = yalt) — 16 c0562(t) A reasonable movie can be constructed by letting t range from Q to 10 in intrements af 0.1 seconds salt) Chapter 7 Review Exercises —_____________ AAO} = [tetas (e-neta= 3 - 2 es 3. False: consider f(t) » 12 . False, since f(t) = (et)! = et - True, since limyago F(s) = 1 90. (See Theorem 4.5 in the text.) . False; consider f(t) = 1 and o(t) = 1 ty) . Efe a A Ich _ 2 etn) = Ehisn 2 = aera is (Pea) ” ae Sean} = ¥fonae— nea ge” 363 Chapter 7 Review Exercises Joe" (hada) 1 e-vir, Love Ws vB | cont + Sea Siem] me cose + Beh sine 2 a connlt=1)(E= 1) tamale) 1 fay fine eee ran DC {acta} dee Fe a1. £{er*) exists for «> =5 . £{ee 70} =- ZF (6-8) 5. (ett Flt — RY AULE — RY} = em Lhe pf a ef [ersinar} = beter) fet i sonar} -s{ i sary . F(t) ULt — to) F(t) — ft) U(t — to} f(t ~ to) M(t - to) F(t) = F(t) U(E = to) + FC) U(E— tr) A() =t-((@— 1) + Ue —1) +e — 1) Aft - 4) = ULE ~ 1) - U4) = eeth elerts(d} = eHO-*) Pls — 0) Fls-a) 2 ar. 28. a Sf) 364 30. 31 32. 33, 34. Chapter 7 Review Exercises 1 a {et = - eo 4010)" ape F{D = sint Ut — x} ~ sin Ut ~ 3a) = —sin(t — 4) Y(t x) + sin{t ~ 3x) U(t — 3x) lw Lars LUO} = gig" gg t i .—n(9=1) ae (010) = Ga ea {= 2— 2U(t— 2) + [(t- 2) + 2]Uee—2) = 24 (e- 2A) Asya 2+ dem He10} Fe) = t= 2B ~ 1) + @~ NUE ~1) ~ (2-H B HE 2 - T]ME~ 1) + C~ )UE-2) LWh= po Gets e™ oe an 1 2 e100} « Gap Baa ‘Taking the Laplace traslorm of the diferentil equation we obtain 2 20 apt eae so that tine motel + te! yn die + 5 ‘Taking the Laplace transform of the differential equation we obtain 1 20h (yew ene he eee ee “75-17 G1? 160 GaP sR GPE so that eee eee ea v= 165° tay 169 2+ Se 2 365 Chapter 7 Review Exercises 35. Taking.the Laplace transform of the given differential equation we obtain P41 2u}= so thet «A 2y— he _-apraee— tpt jet — Bers - Zae—2)- te -arae—9 AM te-mpayey — 9) — 2 e-St-Dayge — + jo ag 2) - Resag—m. 36. Taking the Laplace transform of the differential equation we obtain S42 24254, 20) = SR 5 S—8) ae a 537i 5-8 is 3 so that ee ~ [32 Be-y lees Beseolage— 3 [ge Be) Gear feo 87. Taking the Laplace transform of the integral equation we obtain so that [ 1p u(t) =1st+ 3 238, Taking the Laplace transform ofthe integral equation we obtain 6 L (LP =6-G of Lh = 46-5 so that f(@) = St 39. Taking the Laplace transform of the system gives s L(x} +2} 42 (2} + s2{y} =2 366 40. 4a 42, Chapter 7 Review Exercises so thet 2-254 eye at = eas Then : Lea Mt an Fe feta et and Taking the Laplace transform of the system gives PL {z}+P Ly} = Qs L{z} + PL {y} = so that and ‘Then pads ate The integtl equation roia [epee = 28 00 ‘Taking the Laplace transform we obtain Thus ‘The difterential equation is, ide, de 2 gd ty + 100g = 10 ~ 10%(t - 8). ‘Taking the Laplace transform we obtain » £00 = TarF es Bom Peo oo fo 105 10 GF IOP +I ~ 10 eI +e e 367 Chapter 7 Review Exercises so that ~be 43, Taking the Laplace transform of the given differential equation we obtain Qu (La 1 8 1 up) a 2a at Er e's . — Ler 10t-5) gin 10(¢ = sue -5). tub 2.248.5 so that : Quo [Ly 1 28 Ly) a2, 29] EIL le- i" Art | where (0) = cr and o"(0) = en. Using vt rt) =O and y"(L) =0 we find upL?/24ET, c= —upL/4EI. wo fil La Ball Lys, Bb om pb Lge ft Ge Se 9d 9) ae 9] 44. (a) In this case the boundary conditions ace y(0) = y"(0) = 0 and y(n) 1 = y'0) and ey = y(0) then $8 £(u} ~ s°y(0) — °¥(0) ~ su(0) (0) + 4 2{y} =~ {wo/ ET} Hence {"(n) = 0. TE we let and ie ae Phe Qa ea — Gripe From the table of transforms we get H(sinz cosh + coszsinh 2) + 2(sinzcosh2 ~ cose sinh 2) + 2-1 ~ cos-zcosh 2) 3 ur Using p(x) = 0 and y"(x) = 0 we find = 20 (14 cosh) coche. c= MB teahsjosh=, ep = GHP. (1+ cosh) each Hence 9 = 2(0+ cosh) escha(oin sh + coszsinh2) = GBA + coshayescha{sincoshr ~ cosz sinh2) + 32(1 ~ cos cosh) (b) In this case the boundary conditions ate y(0) = (0) = 0 and a = yl) = 0. Th we let = y"(0) and eg = y"(0) then 34 2{u} — 5440} ~ s*y'(0) — sy(0) — y"(0) + 42{y} = ¥ (5(t— «/2)) 368 Chapter 7 Reviow Exercises acd r ] =o ue vo 8 cant As Q saa’ ad waa TET See From the table of transforms we get = Gsinzsians + 2(oin cosh — coszsinhe y=3 nz + 2 (sin zeost hz) +g fon o~) cnn 2-2) -con(x-$) ann (22) (<2) Using y(n) = 0 and y/(r) = 0 we Gnd two sinh f vo cosh F 9° Er anh" O° Er Sinbx Hence ot. SE sy asinhhs ~ UP, S05 (ain cosh 2 ~ cos sinh 2 Y" SET ions 22 — FE; ng Cnt Cosh — a) + TBs ee fom (2-5) oes F)on (2-3) (9) 369 8 Systems of Linear First-Order Differential Equations 1. te X= (7). Then \) 2. ex (2). thn ; Exercises 8.1 1 1-1 1 i, 1 a |x oa 370 -Q)-G) Exercises 8.1 # ots wy ee a 2 ote by—o2— 82%, Ye ss, de edge 8 Bates sy ort Bates yest: Bm ryt ser else a Gr tcyt tet 3h n+ by 462426 = dz wy = = 3r-Ty+4sint+(t—a)e", Sar sin | Jett 10, Fm Baty tdsints (¢—apetty Hara y+ Bint + (2041) 11. Since val 3) em 3-4 8) x and x=(—)e (a)et (6 o)*= (So) eae that 12, Sinoe xin (Seost— Saint x- (Seta 8i24) 4 Qeost — Asint. Qoost ~4sine © we se that 18, Since “1 1A) 132) oan cei -3 we see that 1 x “1 14. Since we se that 15. 16. qr. 18. 19. 20. 21 22, 23. Exercises 8.1 Sit more tut ia e}e see To 1 Sei to) -20 -1 = ~2e~M 40 and X; and Xo are linearly independent on —c0 0. When cy = ~1and c = 0 we find y = Zr, 2 <0. Letting c1 = 0 and op get x = 7, y = 4eM, Eliminating the parameter we find y = 4x, 2 > 0. Letting «1 = 0 and cy = -1 we find y= 42, 2 <0. (©) The eigenvectors K; = (5,2) and Kp = (1,4) are shown in the figure in part (a) In Problem 2, letting ¢; = 1 and cy = 0 we get = —2e', y= Eliminating the parameter we find y = —}2, 2 <0. When cy =~} and op = 0 we find y = —}2, 2 > 0. Letting ) = 0 and c = 1 we get 2 = eM, y= ef, Bliminating the parameter we find y= 2, 2 > D. When cy = and oy = ~1 we find y = 2, 2<0. In Problem 4, letting ¢ = 1 and cy = 0 we get z= —2e*/?, y= er¥/2, Eliminating the parameter we find y = ~4r, 2 <0. When o; = -1 and ep = 0 we find y = —}2, x > 0. Letting cy =Oand ey = 1 we get 2 3e~t, Eliminating the parameter we find y = $2, 2 > 0, When ¢} = 0 and o2 = ~1 we find y= $x, 2 <0. 379 Exercises 8.2 We have det(A— MI) = 1? =0. For Ay =0 we obtain A solution of (A ~ A)P = K is xea(2)~a[() (2) so that ). We have det(A — AI) = (A +1)? = 0. For Ay = —1 we obtain «-(;) »=(5) xea(etral(jee(j)e] ‘We have det(A — AI) = (A — 2) = 0. For A =2 we obtain. -() o-(2) weal) eal() (2) ‘We have det(A — AI) = (4~ 6)? = 0. For Ay = 6 we obtain «-() oC) soafh) eral] 380 A solution of (A — oP = K is so that A solution of (A ~ AI)P = K is so that A solution of (A ~ uI)P = K is so that Exercises 8.2 23, We have det(A — AI) = (1 ~A)(A~ 2) = 0, For Ar = 1 we obtain For Xp =2 we obtain (fel 24, We have det(A — AI) =(A—8)(A 41)? =0, For A: = 8 we obtain (? K=/ 1}. For y= =1 we obtain Then 7) o a Xnafilese| -2]et+al —2}e 2) 1 0 M(B — A)? = 0. For Ay = 0 we obtain -4 K=|-5 2 -2 K=! 0 1 sp e-[is 0 381 25. We have det(A — For Ap = 5 we obtain A solution of (A. AIP = K is Exercises 8.2 so that +(e 26, We have det(A ~ AL) = (1-)Q~ 0. For dy = 1 we obtain “i Por Ay = 2 we obtain A solution of (A A21)P = K is Geli GH 27. We have det(A — A) = -(~ 1)? =0, For Ar = 1 we obtain “() Solutions of (A — AP = K and (A ~ &uT)Q =P are 0 V2 °-(:} a a-( *) ° ° so that (: o 0 %, (9 yy? 1 i 9, 1 ° ° 28. We have det(A— AI) = (A — 4)? =0, For Ay = 4 we obtain “H 382 so that +3 29. 30. Exercises 8.2 Solutions of (A —yI)P = K and (A — 4y1}Q= P are a8 ~fealie-C4- tO We have det(A — M) = (A= 4)? = 0. For A = 4 we obtain “() *-() - x0)=( a) then ¢) = ~T and ep = 18, We have det(A ~ AI) = —(-+1)(A—1)2=0. For y = ~1 we obtain “() Sl A solution of (A ~ 4I)P = K is so that It For Ag = 1 we obtain 0 that, (ibe i Exercises 8.2 then cr 2, ¢2 = 3, and cy 31. In this case det(A — AI) independent eigenvectors are (2- AIS, and Ay = 2 is an eigenvalue of multiplicity 5. Lineatly zB " a i g x o 32, Letting cy = Land cp =O we gt rely the parameter we find y =, 2 > 0. When cy we find y=2,2<0, liminating PA Lander Phase portrait for Problem 20 2, y = e2, Eliminating the parameter we find y =, 2 > 0. 22<0. Letting cy = 1 and c = 0 we get 2 ‘When c = 1 and cp = 0 we find y Phase parte for Problem 27 In Problems 33-46 the form of the answer will vary according to the choice of eigenvector. For example, *=4 (notrrdce) ** (nants) 2 8h4 17 = 0, For Ay = 441 we obtain 33. We have det(A ~ AI) 384 34. 35. 36. Exercises 8.2 (2) n-ne (etal Domt= aint) a,» (int tot?) 7 oe ‘ (reat et eee) so that Then x= We have det(A — AI) = 4?+1=0. For Ay =i we obtain x= (24) m2 (ee Me) (a) ae 2eost > 2sint ‘We have det(A — AI) =A? — 84417=0. For A = 4+ we obtain we (Y) 0 that ‘Then so that sint— cost) sint ~ 00st) 4 Deost TH asine ‘Then sint — cost —sint ~ cost Xea Be &. aa) 28 = 104-434 =0, For Ay = 543i we obtain we (17 oe 2 7 eo (ome. Ge. so that Dees st 2eos3t Then Xea cosat-+ sin St) a a) : 2eosdt reost )* 385 37. 38, 39. Exercises 8.2 We have det(A — MI) = A?+9.=0. For Ay = = ( . oe co. ‘Scos3t ‘5sin 3t c= ‘Soo 3t ' Ssin3t ‘We have det(A — M) = 7 +2A+5=0. For 4y = —1+ 24 we obtain w-(") t 3 ne e x= (4?) (atta cnnye0s ( PAZ) the a cos 2t a > sin 2t We have det(A — Al) = -4(32 +1) =0, For Ar = 0 we obtain -( Por Xp =i we obtai so that “elSC2) -B(S)-(2) 386 ‘Then Exercises 8.2 40. We have det(A — AI) = ~(1+3)(A2 = 24.45) = 0, For Ay = —3 we obtain E) (3) For da = 1+ 2i we obtain so that Dees 2t + sin dt ~cos2t ~ 2sin 2 X gsintt efi] -Seos2t t 2¢082t 2sin 2 Then 0 ~2eos2t + sin 2t cos 2t ~ 2sin 2¢ 1 2eos 2 2sin 2 41, We have det(A ~ AT) = (1 )(A?— 2442) = 0, For Ay =1 we obtain “fi -( lS} (ES 42, We have det(A — MI) = ~(~ 6)(22 — 8+ 20) = 0. For » “f 387 For p= 1+ we obtain so that wwe obtain 43. . We have det(A ~ AL) = -(A + 2)(07 +4) = 0. For & Exercises 8.2 For Ay 4+ 24 we obtain 2 “i sin 2t ~ 08 2t x 2) 0 etss( 0 Jet 2 20s 2 2sin2t 0 sin 2t cos 2t 9, \ 20082 2sin 2 We have det(A — AI) = (2 A)(? + 44+ 13) =0, Por A: = 2 we obtain 8 K=[-5 5 so that ‘Then ° te 8 4eos3t — 3sin Be Asin3t + 3cosat Xaa]-sfe sal -seosd fe*+aq] —Ssinse Ey 0 0 we obtain 45. 46. Exercises 8.2 teest+ 2a) -tewat— uae sin 2t 0 2eos2t + 28in 2 ~2 cos 2t — 2sin 2¢ Xea|-le%se cos2t ta sin2t q cos 2t sin 2t We have det(A ~ Al) = (1 A)(Q +25) = 0. For Ay “f) cos2t ‘we obtain For dp = Si we obtain 145i cost ~ SsinSt sin St +5 cos5t Xr=] 1 [ha] cosse [+e] singe 1 cos St sin ot 5 cos St — SsinSt sin St + Seos5t Xeal-7]e+e| cos [tal — sinse 6 cosst sin St 4 X(0) (:) -1, then 1 = 69 = —1 and We have det(A — AI) =)? — 104.29 = 0. For Ay = 5+ 2% we obtain 1 me (,14) Bn ( 2a) (csortanon) 7 (sna 20st) cost) sin 2t Je (cat tana) + (sna ae 389 so that Then i so that and x Exercises 8.2 i then ¢1 = =2 and e = 5. © Bi re Prise aR or PB TE rasa pone fe Priam 37 ‘hase porzat tor Pabiem 38 48. (a) Letting 21 = sn, 24 = yp) 22 =U, and 2 = yy we have aad 102 + 429 = —10 + 495 tie = af = day ~ doy = A — Ayn ‘The corresponding linear system is ham t= 10m +495 ham ve= an — Ay Using a CAS, we find eigenvalues 43 and 423i with corresponding eigenvectors evs] [0 Eva4 yet |i}, ,| 0 eVii/2 0] * | eva2 1 1 ° 390 Exercises 8.2 and £V3i/3 0 -Vv3/3 Seah o sv3/6|"| 0|*") vive 1 2 0 Thus 0 ~via 2 0 Yeoman] |? ecevEt— |_| inv 1 0 -viH8 0 +22} | foyg| mvt | anv 0 1 ° vais ta]? |cosavat~ | _ 0 | sinavae 0 -vai6 \La 0 v3/3 o Oe | ee seal ojgfavaes [2] navae 0 1 ‘The initia conditions 4n(0) = 0, vn(0) = 1, 49(0) = 0, and ya(0) = 1 imply oy = -$, 2 = 0, and c4 = 0. Thus, ait) = it) =~ sin t+ YS sn avie a) = w(t) = Ban Vie s sinovit (b) The second-order sytem i af = 1021 +422 afm dey~ te xref Aly 4-4 We assume solutions of the form X = V cosut and X = Vsinut. Since the eigenvalues are ~2 and -12, wy = f=(=2) = VB and ux = y/—(-18) = 2V3.. The corresponding eigenvectors 391 49. (a) From det(A - Al) = A(\ ~2) = 0 we get Ar =O and Ap =2. For Ay Exercises 8.2 wf we wel ‘Then, the general solution of the system is 1 [Ec vae son] vIe+ | T] eave [1] sina x neal so-[] = x0-[ J ¥3/10. Thus ae Yan vier Boinavie Imply e1 = 0, ¢ = ~V2/10, ¢ = 0, and cy = a(t) = —PainvIe~ Bsn ave we obtain Ltfoy _ 1 io “(a ( 1 fo) (a of) | w=( *) For da = 2 we obtain G ey=(4 a) sitar Ka= (7) 1 alo) Co ofo 1 soe( toa) 7 Is not a trajectory of the system. ‘Then ‘The line y ‘Trajectories are y=2-2ch. This is a family of lines perpendicular mer texe™, y= oy + one oF to the line y = ~2. All of the constant solutions ‘of the system do, however, ie on the line y Exercises 8.2 so that All trajectories are parallel to y= —2, but y= ~2 is not a trajectory. There are constant solutions of the system, however, that do lie fon the line y 50. The system of differential equations seinen y= 20 thats ays dea tr5 th = 22s. We see immediately that 2p epe™, 3 = eye, and 25 = oye. Then aha 2a tae! > a= ente™ Hae 2 = 2g+ ese => xy = cte™ + eye 393 Exercises 8.2 ‘The general solution of the system is cate + oe exe xX=| — ae* ceste™ + cye™ cse® 1 1 0 0 0 1 =a] Oe +en]] 0 | t+] 0 |e 0 0 0 9, 0 0 0 0 0 0 0 0 o ° tea] 1 e* +c4] 0 |e +5 || 0 | te +] 0 | e* 0 1 1 o 0 0 ° 1 0 1 = aKye® +p |Kyte® + | 0 | ° 0 0 0 teakxe™ + caKge™ + og | Kate™ + | 0 | 0 1 ‘There are three solutions of the form X = Ke, where K is an eigenvector, and two solutions of the form X = Kte® + Pel, Soe (12) in the text. From (13) and (14) in the text (A-2K, =0 and (A -20)K2 = Ki 394 Exercises 8.2 ‘This implies 0100 0\/n qy o0000/|m] jo ooo 0o}lmi=o], ooo0i1lla| jo 0000 0/\p5) \o 80 pp = 1 and ps = 0, while p1, ps, and py are arbitrary. Choosing pi = ps = pa = 0 we have ‘Therefore a solution is Repeating for Kg we find so another solution is BL. From x= 2cos2t —2sin2t, y= —cos2t we find x +2y = —2sin 2, Then (z+ 2y)? = Asin? 2t = 4(1 ~ 008724) = 4 ~ 40s? 2t Peary tay aay? or 395 Exercises 8.2 ‘This is a rotated conic section and, from the discriminant 6 — dac = 16 ~ 32 < 0, we see that the curve is an ellipse, 52. Suppose the digenalues are a (9, 0> 0. In Problem 36 the eigenvalues are 5 3, n Problem 37 they age #3, ant in Problem 38 they are 1:24, From Problem 47 we deuce thot | | |. gg | Exercises 8.3 1. From ‘ x )x+(4) _ : xaa(i)+a(s)e tel ‘Thea . 7 = and @ a { (1 se) 8 ot (Sh) _ 7 . us ferra~{(Z2)a- (2) ana 2. Pom _ ‘Then 7 te) _ : _ wa fara (er) a= (ete) 4 o\ seroo(i)o() Exercises 8.3 3. Fom as 1 wn (se =2)*+(4) wwe obtain ) ran (2)er (parse; Then 7. wn jew ee (em fen) a} and -t bet so that = 7 um forran {By )a- (Ry) and . nou (82)s00 (89) 4. Prom 2A sin2t a x xo (0a) ta) we obtain : mein) eg (om) a xno (eee) 9 (asnar) LS a ey =e sin2t cos wpe Meine Jeo = (seam 2) ane ote (7 ia re cos2t 2eHsin2t JerMeost Jer ™sin2e sathat ; facta = f (HH) ap (batt us ferra= /(foru) > (jote) a 24 008 4t — 3 cos 2t cos 4t mfsin2tcos4t ~ Joon2eos at) =a é 73 Se) 5. From oa ay x=($ s)x+(A)e ve obtain Thea Exercises 8.3 oo thst va forea=/(_2.,)a=(2) aod sowee (oo) 6. from x=(4 3)x+(4) we oa seal)éee(()# Then aut ot et wet w-(62) we ee 2) so that ae a2ert + dest ve fortran |( Stan (CS) nd x-av=( 4075) 1. Pom x=( S)e+(t): we chal nal t)eea(-2e4 hen a (det je oo o-( ” a) Le -( ) PO eafeea (Se) nove (-A)ee(2) 8. Fom xi a)s+() 398 Exercises 8.3, est ~ oe A) we | - 6) _eeleereed (Secale Hee fe" : srewu- (Eo) ~ x-(3 J ()e Towra) Mee he) ate (“EE FS) ea elie 7) a (Henge) : 00 (G)e - “(3 2) - real J)éeol()e+(%)4 oo ee Ue foreae {ON ae (ON) 399 UL 12. 13. Exercises 8.3 and Pom we obtain Thea o-(a =) oo (a =) int —eot saint eat so that ee wa forte f(.)#= (eed and sow ou a (#08 +t ae {sin ~costin| sed From ae (1 a)*()¢ we obtain ena Sh)era(S)¢ — it t 3 sine c08t) oy get (sat ot) eC Se (SES) oe 3: 3cost ‘3cost + 3sint —fortpar. f (aime Scott) 4 (Seont + ain ue fovea f(Soirroane) > (Sane sens) ood : x-av-(¢ Rom wea(! Heat 11) ** (ane we obtain a(Si)eea(Si)¢ 400 14, Exercises 8.3 - ; (RE Sie me or (CSE SS) ; ve forae/ ("a= (°) a a — 1) te a}t® als VY gente (1?) 6-24 xna(Jerea|(:)iet+ (12 : oo() Briet ma ote (GT? Bh) vs forea= /(*228)a - , a (5. a) ®* (oettane) xea( 8) +0(2) — oe ( 2th leat oe (th ee moe : Us fora & [orden {C= (Sec - x,-ou=( se) t( oe ) = (Get) nteee ~sint]** \sinetant) * (cost 401 16. ar 18. Exercises 8.3 From : wo(4o)*+(cu) =i 0) ** (cote we obtain cost sint seca St) e0( tt Then a= (28 8) at ot ( =) Ssint cost sint cost 0 that 5 t= SP dt = 0 a ) we forte (2) (jet cet and . sintln| eset ~ cote =ou= Xe ae) - a eset x+(S)e (sie 2) ** (2) wwe obtain _ (2sint) (208) Xena (ten (28g Then a. (7 rent) =o (je oe cost sine Joost -aine so that 5 . ea i } u= forma I (seat ane) aes) Ssint 2eost X= au=(; )e+( or Jalsa + ( ) nse Foose =fsine sint Fon 2 “ tant a) (3) we obtain : een cost—sint) fenst +sin xa (Me rea) ‘Then cost sint cost sint — cost, cost —sint — a or. (-tat oe) 402 19. 20. Exercises 8.3 so that . Zeost + sint~ sect) ogee a) Oe re ee, 3 21 —Dsin?t + (sint — cost) In|sect + tant| sin cost ~cos?t = 2sin?¢ + (sint ~ cost) In |sect + tan Bn aon (ee ttceatiubicesuaty) From 110 é 11 0}X+{ & 0 0 3, te sve obtain 1 1 zal -1]+e[1 ° 0 Then an 9 0 and &7 0 so that de he" ie} -j(ieirs }e-(ee4) ‘ we and a yeta eet Xp= BU =| —e + beets tert yee From 3-1-1 0 11 -1)x+| + 1-101 et ve obtain 1 1 1 Kemer|iet+er{1]e+es| 0] e% 1 0 1 Then co oe eafe to | and ata[e® 0 eo e% -e™ 0 403 21, 22. 23. Exercises 8.3 so that and From we obtain : ee we) fede de o-(e &) = (Gee i) ond : 0 X= 97 O)XQ) +0 [oP ds =o ())+* ( = (2) seta (2) hg (2) ett (?) ot. = (sem (Tere (B)et= (5) From feo G2 ve obtain e-(0'8), a and x-se(xo)-0 fete a.( pra 8 [1-2-4 1 and Ao = 1. Corresponding eigenvectors are «-(3) eC) Solving =(A-DQ+1)=0 we obtain eigenvalues ‘Thus Exercises 8.3 Substituting Snto the system yields from which we obtain ay = ~1 and 6; = anea(prea(S}ee() 24. Solving (eo ca jae 914 = (A= 210=7 4 3-a} oA+ A=2E we obtain the eigenvalues 4) = 2 and Ag = 7. Corresponding eigenvectors are 1 1 i= (4) 28 (1) Thus 1 VY 7 neal J)P0()) Substioting @), (2 x= (Mea (P) eG Into the system pls fap +h +6=0 401-4 3m—10=0 Ga, +b, —a2=0 ay + 3b — bp +4 = 0. Solving the fist two equations smultancousy yields a2 = ~2 and by = 6. Scbstitting these two values into the second pair of equations and solving for a1 and by give a1 = 38. Then wal 1)@4a(1\e4 (7 -4 xioma( )tea(ie= (Gey 25. Solving “A U3] ath +21=(-3)A-7 oo fee-mam=0-310-7 405 26. Exercises 8.3 we obtain the eigenvalues Ay = 3 and Ap = 7. ‘Corresponding eigenvectors are w-(3) me -() nea 3)*ea()? Thus Substituting into the system yields Say + 5h =3 901 + 54 = —10 from which we obtain a; = 55/36 and by = -19/4, Then naren( Java )e (Behe jruna “1 Ina we obtain the eigenvalues Ay = 2i and Ay = ~2%. Corresponding eigenvectors are (24) » 6-(.5,) Second) Ssin2e \ cos2t ~ 2sin2t) *°? \ 2e0s2t + sin 2t Solving |= 4s<0 Ky Thus Substituting. into the system yields ay + 5b — oy wan-+ baby nay + Sh Hat na) +h +b =0 406 Exercises 8.3 from which we obtain aj = 3, bg = -2/3, a) =-1/, and i = 1/3. Then 11025 (it aoazt)** (seen rans) * (aya) + (Tn)! ae tc t= (11) so thee 2 Le (7 3), (t00sat ro 3)" . = oy ( 3) im Je +a(4) J, re (48 #8), we oe 10e* sint 2e(2sin t — cost) ue fered { (Sane) (glmunase oxy) = ¥ sint — 8 cost paeu- (Beker) BP sint — 58 cost rea(.)etea( Jets = (? 110) = (3) hen =2and r= 28, (a) Solving - wwe obtain the eigenvalues 2 = 0 and 4p = 2. Correspoxding eigenvectors are ef] et Thus If we substitute Exercises 8.3 into the system, we get [=f fe] fs) facets to} E-1 a} tn}* [5] ~L-a+-s)" ‘which cannot be solved for ay and b1. The difference in the systems is that in Problem 23 the homogeneous system does not have a constant solution, wheress in this problem it does. (b) Trying a solution of the form sf]: _ le], fax xe [ele ft] Substituting into the sytem, we obtain {ar — 6 +3) + (a2 — )t = a2 fails, so we instead try (ar +b: = 5) + (-a2 + ha) = bo and note that @2 = ba, Adding the equations, we find -2 = az + by = 2an, $0 ay = —1 and hhence by = —1. From the first equation, we have a; ~ by +3 = —1, and from the second equation, we have ~aj +6, ~5=—1. These are both equivalent to a; ~ by = —4, so we take w-(JeE] 29. (a) The eigenvalues are 0, 1, 3, and 4, with corresponding eigenvectoss -6 2 3 -1 “4 1 1 [ Slo el ia | 2 0 1 0 6 2 se ett o o -} § ae a ee ae el te 2 eel > et 0 | | 0 fet Abert \2 0 & oO stent Zee og forte wii , feMstet_otahe ( eor= | gee -he* +e JerM a Lett Leertt Je® stew tte 408 Exercises 8.3 ede | fet fet — 2 + bd [ewor(a = 4s oe i ye™— fe et het ets fee mht fet pel feel feet ae Mae hea hels fies he — He G sopod- ‘ a [OR OH tec waft} Ber + Doze! + Seye ~ eg der + cnet + cae + cae! a~QC= : ert dee ee s6)e+ 80) [aOR Ber + Dene! + Sege — eye a ty detyl SSR) (saad : a1 + Pee riers tn tae Fue) he ee a : : +e 1 (@) X=a) 1] 4%) g)+9/0/*4) 6 2 0. 1 0, (gfe ee bea 2 [Den het pet diel dee fe H Foy oe 409 Exercises 8.4 Exercises 8.4 —_ a tora=(1 8) wehow ° and Exercises 8.4 and so on. In general Thor Ae Ae ard May May nts aredigetass = 2 a = Leg lye Jayisy.} ais deedteJoa(edesdte.) crate nents (Se 2H) eect jinht cosht. aod van (SED s,s ae) . sink(—t) cosh(-1))~ (sine cosh 9. Fe we hove 4. For wwe have at Exercises 8.4 { 0 o 3 0 0 0 : i *-(5 &)(Q-«()-9) 6. Using the result of Problem 2, =) Ge A sinnt) (cosh x= (at! an) sinbt cosht) \ep tel ot t a teh t t X=[¢ ter ¢ alaal ¢ |tealeri|ral ¢ —2 = -241/ la 2 2 241 7. Using the result of Problem 3, 8. Using the result of Problem 4, 9. To solve Exercises 8.4 we identify to = 0, F(s) = Gy and use the results of Problem 1 and equation (5) inthe text Xedore [ARs is ik A ‘ x= (5 2) x+ (4) X(}= Moreh [er MR(s hy Exercises 8.4 LL. Ta solve Pn A ao) ** a we tens y= 0, (6) = (7), and ue the sue of Pion 2 and unin (5) ne tet. X(j=MCred £ eM Bs) ds ~( se cone) (ahs ote) (t)* = (cteosht-+ensinht)) | fcoshé sinht ¢/ coshs sinh) 4, araape-regcane) *(sione cont) 4 (sins cos . ane, ee same) ( | ~\evsinht +ezcosht) * (sine cosht ) (cosh s +sink s ane ‘cosh sinh t a ~ (Gee poms) * (shee che) (Someone Geer sinh? ¢ ~ cosh? =) ey 1 ~ (Zatner erence) * Creeomnee) “2 (sane) * Sane) 12. Ta solve x’ 0 1)y, fst (2 0) **(sate) ote sei o 6) = (Sth), nt nt oof ri ad tn nt X(t) = eNO a em [em MP(s) ds ~ (Sgt S8:)(¢)=(s (ks ) (a) -(canetaats) «(Zar St) = -(catetett)-(er V0) exsinht +o cesht lo as Exercises 8.4 Ce ae siokt) (t cisinht + czcosht) * \sinhe coshe} \o. Gee toorht) (cosh) | (sinht)) | (cosht crsnnt-+ peat) * (raane) 7% (Shane) *2 (Coe) (Sine mo fG-600 ‘Thus, the solution of the initial-value problem is t4+1 t : t |-afetifosf ¢ 2 na t+ woeo(ea’)-(3)-(259-0) ‘Thus, 9 ~ 7 and o4 = §, 50 13. We have 14, We have 15, From sT- A and ‘The general solution of the system is then 415 Exercises 8.4 fet tem agtt_ gest jet —per® fe — fer JQ) weet fet get) Le wa (eral joroa( Mlera(Ber ga+ ta) (Ses (-fa-4a) @) om -o(2)2ee(s) s-4 2 -1 8-1 amar 3 ae _ (28 ~20l + 2€* Cee tet we 4 2ett ‘The general solution of the system is then 16, From st a= ( ) me i xoeen (22 E2)(5) a eee) Le wa({)eea(“ese(flera(s)e = =) ({)e+ cara (fe ~a()ee()4 9 17. om sI- A= (= -1 s+ ) wena Exercises 8.4 and we (SSI 910 Oo et ta aie ‘The general solution of the system is then C2" 290) wa({)etra (let ea(t) tral g)e® oe) AP X= eh se. Fomat-a=({ ,71,) roses | _,_ {Grits Gris? (ivayte) Te stit1 Gays GFUPsT and cost + etsint evtsint ‘The general solution of the system is then. to. (Ciettertsint ——evtsine ev Bette Ota tease tent) (a) wa(2}etertsa(SJetentsa(S)etaetsa( Jeter _ (eee ete (ate ine) a7 Exercises 8.4 19. Solving ve find eigenvalues A, e-({ 2). P(E). at 8-89) ppp! (3 ) N= 443=(A-1)A-3) wea eg =. Goong er G) = s=() th in) at =(0 §) Then so that 21. From equation (3) in the text y+ Lapp + T+(PDP™) 4 52°(PDP =P[T+ D+ Hep) + 5 qt) +] Pt = Pep a8 Exercises 8.4 22, From equation (3) in the text 10 0) fy Oo 0 HO 0 or of fo a oO} tafe Be 0 + +h z oo ay oo an 00 a oe 1,{0 #0 Les +ie + 0 0 8 Le ats AO? + ° ° / 0 Te at $02 ° 0 0 + Lb dat + HOt)? + emt 0 0 _[ 0 eM 0 o 0 23. From Problems 19, 21, and 22, and equation (1) in the text X=e*C=PeP IC -2 2 U2 EDO a} \o et) jee tesla -(¢ je one) ) joe Jou 24, From Problems 20-22 and equation (1) in the text ES mh ame | ~(3 (6 2)Ge )() “(Bo Bie) () 19 Exercises 8.4 25. If det(sI— A) =O, then s is an eigenvalue of A. Thus sI—A has an inverse if sis not an eigenvalue of A. For the purposes of the discussion in this section, we take s to be larger than the largest eigenvalue of A. Under this condition sT ~ A has an inverse. 26. Since A? = 0, A is nilpotent, Since if A is nilpotent and A" = 0, then Af = 0 for k > m and of net In this problem A? = 0, so ectomeahefet Jo[tS eft te 1-t-#@2 tee 1-8 tb 1stsep. and the solution of X" = AX is et P)/2) + oat +ea(t + 2/2) ex(-t = 2/2) beat + eat +04 22/2) 27, (a) The following commands can be used in Mathematica: {4 2},{3, 3} 1, 2}; MatrixExp[A t]; sol=Expand(m.c] Collect[sol, {c1, ¢2}] // MatrixForm ‘The output gives sina (26+) +o4(- 26428) ona 8) +(Be se") Ths elves 1a 6 with carmpontng eenectae (3) = G): 420 Chapter 8 Review Exercises s0 the solution of the system is seal 3oen()e 2(t) = ~Bhyet + doe ult) = Bb + bn Ife replace by with —Jer + Jen and by with Bey + 2ex, we obtain the solution found using the ratrix exponential (b) z(t) ult) = % cost — (er + one" sint 6M cost + (2er + exer sine 28. (96% — 26+) + e5(—6er™ + 6e™*) (dem — Se!) + ca(4e-2 — de“*) (er — e4) eg(—2e-® + Se) a( Be" + Be™#) + cg (—Bev + de” Chapter 8 Review Exercises se arxau(2), sien oat G )@Q-Q-G)-G)-@) 2 )0-G-Q) ‘we see that A= 4 is an eigenvalue with eigenvector Ks. The corresponding solution is X = Ket ‘We see that k 3. Since 4. The other eigenue isto = 1 2 with coesponding eigenvector Kz = ( ‘) The general xora(_ Se )era(eae 421 solution is 10. We have det(A ~ A) = Chapter 8 Review Exercises vietoe ania 29-01? -easa = ( 1) sateen (a ane aca = (°) xea(A)eral(dee+(2)4 We have det(A ~ A) = (A+6)(A+2) =0s0 that seo Sete’) so that + We have d(A— AD) = 38 2A+5-=0, For A= 1+ we obtain Ky = (1) ant v= (7) (Sa) (Sa) fre (Sa) ‘Then 2 ee a(n At x0 (8 ere (er ee aes We ave da(A ~ AI) = 38-2420 For A= Line obtain Ks = (5) and Then Beost+sint) | /—cost+3sint) rma (ee ere eae) (\—2)(A=4)(A +3) =0 0 that ~2 0 7 X=al 3 esa ifetses| 12 ]e% 1 L 16, We have det(A — AI) = (s+ 2)(0? — 2543) =0. The eigenvalues are Ay = and 2 = 1 ~ V%, with eigenvectors a : «-( :) we va) xt «+ (-ve) a f 422 2, dg = 1+ V2, Chapter 8 Review Exercises Thus , xeo[ereal( a. (taal i , cade) ee (ona cos Vt sin Jt oo( t]erea( tithe) eve ssa) 4 cos Vit sin Be ena (t)etoa(te o-(F S) oS 8), ee fortran /(#A an (BELEN), = wou. (118) reve (208) ee (2) _ a (20088 Bint) gaa _ ( foost want) ue fernan f(a sonny ~2eostln sect + tant Hn 80 = ( aera)” 423 Chapter 8 Review Exercises 38. We bave cost +sint\) sine ~cost xno) Maca) Then . = oe) ( sint Joost dsint 2eost Qsint cost feost + bin and : ‘ a Jsint — boost + fesct u-/* ra=/(4 — faint — foost+ poset | Giese) OA. feost = fsint + $n] eset - cot] }" so that )= (actin, iorooee 14. We hove ' : L x=a(l)e wre (el (a)eeel( eG) = et te +e ay [ct = a(S) eo and . f-1)),_ (fiat um fateam f° a= (#74), so that xnav (U2) eet (2) et a ‘we note that (A ~21)K = 0 implies that 3 + 3k: + 3ky = 0, go fx = —(ky + hy). Choosing. sky = 0, ky = 1 and then fp = 1, ky = 0 we get o-(3) a e-(7) 424 15. (a) Letting Chapter 8 Review Exercises respectively. Thus, are two solutions (b) From det(A ~ AI) = two. Letting 2(3 ~ A) = O we see that y = ky K=| km], ig as in part (a), we note that (A — O1)K = AK = 0 implies that fi + ka + ky = 0, s0 hy = ~(ha + hs). Chocsing ky = 0, ky = 1, and then hy = 1, a = O we get (==) respectively. Since the eigenvector corresponding to \y = 3 is «() , and 0 is an eigenvalue of multiplicity the general solution of the system is fe(o() Se weave X= . 26. Fer x= (( 1% 425 Chapter 8 Related Exercises Chapter 8 Related Exercises 1. (a) In operator notation we have (D? +4)z1 - 222 = 0 2x, - (DP +2)29 = 0. 2p we have (D* + 6D? + 4)2j = 0. The roots of the auxiliary equation are and 4/3 VBi, so y(t) = 0,008 V3 ~ Vit + cgsin V3 — Vt cp cos V3 + VEE + casin V+ VEC ‘Thea al) = HO nu) 14 V5 cos = Vot+ - Beasin Va Ve + YS a cos ae vats Besta Va Vir ‘The frequencies ae un = YO= VE and un = V4 VB. (b) We write the system in the form x ‘and assume solutions of the form X=Veosut and X= Vsinut Since the eigenvalues are ~8+ vB and -3 ~ V8, = /3~ VB and uy = 3+ V5. The corresponding eigenvectors are “elosenl [asia ‘Then, the general solution of the system is. xo=a| ]osV8-Were[ Jano=Vee ave. t9[o ayn] ener 2. JFK is multiplied by a factor of 10, then 1 (a+ v5y/2 1 | a Bynlevenee 426 Chapter 8 Related Exercises “my 2 0 0 0 0 0 @ 0 oO 20 2 0 0 6 0 0 0G 70 fo 6 0 oo ooo 8 @ 0 0 0 oo Pee 0 0 2 2 oo 060 0 220 9 6 GO Oo 0 0 0 0 2 i 2 06 0 0 60 0 0 0 2-0 2 0 O 0 0 0 0 0 9 3 2 0 6 0 0 0 0 6 0 2 10 ‘The table below shows the eigenvalues A; with the corresponding frequencies wi = =%; and periods 7} = 2m /. TPE e [SS BS aa ea aT [vo core] ca aeoe | To Tase | Woes [eas [Sb fa_[s.7n6e | 3.6020 13.5225 | s.s6an | a.i776 | 2.s7es | 2.7795 | cons | 2.ane [a.z7e7 Hf 6907 [3.7289 | 3.7008 | x-007t Pacsr73 [2.1208 | 2.7605 |? soe [ 2.s216 [5.2902 We see that the frequencies are higher and consequently, the periods are smaller. 4. (a) For this exercise we assume e building of four stories, each with mass 10,000 kg and restoring force constants each hi = 5,000 ke/s*. Then 100 1 F—10,000 5,000 0 o e100 5,009 -10,000 5,000 0 M=1, K : 2 ho 0 2 of” 0 5,009 10,000 5,000)" ooo o 0 5,000 ~5,000. and “105 a 0 05-105 0 0 a5 1 08 0 0 08 ~95} As (b) The table below shows the eigenvalues \y with the corresponding frequencies uy = VK and periods 7} = 2n To Eve [aS | HSE | OE ‘o, | 3.3709 | a.00se [e.7073 | 0.2056 T_| 7280] 5.7998 | 6.056 | 25.5055 427 Chapter 8 Related Exercises Since the periods are all larger than 4, the building should be safe in a period-2 earthquake. (c) IE kis multiplied by 10, then a ee 50-109 500 0 50-100 50 0 0 80-50. From the table below we soe that the periods are generally in the range from 1.5 to 3 and the building is unsafe from a period-2 earthquake, TLL aT ee | es | 42024 | 3.4289 | 2.261 | 0.7766 Tif 2.4951 | 2.9349 | 2.9098 | 8.0909 Experimenting, we see that if the building is made stiffer by a factor of 8, the periods are entering the danger zone. Ef eee [BS [ ao ee ‘o, | 3.7508 | 3.0842 [7.0000 | 9.6946 FP ravers | 2.0508 [3.1416 | 2.0059 4, The related homogencous system is X" = M“KX = AX, where A is shown in the solution of Problem 3(a). We assume solutions of the form X = Vcosw and X = Vsinut. The eigenvalues are —1.7660, ~1.1737, -0.5, and ~0.0603 with corresponding frequencies 1.3289, 1.0834, 0.7071, and 0.2456. The eigenvectors are 0.4285 0.6565 osm 0.280 0.6565 0.2280 osm 0.4288 7 é and Ve= 2 0.5774 |" - 0.5774 0 . : 05774 0.2280 0.4285, osrre 0.6565 and the general solution is X{t) = o1 Vi cos 1.3289¢ + c2V; sin 1.3289¢ + cpV3 cos L.0834¢ + c4Vpsin LO834t + ¢5V3 c0s0.7071¢ + ces sin0.707It + eV 005 0.24568 + gV4 sin 0.24568, Since 10000 cos 3t ° o 0 FO) = 428 Chapter 8 Related Exercises to solve the nonhomogenesus system we look for a particular solution of the forma a1 ts Xp= |e ome [P| sna ou he Substituting into X" = AX +M-'F(1) we obtain M9) == 9, +0809 14 Oy Yaz = 0.50; — a2 + 0.543 ~9by Shy — b + 0.5by and Sag = 0.502 — ay + 0.504 —9b3 = 0.5b2 ~ b3 + 0.5bq 804 = 0.03 ~ 050g = hy = 0.5% ~ 0.58 Solving these systems gives ay = 0.12548, a2 = 0.0787, ay = 0.00049, a¢ = 0.00008, and by = bam y= by = 0. Thus, 0.12549 oora7 cost 0.00049 1 o.0w003, Applying the instil conditions X(0) = 0 and X'(0) = 0, we ind e = ~0.0592, cp = 0, ey = ~0.0889, 0879, ce = 0,65 0, cr = 0.0258, and cg = 0. Thus, the solution of the initial-value problem: 0.4285 0.6565, 0.6565, 0.2280 X(t) = 0.0592 . cos $.0834t (8) = 0.0592 | 2 | 008 1.32891 — 0.0899 | 55 | cos 1.08941 0.2280, 0.4285, 0.5774 0.2280 0.12549 osrra 0.4285 0.00787 + cos + i. Qoe7e | 97" | eas a.oTae + 0.0255 |“ | ens 0.24588 + | aan | e089 ost 0.6568, 0.00003. 429 9 Numerical Solutions of Ordinary Differential Equations Exercises 9.1 All tables in this chapter were constructed in a spreadsheet prograin which does not support subscripts Consequently, z_ and ya will be indicated as 2(n) and y(n), respectively. 1. (oe=ar Eo ad] 97a my yar Too] 3-0 Taos Sabo ria | 3!3800 | | ales | aceers aiz0 | 3lasas | | acto | Sister aso | 2ip2ae | | alas | aisrst tao | 2:4asi | | 2:20 | 3.2342 aiso_| d'osox |} ates | 2 ro 3007 203 308 a4 430 Siar] aya 9-08 | 1-000 | [0.00] 1-000 oro | iinto | | los | tlesze 0:20 | 1251s | | ols | rani o:30 | raze | | ovas | 22975 orao | ieee | | 0:20 | iiasze 0:50 pe} | 0.25 | 1/3308 ol30 | ilaser Ol3s | isss6 0.50 613 | Et os ae ro xin] vtay 3-00] 0-000 | |~0.00-] 0.coa0 orto | o:oss2 | | 0:08 | o:oaee 0°20 | olae22 | | o:10 | oceess 0:30 | 0.2622 | | o.s | 02397 oro | 0.3363 | | 0:20 | ocae23 403 | | 0:25 | 0.2232 0:30 | 0.2623 0°35 | o°3002, o-ao | 013360 oras | ocsnis Reo BEoe er] yar se] ya 9-08 | o-0050| | 0-00] 0-cooo 0:30 | 070050 | | 0:05 | 0.0013 0:20 | o:o200 | | orto | o:c0s0 0:30 | o.o4s | | ors | ocaa3 e140 | ocoees | | 0:20 | o.ez00 oso | ou eras | o:033 0:40 | 0-0805, ocas | olto22 | 01266 a mE ie res 0.00 9.00 | 0-5000 0:30 eros | ovsiie 0:30 ore | ocszie 0.30 eras | ovszea bie 0°20 | olsase 0.50 0128 | o:seoe ol30 | ovseca e135 | olsess 040 | 015484 ols | olsasz oso | o1se 431 Exercises 9.1 10. un. Exercises 9.1 Raat en xia yar] ep viae ‘9-09 | i-osae] [9-0] 1.0000 ore | ites | | o:es | tosis o:20 | vasa | | o:a0 | 099, 0:30 | 'se06| | 013s | iiieae aleo | i’ssas | | 9120 | 212837 357 || 145 | 2.3003 8:30 | Liar 013s | Llaesa oo | issi0 aise | 17560 aay] (are var Tos] 3-a00r| [~i.0e [7 coor ato | loos | | ios | Lovee 120 | ioeos | | ato | Loves al30 | xlose7 | | alas | oz28 ivao | Gliese | | i120 | losis Lacse | ia 325 | toses 230 | Tose a3s | ines aise | uu = oT Fy a xi] “viar 9.00] 9.5000] [0-00 | a-3000 oso | ovsaso | | cos | ocsizs 0120 | o'sase | | 020 | 075250 9.30 | 9: aris | o'sare ores | ovei08 Eo EEO mal veer] Caer 3.00] 2 e005] 2.0000] | 0-00] 2-000 ps0 | 21220 | 21230 | | olos | 2/0553 8:20 | 2.3049 | 2/3085 | | octo | 2.3220 0:30 | 2:sese | 2:sssa || ols | 2.2056 o'40 | 3e37e | 3io6s0 |] 0°20 | 23078 sisas | 390s J | 0125 | 2.asez 0°30 | 2'ssa eras | 3ca059 432 Exercises 9.1 32. (a) oy () TTD peo | euee | "zou Parsee tt | a5 1a0 | 122000 | 3!2ae8 ad 120 | ilasse | 1ese5 130 | isnt | 2627 5 Liss soso | 6 7580 eg aaa 13. (a) Using the Buler method we obtain y(0.1) = 1 = 12 14. (a) Using the improved Euler method we obtain y(0.1) = a1 (b) Using y"" = de? we see that the local truncation error is 1 (6) = ge OY? = o.gner oh = 4ee 22" 000 Since 2 is an inereasing function, e% < e201) = ¢?? for 0< e < 0.1. Thus an upper bound for the local truncation error is 0.02¢9? = 0.0244. (©) Since y(0.1) = 9? = 1.2214, the actual error is y(0.1)~ yn = 0.0214, which is ess than (0244 (d) Using the Euler method with h = 0.05 we obtain y(0.1) 120 (e) The error in (d) is 1.2214 ~ 1.21 = 0.0114. With global truncation error Oh), when the step size is halved we expect the error for h = 0,05 to be one-half the exror when h = 0.1. Comparing 0.0114 with 0.214 we see that this is the cus. 1.22, (b) Using »"" = 8e2 we see that the local truncation error is sng) ox gore OL Pe a" =e SY = ocorssse Since e% is an increasing function, e% < e204) = 9? for 0 < ¢< G.1. Thus an upper bound for the local truncation error is 0.001333¢% = 0.001628, (c) Since y(0.1) = ¢0? = 1.291403, the actual error is y(0.1) ~ v1 = 0.001403 which is less than 0.001628, () Using the improved Buler method with h = 0.05 we obtain y(0.1) * yp = 1.221026. (e) The error in (a) is 1.221403 ~ 1.221025 = 0.000378. With global truncation error O(h?), when the step size is halved we expect the error for h = 01.05 to be one-fourth the errar for h = 0.1. Comparing 0.000378 with 0.001403 we see that this is the case. 15. (a) Using the Euler method we obtain y(0.1) = an = 08. (b) Using y” = 5e~ we see that the local truncation error is ser OOP gate 433 Exercises 9.1 Since e~® is a decreasing function, e-® < ¢ the local truncation error is 0.025(1) = 0.025. (6) Since y(0.1) = 0.8284, the actual error is y(0.1) ~ yx = 0.0234, which is less than 0.025 (d) Using the Buler method with h = 0.05 we obtain (0.1) = yo = 0.8128. (e) The error in (d) is 08234 ~ 0.8125 = 0.0109. With global truncation error O{h), when the step size is halved we expect the error for h = 0.05 to be one-half the error when ht = 0.1 Comparing 0.0109 with 0.0234 we see that this is the case 1 for 0<¢< 0.1, Thus an upper hound for 16. (a) Using the improved Buler method we obtain y(0.1) v1 = 0.825, (b) Using y!” = —102-% we see that the local truncation error is Go01667en*, Since e-® is a decreasing function, ¢-* ) From the graph we estimate A(1) + 1.68, A(2) = 13.2, A(3) = 36.8, A(4) = 469, and A(5) = 48.9, ater ‘ aq ad ri Cece ae age (6) Let a= 2.128 and 9 = 0.0432. Separating variables we obtain aa ath mat eam >" Tae 3 (G+xfa)ene pala 6A) =t+6 =alt +6) Ac aetttt0) — pacts (a+ aeste] A = aesttre ‘Thus aottt9) AO) = Ty pees = FE From A(0) = 0.24 we obtain so that e-®° = a/0.24 ~ 6 = 8.8235 and 438 Exercises 9.2 2128 Oe 00485 + 5.8235e-2 7 re ees = br tebservea) | 378 assy 36-30 asa a7 lepproximatea)| 193 12/50 36.46 47.23 49.00 : 95126 24 15. (a) Rete or Ge mare er {00-] — 10000 “70009 2 rie | 3513} a.asuy as| Bis | resee| is | dioa] 7% “| 230 | 2/95e0] 2. seas 5) 538 | 3iapen] 3.40 | s.o6es| 903 ozaal a 16. (a) Using the fourth-order Runge-Kutta method we obtain y(0.1) © yy = 1.2214 (b) Using v(x) = 32% we see that the local truncation error is yo) - = nage 0at = 0,0000026676, Since eis an increasing function, e2 < e400) = °2 for 9 < e< 0.1. Thus an upper bound for the local truncation error is 0.000002667e"? = 0.000003257. (6) Since y(0.1) = e®# = 1.221402758, the actual error is y(0-1) ~ y tha» ¢,000008267. (a) Using the fourth-order Runge-Kutta formula with h = 0.08 we obtain y(0.1) = yw = .223400571. (©) The error in (a) is 1.221402758 ~ 1.221402571 = 0.000000187. With globsl truncation erxer O(h*), when the step size is halved we expect the error for h = 0.05 to be one-sixteenth the error for h = 0.1, Comparing 0,000000187 with 0,000002758 we see that this i the case 17. (a) Using the fourth-order Runge-Kutta method we obtain y(0.1) = a1 = 0.823416667. (b) Using 9a) 0000002758 which is less 402" we see that the local truncation error is (0a)s 120 Since e-* is a decreasing function, % < ¢° = 1 for 0.< ¢< 0.1. Thus an upper bound for the local truncation error is 0.000003332(1) = 0000003333, 4gew* SY" = 0.000003333. 439 18. 19. 20. Exercises 9.2 (c) Since y(0.1) = 0.823413441, the actual erro is |y(0.1) ~ vs] = 0.000003225, which is less than 0.000003333, (d) Using the fourth-order Runge-Kutta method with Ah = 0.05 we obtain y(0.1) ~ vz = 0.823413627. (e) The error in (d) is [0.828413441 ~ 0.828413627] = 0.000000185. With global truncation error Olt), when the step size is halved we expect the error for h = 0.05 to be one-sixteenth the error when h = 0.1. Comparing 0.000000185 with 0.000005225 we see that this is the case (a) Using y) = -1026e~%-1) we see tha the local truncation error is yale : (91) © | = g.s5hie-Se-0, we) 55 | 8.55h%e (b) Since e~M=- jg a decreasing function for 12 < 15, e-MY O for (0,0) to be a center. Therefore |j| <1. 18. Note that A= 1 and 7 = yz. Therefore we need both 7 =u < Oand 7?—48 = y?—4 < 0 for (0,0) to be a stable spiral point. These rwo conlitions may be written as ~2.< p< 0. 19, Note that A= y-+1 end 7 = +1 and s07?—4A = (w+ 1)?=4(u+1) = (u+1)(u—2). I follows ‘that r? — 4A < 0 if and only if -1 < 4 <3. We may conclude that (0,0) will be a saddle point when 4 <1, Likewise (0,0) will be an unstable spirai point when T = y+1> 0 and 17-44 <0. This condition reduces to -1< y< 20, 7 = 2a, A= a? +6? > 0, and 7? = 40 = 49 <0. Ia <0, (0,0) isa stable spiral point. a > 0, (0,0) is an unstable spiral point. Therefore (0,0) cannot be a node or saddle point. 21. AX) +F =0 implies that AX) = -F or X; = —A7IF. Since X,(t) = —ATF is a particular solution, it follows from Theorem 8.6 that X(t) = X¢(t)+X1 isthe general solution to X’ = AX+F. I's Othen Xe(t) approaches (0,0) by Theorem 10.1(a). It follows that Xj approaches Xi; a8 t= 00. 22. Ih be< 1, A= odfi{l ~ be) > 0 and 7? — 40 = (a — dj)® + dabediy > 0. Therefore (0,0) is @ stable node 467 Exercises 10.2 23. (a) The critical point is Xi (b) From the graph, Xi appears to be an unstable node or a saddle point (c) Since A = —1, (0,0) is a saddle point 24, (a) The critical point is X, = (~1,—2) stable node (b) From the graph, X1 appears to be a stable node or a degenerate AY (c) Since r= ~16, A = 64, and r? — 4A = 0, (0,0) is a degenerate stable node. 5,2) 25. (a) The critical point is Xt (bj From the graph, X; appears to be an unstable spiral point =) =) ~~ (6) Since 7 = 0.2, A = 0.03, and r2— 4A = ~0.08, (0,0) is an unstable spiel point 26. (a) The ctitical point is X1 (b) From the graph, X1 appears to be a center, (c) Since r= A=1, (0,0) is a center, 468 Exercises 10.3 Exercises 10.3 1. Switching to polar coordinates, de ifide dy @ ($+) ‘Therefore r= ce and sor —Q if and only if'a <0. Jae? ~ Aey + ay? + By + oy" — 29°) 2. The differential equation GF = ar(S—r) isa logistic differential equation. [See Section 32, (4) and (8)) It follows that r and @=-t+er T+ cet Ha >0,r—5 ast — +00 and co the critical point (0,0) is unstable. If < 0, r+ 0.ast + +00 and so (0,0) is asymptotically stable 8. The critical points are 2 = 0 and x = ant 1) = Rin +1) <0. Therefore See Theorem 10.2. 4, Note that «= & is the only critical point since In(z/k) is not defined at z = 0. Since gz) = k~ kln(z/k), g/(k) = —k <0, Therefore 2 = i is an asymptotically stable critical point by ‘Theorem 10.2. 5. The only critical point is 7 = 7, Since g/(F) = h, (To) = k > 0. Therefore T = 7p is unstable by ‘Theorem 10.2. 41, Since g'(2) = k(n+1) = 2k. g!(0) = Hn-43) > and = 0s unstable while 2 = n+ 1 is asymptotically stable 6. ‘Phe only critical point is v = mg/k, Now g(v) = g— (k/m)v and so g/(v) = ~k/m < 0. Therefore v= mg/k is an asymptotically stable critical point by Theorem 10.2. T. Critical points occur at ¢ = a, 9. Since g'(z) = k(—a~ 8+ 2x), g(a) = Ka ~ f) and g(A) H(B—a), Since a > 8, g(a) > Oand soz = ais unstable. Likewise 2 = 9 is asymptotically stable. 8. Critical points occur at 2 =a, 8,7. Since al (2) = Ke — 2)(-8— 9-22) + (8-2) 2)(-1), g(a) = -K(8~a)(—a) <0 since a > 9 > 7. Therefore (8) > 0 and g'(9) < 0. Therefore x = f is unstable while x 9. Critical points orcur at P = a/b, c but not at P = 0. Since g/(P} = (a — bP) + (P — e)(—6), a'(e/b) = (ofa Since a < bc, —a + be > 0 and a~ be < 0. Therefore P = a/b is unstable while P saymptotically stable a is asymptotically stable. Similarly jis asymptotically stable athe and g(e)=e—te y= 469 10. ul ae. 13. 1a, 16. Exercises 10.3 Since A > 0, the only critical point is A= K® Since g/(A) = }AKA~Y? &, g!(K2) ‘Therefore A = K? is asymptotically stable. ‘The sole critical point is (1/2, 1) and =2y 2 ) £00= (Fe 2 and A = 20 that P40 = ‘Computing g'((1/2,1)) we find that (1/2,2) is a stable spiral point. Critical points are (1,0) and (—1,0), and 4<0. Therefore eX) Qe =2y oN o AtX = (1,0), 7=4, A =4, and so 7? — 4A = 0, We may conclude that (1,0) is unstable but we are unsble to classify this ertical point any further. At X = (~1,0}, 4 = 4 <0 and so (~2,0) is a saddle point ya dyny 21/2 and y~z?-+2-= 0 implies that (c,y) = (1/2,—7/4). The case y = points (V2, 0) and (V3, 0). We next use the Jecobien matrix og e(—2= eo =( > ) to classify these three critical points. For X = (v2,0) or (-v2,0), 7 = 1 and A <0. Therefore both critical points are saddle points For X = (1/2,~7/4), r= 1, A= 7/2and 90 7? — 40 = =13 <0. Therefore (1/2,—7/4) isa stable spiral point (22 ~ 1). Therefore if (x,y) is a critical point, either 2 = 1/2 or y = 0. The case leads to the eritical Y= -y-t ey = y(-1+2). Therefore if (,y) is rte point, either y = O or z= 1. The case y= 0 and 2x —y? = 0 implies that (z,y) = (0,0). ‘The case x = 1 leads to the critical points (1, v2) and (1, V3). We next use the Jacobian matrix 2 -2y eoo-(7 2) to clasify thes critical points, For X = (0,0), A = ~2 < 0 and so (0,0) sa suddle point. For either (1, v2) or (1, v2), 7 = , and so 724A = ~12. Therefore (1, V2) and (1,-v3) are unstable spiral pont, Since 2? ~ y? = 0, y? = 2? and so 2? ~ 32-42 = (z—1)(e-2) = 0. It follows that the critical points are (1,1), (1,~1), (2,2), and (2,2). We next use the Jacobian Zo & =) 2 -2y 470 16. a. 18. 19. Exercises 10.3 to classify these four critical points. For X = (1,1), > = -5, A= 2, and so7?—4A = 17 > 0, ‘Therefore (1,1) ie a stable node. For X = (1,~1), A = -2 < 0 and so (1,~1) is a saddle point, For X = (2,2), A = 4 < 0 and so we have another saddle point. Finally, sf X = (2,-2), 7 A <4, and so 7?~ 40 = 15 <0. Therefore (2,—2) is an unstable spiral point. From g?— 2 = 0, y= zory= 2. The case y= leads to (4,4) and (~1,1) but the case y = —x leads to 2 — 3244 = 0 which has no real solutions. Therefore (4,4) and (—1,1) are the only critical points. We next use the Jacobian matrix vo-(4, “') to classify these two critical points. For X = (4,4), 7 = 12, A =40, and 90 7?— 4A < 0. Therefore (4,4) is an unstable spiral point. For X = (~1,3}, = ~3, A = 10, and s0.2?— 4A <0. It follows that (~1,—1) is a stable spiral point Since 2! = -2ry = 0, either + =Oory=0. Ifz = 0, y(1—y") = 0 and s0 (0,0), (0,2), and (0,—1) are critical points. The case y = 0 leads to x = 0. We next use the Jacobian matrix co( =.) ae ) \atey 1¢2-ayt to classify these three critical points. For X = (0,0),7 = 1 and A = 0 and so the testis inconclusive. For X= (0,2), 7 = 4, A =4 and so 7? — 4A =0, We can conclude that (0,2) is a stable critical point but we are unable to classify this critical point further in this borderline case. For X = (0,—1), A= ~4.< 0 and so (0,1) isa saddle point We found that (0,0), (0,2, (0,-2} 0,0) and (1,0) were the teal pits in Bete 15, Section 10., The Jasbin is 1-323? Ezy wtoy 3-2? ay?) For X = (0,0), 7 = 4, A=3 and so 72-44 = 4 > 0. Therefore (0,0) is an unstable node. Both (0,1) and (0,1) give 7 = —8, A = 12, and x? — 4A = 16 > 0. These two critical points are therefore stable nodes. For X = (1,0) or (~1,0), A = —4 < 0 and s0 saddle points occur. eX) = ( We found the eiteal points (0,0), (10,0), (0,16) and (4,12) in Brereise 11, Setion 10.1, Since the Jacobian is 10-22— by ~v eon ( art Exercises 10.3 ‘we may classify the critical points as follows: x tr A 24h Conelusion ey aa 55 nstable node (10,0) 4-60 - saddle point (0,16) -14 -32 - saddle point (4,12) -16 a 160 stable node 20. We found the sole critical point (10,10) in Exercise 12, Section 10.1. The Jacobian is 2 1 (X= ( 2 31-38 ). wy g'((10,10)) has trace r = 46/15, A = 2/15, and 7? — 4A > 0. Therefore (0,0) is a stable node. 21. The corresponding plane autonomous system is ¥ 1 cos 9 — 5) sin? Since |@] <1, it follows that critical points are (0,0), (n/3,0) and (—7/3,0). The Jacobian matrix is : : ge = (cosso Jeose a) and soa (0,0), r= 0 and A = 1/2. Thecefore (00) sa sade point. For X = (4/3,0),7 and A = 3/4. It is not possible to classify either critical point in this borderline case. 22. The corresponding plane autonomous system is vow yerse(haa)y If (¢,) is critical point, y = 0 and so —z — 2" = -z(1+2) = 0. Therefore (0,0) and (—1,0) are the only two critical points. We neat use the Jacobian matrix _ #00= (25, yo) to classify these critical points. For X = (0,0), 7 = 1/2, A = 1, and 7? — 44 < 0. Therefore (0,0) is an unstable spiral point. For X = (—1,0), r = 1/2, A = —1 and so (—1,0) is a saddle point. 23. The corresponding plane autonomous system is wiv andthe only ertial point is 0,0), Since te Jacobian matrix is : co 7 g)= Ce a ): 472 # -Wl-2) 24, 25. 28. 27. 28. Exercises 10.3 7 = =I and 4 =U, and we are wnable to classify the critical point in this borderline case. ‘The corresponding plane autonomous system is te day foo ler td and the only critical point is (0,0). Since the Jacobian matrix is an g(X)= ( 1-2? ) wer 7 2, A= 4,72 44 = ~12, and so (0,0) isa stable spiral point. In Exercise 5, Section 10.1, we showed that (0,0), (x/1/¢,0) and (— \/I/e, 0) are the critical points. ‘We will use the Jacobian matin \ Go 1 #00- (soar a) 1 classify these three eritcal points. For X = (0,0), r = @and A = 1 and we are unable to classify this rtical point. For (4/1/é,0), r= 0 and A = ~2 end ao both of these cttieal points are saddle points Is Exercise 6, Section 10.1, we showed that (0,0), (1/e,0), and (—1/¢,0) are the critical points Since Dez|2} |x|, the Jacobian matrix is #0 (sect 0): For X= (0,0), 70, A=1 and we are unable to classify this eritieal point. For (1/e,0), 7 ‘4 ==1, and so both of these critical points are saddle points ‘The corresponding plane autonomous systera Is = reise a. T+ ota? and the Jacobian matrix is Por X (0,0), 7 =O and A = 2. Since 9 < 0, we may conclude that (0,0) isa saddle point From 2! = ~or +ay = 2{-a-+y) =0, either 2 = 0 ory =o. r= 0, then 1~ Ay = 0 and so /9. The case y = a implies that 1 — fla ~2? 1 =a. Since a8 > 1, this equation has no rea solutions. It follows that (0, 1/3) isthe unique ertieal point. Since the Jacobian matrix : goo=(22" 3) 473 29. 30. 3u 32, Exercises 10.3 Logg teat re na-p+ 0, Therefore (0,1/8) is a stable critical a point (a) The graphs of —r+y~29 = 0 and -2—y+y’ in the figure. The Jacobian matrix is voo- (712 1 ) oS x -1+2y For X = (0,0), r= -2, A= 2, 77-40 = -4, and s0 (0,0) is : 1 stable spiral point. oak = a (b) For Xy, A = 6.07 <0 and so # saddle point occurs at X1 are shown q (a) ‘The corresponding plane autonomous system is 1 vey ved W)-2 and so the only critical point is (0,0). Since the Jacobian matrix is sof? #9=(F ayy): rae, Am Land 90 124A = 2 —4 at the critical point (0,0) (b) When r= > 0, (0,0) isan unstable tcl pon (c) When ¢ <0 and r? ~ 4A = 4 <0, (0,0) is a stable spiral point. These two requirements ‘can be written as -2<¢ <0, (d) When ¢=0, 2” +2-=0 and so x= c1 cost +cnsin£. Therefore all solutions are periodic (with period 2r) and so (0,0) is a center. 8H n= say be a by sepnng e. 1llontakyeat= eX) = (0,0) where 2 > 0, then © = 2f so that y= 2f—24. Therefore if -29 <2 < 20, y? > O and so there aze two values of y corresponding to each value of 2. ‘Therefore the solution X(t) with x0) (20,0) is periodic and so (0,0) is @ center. may be solved by separating variables. It follows that 1 2 as v since X(0) = (¢(0),2"(0)) = (40), ¢= 2. Therefore Z +2 (e—1){a?— 22-2) 2 3 — 474 33. (a) 34. (a) Exercises 10.3 But (2-1)(2? 22 ~2) > 0 for 1~ V3 <2 <1 and so each = in this interval has 2 corresponding values of y. therefore X(t) isa period solution. zy = 0 implies that either =O or y= 0. If z= 0, then from 1-2? +y?=0, y= —1 and so (1,0) and (1,0) are critical points land there are no real solutions. Ify = 0, 1~: The Jacobian matrix is and 807 =O and A = 4 at either X = (1,0) critical points in this borderline case. dy_y_inwey dy Oe ay Ge w= ¥, ivfotowsthat 4 = 3 1 and co p= -2-2426 ve L Therefore © = —1—24+2e which ean be putin the form (eet y= 2-1 ‘The solution curves are shown and so both (2,0) and (1,0) are centers. z dy a1 En yandeo Bye ay (b) Let w= yi = oP. 1 flows that 22 + 20 = 22, a linear fiat order dierential equation @ ‘whose sotution is rie (2a vawae™+ (5-2) Since ¢(0) = } and (0) = 2/(0) = 0, 0= c and so 1 2 a parabola with vertex at (1/2,0). Therefore the solution X(t) with X(Q) = (1/2,0) is not periodic aoe oo F-Scory = S22 4a, Since o(0) = 0 and (0) = 2'(0) = vo, it follows that oy = uf and so 475 Exercises 10.3 (214 ng—1 a Pag s ‘The r-intercepts on this graph satisly re ft 2h and so we must require that 1 ~ 21 > 0 (or [vol $ 4VE) for real solutions to exist. If af = 1~ T= 20h and —2 O and so there are two corresponding values of y. Therefore X(t) with X(0) = (0,19) is periodic provided that |uo| < }v2. 36. The corresponding plane autonomous system is ‘Therefore we must requte that €< | for nial solutions to exist, We will use the Jacobian matric #=(,2 5 9) to attempt to classify ((1 VT=4)/26,0) when € < 1/4. Note that r= 0 and A= FVT= For X = ((1 + vT=4)/269) and ¢ < 1/4, A < 0 and so a saddle point occurs. For X = ((.~ VT=AE)/2e,0), A > 0 and we are not able to classify this critical point using linearization, 87. The corresponding plane autonomous system is . IX = (x,y) is a critical point, y = 0 and ~a ~ fx* = —2(a+ 82") = 0. has no real solutions and so (0,0) is the only critical point. Since w0-( =R/L <0 and A= a/L > 0. Therefore (0,0) is a stable critical point. 1f 6 < 0, (0,0) and (42,0), where #? = 0/8 are critical points, At X(42,0), r= —R/L <0 and A= ~2a/L <0. ‘Therefore both critical points are saddles, 38. If we let dz/dt = y, then dy/dt = —2*—z. From this we obtain the first-order differential equation dy duet Pte de ~ deja v and IG >0,a+6x* 476 Exercises 10.3 Separating variables and integrating we obtain fudu=- [(S+2)é and ye jst 52 +o. a 2 Completing the square we can write the solution as y? = ~¥(22 + 1)? + ep. IEX(0) = (20,0), then 2 = Ha§ +1)? and 50. Lata yta Mga pp? tebe lat 20? 30 +0? + 58+) ; (eh+ 22)(03 — 2°) + 2008-2) _ (oh 2? + 2)008 z 2 Note that y =O when 2 = zo. In addition, the right-hand side is postive for ~2 << zo, and 40 there are two corresponding values of y for each 2 between —z9 and 29. ‘The solution X = X{¢) ‘that satisfies X(0) = (10,0) is therefore periodic, and so (0,0) is center 39. (a) Letting z= 6 and y = 2! we obtain the system 2" = y and of = 1/2 sinz. Since sinx/6 = sintin/6 = 1/2 we see that (1/6,0) and (61/6,0) are critical points of the system. (b) The Jacobian matrix is eo a ( a) and so Ar=e=(0r150)=(_ sje 4) md Arm e'=(6r/60)=( $0 6) -Vv3/2 0. v3/2_ 0 Since det Ay > 0 and the tzace of Ay is 0, no conclusion can be drawn regarding the critical oint (*/6,0). Since det A <0, we see that (5%/6,0) is a saddle point (©) From the system in part (a) we obtain the first-order differential equation dy _12-sinz a v Separating variables and integrating we obtain [ove [(f-ses)e it p= je toozta and B+ 2e082 + cp aqT 40. (a) Weiting the system as 2 Exercises 10.3 For 29 near 7/6, if X(0) = (20,0) then e Thus, there aro two values of y for each z in a suflicently small interval around /6. Therefore x9 ~ 2oosao and y? = z+ 2coss— x9 -2cosa0. (7/6,0) is a center. a(x? — 2y8) and yf = y(2z4— v we see that ©, 0) is a critical point. Setting 2° ~ 2y = 0 we have 2° = 2y4 and 2r5 — the only critical point of the system, (b) From the system we obtain the first-order differential equation dy ayy ae es (y= "adr + Qn —24) whichis homogeneous, If we let y = ur it follows that (224u — atu) de + (2e4u ~ 2")(ude + edu) = ahu(1 +3) de + 28(2u5 — 1) du Lae Bah an Integrating gives In| + nf +1] ~Inful + nu? a4] = er ie 2(M) (e-uen=a C52) -te)-> (ey + 2)? — ay +22) = o2n’y ay at hy? = Beaty, 478 Exercises 10.4 (e) We see from the graph that (0,0) is unstable, It isnot possible to classify the eiticl point as a node, saddle, center, oF spiral point Exercises 10.4-—__ 1, We are given that 2(0) = 0(0) = % and (0) = (0) = uy. Since y 2 eosz +e, ub = 8 cosk pend wand erefore f cost tem $+ eand socm ug 2, There ait l and the a-intercepts occur where cos = 4 — = uf anc wf must be greater than —1 for dt pts here cos = 5 ~ 95 wh an 3g wd mst be greater than —1 a “7 slvons to ext. The een eqn tool < 2 (a) Since o? = 72 cosa +c, (0) = 8(0) = Bp and v(0) = #10) = 0, ¢ = ~*Zcosty and so ¥ 22(coso ~ costs). When # = 8, ¥ 72 (can) ~ cos@)) = 0, Therefore y = F = 0 # when #= () Since y= % and 0 is decreasing between the time when 9 = dy, t= 0, and @ = ~G tht i te=f, . 9 ae $= 9 (esta T 3 yarn V5 Lo verre? Prete” 3. The corresponding plane autonomous system is ae therefore cy and Exercises 10.4 z( fz) 8 ) G+ Lei F"(z) - F@)2f te) Fa) ae SPE mY, +P @RF : If X1 = (z,y2) is a etitical point, yr = 0 and f"(z1) = 0. The Jacobian at this critical point is therefore om) =( 4, When 6 = 0 the Jacobian matrix is ~9f"(a1) (-srtey 4) which has complex eigenvaluss \ = /9f"(ai)i. The approximating linear system with 2/(0) = 0 has solution a(t) = x(0)eos Va"(@t and period 2n/\/9f""(a1). Therefore p = 2n/,/gf"(x1) for the actual solution. 5. (a) Ff) =F, f@) =z ands ay BZ By We may separate variables to show that y? = —gln(1 + 2?) +c. But 2(0) = zp and y(0) = 2'(0) =v, Therefore c= of + gln(1 +28) and 20 ae 1+? aari-oie(23) Now Gan (145) 20 and only if 2? < e8/9(1 +28) —1 ‘Therefore, f |x| < [e'8/9(1 + 23) — 1}4/, there are two values of y for a given value of z and so the solution is periodic. 2 (b) , the maximum height occurs at the largest value of x on the cycle, From (3), Emax = [ett/9(1 + 28) ~ 1]! and so Fax _ Lp nd/oy Fax = ABM = 5letH/9(1 + 29) — I] 6. (a) If f(a) = coshe, f'{z) = sinhz and [f'(c))? +1 = sinh? +1 = cosh? x, Therefore ty _, sah 1 & costs ¥ 480 Exercises 10.4 ‘We may separate variables to show that y? = 22 +e. But 2(0) = zy and v0} = 210) = 00 Therefore c= 08 3: and so. a veo oye cosh” Sash z5 Now 2 2g cosh 20 ~ 2A 4820 itand only if cose < PORE ZD_ cosh ~ Gtahay * U9 $35 —wfoonm and the solution to this inequality is an interval [aa]. Therefore each x in {2,4} has two corresponding values of y and so the solution is periodic (b) Since z= cosh, the maximum height occurs at the largest Value of z on the cycle. From (a), 2gcosh r9 tmax =a where cosha = —~2°5P29_ Therefore wees 29 cosh-29 tow = Bg ef cosh . If tm < 21 < tn, then Flat) > F(tm) = F(zq). Letting z= x1, co Flim)G(a/b) Fey ~~ Fe ae Fay Gy) = << Glo/, ‘Therefore from (2) on page 474, Gly) thas two solutions yr and yp that satisfy wm 2, there is no corresponding value of y. Therefore the maximum number of predators is zq and tn Occurs when y= o/b (a) In the Lotka-Velterra Model the average number of predators is d/e and the average number of prey is a/8. But as nar they q2=—(ate)r + bey o = ery + dy ~ ay = ~cxy + (- ey a/b + e1/b). (b) The average number of predstors d/c— ez/e has decreased while the average number of prey a/b +4 /b has increased. The fishery science model is consistent with Volterza's priniple. fand so the new ctitia point in the frst quadrant is (d/e— e/ 481 10. a 12. 13. Exercises 10.4 (2) Soiving . : 2(-0.1 + 002y bd > : a (0.2 ~ 0.0252) = 0 in the first quadrant we obtain the critical » 4% © © 10 point (8,5). The graphs are plotted using (0) = 7 and 4(0) = 4 (b) ‘The graph in part (a) was obtained using NDSolve in Mathematica, We see that the period ' around 40. Since (9) = 7, we use the FindRoot equation solver in Mathematica to approximate the solution of 2(t) = 7 for t near 40. From this we see that the period is more closely approximated by t = 44.65. Solving (20 - 0.42 — 0.8y) 50 y(10—0.1y 0.32) =0 ‘we see that critical points are (0,0), (0,100), (50,0), and (20,40). The Jacobian matrix is 008(20-082-03y) —— ~aurae aX) = ( ~G.018y caceese) snd s0 =o A=eon=(') J.) ome) = (“TF 34) an 6 OM As =w'(60.0) = ( ad ) Ag= 6'(20,40) (am on): Since det(As) = Ay = 0.98 > 0, r= 2.2 > 0, and r?—441 = 1 > 0, we see that (0,0) is an unstable node. Since det(Az) = 2 = 0.48 > 0,7 = -1.4 <0, and 7f ~442 = 0.04 > 0, we se that (0, 109} is stable node, Since det(Aa) = As = 0.48 > 0, r = ~19-< 0, and 7f — 40g = 1.69 > 0, we see that (60,0) isa stable node. Since det(A,) = ~0.192 < 0 we see that (20,40) is saddle point Marina rary traand 7! 40 = (r+ra)? arava = (ru ry). Therefore when 7 #72 (0,0) is an unstable node, ForX=(Kj,0),7= =n¢ra (1 Hon) aida ona (1~ ft 2-40 = (orn + r1)8 > 0. Now thy > A, ¢-< 0 and sor < 0, & > 0. Therefore (Ki) 8 K Ky 2). teen 1-H, sede 1, < £2,050 ade 8 ath (0) ee po 482 Exercises 10.4 14, (é\8) is a stable node if and only if a > Ke and 2 > K,, [See Figure 10,38(e) in the text] From Problem 12, (00) isan unstable node and from Problem 19, sinoe Ky < 22, (4,0) is 0 saddle point. Finally, when Ko < £1 (0, Ke) is «saddle point. This is Problem 12 with the roles ain of 1 and 2 interchanged. Therefore (0,0), (41,0), and (0, Kz) are unstable. Ea ag tit as 15. =) < Ka < Kian, and so ayz0ni > 1. Therefore & = (2 — axon) PE <0 and s0 (2,8) is an Rk a saddle point 16. (a) The corresponding plane autonemous system is ‘ a 8 vay vestane— Sy and so critical points must satisfy both y = 0 and sinz = 0. Therefore (nr, 0) are critical points (b) The Jacobias matrix > Oat (0,0). Therefore po Basin? i me We may conclude that (0,0) is stable spiral point provided 6? ~ dglm® < 0 or 6 < 2my/gl 17. (a) ‘The corresponding plane autonomous system is zen vt L 2 yoi- Ke and so a critical point must satisfy both y= 0 and x = 0, Therefore (0,0) is the unique critical pein, (3 ) Baty) sutra oand a= £50, Thar cee, sabe it plo an uae tpt put, Psa eatin sages a (2.0) mut be asap sae nd (0) st be ales ae (b) The Jacobian matrix is 483 Exercises 10.4 18. (a) The magnitude of the frictional force between the bead and the wire is (mg cos ) for some > 0. The component of this fictional force in the z-direction is (umgcos8) cos = umg cos? & But ime i+F@P cos = and so ymg) It follows from Newton's Second Law that , (oes oe melo ma epee AY TSE | wag tcl@ By FUP (b) A critical point (zy) must satisfy y = 0 and (x) = 4. Therefore critical points occur at (1,0) where /“(21) =. The Jacobian matrix of the plane autonomous system is 0 1 e(X)= (, +2) P"() == PFS") 3) a+ Fey = and so ata critical point X;, 0 1 g®= ( fia) _8 Tee Tm Therefore r= -2 < Oand a= she) When /"(21) <0, A <0 and so a saddle point ceurs. When /"(t1) > 0 and (21,0) is a stable spiral point. This condition may also be written as tone LEY 4 ¥_ 10) Zoe 24 2F{s) = 19, B= F = £2 and so using separation of variables, % =~ f° flu) du teor y? +2F ts) =e ‘We may conclude that for given value of x there are at most two corresponding values of y. If (0,0) were a stable pial point there would exist an z with more than two coresponing values of Rote thatthe condition f(0) =O is ruled for (0,0) to be a critical point ofthe coresponding plane autonomous systecn 2! = y, = ~f(2). 484 Exercises 10.4 20. (a) x = 2(-0-+ by) = implies that 2 =O or y= a/b. Wz = nary + FWK~y) =0, y= or K, Therefore (0,0) and (0,4) are critical pois. 1 ieee Zex—a] =o ‘The cotresponding value of x, x = &, therefore satisfies the equation c (b) The Jacobian mats is watby be (X= “ ( K- a) 01), -w and 50 at X; = (0,0), A= —ar < 0. For Xj niKb~ a) =10(~f). Since we are given that K > $, A < On this case, Therefore (0,0} and (0,4) ave each saddle $ and o& ote s'(Ga) ( ot ) -o —RF, andsor = -2 9 0. Therefore (2,4) is a stable critical point and so it is points. For X; as (E.9) where 9 CK ~ A), we may write the Jacobian matrix either a stable node (perhaps degenerate) or a steble spiral point, (c) Write using ‘Therefore 1? — 4A <0 if and only if jc fOK aK? US Fea aK Note that 4K? ab THOR FRR F < K. Therefore 7? - 4A < 0 when K is large and a stable spiral (KEK where K is large, and point will result. 485 a. 22, Exercises 10.4 ‘The equation - az Ynarye-2= implies that x = 0 or y= When @ > 0,9 = 7 equation for y', y = 9. On the other hand, if I: follows that sea(o-y)= <2 yllo-n0-1 and if (a ~ 1) > 1, #>0, Therefore (2,9) is the unique critical point in the fist quadrant. The Jacobian matrix is fof oe 7 oye) Wey g(X)= ( : = ) “ity Wy and for X =: (2,9), the Jacobian can be written in the form o «(a (9) = 4A = (441) = 4A = (4-1)? > 0. Therefore (4,9) i @ It follows that and sor = —(A +2). Therefore +! stable node. Letting y = 2 we obtain the plane autonomous system Solving 2° ~ Gx? + Br = 2(22 — 4)(2? — 2) = 0 we see that cxiticl points axe (0,0), (0,2), (0,2), (0,-V3), and (9, v2) ‘The Jacobian matrix is ax 0 1 (onsee—set a) and we se that det(g/(X)) = S2'~ 182-48 and the trace of ¢'(K) is D. Since det(a((+V3,0))) = 8 < 0, (£V2,0) ace saddle points, For the other extical points the determinant is positive and Xinearizationeislases no information, The graph of che phase plane suggests that (0,0) and (2,0) ate center 486 Chapter 10 Review Exercises Chapter 10 Review Exercises —_________ . True ‘True a center or a saddle point complex with negative real parts . alee; there are initial conditions for which jim X(t) = (0,0) Tue False; this is a borderline case. See Figute 10.25 in the text . False; see Figure 10.29 in the text . The system is near and we identify 4 when A> 0 and 7 =0 we see that for -aandr +1, Since a critical point will be a center =1 critical points will be centers and sotutions will be petiodic. Note also that when a = —1 the system is d=-2-2y vaz+y which does have an isolated critical point at (0,0) 10, We identify g(2) = sinz in Theorem 10.2 Then 21 = nom is a critical point for n an integer and 9 (nm) ‘when m is an odd integer. cosn <0 when n is an odd integer. Thus, nx is an asymptotically stable critical point 11. Switching to polar coordinates, ae @ a a 1 Using separstion of variables it follows that r = and = ince X(0) = (1,0), 6 Sep variables i folows that r= pti and @= t-+en, Since X(0) = (1,0), and @ = 0, It follows that = 1, c2 = 0, and s0 L your Ast ~+00, r+ 0 and the solution spirals toward the origin, 12, (a) IFX(0) = Xe les on the line y = ~22, then X(t) approaches (0,0) along this bine. Far al other Initial conditions, X(¢) approaches (0,0) ftom the direction determined by the line y = 2 487 13. 4. 15. 16. Chapter 10 Review Exercises (b) IE X(0) = Xo lies om the line y = ~z, then X(t) approaches (0,0) along this line. For all other initial conditions, X(t} becomes unbounded and y = 2z serves as an asymptote. (a) r= 0,8 = 11> 0 and so (0,0) is a center. (b) += -2, A= 1, 7 — 40 =0 and s0 (0,0) isa degenerate stable node. From 2’ = 2(L+y—32) =0, either ¢ =0 or 1+y—32=0. Itz =0, then, ftom y(4—2e—y) =0 wwe obtain y(4— y) = 0. It follows that (0,0) and (0,4) are critical points. If 1+y—3z = 0, then y(5 ~ St) = 0. Therefore (1/3,0) and (1,2) are the remaining critical points, We will use the Jacobian matrix Leyte x)= eo ( ty 4-2 | to cast these four eral pot, The rents area allows A rF48 Conclusion 7 unstable node -20 ~ ——sadile point ~~ aadele point 10-15 stable spiral point From z= rcos8, y = rsind we have rind oT cose a ata 4 eos 4 © sin Ce ae ‘Then 1! = ar, 6 = 1 gives = -rsind + ar cos de @ 8 ycos8 + arsin 3 8 +arsind. We see that r = 0, which corresponds to X = (0,0), is a critical point. Solving 1’ = ar we have re. Thos, when a <0, liao r(t) = 0 and (0,0) is a stable critical point. When a = 0, and r = ey. In this case (0,0) is a center, whichis stable. ‘Therefore, (0,0) is a stable critical point for the system when a <0 ‘The corresponding plane autonomous system is 2! = (1 22) ~2 and so the Jacobian at the critical point (0,0) is gcoo=($ 1) “1 488 Chapter 10 Review Exercises Therefore + =u, A= Land 7? 48 = y?—4. Now y?—4.< 0 ifand only if -2 2. IT. Critical points occur at x = 41. Since ge) =- 572" — ae - 1), (1) > 0 and of(-1) <0, Therefore x = 1 is unstable and x = —1 is asymptotically stable. ww. 2. We may separate variables to show that yy@ti= 2? +e But (0) = 29 and y(0) = 2/(0) = 0. It follows that c= 2+ xf so that 2p Potde2 Note that 1423-2? > 1 for ~ag < x < ap and y = 0 for z= +2, Bach x with —ry <2 < zo hhas two corresponding values of y and so the solution X(t) with X(0) = (20,0) is periodic 19. The corresponding plane autonomous system ‘and so the Jacobian is For X = (0,0), = 4e> 0, therefore ky 2B a Sig? — 10st) ‘Therefore (0,0) is a stable node if 6? > 12hms" and a stable spiral point provided 4 < 12kms?, where ks? = mg. 20. (a) If (z,y) is a critical point, y = 0 and so sinz(ui? cosx ~ g/t) = 0. Bither sinz = 0 (in which case x = 0) of cosz = g/uHl. But if u* < g/l, g/u*l > 1 and so the latter equation hs no real solutions. Therefore (0,0) is the only critical point if w? < g/l. The Jacobian matrix is 0 1 #00 = ( ) wPeosds —feosr —2, and sot = —Sjmi < 0 and A = g/l—w? > 0 for X = (0,0). It follows that (0,0) is asymptotically stable and so after a small displacement, the pendulum will return to # o=0. 489 Chapter 10 Review Exercises (b) fw? > g/l, cose = g/l will have two solutions x = +2 that satisfy —7 <2 °- Poa ‘Therefore (2,0) and (2,0) are each stable. When (0) = 6, 6'(0) = 0 and ép is small we expect the pendulum to reach one of these two stable equilibrium positions. 2 2 9 2 alsa A= 9 cost - uP eos2t wf (-fn-1) 490 11 Orthogonal Functions and Fourier Series Exercises 11.1 2 tetanic) 3. [elec — ede = 4. [lowsentrécn hte 7. Form#n [satan +1)2 snlom + t)eds 1 pe ae 23 (C eetin = me oman + sd 1 i, ra ~ ggg me [= galanin nell Form =n, at | L(G - pene2ten + noe) ae Lye lo ~ We+D) sin 2(2n + 1)z lo 491 Exercises 11.1 so that Form#n J coston + 1)zcostam + 1)2d2 = 4 c0s2(n—m)e + eos2{n-+m + Halas |_| ra aap Me |S + Tp snain tm + De <0 Forman [Poteans eden [0 (b+ ecexan 02) Lp oy fa + gphpsnzant nel! so that 1 Moos(an + 1)xli = 5V%. Formén [(sanesimede-=} [esin ma cas + mde aeray tint mel’ For m so that Exercises 11.1 10. Form #n [sins in Meee} [cos Ps con SAMS) ae ae ap gree pg, Intmn, P area Sl aera Sl fam=n flow Eaten ff [E Pooalacn be Ean Bel E ott edi 1. Fermgn erm. (n- (enn, (nme [cos cos Mae = 5 [? (cs 00 RIS) ae a ue mel —_ Ja mye P Imam “0 Frmen LP 4g Pg 2 [oat Mhoaem f(b foots) den Zo + Panes “ [ewMecee Ein Za[ao ant [Mee=p oo f count Wh = yp and jst 12. For m# n, we use Problems 11 sud 10: ii pepe la osMedr=o [frites Mate =2 [sn Ts an™oar=o ae oom gan AE.) domo 493 Exercises 11.1 [vostEzde= Lane [r-sn%ete= Zoos = Qn Boog MEAP [sins cos eden f° fon rds» Pew a wo Reman [08 adem [* ( a Joos 2) as = [iso? Made= J (3 ieos™Es) don ond [ire- so that In yap. fow%s|= vo, an Jet - 0 Sine [lettin and wo (te -aidens [2 (tne?) tea [nett wa(-te? Pat feet) a fete ° re) pe ee a =4( Le [+2 fax ‘dz=0, the Fanetions are orthogonal 494 14, Exercises 11.1 since fetaa- aér=(e-Ye* | - [eterno [eta (Jet ae41) dem (22-1 ~ Ft} [P+ [ere 2)ae a1 2-2)" | Pf eds =0, and [feto-a (hears t)ar Co [oet(feve-s)ee nveQ2 -5e43 yf 1 [rere-se wt-aeeer-o ff -a Coreen, tthe functions are orthogonal . By orthogonality j* do(z)da(z}dz = 0 for n=1,2,3,... 5 that is, {2 da(e)dz = 0 for n=1,2,3, }. Using the facts that dp and éy are orthogonal to @, for n > 1, we have [ez Bintz)dz= af s0q(a)de+ 0 [2-dn(e) ar maf oxlaiénte)ér+ a f° doladén(a)éx =a-04+9-0=0 for n= 2,3, 4)... Using the fact that gy and dy are orthoganal for n # m we have Ventz)-+ bei? = fnta) + bea J [62) + 24m (elén(z) + 68) dr = [lena +2 [ ontarintclee+ abla) ae = llema(2)]P + lho)? 495 Exercises 11.1 18. Setting 2 2 Ala\f(ajae= [) (2 +e0 +e!) de = Bp Soy and f fale) pla) ds= f(b ot tenet) de = 5/12. _—. ‘we obtain cy = 0 and (usnne) = [{saneds 0 and f(z) = Lis orthogenal to every member of {sinnz}. Thus {sin nz} is not complete. 20. (Fi of) = [tiled + ACeialeyae = f° fte) fle) de+ [fle ale) de = (fi B)+ (Oe) 21, (a) The fundstental petit is 2/2 (b) The fundamental pei 3 2n/(4/2) =o (6) The fundamental period of sin + sin x is 2n (4) ‘The fundamental period of sin 2r + cosdz is 22/2 = (©) ‘The fundamental period of sin3x + cosdx is 2x since the smallest integer multiples of 2x/3 and 2n/4 = 1/2 that are equal ace 3 and 4, respectively. (£) The fundamental period of f(z) is 2n/(nx/p) = 20/n. Exercises 11.2 ————__ Ab-cn) LS IE CY sna aon tf" seyret? rde4t a=} ft sa)corneds = [? —cosnzds+ [" pxeneie 1 ce 1 pejainedse} ? -sianeder} [aninneds = Spa) 496 Exercises 11.2 fee 5e8 Peers {Mone 1 ed Majds= Piast fi zdr= 5 [, Fs) cosnes em f connnzds + [' xecennsde = alr [,Ala) sinned fe)= 3.8 [c ape saans- Laan 4. apm [' se)dem f'zde~} [/,fe)eosnrade = [' zcosnazde = Pl(-1)" 1 cyt (@)sinnezde = [' esinnrzds = peje [RR cone 4 EO" sinar| veo} ayden} [stern bet " fa)cansdze} [°Peosneds «2 (Z sinnz [-2 [fesencee) 20" ( Pennede= 2 (—E cone [+2 ff eomnear) = Zaye Secu —at Sa) = BERS Da (E-uret + =) sna] “jayden feared [aa arabe? "eanies}f gesmcees![( (2-22) cosnedz =? inne 42 rsinntde) = 3(0™ iw " sle)sinnzdr= 2 f? sPeinnede + 2 f(x? 2°) sinnads 497 a Exercises 11.2 = Hemr-y+2( oie 2 [fsenede) = Hoyts Eu-) 4) 8 $ [Rca tonnes (Hens LEE) a= 2 f" peyae (24m) cosnede an =? (" fle)cosneds = + ay fears 2 Hap eane =1f soyae=} [(s-2dr=6 ee yt beat [(e-20)sanrde = 4-39 ye fanaa sinne co= 2" fede} [sinzde=2 2 facosnede => [sinz cosnrdz = 2 ["pin(n + te + sin(1 —n)a}dz boat [" se)sinneas = =f [eostt—n)z—cos(t + nlzhde = 0 for =2,3,4, oo #f0-maner} 498 Exercises 11.2 2 i 10. ap= 2 ("7 a= 2 (7 a)cosanede = 2 ay” cos cos nz dz = 2 af? [cos(2n — Iz + cos(2n + 1)z}dz feyde= 2 f°" coszae = 2 __“ Hata 2-1)" 4n fla) = $+ S [APS cotnes it en 4 =} fi sede 3 (faa f° ur) =-} wo Ufo (f fata fea) dat Lfisrn tare} (fami et [ate «2 (to) fone sin Foon at 3 (1 ~ conf) sin Fa] a a(fises fre) =§ ; (floes Frees [feeds] = aha (oF -1) f ai cia [iso%zas) teks Fea) sey=5+S [aa (cet sua B+ (sin +E (-ay*) sin] be wont poer=2(ft tée+ Passes) = : [7 se)eus Fede =F E(fiontBeaes ff aes) on eds) = Sy") agg 1 1 v Exercises 11.2 bad [Mean Baden (fia Bede ['a+a)om eas) = S(ye fla) = Boll" — toon Me + 5 (1) sin a] a= 5 f'sfez=} }(Liernaes ['2ee) = 3 oon jf seeos Bede = 3 (24s)cos Binds + [2008 ede) = alt (-U"| nj [,feban Bade 3 (fi esaysin ears [ain eas) = 2 Ne) = 5+ ¥ [geal - rea Bet By ain Fe] oat" feydent f etae= dere) an=2[" sa)cosnzde = ee a [Lse@)sinnede =} [ @sinnzdr = crates 6 +E [ERE aes EG a . ay 2 f” pajdee? [er ~ 1dr = Her 2-0) [l fe2pcosneds = 2 ["(et — 1yoosnzds = co = os F(a)sinnrde =} [e* ~a)sinnede = 2 (PECU a, CU" 2) f(a) = 4 So oes oe+(a fee +1] =) sna] + The function in Problem 5 is discontinuous at x = x, s0 the corresponding Fourier series converges 500 18. 19. 20. Exercises 11.2 to 22/2 at z=. That is, Hot) nnn] At z= 0 the series converges to Cand 2? Say 2 eG rors Problem 17 wads a 2 s (5 5) 3Ct# ‘The function in Problem 7 is continuous at z= x/2.s0 a and Exercises 11.2 BL. (a) Letting a = 49/2, en = (On ~ iby), and cn = (Qn + ibn)/2 we have eole 694 FE (eel eaggl-MIH) E exehvel, (b) Multiplying both sides of the expression in (a) by e~*""*/P and integrating we obtain L Hayes ie (= cin) os E a ftonnete 2 P aslo-mrslPde + on [? elimi, ze fre da tem f le = Sn f clmtslrds son [? = ie do fie = Boo fetite + 20m Recalling that e meosytisiny and <= cosy ~isiny we have for nm an integer and nm [oma Ponies Tan mie (om eum) [cos(n — m)x + sin(n — m)r ~ cos(n ~ m}r + ésin(n ~ m)x] 502 Exercises 11.3 Thus [1 Feels = 27m and — \-imrz/P, on 5 [fleets we have [Lote d= 22. Identifying f(2) = e-* and p ent ge 2 Ble 1 : 7 atin Was He *(cosnn — tsinnen) en De _ (ete Foosne _ fen~e")(-1 Bint ie ine) ‘Thus fla) = So (BE oie nee Bin + Uy Exercises 11.3 . Since f(—2) = sin(—32) = -sin3x = ~f(2), f(z) Is an odd function, Since (2) =2 cos 2 = ~f(2), f(z) is an odd function. Since f(—2} 1, f(z) is neither even nor odd Since f(—2} (29 — 42) = (2), #(2) is an odd function, Since f(-2 F(a), f(2) is an even function. Since f(-2 ~s(z), f(z) is an odd function 1 For 0< 2 <1, f(z) = (-z)? = 2? = ~f(2), f(z) s an odd function. For 0$2 <2, f(-z)=-2+5= f(a) f(e) is an even function Since f(z) is not defined for 2 < 0, itis neither even nor odd, 503 u L . Since’ f(—z) = |(-2) Exercises 11.3 [=| = #2), F() is an even function. Since f(z) is an odd function, we expand in a sine series: [[vesancde= 20-7) ‘Thus * S| 100) = Slt (anne . Since f(z) is an even function, we expand in a cosine series: ‘Thus Te ne ne felag+L Gan Zon Fe }. Since f(z) is an even function, we expand in a cosine series: a = Ths . A X aiel-" ~ Yeosnz. Since /(z) is an odd function, we expand in a sine series: eat ff sianade = 24 fe)= DE -1)"*" sinns. Since f(2) is an even function, we expand in a cosine series: wnafiteen? (Zesanee|l- 2 ssianesae Thus a He) = 5+ Beat cone 504 16. ay. 28. 19. Exercises 11.3 Since jx} is an odd function, we expand in a sine series 2 A ba2 [| einmede=2(—E coenne fre conned) et ar oh Thus i= ‘— + galt - 1) sinnaz. Since f(z) is an even function, we expand in a cosine series: wa? [escent aye " ) = ays Bay. Thus a 12 (x) = (23) + (-1)" | sin nz. 1e)=¥ (Beas Bea) Since (2) isan od funtion, we expand in a sine sere: 2 ned [ter snnede = 22D ape 2 1 tos £ (Dears 2) sos 505 20. a1. 22. 23, Exercises 11.3 Since f(z) is an odd function, we expand in a sine series: 1 m2 f'(e-nsnereds=2[[!ssinnnede ~ f'sinnrzaz] =2[ghrsinnre ~Zcosnre + 2 cosney], Ths s(2) : Zsiunee. Sint f() i a ee union, we expand ini eie om fisare ['rae~8 on = fxm Eade + [cos Fade = (oi “Ths ee (ee ee se= 7+ 3 aia (oF -!) Since f(e) 6 an ode function, we expand in sine series Ve ee bal fl ennBeaes [nn Bede ‘Thus 1 = 3 (lesa Eg 20 ‘Since f(z) is an even function, we expand in a cosine series o=2 [lonsee= oxo? [sae canede [ino erent aiid walaglt+ Can on=3.904 ‘Thus Exercises 11.3 24, Since f(2) is an even function, we expand in a cosine series. [See the solution of Problem 10 in Exercise 11.2 for the computation of the integrals) 2 pe oe Tih OTR 2 pn nn Aue ona By [wore Bede SP Thus S@= ‘cos 2nz. vA 25. a2 [rae phe oe tan? [sioner 2 (1c 1,82 one flel= 5+ 2 psn F cosnns f0)= 2 3 (cos =(-1)") sinnez a7, 2 (Pon ected A ive [one snmede =? [poten si)e1 stn tees 8 507 Exercises 11.3 acy" a4) fan2s3 cos nz fa)= 5 oy sin 2nz 28, sinzde = 4 "ane oanede =} [nnn santa valde MV ben n.a.s bon? f sing sianede = + {"[oos(n—1)2 —cos(n+ 1)z]dz = 0 for n = 3,3,4, 29. fle) = iain E inne a eS (e-n)de=5 al See nm] zf (2) 08 Fade = =~ [(-1)" ~ cos byob fem aysin ede = 2(-1y - in 1)" ~ cos cos 52 508 a1. 3: 3 3. 1a =¥ (Zi = psn) an = [[xte+ ffrden’ osm [free fla-ayiend Exercises 11.3 oe sin + (1 ane [tose "Eade + fa spo tede = of (onl 4) by = fr sn dr ["(2— a)sin Fay Heye SS a (conf +) oe 1) = ¥ (Z + gaeysin"F) ain Fe co=2 "zt +a)de=$ on 2 [ (ets) emnrede = M242) mar [tha )sinnnede = Ete) = Sa + eal) Hee)= +E Spal" — dees =F (Score Aico 4 te dn Stee os Fe snore|,- 2 florenydannede= 2yh-9-1 cone | + 2 fee Noosnacd: ) snnes 509 Exercises 11.3 a 4 34. ap = f (22-22) = 5 oS fee = 22) cos Side = aay -y 6 bye [i ee-2sin ede Ssh - (0) fe) +E seal — teas Fe fle) = Saat (-y%}sin Ze 1 96.0} [sae 37, a =2 [exile =3 on=2 {ler Neon2mede—0 wen [tes dennzede= 2 f(z)= 38. aq =2 f2-z}de=2 510 39. Exercises 11.3 on = 2 ['2—z)cosnredr = 0 n=2 fo-ainmete= 2 e1er+ Sano We have so that sinnt ¥ Basin nt into the differential equation then gives ai Substituting the assumption p(0) Bh 10a, = 5 B,(10—n?)sinnt = Se = CU sinnt ont nel ca 301 — (-1)"} sed oy = SES ta eee tolt) = eB ato a8 We have 2 Pa -gsinnnedt so that sf) = 5 > sinnat. at Substituting the assumption z,(t) = 3° By inert into the differential equation then gives af tsp = 2,40-r2eanane= 8 ane at _ 2 and 30 Be = aay Thus 2 1 2 mio ata Ant z(t) = 511 Exercises 11.3 41. We have . ag = 2 [core eat =2 [cart -#)cosntde = ~an? waa ) so that, 1922s ESD cant Subattting tn assumption ay) = 24S Ancosnt q toto the eiferential equation thn gives = 60+ Fy 2-412) coae ‘Thus 2 209 +8 apt 42. We have pal) so that 1 &cyr-i f=5+ nie cos 2nat. Substituting the assumption Aq cos nat into the differential equation then gives, F204 tp = 6g + Fe An(22 ~ ne) costa (yr An amram) Ths a= 3 Exercises 11.3 48, (a) The general slaton is 2(t) = 6 cos VT0¢-+ op sin VO + alt), where 10 & 1-(-1)" ayy = ao sina ‘The initial condition 2(0) = 0 implies c1 + 29(0) = ©. Since zp(0) we have ¢, = 0 and 2() = spain VTbe + 2p), Then 2() = epVTcos VIDE + 24) and 2(0) =O pin ev + BS TCV cos 0 Ths = 10 S 1-(-1)" « cao wd w= BE ESP [Lanne gneve] (b) The graph is plotted using eight nonzero terms in the series expansion of 2(t), 44, (a) Te general solution is (t) = cos 43+ cn sin VEE + ap), where oe ait) G8 oom ooomt ‘The initial condition <(0) = 0 implies cy + 25(0) = 1 or 2 o-wy a 1 t=) = 1 zplO) = Now 2!(¢) = ~4VIey sin 43+ deg cos 4V31-+29(t), £0 2/(0) = O implies dey +2'(0) = 0. Since (0) = 0, we have c =O and oS | oe, ae (1-103 apts erate F +10 cepa conn ~ F) cose re S [cosnt ~ cos v3 513 Exercises 11.3 (b) The graph is plotted using five nonzero terms in the series expansion of 2(¢). 45. (a) We have so that (©) th we assume v(x) = > Bysin Me then we ntet aE and so the differential equation Bly!) = w(z) gives en, py = Beate = ETS Thos Pool & (ey! ae ve) Fis Le nT 46. We have so that If we assume y(z] a7. 48, Exercises 11.3 and so the differential equation Ely\® = w(z) gives Ba = tual — EES Thus Bank $ conf — co BE ne V0) = pe a T ‘We note that w(r)is 2r-periodic and even. With p = = we find the cosine expansion of _ [uw O 0 we have cos Viz + asin Viz. Now Y(a) = -aVisinVN2 + gVF008 Viz and y‘(0) = 0 implies cz = 0, so u(t) + (1) = (cos VI~Visin VI) <0 or cotVI= VI. ‘The eigenvalues are Ay = 22 where zy, 32,43, ... are the consecotive positive solutions of cot V3 = VX. The corresponding eigenfunctions are cos Vn: 12, 3, --. - Using CAS os yt for n = 517 Exercises 11.4 wo find that the first four eigenvalues are 0.7402, 11.7349, 41.4988, and 90.8082 with corresponding. ‘eigenfunctions cos 0.86032, cos3.4256z, cas6.4373r, and cos 9.5298n. 2. For <0 the only solution of the boundary-value problem is y = 0. For A= 0 we have y = ci2-tex. Now y’ = cy and the boundary conditions both imply ¢) + ¢ = 0. Thus, A= 0 is an eigenvalue with corresponding eigenfunction yp = 2 ~ 1. For \ > 0 we have eycos Vaz + epsin Vaz y and pee ee "The boundary conditions imply a tavi=0 1008 VR + epsin VX = 0 which gives ~eeVKeos VA +epsinvX =O or tan v= VX. ‘The eigenvalues are Ay = 2h where zi, 2, ma, ... are the consecutive positive solutions of tan VX = VX. The corresponding eigenfunctions are VTqcos Vaz ~ sin Tq (obtained by taking ey = —1 in the first equation of the system.) Using a CAS we find that the Rist four positive eigenvalues are 20.1907, 5916795, 118.9000, and 197.858 with corresponding eigenfunctions 4.4934 cos 4 4984x —sin 4.499, 7.7253 008 7.72532~sin 7.7253z, 10.9041 cos 10,9041~sin 10.9041, and 14,0662cos 14.0662r — sin 4.06622 3. For X= 0 the solution of y" = 0 is y = erz +62. The condition y/(0) = 0 implies ey = 0, 80 is an eigenvalue with corresponding eigenfunction L excosh ¥=Ke-tep sin V=Kx and y = eyy/=Nsinh V=Kr-tepy—Keosh YK, a For A< Owe have ‘The condition y'(0) = 0 implies ¢» = 0 and so y =e; cash YX, Now the condition y/(L) Implies ey = 0. Thus y= 0 and there are no negative eigenvalues ~oVAsin Viz + eWVAc0s VAz. The implies For A> 0 we have y = eyes Viz + epsin Viz and y/ condition y/(0) = 0 implies cy = 0 and so y = cicosVXz, Now the condition »/(Z) = ~oVisin VRE = 0, For oy #0 this condition will hold when V/A. = nr or X = n?n2/2, where n=1,2,3, ‘These are the positive eigenvalues with corresponding eigenfunctions cos(nm/L)z, n=1,23, 4. For A< Owe have y=cyoosh Na + egsinh VX of =cv=Xsink V=Az + av—Xeosh V=Az. 518 Exercises 11.4 Using the fact that cosh is an even function and sinh 2 is odd we have y(-L) = ¢ cosh(—v=3 L) + ep sink(—V=K L) = ccosh V=XL — ep sinh V=XL and 4 VT Asinh(-V=RL) + e2V=Neosh(-Y=HB) = -c,¥—Asinh VEAL + egV=Neosh v= L. The boundary conditions imply cy cosh VAL = cosinh V=XL = €y cosh VAL + epsinh V=AL vCh 2ep sinh V=XL = 0 xd 0, V=Xsink VAL + V—Keosh V=AL = er V=Asinh VAL + cpv—oosh V=AL dey/ZReinh VEEL = 0 Since V=AL #0, e and the only solution of the boundary-value problem in this case is peo. For \=0.we have yearte yea From y(~L) = y(L} we obtain -ab+e=ab+e ‘Then ¢; = 0 and y= 1 is an eigenfunction corresponding to the eigenvalue A= 0. For A> Owe have ye cra Vie + sin Vie Wisin Vix + V3 cos Viz. ve ‘The first boundary condition implies eycos VAL cpsin VAL =e) cos VAL + egsin VAL 2egsin VAL 519 Exercises 11.4 ‘Thus, fey =O and o #0, : wn? ViL=ne or X=", n= 1,23, ‘The corresponding eigenfunctions are sin Fa, for n= 1,2, 3,.... Similarly, the second boundary condition implies 2eyVXsin VAL = 0. Ifeq = Oand a £0, eo vi eee nena, and the corresponding eigenunctions are cor Ms, for m= 12,3 5. The eigenfunctions are cos V%qx where cot V%q = W3n- Thus loos Pinaft= fons! Yinede = 1! (1+ cos2yina) de =} (c+ ghesnases) [+3 (1+ ghana) = hf de (ten Virco vn) levee Feat asin yo] a3he re (eV) ve («i vin)) = £ (1 +m? Ym) 6. The eigenfunctions exe sin /Xq x where tan /%q = ~Aq. Thus Usia Vinal? = ['sin® Masao = 3 f' (1 ven) ae sees) -aiea) are (#80 Vom Vin) ] Ft Vino Pinson fa] ~ Fa (-Vro F)] =} (94) 520 Exercises 11.4 7. (a) HEX-<0 the initial conditions imply y = 0. For >> O the geveral solution of the Cauchy-Buler Aiferential equation is y = o,cos(VX In) + epsin(V/X Inc). ‘The condition y(1) = 0 implies ¢, = 0, s0 thet y = exsin(V Inz). ‘The condition y(5) = 0 implies VI In = nn, n= 1, nn Thus, the eigenvalues are n?x?/(In5)* for n = 1, 2, 3, ..., with corresponding sigenfunctions sin{{07/103) nz} (b) The self-adjoint form is fevitdy (€) An orthogonality relation is s0(@Eiae) sin (@ ins) ae =o 8. (a) The roots ofthe auxiliary equation 4m += 0 are H(—1 4 VT=AR). When A= 0 the general solution ofthe diferential equation i c1-+ene~®. The initial conditions imply ey+e2 = 0 and ¢; + cze7? = 0. Since the determinant of the coefficients is not 0, the only solution of this homogeneous syttem is ¢1 = cp = 0, in which case y = 0. Similarly, if 0 < A-< J, the general solution is eh IAVEB = 4 eyeh(I-VRe ¥ In this case the initial conditions again imply 1 = co = eneral solution of the differential equation i y= ce cos Vika 12+ one"? ehn VINO Te, ‘The condition y(0) = 0 implies cy = 0 s0 y= @2¢~*/?sin VEN Tz. From (2) = ge-sin VENT and 20 y= 0, Now, for A> }, the ‘we see that the eigenvalues are determined by 2VIV—1 = nx for n = 1, 2, 3, .... Thus, the eigenvalues are n2n2/4? + 1/4 for n = 1, 2, 3, -.. , with corresponding eigenfunctions er ain ME +e (b) The self-adjoint form is fie - Bley + ey =0. (6) An orthogonality relation is es (e-#?% sin ™ 2) (e-=!eos 2) de= [ (sin 2) (0082) de = fe (eran) (e*Pom Es) dom fl (ans) (con Fs) = 0 9. To obtain the self-adjoint frm we note that an integrating factor is (1/2)e/OM#H/= =e, Thus the dieretia equation i rety! + (I= a)ety tne ty 521 10. uw Exercises 11.4 and the self-adjoint form is, 4 fe-*y/] L feey| + ne enilyng the weight facia a) = e°* and noting that sine r(e) = 2%, (0) = 0 an lims-co "(2) = 0, we have the orthogonality relation [fF tala im(e) dr = 0, m To obtain the self-adjoint form we note that an integrating factor is ¢/ ~2*4* Thus, the differential equation is Dee" y + One ‘and the self-adjoint form is lim oe M(t) = dentifying the weight function p(2) = 2e~** and noting that since r(x) = | we have the orthogonality relation Jims oe 2) [26°F Hola) Hin(a) de = 0, m x (a) The differential equation is (ato! + 2ay + d Tre? Letting 2 = tand we have = tan” x und, ay = &y __ 2 dy we aye Weep ae ‘The dilferentil equation can then be written in terms of y(@) as, 1 fy ay yf 1 ay, ase Tray a, Ts3 aw) *isa? ase 12. 15. Exercises 11.4 The boundary conditions become y(0) = y(n/4) = 0. For A < 0 the only solution of the boundary-value problem is y = 0. For A> Othe general solution ofthe diferential equation is y= cy cosVX@ + cpsin VA6. The condition y(0) = 0 implies cy) = @ so y = cp sin WA8. Now the condition y(x/4) = O implies opsin Vi7/4 = 0. For cp #0 this condition wil old when Vin/d = ne or A= 16n?, where n = 1,2,8,.... These are the eigenvalues with erresponding =12,3, eigenfunctions sin 4n@ = sin(4n tan! 2}, for (b) Ap orthogonality relation is el eld [ physotiniar!yentintalper=a (a) This is the parametric Bessel equation with » = 1. The general solution is y= ers (Aa)ea¥i (Az), Since ¥ is bounded at 0 we must have co = 0, so that y = c14(2). The condition J3(3A) = 0 defines the eigenvalues 22, A2, Ag)...» (When i= 0 the differential equation is Cauchy-Euler and the only solution satisfying the boundary condition is y = 0, so A = 0 is not an eigenvalue.) (b) From Table 6.1 in Section 6.4 in the text we see that eigenvalues are determined by 341 = 3.892, (016, 34s = 10.173, and 34 = 19,323, The first four eigenvalues are thus &y = 1.2773, 2.9987, Ag = 3.391, and Ay = 4.441. When A= 0 the differential equation is r(z}y” + (z)p! = 0. By inspection we soe that y = 1 is @ solution of the boundary-value problem. Thus, X= 0 s an eigenvalue (a) An cxthogonality relation is ff cosas cosznt =0 fo where tm # ty are positive solutions of cot = =. (b) Referring to Problem 1 we use a CAS to compute 1 (f (e0s0, 86082) (ce03.42562) dr = 1.8771 x 1076 (a) Ab orthogonality relation is ff m8 tnt sin 392)(24 conn ~ $n 242) de = 0 where 2m # tq ate positive solutions of tan = 2 (b) Referring to Problem 2 we use a CAS to compute 1 [soso sate nse 7 sos 1.59 — sin. 5) de = -2s80 2 104 523 Exercises 11.5 Exercises 11.5 1. Identifying b = 3, the first four eigenvalues are 3.832 we SR waar dy = 26 2.330 dq = 10278. 3.301 13.333 “ 441. 3 2. We fist note from Case IIT inthe text that 0 is an eigenvalue. Now, since Jf(2A) = 0 is equivalent to Ji(2X) = 0, the next three eigenvalues aré ‘8, The boundary condition indicates that we use (15) and (16) in the text. With & = 2 we obtain a = GREK i 2Jo(Mz) dz tone demas 1 aps = aan xh tdot) dt 1 pea = aaron b glo] ae [From (4) in the text] 1a - ah a WAM Tous Exercises 11.5 4. The boundary condition indicates that we use (19) and (20) in the text, With b= 2 we obtain Ae Tim ff wolda) de 1p = ORD xh suJo(t) dt 24 = aero h Og (From (4) in the text] ' as = aol A(2d) XRD) Now since J§(24) = 0 is equivalent 10 Ji(2) = 0 we conclude 6 = Ofor {= 2,3, 4,.... Thus 5. The boundary condition indicates that we use (17) and (18) in the text. With b= 2 and h=1 we obtain mop DEE AG fb MRA 2 1 py ED xh tot) at = a f ie Sin ionae [From (4) in the text) 2 ms ~ are ANJA) DP + IWR) Thus Aid(2%) We) = 43 ary yaaa 525 Exercises 11.5 6. Writing the boundary condition in the form 2dp(2A) + 2AJ§(2A) = 0 ‘we identify b = 2 and h = 2. Using (17) and (18) in the text we obtain af an wraFaoh edo( Miz) dz x 1 ps “aria eh ode =m a ati f Siwau(oae (From (4) in the text] A ae = Ee ra, __Awhl2) = De DFR) Thus SB _Ai(2M) 10) =F GP pany) 7. The boundary condition indicates that we use (17) and (18) in the text. With n= 1, 6 = 4, and = 3 we obtain 2 &* Tei + Daa sh ash(e)6ede . 5a Ll an eat “aarti Bh Ol 5 HN do P “pera cf Hit alellde [From (4) in the text] 5 oh - merry 2, = 20nJeLan) * OY D AO) Thus S_sh(tn) 1) = 205 yn 526 Exercises 11.5 8. The boundary condition indicates that we use (15) and (16) in the text, With n = 2 and b= 1 we obtain ao * a)z" dz a= Fay h shot ee * HO) 3 fo Pn{t) at = aay h Gene [Pro (4) in the text » — YedsF Thus He) #23. say 20 9. The boundary condition indicates that we use (19) and (20) in the text, With = 3 we obtain 2 2 = apptany fh semaistde ee ear ~ aman Hh Pe fea am h Pao ual d= Epa du=2d — v=th(t) = arian (C20 2p ence) With n = 0 in equation (5) in the text we have J§(z) = —vi(z), so the boundary condition 527 Exercises 11.5 Jj(3X:) = 0 implies J1(3A,) = 0. Then 2 OM da : om agam (20 $201 4) = ar 842.2(8N)] = ae 2 DION, ‘Thus (3%) 9 1) = 3-45 PE he) 10, The bocndary condition indicates that we sse (15) snd (16) in the text. With 8 =1 it follows that a spay ff 2(t- 2) slae)ae = sing | meee [ Pn0.2r4s] (ee _ ae] = aH [ap fae 5x [erate] Sta (at) 705 | lz f Sento ae wah wa EPAO a dua%d — v=thi) “Fos [ene - Alea), -2 {ene «)) ~ 705 EY ge - 3) ah £ Pato] a] a 420) “7 iF 20) "ROY f= 45 8 0. 528 Exercises 11.5 (b) Using FindRoot in Mathematica we find the roots 2 = 2.9496, 72 = 5.84113, zy = 8.87273, 24 = 11.9561, and zp = 15,0624. (©) Dividing the roots in part (b) by 4 we find the eigenvalues Ay = 0.7374, Ay = 1.46028, dg = 2288, 4 = 2.98004, and Ag = 9.76559. (A) The next five eigenvalues are Xe = Do = 7.6749, 54508, ry w 5.32626, Xe = 6.1085, Ag = 689145, and 12. (a) From Problem 7, the coefficients of the Fourier-Bessel series are = _20mh(4¥) 1 AFF 1) IRAN)” Using a CAS we find o = 26,7896, ep = —12.4624, cy = 7.1404, cy = ~4.68705, and y= 3.88609. (b) og 32 3 se ss 4 : : 8 : V\ : : I ; sak » Yaw: Y »& ©) so 3 529 Exercises 11.5 13, We compute te Lp tf y % 5 ff 2Polzyae 3h = oa 3yf 7 3p mas al a5 f) erlside= 5 [star = 5 ef A A ee 5 —T* ff 2Pata) de =5 [5182 - 2)dz = 7 Tp Leese ff xPale)de = 5 [50628 -32")a2 =0 1 9 flees ed Pa(x) da 3h 052" — 802° + 32)dr = — 5 1 1 2 ['steyde= 2 [ He3e! — roxt + 152"\de =o Ce a=! ftorieide= 3 ff Leone —oise + 082 50) = 3 Thus (2) = 1A) +2 At) + Lae) - Sa)+ Baie a 6 32 Pla) + 5g Ole) + 14, We compute tay 1 1 hy " on} fener [ten feet) a dn oA. - 4 a= 5f eAlara3 [zeae = 36 a : : anSflenar=$' Yate ees oo = Ne-16) Th pa 1 - engl chilean ff) Lente —aret\de = F(—5e-+ 3764) 530

You might also like